ALL MED SURG NOTES FOR FINAL

Réussis tes devoirs et examens dès maintenant avec Quizwiz!

In teaching a patient about coronary arter disease, the nurse explains that the changes that occur in this disorder include (select all that apply)

- Abnormal levels of cholesterol, especially ow-density lipoproteins - Accumulation of lipid and fibrous tissue within the coronary arteries - Development of angina due to a decreased blood supply to the heart muscle Atherosclerosis is the major cause of coronary artery disease (CAD) and is characterized by a focal deposit of cholesterol and lipids, primarily within the intimal wall of the artery. The endothelial lining of the coronary arteries becomes inflamed from the presence of unstable plaques and the oxidation of low-density lipoprotein (LDL) cholesterol. Fibrous plaque causes progressive changes in the endothelium of the arterial wall. The result is a narrowing of the vessel lumen and a reduction in blood flow to the myocardial tissue.

Lifestyle modifications for Hiatal hernia:

- Eliminating dietary factors (such as caffeine-containing beverages, chocolate, peppermint) - Small frequent meals - Elevating the head of the bed on blocks -Teach the patient to reduce intraabdominal pressure by eliminating constricting garments and avoiding lifting and straining.

You are caring for a patient with ADHF who is receiving IV dobutamine (Dobutrex). You know this drug is ordered because it (select all that apply)

- Improves contractility - Works on the B1-receptors in the heart Dobutamine (Dobutrex) has a positive chronotropic effect and increases heart rate and improves contractility. It is a selective β-adrenergic agonist and works primarily on the β1-adrenergic receptors in the heart. It is frequently used in the short-term management of acute decompensated heart failure (ADHF).

A patient is admitted to the hospital in hypertensive emergency (BP 244/142 mmHg). Sodium nitroprusside is started to treat the elevated BP. Which management strategy(ies) would be most appropriate for this patient? (select all that apply)

- Measure hourly urine output - Continuous BP monitoring with an arterial line - Assessing the patient for signs and symptoms of heart failure and changes in mental status Measure urine output hourly to assess renal perfusion. Patients treated with IV sodium nitroprusside should have continuous intraarterial BP monitoring. Hypertensive crisis can cause encephalopathy, intracranial or subarachnoid hemorrhage, acute left ventricular failure, myocardial infarction, renal failure, dissecting aortic aneurysm, and retinopathy. The initial treatment goal is to decrease the mean atrial pressure (MAP) by no more than 25% within minutes to 1 hour. Patients receiving IV antihypertensive drugs may be restricted to bed rest. Getting up (e.g., to use the toilet/commode) may cause severe cerebral ischemia and fainting.

Which are clinical findings in a person with an acute lower extremity VTE? (select all that apply)

- Mild to moderate calf pain and tenderness - Unilateral edema and induration of the thigh The patient with lower extremity venous thromboembolism (VTE) may or may not have unilateral leg edema, extremity pain, a sense of fullness in the thigh or calf, paresthesias, warm skin, erythema, or a systemic temperature greater than 100.4 F (38 C). If the calf is involved, it may be tender to palpation.

A patient with chronic HF and atrial fibrillation is treated with a digitalis glycoside and a loop diuretic. To prevent possible complications of this combination of drugs, what does the nurse need to do? (select all that apply)

- Monitor serum potassium level - Teach the patient how to take a pulse rate Hypokalemia, which can be caused by the use of potassium-depleting diuretics (e.g., thiazides, loop diuretics), is one of the most common causes of digitalis toxicity. Low serum levels of potassium enhance the action of digitalis, causing a therapeutic dose to achieve toxic levels. Hypokalemia can also precipitate dysrhythmias. Monitoring the serum potassium levels of patients receiving digitalis preparations and potassium-depleting diuretics is essential. Patients taking digitalis preparations should be taught how to measure their pulse rate because bradycardia and atrioventricular blocks are late signs of digitalis toxicity. In addition, patients should know what pulse rate would require a call to the HCP. Patients should not independently decide to skip a dose of digitalis.

Which nursing responsibilities are priorities when carinf for a patient returning from a cardiac catheterization? (select all that apply)

- Monitor vital signs and ECG - Checking the catheter insertion site and distal pulses The nursing responsibilities after cardiac catheterization include assessment of the puncture site for hematoma and bleeding; assessment of circulation to the extremity used for catheter insertion and of peripheral pulses, color, and sensation of the extremity; and monitoring vital signs and electrocardiographic rhythm. Other nursing responsibilities are described in Table 31-6.

Which BP-regulating mechanism(s) can result in the development of hypertension if defective? (select all that apply)

- Release of norepinephrine - Stimulation of the sympathetic nervous system - Activation of the renin-angiotensin-aldosterone system Norepinephrine (NE) is released from the sympathetic nervous system nerve endings and activates receptors located in the vascular smooth muscle. When the α-adrenergic receptors in smooth muscle of the blood vessels are stimulated by NE, vasoconstriction results. Increased sympathetic nervous system stimulation produces increased vasoconstriction and increased renin release. Increased renin levels activate the renin-angiotensin-aldosterone system, leading to elevation in BP.

Which clinical manifestations are seen in patients with either Buerger's disease or Raynaud's phenomenon? (select all that apply)

- Sensitivity to cold temperatures - Gangrenous ulcers on fingertips - Color changes of fingers or toes Both Buerger's disease and Raynaud's phenomenon have the following clinical manifestations in common: cold sensitivity, ischemic and gangrenous ulcers on fingertips, and color changes of the distal extremity (fingers or toes).

When collecting subjective data related to the cardiovascular system, which information should be obtained from the patient? (select all that apply)

- Smoking history - Religious preference - Number of pillows used to sleep The health history should include assessment of tobacco use. The patient should be asked about any cultural or religious beliefs that may influence the management of the cardiovascular problem. Patients with heart failure may need to sleep with the head elevated on pillows or sleep in a chair.

How to decrease dumping syndrome

-A short rest period after each meal reduces the chance of the syndrome -Divide meals into 6 small feedings to avoid overloading the stomach -Do not take fluids with meals but at least 30-45 min before or after meals. This helps prevent distension -Avoid concentrated sweets (honey, sugar, jelly, jam, candies, sweet pastries) because they can cause dizziness, diarrhea, and a sense of fullness -Avoid dairy and simple carbs -Increase protein and fats and carbohydrates to promote rebuilding of body tissues and to meet energy needs. Meat, cheese, and eggs are specific foods to increase in the diet

S/S of IBS

-All patient experience different symptoms, most are going to have diarrhea or constipation, or both. This is diagnosed solely on these symptoms. Other common symptoms include abdominal distention, nausea, flatulence, bloating, urgency, mucus in the stool, and sensation of incomplete evacuation. Non-GI symptoms may include fatigue, headache, and sleep disturbances.

Symptoms associated with hyperthyroidism:

-Anxiety - Tremor -Restlessness - Tachycardia -Goiter - Feeling warm - Loss of weight -Bruit over gland -Arrhythmias -Decreased fertility

Prevention of aspiration for tube feeds:

-Aspiration risk can be decreased by checking volume of stomach (if volume is to high chance of aspiration increases), keep HOB elevated, and flush tubes and make sure they are in the proper place -Check gastric contents every 4 hours in first 48 hours of placement and in critically ill • Every 6-8 in those who are not critically ill • Can give erythromycin and metoclopramide to increase gastric emptying if getting too full.

H. pylori

-Bacterial Infection mostly occurs during childhood and given through a fecal-oral or oral-oral route. -The bacteria can live in the stomach for a long time by colonizing the gastric epithelial cells within the mucosal layer. -The bacteria produce urease, which metabolizes urea-producing ammonium chloride. This leads to increased gastric secretion and produces tissue damage, leading to PUD.

Complications of E. Coli

-Can cause hemorrhagic colitis and kidney failure -Can be life threatening for the very young and old -Hemolytic uremic syndrome - the RBCs are destroyed and the kidneys fail. Blood transfusions and kidney dialysis are often required. Also cause HTN, seizures, blindness, and paralysis

Why does dumping syndrome occur:

-Can occur after peptic ulcer disease surgery -Normally, gastric chyme enters the small intestine in small amounts -After surgery, the stomach no longer has control over the amount of gastric chyme entering the small intestine -A large bolus of hypertonic (pull fluid to the area) fluid enters the intestine and causes fluid to be drawn into the bowel lumen. This creates a decrease in plasma volume, distention of the bowel lumen, and rapid intestinal transit

Medication Tx for GERD

-Cholinergic (increases LES pressure) -Reglan & Propulsid ( stimulate increased GI motility) -Histamine H2 receptor antagonist: Pepcid, Tagament, Zantac -Antacids -PPI (increase calcium intake to avoid calcium deficiency): Prilosec, Prevacid, Protonix

Perforation of PUD

-Considered the most lethal complication of PUD -Immediate action is to stop the spillage of gastric or duodenal contents into the peritoneal cavity and to restore blood volume -An NG tube can provide continuous aspiration and gastric decompression to stop spillage through the perforation. -Circulating blood products are replaced with LR -Sudden and acute pain, hard rigid abdomen, considered and emergent acute situation

Gastric outlet obstruction of PUD

-Decompress the stomach, correct any existing fluid and electrolyte imbalances, and improve the patient's general state of health. -This decompression can allow the ulcer to heal -Obstruction occurs at the site of the pyloric sphincter and can cause the patient to vomit, feel full or be nauseous as the food in their stomach cannot pass -Abdominal distention, hypoactive or absent bowel sounds

Examples of PPIs

-Esomeprazole (Nexium) -Lansoprazole (Prevacid), -Omeprazole (Prilosec)

H2 Blockers:

-Famotidine (Pepcid) -Ranitidine (Zantac)

E. Coli

-Found primarily in undercooked meats, especially poultry and hamburger. Outbreaks have been with leafy veggies, fruits, and nuts -Infection can occur after drinking raw milk, unpasteurized juice, or contaminated fruit juices and after swimming in or drinking sewage-contaminated water. -Most strains are harmless and live in the intestines of healthy humans and it produces a powerful toxin

S/S of GERD

-Heartburn (dyspepsia), substernal pressure -Regurgitation, dysphagia -Laryngospasm, bronchospasm -Aspiration pneumonia -Atypical complaints

Symptoms associated with hypothyroidism:

-Hypocholesterolemia - Gains weight easily - Decreased fertility -Delayed reflexes - Sluggishness - Feeling cold -Thick tongue - Constipation - Lethargy - Fatigue -Can cause yellow tint to skin -Puffy face -Poor attention span

MOA for Antacids:

-MOA: Neutralize HCl acid, taken 1-3 hours after meals and at bedtime

Safety administrations for Enteral nutrition:

-Patients can remove their tubes accidentally, we need to ensure they are getting adequate nutrition, check site for skin irritation, flush tube every 4-6 hours with 30 ml of saline to keep line patent & check for residual -Check tube positioning in insertion and prior to feeding. -Monitoring nutritional status and tolerance of feeding -Obtain abdominal X-ray to confirm placement (if done blindly) -Monitor patient closely with diabetes

Symptoms of Type 2 Diabetes

-Polyuria- abnormal large amounts of diluted urine -Polydispia- great thirst -Polyphagia- excessive hunger, increased appetite Obesity, lack of physical activity, age, gestational diabetes Metabolic Syndrome: hypertension, dyslipidemia, hyperinsulinism, "apple shaped" obesity, glucose intolerance Improves with weight reduction DKA is usually not a symptom

Treatments of E. coli

-Supportive care to maintain blood volume -Antibiotic therapy is not proven to be effective -Should avoid antidiarrheal agents -Possible dialysis and plasmapheresis

Symptoms of E. coli

-Symptoms are bloody diarrhea, cramping pain that lasts from 2-8 days after infection. Hemolytic uremia and thrombocytopenia purpura

Dumping Syndrome:

-Symptoms begin 15 to 30 minutes after eating. There is weakness, sweating, palpitations, and dizziness. These are due to the sudden decrease in plasma volume and there are abdominal cramps, borborygmi, and the urge to defecate (these usually last less than 1 hour after eating. -Gastric contents empty too rapidly into the small intestine, overwhelming its ability to digest nutrients. Symptoms include vomiting, nausea, weakness, sweating, faintness, and on occasion, diarrhea. Patients are discouraged from eating sugary foods after surgery to avoid

Sliding hiatal hernia

-The junction of the stomach and esophagus is above the diaphragm, and a part of the stomach slides through the hiatal opening in the diaphragm. This occurs when the patient is supine -The hernia usually goes back into the abdominal cavity when the patient is standing upright.

Hemorrhage condition with PUD:

-The most common complication of PUD -They frequently experience bleeding, rigid abdomen, dropping BP, increased pain, and nausea and vomiting -Can lead to shock

A 42-year-old patient is admitted to the emergency department (ED) following being mugged. The patient received blunt trauma to the face and has clear fluid draining from the right nostril. What does the ED nurse know to assess this fluid for? Sodium Protein Calcium Glucose

...

In addition to routine postoperative care, immediate plans by the nurse for a laryngectomy patient will focus on the need for: Adequate fluid and electrolyte balance Prevention of hemorrhage Pain control Patent airway

...

The return of peristalsis in the postoperative period can be determined by the presence of ___________________________ and _________________________________, both of which are assessed by the nurse

...

Two potential postoperative complications following abdominal surgery are: ____________ and _________________.

...

Basophils WNL

0 to 0.02

Eosinophils WNL

0 to 0.4

A Mantoux skin test is considered not significant if the size of the induration is: a. 0 to 4 mm. b. 5 to 6 mm. c. 7 to 8 mm. d. 9 mm.

0-4mm

Monocytes

0.04 to 0.08

lymphocytes

0.20 to 0.40

Neutrophils WNL

0.50 to 0.70

A patient is admitted to the ICU with a diagnosis of unstable angina. Which drug(s) would the nurse expect the patient to receive? (select all that apply)

- ACE inhibitors - Antiplatelet therapy - Intravenous nitroglycerin In addition to oxygen, several drugs may be used to treat unstable angina (UA): IV nitroglycerin, aspirin (chewable), and morphine. For patients with UA with negative cardiac biomarkers and ongoing angina, a combination of aspirin, heparin, and a glycoprotein IIb/IIIa inhibitor (e.g., eptifibatide [Integrilin]) is recommended. Angiotensin-converting enzyme (ACE) inhibitors decrease myocardial oxygen demand by producing vasodilation, reducing blood volume, and slowing or reversing cardiac remodeling.

PPN is used when

1) nutritional support is needed for only a short time 2) protein and caloric requirements are not high 3) the risk of a central catheter is too great 4) used to supplement inadequate oral intake

The assumption behind behavior modification is twofold:

1) obesity is learned disorder caused by overeating 2) often the critical difference between an obese person and person of normal weight is the cues that regulate eating behavior

Fats in the diet can be divided into

1) potentially harmful (saturated fat and trans fat) and 2) healthier dietary fat (monounsaturated and polyunsaturated fat)

The client diagnosed with sickle cell anemia is admitted to the hospital for a vaso-occlusive crisis. The nurse reviews admission prescriptions. Which prescriptions does the nurse question? (Select all that apply.) 1. Restriction of oral fluid intake 2. Water aerobic therapy for joint pain 3. High calorie, high protein diet 4. Acetaminophen 10 mg/kg every 6 hours as needed for pain 5. Oxygen via nasal cannula at 2 L/min

1, 2, 4 1) CORRECT - In vaso-occlusive crisis, IV and oral fluids are the primary treatment to reduce blood viscosity. The nurse needs to question this prescription. 2) CORRECT - In vaso-occlusive crisis, activity is limited to decrease oxygen needs, which prevents further sickling. The nurse needs to question this prescription. 3) INCORRECT - In vaso-occlusive crisis, the client needs to eat a high calorie, high protein diet, which supports metabolic demands. 4) CORRECT - Vaso-occlusive crisis is characterized by severe pain. A combination of opioid and nonopioid therapy are used for management of mild to moderate pain, whereas opioid analgesics are used for severe pain. The nurse needs to question this prescription. 5) INCORRECT - In vaso-occlusive crisis, oxygen is indicated to prevent further sickling of cells.

The nurse provides care for a client experiencing diabetic ketoacidosis (DKA). Which findings will the nurse expect when assessing this client? (Select all that apply.) 1. Poor skin turgor 2. Decreased urine output 3. Elevated blood glucose 4. Tachycardia 5. Orthostatic hypotension

1, 3, 4, 5 1) CORRECT - Poor skin turgor results from the dehydration that occurs in DKA. 2) INCORRECT - The client would experience polyuria because the body attempts to rid itself of glucose through the kidneys. 3) CORRECT - Elevated blood glucose levels result from the profound deficiency of insulin. 4) CORRECT - Tachycardia results from the dehydration that occurs in DKA. Tachycardia is a compensatory mechanism to keep the cardiac output adequate in times of fluid volume deficit. 5) CORRECT - DKA results in dehydration. Fluid is eliminated by the kidneys in an attempt to excrete the high levels of glucose. Dehydration causes orthostatic hypotension.

The nurse notes that a client with type 2 diabetes mellitus has a glycosylated hemoglobin A1c level of 6.5% (0.065). Which action will the nurse implement based on the current data? 1. Inform the client that the current treatment is effective 2. Obtain a fingerstick blood sample for glucose testing 3. Counsel the client about measures to better control blood glucose 4. Notify the health care provider immediately of the test result

1.Inform the client that the current treatment is effective 1) CORRECT- Normal glycosylated hemoglobin A1c values typically range from 4% to 6%. The target range for people with diabetes is less than 7%, so the nurse should inform the client that the current treatment regimen is effective. 2) INCORRECT - There is nothing to suggest that a fingerstick blood sample for glucose testing is needed. 3) INCORRECT - The client's glycosylated hemoglobin A1c falls within the target range for clients with diabetes, which is less than 7%. 4) INCORRECT - It is not necessary to notify the health care provider of the test result because it falls within the target range.

When reviewing the results of an 83-year-old patient's blood tests, which finding would be of most concern to the nurse? a. Platelets 150,000/μL b. Serum iron 50 mcg/dL c. Partial thromboplastin time (PTT) 60 seconds d. Erythrocyte sedimentation rate (ESR) 35 mm in 1 hour

10. c. As a person ages the partial thromboplastin time (PTT) is normally decreased, so an abnormally high PTT of 60 seconds is an indication that bleeding could readily occur. Platelets are unaffected by aging and 150,000/μL is a normal count. Serum iron levels are decreased and the erythrocyte sedimentation rate (ESR) is significantly increased with aging, as are reflected in these values.

11. A patient with a bone marrow disorder has an overproduction of myeloblasts. The nurse would expect the results of a complete blood count (CBC) to include an increase in which cell types (select all that apply)? a. Basophils b. Eosinophils c. Monocytes d. Neutrophils e. Lymphocytes

11. a, b, d. The myeloblast is a committed hematopoietic cell found in the bone marrow from which granulocytes develop. A disorder in which myeloblasts are overproduced would result in increased basophils, eosinophils, and neutrophils.

12. During the nursing assessment of a patient with anemia, what specific information should the nurse ask the patient about? a. Stomach surgery b. Recurring infections c. Corticosteroid therapy d. Oral contraceptive use

12. a. The parietal cells of the stomach secrete intrinsic factor, a substance necessary for the absorption of cobalamin (vitamin B12), and if all or part of the stomach is removed, the lack of intrinsic factor can lead to impaired RBC production and pernicious anemia. Recurring infections indicate decreased WBCs and immune response and corticosteroid therapy may cause a neutrophilia and lymphopenia. Oral contraceptive use is strongly associated with changes in blood coagulation.

What are the ranges for fasting glucose for a diabetic person?

125 mg/dL and above

Hemoglobin WNL - male

13.2 to 17.3

Electrolyte Review - Sodium - important in cardiac function

135 to 145

It is estimated that health-care associated infections occur in the surgical site in what percentage of surgical patients? a. 5% to 10% of surgical patients. b. 14% to 16% of surgical patients. c. 20% to 30% of surgical patients. d. Approximately 57% of surgical patients.

14% to 16% of surgical patients.

What are the ranges for postprandial (after eating) glucose for a prediabetic person?

141-200 mg/dL

15. During physical assessment of a patient with thrombocytopenia, what would the nurse expect to find? a. Sternal tenderness b. Petechiae and purpura c. Jaundiced sclera and skin d. Tender, enlarged lymph nodes

15. b. Petechiae are small, flat, red, or reddish-brown pinpoint microhemorrhages that occur on the skin when platelet levels are low. When petechiae are numerous, they group, causing reddish bruises known as purpura. Sternal tenderness is associated with leukemias. Jaundice occurs when anemias are of a hemolytic origin, resulting in accumulation of bile pigments from RBCs. Enlarged, tender lymph nodes are associated with infection or cancer.

Platelets WNL

150,000 to 400,000

16. A patient with a hematologic disorder has a smooth, shiny, red tongue. Which laboratory result would the nurse expect to see? a. Neutrophils 45% b. Hgb 9.6 g/dL (96 g/L) c. WBC count 13,500/μL d. Red blood cell (RBC) count 6.4 × 106/μL

16. b. A smooth, shiny, reddened tongue is an indication of iron-deficiency anemia or pernicious anemia that would be reflected by a decreased hemoglobin level. The decreased neutrophils would be indicative of neutropenia. The increased WBC count could be indicative of an infection and the increased RBC count of polycythemia.

Normal BMI:

18.5 - 24.9

When planning care for adult patients, which oral intake is adequate to meet daily fluid needs of a stable patient?

2000 to 3000 mL Daily fluid intake and output is usually 2000 to 3000 mL. This is sufficient to meet the needs of the body and replace both sensible and insensible fluid losses. These would include urine output and fluids lost through the respiratory system, skin, and GI tract.

21. When teaching a patient about a bone marrow examination, what should the nurse explain? a. The procedure will be done under general anesthesia because it is so painful. b. The patient will not have any pain after the area at the puncture site is anesthetized. c. The patient will experience a brief, very sharp pain during aspiration of the bone marrow. d. There will be no pain during the procedure, but an ache will be present several days afterward.

21. c. The aspiration of bone marrow content is done with local anesthesia at the site of the puncture but the aspiration causes a suction pain that is quite painful but very brief. There generally is no residual pain following the test.

24. Molecular cytogenetics and gene analysis may be done to diagnose, stage, and help to determine treatment options for various hematologic disorders. Which sites are preferred to obtain the sample for this testing (select all that apply)? a. Skin sample b. Lymph node c. Bone marrow d. Arterial blood e. Inner cheek mucosa

24. b, c. Bone marrow and lymph node biopsies are preferred methods to obtain the sample for gene analysis. If a large number of abnormal cells are circulating in the blood, peripheral blood may be used. The other options will not provide the desired information.

Physiological Integrity COMPLETION 1. When analyzing an electrocardiographic (ECG) rhythm strip of a patient with a regular heart rhythm, the nurse counts 30 small blocks from one R wave to the next. The nurse calculates the patient's heart rate as ____.

50 There are 1500 small blocks in a minute, and the nurse will divide 1500 by 30. DIF: Cognitive Level: Remember (knowledge) REF: 789-790 TOP: Nursing Process: Assessment MSC:

Serum Iron

50 - 175

. Acute respiratory failure (ARF) occurs when oxygen tension (PaO2) falls to less than _____ mm Hg (hypoxemia) and carbon dioxide tension (PaCO2) rises to greater than ______ mm Hg (hypercapnia). a. 50 and 50 b. 60 and 60 c. 75 and 75 d. 80 and 80

50 and 50

A dietary reduction of at least

500 to 1000 cal/day is recommended for an expected weight loss of 1 to 2 lb/wk

Prophylactic isoniazid (INH) drug treatment is necessary for about how many months? a. 3 months b. 3 to 5 months c. 6 to 12 months d. 13 to 18 months

6-12 months

7. Which component of normal hemostasis involves the processes of protein C and protein S and plasminogen? a. Lysis of clot b. Vascular response c. Plasma clotting factors d. Platelet plug formation

7. a. Protein C and protein S are examples of anticoagulants that are involved in the lysis of clots. Fibrinolysis is also achieved by thrombin-activating conversion of plasminogen to plasmin, which attacks fibrin or fibrinogen and splits it into smaller elements known as fibrin split products (FSPs) or fibrin degradation products (FDPs).

Pharyngitis: Acute

70% are viral 15-20% are beta hemolytic streptococcus Or gonorrhea, diphtheria, fungal Culture before treatment

What are the ranges for fasting glucose for a normal person?

70-110 mg/dL

What are the ranges for postprandial (after eating) glucose for a normal person?

70-140 mg/dL

IV fat emulsion administered separately should be administered over a course of

8 to 10 hours, and the infusion rate should not exceed 0.11 g/kg/hour

8. A patient who was in a car accident had abdominal trauma. Which organs may be damaged and contribute to altered function of the hematologic system (select all that apply)? a. Liver b. Spleen c. Stomach d. Gallbladder e. Lymph nodes

8. a, b, e. The abdominal organs that are primarily involved in hematologic function are the liver, spleen, and lymph nodes. The liver filters the blood, produces procoagulants, and stores iron. The spleen removes old and defective erythrocytes and filters iron for reuse. The lymph nodes filter pathogens and foreign particles from lymphatic circulation.

In the vertical sleeve gastrectomy

85%of the stomach is removed, leaving a sleeve shaped stomach. It's function is preserved. The removal of the majority of the stomach results in elimination of hormones produced in the stomach that stimulate hunger, such as ghrelin

Underweight BMI:

<18.5

A 58-year-old woman has just returned to the nursing unit after an esophagogastroduodenoscopy (EGD). Which action by unlicensed assistive personnel (UAP) requires that the registered nurse (RN) intervene? a. Offering the patient a drink of water b. Positioning the patient on the right side c. Checking the vital signs every 30 minutes d. Swabbing the patient's mouth with cold water

A

The nurse recognizes that primary manifestations of systolic failure include a. ↓ EF and ↑ PAWP. b. ↓ PAWP and ↑ EF. c. ↓ pulmonary hypertension associated with normal EF. d. ↓ afterload and ↓ left ventricular end-diastolic pressure.

A

To palpate the liver during a head-to-toe physical assessment, the nurse a. places one hand on the patient's back and presses upward and inward with the other hand below the patient's right costal margin. b. places one hand on top of the other and uses the upper fingers to apply pressure and the bottom fingers to feel for the liver edge. c. presses slowly and firmly over the right costal margin with one hand and withdraws the fingers quickly after the liver edge is felt. d. places one hand under the patient's lower ribs and presses the left lower rib cage forward, palpating below the costal margin with the other hand.

A

Physiological Integrity 25. Which action will the nurse include in the plan of care for a patient who was admitted with syncopal episodes of unknown origin? a. Instruct the patient to call for assistance before getting out of bed. b. Explain the association between various dysrhythmias and syncope. c. Educate the patient about the need to avoid caffeine and other stimulants. d. Tell the patient about the benefits of implantable cardioverter-defibrillators.

A A patient with fainting episodes is at risk for falls. The nurse will plan to minimize the risk by having assistance whenever the patient up. The other actions may be needed if dysrhythmias are found to be the cause of the patient's syncope, but are not appropriate for syncope of unknown origin. DIF: Cognitive Level: Apply (application) REF: 807 TOP: Nursing Process: Planning MSC:

Safe and Effective Care Environment 27. Which action by a new registered nurse (RN) who is orienting to the progressive care unit indicates a good understanding of the treatment of cardiac dysrhythmias? a. Injects IV adenosine (Adenocard) over 2 seconds to a patient with supraventricular tachycardia b. Obtains the defibrillator and quickly brings it to the bedside of a patient whose monitor shows asystole c. Turns the synchronizer switch to the "on" position before defibrillating a patient with ventricular fibrillation d. Gives the prescribed dose of diltiazem (Cardizem) to a patient with new-onset type II second degree AV block

A Adenosine must be given over 1 to 2 seconds to be effective. The other actions indicate a need for more education about treatment of cardiac dysrhythmias. The RN should hold the diltiazem until talking to the health care provider. The treatment for asystole is immediate CPR. The synchronizer switch should be "off" when defibrillating. DIF: Cognitive Level: Analyze (analysis) REF: 795 OBJ: Special Questions: Multiple Patients TOP: Nursing Process: Evaluation MSC:

Physiological Integrity 9. A patient with dilated cardiomyopathy has new onset atrial fibrillation that has been unresponsive to drug therapy for several days. The priority teaching needed for this patient would include information about a. anticoagulant therapy. b. permanent pacemakers. c. electrical cardioversion. d. IV adenosine (Adenocard).

A Atrial fibrillation therapy that has persisted for more than 48 hours requires anticoagulant treatment for 3 weeks before attempting cardioversion. This is done to prevent embolization of clots from the atria. Cardioversion may be done after several weeks of anticoagulation therapy. Adenosine is not used to treat atrial fibrillation. Pacemakers are routinely used for patients with bradydysrhythmias. Information does not indicate that the patient has a slow heart rate. DIF: Cognitive Level: Apply (application) REF: 796 OBJ: Special Questions: Prioritization TOP: Nursing Process: Planning MSC:

Physiological Integrity 14. A 20-year-old has a mandatory electrocardiogram (ECG) before participating on a college soccer team and is found to have sinus bradycardia, rate 52. Blood pressure (BP) is 114/54, and the student denies any health problems. What action by the nurse is most appropriate? a. Allow the student to participate on the soccer team. b. Refer the student to a cardiologist for further diagnostic testing. c. Tell the student to stop playing immediately if any dyspnea occurs. d. Obtain more detailed information about the student's family health history.

A In an aerobically trained individual, sinus bradycardia is normal. The student's normal BP and negative health history indicate that there is no need for a cardiology referral or for more detailed information about the family's health history. Dyspnea during an aerobic activity such as soccer is normal. DIF: Cognitive Level: Apply (application) REF: 793 TOP: Nursing Process: Implementation MSC:

The patient with chronic heart failure is being discharged from the hospital. What information should the nurse emphasize in the patient's discharge teaching to prevent progression of the disease to acute decompensated heart failure (ADHF)? A. Take medications as prescribed. B. Use oxygen when feeling short of breath. C. Only ask the physician's office questions. D. Encourage most activity in the morning when rested.

A The goal for the patient with chronic HF is to avoid exacerbations and hospitalization. Taking the medications as prescribed along with nondrug therapies such as alternating activity with rest will help the patient meet this goal. If the patient needs to use oxygen at home, it will probably be used all the time or with activity to prevent respiratory acidosis. Many HF patients are monitored by a care manager or in a transitional program to assess the patient for medication effectiveness and monitor for patient deterioration and encourage the patient. This nurse manager can be asked questions or can contact the health care provider if there is evidence of worsening HF.

Physiological Integrity 12. Which intervention by a new nurse who is caring for a patient who has just had an implantable cardioverter-defibrillator (ICD) inserted indicates a need for more education about care of patients with ICDs? a. The nurse assists the patient to do active range of motion exercises for all extremities. b. The nurse assists the patient to fill out the application for obtaining a Medic Alert ID. c. The nurse gives amiodarone (Cordarone) to the patient without first consulting with the health care provider. d. The nurse teaches the patient that sexual activity usually can be resumed once the surgical incision is healed.

A The patient should avoid moving the arm on the ICD insertion site until healing has occurred in order to prevent displacement of the ICD leads. The other actions by the new nurse are appropriate for this patient. DIF: Cognitive Level: Apply (application) REF: 803 TOP: Nursing Process: Evaluation MSC:

Physiological Integrity 18. A patient's cardiac monitor shows a pattern of undulations of varying contours and amplitude with no measurable ECG pattern. The patient is unconscious and pulseless. Which action should the nurse take first? a. Perform immediate defibrillation. b. Give epinephrine (Adrenalin) IV. c. Prepare for endotracheal intubation. d. Give ventilations with a bag-valve-mask device.

A The patient's rhythm and assessment indicate ventricular fibrillation and cardiac arrest; the initial action should be to defibrillate. If a defibrillator is not immediately available or is unsuccessful in converting the patient to a better rhythm, the other actions may be appropriate. DIF: Cognitive Level: Apply (application) REF: 801 OBJ: Special Questions: Prioritization TOP: Nursing Process: Implementation MSC:

The patient at highest risk for venous thromboembolism (VTE) is

A 32-year-old woman who smokes, takes oral contraceptives, and is planning a trip to Europe Three important factors (called Virchow's triad) in the etiology of venous thrombosis are (1) venous stasis, (2) damage of the endothelium (inner lining of the vein), and (3) hypercoagulability of the blood. Patients at risk for venous thrombosis usually have predisposing conditions for these three disorders (see Table 37-8). The 32-year-old woman has the highest risk: long trips without adequate exercise (venous stasis), tobacco use, and use of oral contraceptives. NOTE: The likelihood of hypercoagulability of blood is increased in women older than 35 years who use tobacco.

When providing nutritional counseling for patients at risk for coronary artery disease (CAD), which foods would the nurse encourage patients to include in their diet (select all that apply.)? A. Tofu B. Walnuts C. Tuna fish D. Whole milk E. Orange juice

A B C Tuna fish, tofu, and walnuts are all rich in omega-3 fatty acids, which have been shown to reduce the risks associated with CAD when consumed regularly.

When caring for elderly patients with hypertension, which information should the nurse consider when planning care (select all that apply.)? A. Systolic blood pressure increases with aging. B. Blood pressures should be maintained near 120/80 mm Hg. C. White coat syndrome is prevalent in elderly patients. D. Volume depletion contributes to orthostatic hypotension. E. Blood pressure drops 1 hour postprandially in many older patients. F. Older patients will require higher doses of antihypertensive medications.

A C D E Systolic blood pressure increases with age and patients older than age 60 years should be maintained below 150/90 mm Hg. Older patients have significantly higher blood pressure readings when taken by health care providers (white coat syndrome). Older patients experience orthostatic hypotension related to dehydration, reduced compensatory mechanisms, and medications. One hour after eating, many older patients experience a drop in blood pressure. Lower doses of medications may be needed to control blood pressures in older adults related to decreased absorption rates and excretion ability.

What medications should the nurse expect to include in the teaching plan to decrease the risk of cardiovascular events and death for PAD patients (select all that apply.)? A. Ramipril (Altace) B. Cilostazol (Pletal) C. Simvastatin (Zocor) D. Clopidogrel (Plavix) E. Warfarin (Coumadin) F. Aspirin (acetylsalicylic acid)

A C F Angiotensin-converting enzyme inhibitors (e.g., ramipril [Altace]) are used to control hypertension. Statins (e.g., simvastatin [Zocor]) are used for lipid management. Aspirin is used as an antiplatelet agent. Cilostazol (Pletal) is used for intermittent claudication, but it does not reduce CVD morbidity and mortality risks. Clopidogrel may be used if the patient cannot tolerate aspirin. Anticoagulants (e.g., warfarin [Coumadin]) are not recommended to prevent cardiovascular disease events in PAD patients.

Von Willebrand Disease

A bleeding disorder caused by low levels of clotting protein in the blood. Is a genetic disorder transmitted as an autosomal trait that causes a deficiency of or defect of vWF. vWF is a protein that connects platelets to the endothelial lining of blood vessels and bings to factor VIII of the coagulation cascade. This definciency of vWF causes decreased platelet adhesion and reduced levels of active factor VIII, which result in defective clot formation. Males and females are equally affected, but it's more evident in females due to menstrual bleeding.

The nurse on a medical-surgical unit identifies that which patient has the highest risk for metabolic alkalosis?

A patient with a traumatic brain injury A patient with type 1 diabetes mellitus A patient with acute respiratory failure A patient with nasogastric tube suction (correct) Excessive nasogastric suctioning may cause metabolic alkalosis. Brain injury may cause hyperventilation and respiratory alkalosis. Type 1 diabetes mellitus (diabetic ketoacidosis) is associated with metabolic acidosis. Acute respiratory failure may lead to respiratory acidosis.

When preparing a patient for a capsule endoscopy study, what should the nurse do? A) Ensure the patient understands the required bowel preparation. B) Have the patient return to the procedure room for removal of the capsule. C) Teach the patient to maintain a clear liquid diet throughout the procedure. D) Explain to the patient that conscious sedation will be used during placement of the capsule.

A) A capsule endoscopy study involves the patient performing a bowel prep to cleanse the bowel before swallowing the capsule. The patient will be on a clear liquid diet for 1 to 2 days before the procedure and will remain NPO for 4 to 6 hours after swallowing the capsule. The capsule is disposable and will pass naturally with the bowel movement, although the monitoring device will need to be removed.

After administering a dose of promethazine (Phenergan) to a patient with nausea and vomiting, the nurse explains that which of the following may be experienced as a common temporary adverse effect of the medication? A) Drowsiness B) Reduced hearing C) Sensation of falling D) Photosensitivity

A) Drowsiness (Although being given to this patient as an antiemetic, promethazine also has sedative and amnesic properties. For this reason, the patient is likely to experience drowsiness as an adverse effect of the medication.)

A patient had a stomach resection for stomach cancer. The nurse should teach the patient about the loss of the hormone that stimulates gastric acid secretion and motility and maintains lower esophageal sphincter tone. Which hormone will be decreased with a gastric resection? A) Gastrin B) Secretin C) Cholecystokinin D) Gastric inhibitory peptide

A) Gastrin is the hormone activated in the stomach (and duodenal mucosa) by stomach distention that stimulates gastric acid secretion and motility and maintains lower esophageal sphincter tone. Secretin inhibits gastric motility and acid secretion and stimulates pancreatic bicarbonate secretion. Cholecystokinin allows increased flow of bile into the duodenum and release of pancreatic digestive enzymes. Gastric inhibitory peptide inhibits gastric acid secretion and motility.

The patient with cirrhosis is being taught self-care. Which statement indicates the patient needs more teaching? A) "If I notice a fast heart rate or irregular beats, this is normal for cirrhosis." B) "I need to take good care of my belly and ankle skin where it is swollen." C) "A scrotal support may be more comfortable when I have scrotal edema." D) "I can use pillows to support my head to help me breathe when I am in bed."

A) If the patient with cirrhosis experiences a fast or irregular heart rate, it may be indicative of hypokalemia and should be reported to the health care provider, as this is not normal for cirrhosis. Edematous tissue is subject to breakdown and needs meticulous skin care. Pillows and a semi-Fowler's or Fowler's position will increase respiratory efficiency. A scrotal support may improve comfort if there is scrotal edema

The wound, ostomy, and continence (WOC) nurse selects the site where the ostomy will be placed. What should be included in the consideration for the site? A) The patient must be able to see the site. B) Outside the rectus muscle area is the best site. C) It is easier to seal the drainage bag to a protruding area. D) The ostomy will need irrigation, so area should not be tender.

A) In selection of the ostomy site, the WOC nurse will want a site visible to the patient so the patient can take care of it, within the rectus muscle to avoid hernias, and on a flat surface to more easily create a good seal with the drainage bag.

The nurse is conducting discharge teaching for a patient with metastatic lung cancer who was admitted with a bowel impaction. Which of the following instructions would be most helpful to prevent further episodes of constipation? A) Maintain a high intake of fluid and fiber in the diet. B) Reduce intake of medications causing constipation. C) Eat several small meals per day to maintain bowel motility. D) Sit upright during meals to increase bowel motility by gravity.

A) Increased fluid intake and a high-fiber diet reduce the incidence of constipation caused by immobility, medications, and other factors. Fluid and fiber provide bulk that in turn increases peristalsis and bowel motility. Analgesics taken for lung cancer probably cannot be reduced. Other medications may decrease constipation, but it is best to avoid laxatives. Eating several small meals per day and position do not facilitate bowel motility. Defecation is easiest when the person sits on the commode with the knees higher than the hips.

A stroke patient who primarily uses a wheelchair for mobility has diarrhea with fecal incontinence. What should the nurse assess first? A) Fecal impaction B) Perineal hygiene C) Dietary fiber intake D) Antidiarrheal agent use

A) Patients with limited mobility are at risk for fecal impactions due to constipation that may lead to liquid stool leaking around the hardened impacted feces, so assessing for fecal impaction is the priority. Perineal hygiene can be assessed at the same time. Assessing the dietary fiber and fluid intake and antidiarrheal agent use will be assessed and considered next.

A patient with type 2 diabetes and cirrhosis asks the nurse if it would be okay to take silymarin (milk thistle) to help minimize liver damage. The nurse responds based on knowledge that: A. Milk thistle may affect liver enzymes and thus alter drug metabolism. B. Milk thistle is generally safe in recommended doses for up to 10 years. C. is unclear scientific evidence for the use of milk thistle in treating cirrhosis. D. Milk thistle may elevate the serum glucose levels and is thus contraindicated in diabetes.

A) There is good scientific evidence that there is no real benefit from using milk thistle to protect the liver cells from toxic damage in the treatment of cirrhosis. Milk thistle does affect liver enzymes and thus could alter drug metabolism. Therefore patients will need to be monitored for drug interactions. It is noted to be safe for up to 6 years, not 10 years, and it may lower, not elevate, blood glucose levels.

Two days following a colectomy for an abdominal mass, a patient reports gas pains and abdominal distention. The nurse plans care for the patient based on the knowledge that the symptoms are occurring as a result of A) impaired peristalsis. B) irritation of the bowel. C) nasogastric suctioning. D) inflammation of the incision site.

A) Until peristalsis returns to normal following anesthesia, the patient may experience slowed gastrointestinal motility leading to gas pains and abdominal distention. Irritation of the bowel, nasogastric suctioning, and inflammation of the surgical site do not cause gas pains or abdominal distention.

A 90-year-old healthy man is suffering from dysphagia. The nurse explains what age-related change of the GI tract is the most likely cause of his difficulty? A) Xerostomia B) Esophageal cancer C) Decreased taste buds D) Thinner abdominal wall

A) Xerostomia, decreased volume of saliva, leads to dry oral mucosa and dysphagia. Esophageal cancer is not an age-related change. Decreased taste buds and a thinner abdominal wall do not contribute to difficulty swallowing.

When caring for a patient with liver disease, the nurse recognizes the need to prevent bleeding resulting from altered clotting factors and rupture of varices. Which of the following nursing interventions would be appropriate to achieve this outcome (select all that apply)? A. Use smallest gauge possible when giving injections or drawing blood. B. Teach patient to avoid straining at stool, vigorous blowing of nose, and coughing. C. Advise patient to use soft-bristle toothbrush and avoid ingestion of irritating food. D. Apply gentle pressure for the shortest possible time period after performing venipuncture. E. Instruct patient to avoid aspirin and NSAIDs to prevent hemorrhage when varices are present.

A,B,C,E) Using the smallest gauge needle for injections will minimize the risk of bleeding into the tissues. Avoiding straining, nose blowing, and coughing will reduce the risk of hemorrhage at these sites. The use of a soft-bristle toothbrush and avoidance of irritating food will reduce injury to highly vascular mucous membranes. The nurse should apply gentle but prolonged pressure to venipuncture sites to minimize the risk of bleeding. Aspirin and NSAIDs should not be used in patients with liver disease because they interfere with platelet aggregation, thus increasing the risk for bleeding.

The patient with cirrhosis has an increased abdominal girth from ascites. The nurse should know that this fluid gathers in the abdomen for which reasons (select all that apply)? A) There is decreased colloid oncotic pressure from the liver's inability to synthesize albumin. B) Hyperaldosteronism related to damaged hepatocytes increases sodium and fluid retention. C) Portal hypertension pushes proteins from the blood vessels, causing leaking into the peritoneal cavity. D) Osmoreceptors in the hypothalamus stimulate thirst, which causes the stimulation to take in fluids orally. E) Overactivity of the enlarged spleen results in increased removal of blood cells from the circulation, which decreases the vascular pressure.

A,B.C) The ascites related to cirrhosis are caused by decreased colloid oncotic pressure from the lack of albumin from liver inability to synthesize it and the portal hypertension that shifts the protein from the blood vessels to the peritoneal cavity, and hyperaldosteronism which increases sodium and fluid retention. The intake of fluids orally and the removal of blood cells by the spleen do not directly contribute to ascites.

A 52-yr-old male patient has received a bolus dose and an infusion of alteplase (Activase) for an ST-segment elevation myocardial infarction (STEMI). Which patient assessment would determine the effectiveness of the medication? A. Presence of chest pain B. Blood in the urine or stool C. Tachycardia with hypotension D. Decreased level of consciousness

A. Alteplase is a fibrinolytic agent that is administered to patients who have had an STEMI. If the medication is effective, the patient's chest pain will resolve because the medication dissolves the thrombus in the coronary artery and results in reperfusion of the myocardium. Bleeding is a major complication of fibrinolytic therapy. Signs of major bleeding include decreased level of consciousness, blood in the urine or stool, and increased heart rate with decreased blood pressure.

The nurse is admitting a 68-yr-old preoperative patient with a suspected abdominal aortic aneurysm (AAA). The medication history reveals that the patient has been taking warfarin (Coumadin) on a daily basis. Based on this history and the patient's admission diagnosis, the nurse should prepare to administer which medication? A. Vitamin K B. Cobalamin C. Heparin sodium D. Protamine sulfate

A. Coumadin is a vitamin K antagonist anticoagulant that could cause excessive bleeding during surgery if clotting times are not corrected before surgery. For this reason, vitamin K is given as the antidote for warfarin (Coumadin).

When the patient is being examined for venous thromboembolism (VTE) in the calf, what diagnostic test should the nurse expect to teach the patient about first? A. Duplex ultrasound B. Contrast venography C. Magnetic resonance venography D. Computed tomography venography

A. The duplex ultrasound is the most widely used test to diagnose VTE. Contrast venography is rarely used now. Magnetic resonance venography is less accurate for calf veins than pelvic and proximal veins. Computed tomography venography may be used but is invasive and much more expensive than the duplex ultrasound.

Assessment of a patient's peripheral IV site reveals that phlebitis has developed over the past several hours. Which intervention should the nurse implement first? A. Remove the patient's IV catheter. B. Apply an ice pack to the affected area. C. Decrease the IV rate to 20 to 30 mL/hr. D. Administer prophylactic anticoagulants.

A. The priority intervention for superficial phlebitis is removal of the offending IV catheter. Decreasing the IV rate is insufficient. Anticoagulants are not normally required, and warm, moist heat is often therapeutic.

Despite a high dosage, a male patient who is taking nifedipine (Procardia XL) for antihypertensive therapy continues to have blood pressures over 140/90 mm Hg. What should the nurse do next? A. Assess his adherence to therapy. B. Ask him to make an exercise plan. C. Instruct him to use the DASH diet. D. Request a prescription for a thiazide diuretic.

A. A long-acting calcium-channel blocker such as nifedipine causes vascular smooth muscle relaxation, resulting in decreased systemic vascular resistance and arterial blood pressure and related side effects. The patient data the nurse has about this patient is very limited, so the nurse needs to begin by assessing adherence to therapy.

An older adult patient with chronic heart failure (HF) and atrial fibrillation asks the nurse why warfarin has been prescribed to continue at home. What is the best response by the nurse? A. "The medication prevents blood clots from forming in your heart." Correct B. "The medication dissolves clots that develop in your coronary arteries." C. "The medication reduces clotting by decreasing serum potassium levels." D. "The medication increases your heart rate so that clots do not form in your heart."

A. Chronic HF causes enlargement of the chambers of the heart and an altered electrical pathway, especially in the atria. When numerous sites in the atria fire spontaneously and rapidly, atrial fibrillation occurs. Atrial fibrillation promotes thrombus formation within the atria with an increased risk of stroke and requires treatment with cardioversion, antidysrhythmics, and/or anticoagulants. Warfarin is an anticoagulant that interferes with hepatic synthesis of vitamin K-dependent clotting factors.

At a clinic visit, the nurse provides dietary teaching for a patient recently hospitalized with an exacerbation of chronic heart failure. The nurse determines that teaching is successful if the patient makes which statement? A. "I will limit the amount of milk and cheese in my diet." B. "I can add salt when cooking foods but not at the table." C. "I will take an extra diuretic pill when I eat a lot of salt." D. "I can have unlimited amounts of foods labeled as reduced sodium."

A. Milk products should be limited to 2 cups per day for a 2500-mg sodium-restricted diet. Salt should not be added during food preparation or at the table. Diuretics should be taken as prescribed (usually daily) and not based on sodium intake. Foods labeled as reduced sodium contain at least 25% less sodium than regular.

The nurse teaches a patient with hypertension that uncontrolled hypertension may damage organs in the body primarily by which mechanism? A. Hypertension promotes atherosclerosis and damage to the walls of the arteries. B. Hypertension causes direct pressure on organs, resulting in necrosis and replacement of cells with scar tissue. C. Hypertension causes thickening of the capillary membranes, leading to hypoxia of organ systems. D. Hypertension increases blood viscosity, which contributes to intravascular coagulation and tissue necrosis distal to occlusions.

A. Hypertension is a major risk factor for the development of atherosclerosis by mechanisms not yet fully known. However, when atherosclerosis develops, it damages the walls of arteries and reduces circulation to target organs and tissues.

Which individuals would the nurse identify as having the highest risk for coronary artery disease (CAD)? A. A 45-yr-old depressed man with a high-stress job B. A 60-yr-old man with below normal homocysteine levels C. A 54-yr-old woman vegetarian with elevated high-density lipoprotein (HDL) levels D. A 62-yr-old woman who has a sedentary lifestyle and body mass index (BMI) of 23 kg/m2

A. The 45-yr-old depressed man with a high-stress job is at the highest risk for CAD. Studies demonstrate that depression and stressful states can contribute to the development of CAD. Elevated HDL levels and low homocysteine levels actually help to prevent CAD. Although a sedentary lifestyle is a risk factor, a BMI of 23 kg/m2 depicts normal weight, and thus the patient with two risk factors is at greatest risk for developing CAD.

Which statement by a patient indicates good understanding of the nurse's teaching about prevention of sickle cell crisis? A. "Risk for a crisis is decreased by having an annual influenza vaccination." B. "Routine continuous dosage narcotics are prescribed to prevent a crisis." C. "There are no effective medications that can help prevent sickling." D. "Home oxygen therapy is frequently used to decrease sickling."

A. "Risk for a crisis is decreased by having an annual influenza vaccination." Because infection is the most common cause of a sickle cell crisis, influenza, Haemophilus influenzae, pneumococcal pneumonia, and hepatitis immunizations should be administered. Although continuous dose opioids and oxygen may be administered during a crisis (pain management is a priority), patients do not receive these therapies to prevent crisis. Hydroxyurea (Hydrea) is a medication used to decrease the number of sickle cell crises.

Long- term administration at therapeutic doses often leads to serious complications and side effects. Therapy should be reserved for diseases that have a risk of death or permanent loss of function and for conditions in which short-term treatment is likely to produce remission or recovery. Patient education must include teaching about rinsing the mouth out after use to prevent oral fungal infections, they can get thrush, pharyngeal irritation, hoarseness, oral fungal infections, and dry mouth Can cause cushing's syndrome, hyperglycemia especially with diabetic patients, avoid taking NSAIDs, osteoarthritis, bone weakening, and PUD

AE of corticosteroid medications

A patient is to receive methylprednisolone (Solu-Medrol) 100 mg. The label on the medication states: methylprednisolone 125 mg in 2 mL. How many milliliters will the nurse administer?

ANS: 1.6 A concentration of 125 mg in 2 mL will result in 100 mg in 1.6 mL.

In which order will the nurse take these steps to prepare NPH 20 units and regular insulin 2 units using the same syringe? (Put a comma and a space between each answer choice [A, B, C, D, E]). a. Rotate NPH vial. b. Withdraw regular insulin. c. Withdraw 20 units of NPH. d. Inject 20 units of air into NPH vial. e. Inject 2 units of air into regular insulin vial

ANS: A, D, E, B, C When mixing regular insulin with NPH, it is important to avoid contact between the regular insulin and the additives in the NPH that slow the onset, peak, and duration of activity in the longer-acting insulin.

Which menu choice by the patient with diverticulosis is best for preventing diverticulitis? a. Navy bean soup and vegetable salad b. Whole grain pasta with tomato sauce c. Baked potato with low-fat sour cream d. Roast beef sandwich on whole wheat bread

ANS: A A diet high in fiber and low in fats and red meat is recommended to prevent diverticulitis. Although all of the choices have some fiber, the bean soup and salad will be the highest in fiber and the lowest in fat.

The nurse admitting a patient with acute diverticulitis explains that the initial plan of care is to a. administer IV fluids. b. give stool softeners and enemas. c. order a diet high in fiber and fluids. d. prepare the patient for colonoscopy.

ANS: A A patient with acute diverticulitis will be NPO and given parenteral fluids. A diet high in fiber and fluids will be implemented before discharge. Bulk-forming laxatives, rather than stool softeners, are usually given, and these will be implemented later in the hospitalization. The patient with acute diverticulitis will not have enemas or a colonoscopy because of the risk for perforation and peritonitis.

A 50-year-old female patient calls the clinic to report a new onset of severe diarrhea. The nurse anticipates that the patient will need to a. collect a stool specimen. b. prepare for colonoscopy. c. schedule a barium enema. d. have blood cultures drawn.

ANS: A Acute diarrhea is usually caused by an infectious process, and stool specimens are obtained for culture and examined for parasites or white blood cells. There is no indication that the patient needs a colonoscopy, blood cultures, or a barium enema.

A 34-year-old female patient with a new ileostomy asks how much drainage to expect. The nurse explains that after the bowel adjusts to the ileostomy, the usual drainage will be about _____ cups. a. 2 b. 3 c. 4 d. 5

ANS: A After the proximal small bowel adapts to reabsorb more fluid, the average amount of ileostomy drainage is about 500 mL daily. One cup is about 240 mL.

A patient complains of gas pains and abdominal distention two days after a small bowel resection. Which nursing action is best to take? a. Encourage the patient to ambulate. b. Instill a mineral oil retention enema. c. Administer the ordered IV morphine sulfate. d. Offer the ordered promethazine (Phenergan) suppository.

ANS: A Ambulation will improve peristalsis and help the patient eliminate flatus and reduce gas pain. A mineral oil retention enema is helpful for constipation with hard stool. A return-flow enema might be used to relieve persistent gas pains. Morphine will further reduce peristalsis. Promethazine (Phenergan) is used as an antiemetic rather than to decrease gas pains or distention.

Which finding for a patient who has hypothyroidism and hypertension indicates that the nurse should contact the health care provider before administering levothyroxine (Synthroid)? a. Increased thyroxine (T4) level b. Blood pressure 112/62 mm Hg c. Distant and difficult to hear heart sounds d. Elevated thyroid stimulating hormone level

ANS: A An increased thyroxine level indicates the levothyroxine dose needs to be decreased. The other data are consistent with hypothyroidism and the nurse should administer the levothyroxine

Which question during the assessment of a diabetic patient will help the nurse identify autonomic neuropathy? a. "Do you feel bloated after eating?" b. "Have you seen any skin changes?" c. "Do you need to increase your insulin dosage when you are stressed?" d. "Have you noticed any painful new ulcerations or sores on your feet?"

ANS: A Autonomic neuropathy can cause delayed gastric emptying, which results in a bloated feeling for the patient. The other questions are also appropriate to ask but would not help in identifying autonomic neuropathy

In order to assist an older diabetic patient to engage in moderate daily exercise, which action is most important for the nurse to take? a. Determine what type of activities the patient enjoys. b. Remind the patient that exercise will improve self-esteem. c. Teach the patient about the effects of exercise on glucose level. d. Give the patient a list of activities that are moderate in intensity.

ANS: A Because consistency with exercise is important, assessment for the types of exercise that the patient finds enjoyable is the most important action by the nurse in ensuring adherence to an exercise program. The other actions will also be implemented but are not the most important in improving compliance

Which nursing action will be included in the plan of care for a 27-year-old male patient with bowel irregularity and a new diagnosis of irritable bowel syndrome (IBS)? a. Encourage the patient to express concerns and ask questions about IBS. b. Suggest that the patient increase the intake of milk and other dairy products. c. Educate the patient about the use of alosetron (Lotronex) to reduce symptoms. d. Teach the patient to avoid using nonsteroidal antiinflammatory drugs (NSAIDs).

ANS: A Because psychologic and emotional factors can affect the symptoms for IBS, encouraging the patient to discuss emotions and ask questions is an important intervention. Alosetron has serious side effects, and is used only for female patients who have not responded to other therapies. Although yogurt may be beneficial, milk is avoided because lactose intolerance can contribute to symptoms in some patients. NSAIDs can be used by patients with IBS.

Which finding by the nurse when assessing a patient with Hashimoto's thyroiditis and a goiter will require the most immediate action? a. New-onset changes in the patient's voice b. Apical pulse rate at rest 112 beats/minute c. Elevation in the patient's T3 and T4 levels d. Bruit audible bilaterally over the thyroid gland

ANS: A Changes in the patient's voice indicate that the goiter is compressing the laryngeal nerve and may lead to airway compression. The other findings will also be reported but are expected with Hashimoto's thyroiditis and do not require immediate action

Which nursing action can the nurse delegate to unlicensed assistive personnel (UAP) who are working in the diabetic clinic? a. Measure the ankle-brachial index. b. Check for changes in skin pigmentation. c. Assess for unilateral or bilateral foot drop. d. Ask the patient about symptoms of depression.

ANS: A Checking systolic pressure at the ankle and brachial areas and calculating the ankle-brachial index is a procedure that can be done by UAP who have been trained in the procedure. The other assessments require more education and critical thinking and should be done by the registered nurse (RN).

Which finding indicates to the nurse that the current therapies are effective for a patient with acute adrenal insufficiency? a. Increasing serum sodium levels b. Decreasing blood glucose levels c. Decreasing serum chloride levels d. Increasing serum potassium levels

ANS: A Clinical manifestations of Addison's disease include hyponatremia and an increase in sodium level indicates improvement. The other values indicate that treatment has not been effective

A 56-year-old female patient has an adrenocortical adenoma, causing hyperaldosteronism. The nurse providing care should a. monitor the blood pressure every 4 hours. b. elevate the patient's legs to relieve edema. c. monitor blood glucose level every 4 hours. d. order the patient a potassium-restricted diet.

ANS: A Hypertension caused by sodium retention is a common complication of hyperaldosteronism. Hyperaldosteronism does not cause an elevation in blood glucose. The patient will be hypokalemic and require potassium supplementation before surgery. Edema does not usually occur with hyperaldosteronism

A 27-year-old patient admitted with diabetic ketoacidosis (DKA) has a serum glucose level of 732 mg/dL and serum potassium level of 3.1 mEq/L. Which action prescribed by the health care provider should the nurse take first? a. Place the patient on a cardiac monitor. b. Administer IV potassium supplements. c. Obtain urine glucose and ketone levels. d. Start an insulin infusion at 0.1 units/kg/hr.

ANS: A Hypokalemia can lead to potentially fatal dysrhythmias such as ventricular tachycardia and ventricular fibrillation, which would be detected with electrocardiogram (ECG) monitoring. Because potassium must be infused over at least 1 hour, the nurse should initiate cardiac monitoring before infusion of potassium. Insulin should not be administered without cardiac monitoring because insulin infusion will further decrease potassium levels. Urine glucose and ketone levels are not urgently needed to manage the patient's care

Which patient action indicates a good understanding of the nurse's teaching about the use of an insulin pump? a. The patient programs the pump for an insulin bolus after eating. b. The patient changes the location of the insertion site every week. c. The patient takes the pump off at bedtime and starts it again each morning. d. The patient plans for a diet that is less flexible when using the insulin pump.

ANS: A In addition to the basal rate of insulin infusion, the patient will adjust the pump to administer a bolus after each meal, with the dosage depending on the oral intake. The insertion site should be changed every 2 or 3 days. There is more flexibility in diet and exercise when an insulin pump is used. The pump will deliver a basal insulin rate 24 hours a day.

A 51-year-old male patient has a new diagnosis of Crohn's disease after having frequent diarrhea and a weight loss of 10 pounds (4.5 kg) over 2 months. The nurse will plan to teach about a. medication use. b. fluid restriction. c. enteral nutrition. d. activity restrictions.

ANS: A Medications are used to induce and maintain remission in patients with inflammatory bowel disease (IBD). Decreased activity level is indicated only if the patient has severe fatigue and weakness. Fluids are needed to prevent dehydration. There is no advantage to enteral feedings.

A patient in the intensive care unit with acute decompensated heart failure (ADHF) complains of severe dyspnea and is anxious, tachypneic, and tachycardic. All of the following medications have been ordered for the patient. The nurse's priority action will be to a. give IV morphine sulfate 4 mg. b. give IV diazepam (Valium) 2.5 mg. c. increase nitroglycerin (Tridil) infusion by 5 mcg/min. d. increase dopamine (Intropin) infusion by 2 mcg/kg/min.

ANS: A Morphine improves alveolar gas exchange, improves cardiac output by reducing ventricular preload and afterload, decreases anxiety, and assists in reducing the subjective feeling of dyspnea. Diazepam may decrease patient anxiety, but it will not improve the cardiac output or gas exchange. Increasing the dopamine may improve cardiac output, but it will also increase the heart rate and myocardial oxygen consumption. Nitroglycerin will improve cardiac output and may be appropriate for this patient, but it will not directly reduce anxiety and will not act as quickly as morphine to decrease dyspnea.

Which action should the nurse include in the plan of care when caring for a patient admitted with acute decompensated heart failure (ADHF) who is receiving nesiritide (Natrecor)? a. Monitor blood pressure frequently. b. Encourage patient to ambulate in room. c. Titrate nesiritide slowly before stopping. d. Teach patient about home use of the drug.

ANS: A Nesiritide is a potent arterial and venous dilator, and the major adverse effect is hypotension. Because the patient is likely to have orthostatic hypotension, the patient should not be encouraged to ambulate. Nesiritide does not require titration and is used for ADHF but not in a home setting.

A female patient is scheduled for an oral glucose tolerance test. Which information from the patient's health history is most important for the nurse to communicate to the health care provider? a. The patient uses oral contraceptives. b. The patient runs several days a week. c. The patient has been pregnant three times. d. The patient has a family history of diabetes.

ANS: A Oral contraceptive use may falsely elevate oral glucose tolerance test (OGTT) values. Exercise and a family history of diabetes both can affect blood glucose but will not lead to misleading information from the OGTT. History of previous pregnancies may provide informational about gestational glucose tolerance, but will not lead to misleading information from the OGTT

A 32-year-old patient with diabetes is starting on intensive insulin therapy. Which type of insulin will the nurse discuss using for mealtime coverage? a. Lispro (Humalog) b. Glargine (Lantus) c. Detemir (Levemir) d. NPH (Humulin N)

ANS: A Rapid- or short-acting insulin is used for mealtime coverage for patients receiving intensive insulin therapy. NPH, glargine, or detemir will be used as the basal insulin

Which statement by the patient indicates a need for additional instruction in administering insulin? a. "I need to rotate injection sites among my arms, legs, and abdomen each day." b. "I can buy the 0.5 mL syringes because the line markings will be easier to see." c. "I should draw up the regular insulin first after injecting air into the NPH bottle." d. "I do not need to aspirate the plunger to check for blood before injecting insulin."

ANS: A Rotating sites is no longer recommended because there is more consistent insulin absorption when the same site is used consistently. The other patient statements are accurate and indicate that no additional instruction is needed

A 51-year-old woman with Crohn's disease who is taking infliximab (Remicade) calls the nurse in the outpatient clinic about new symptoms. Which symptom is most important to communicate to the health care provider? a. Fever b. Nausea c. Joint pain d. Headache

ANS: A Since infliximab suppresses the immune response, rapid treatment of infection is essential. The other patient complaints are common side effects of the medication, but they do not indicate any potentially life-threatening complications.

The nurse has administered 4 oz of orange juice to an alert patient whose blood glucose was 62 mg/dL. Fifteen minutes later, the blood glucose is 67 mg/dL. Which action should the nurse take next? a. Give the patient 4 to 6 oz more orange juice. b. Administer the PRN glucagon (Glucagon) 1 mg IM. c. Have the patient eat some peanut butter with crackers. d. Notify the health care provider about the hypoglycemia.

ANS: A The "rule of 15" indicates that administration of quickly acting carbohydrates should be done 2 to 3 times for a conscious patient whose glucose remains less than 70 mg/dL before notifying the health care provider. More complex carbohydrates and fats may be used once the glucose has stabilized. Glucagon should be used if the patient's level of consciousness decreases so that oral carbohydrates can no longer be given

A 38-year-old patient who has type 1 diabetes plans to swim laps daily at 1:00 PM. The clinic nurse will plan to teach the patient to a. check glucose level before, during, and after swimming. b. delay eating the noon meal until after the swimming class. c. increase the morning dose of neutral protamine Hagedorn (NPH) insulin. d. time the morning insulin injection so that the peak occurs while swimming.

ANS: A The change in exercise will affect blood glucose, and the patient will need to monitor glucose carefully to determine the need for changes in diet and insulin administration. Because exercise tends to decrease blood glucose, patients are advised to eat before exercising. Increasing the morning NPH or timing the insulin to peak during exercise may lead to hypoglycemia, especially with the increased exercise

A 55-year-old female patient with type 2 diabetes has a nursing diagnosis of imbalanced nutrition: more than body requirements. Which goal is most important for this patient? a. The patient will reach a glycosylated hemoglobin level of less than 7%. b. The patient will follow a diet and exercise plan that results in weight loss. c. The patient will choose a diet that distributes calories throughout the day. d. The patient will state the reasons for eliminating simple sugars in the diet.

ANS: A The complications of diabetes are related to elevated blood glucose, and the most important patient outcome is the reduction of glucose to near-normal levels. The other outcomes also are appropriate but are not as high in priority

A 54-year-old patient is admitted with diabetic ketoacidosis. Which admission order should the nurse implement first? a. Infuse 1 liter of normal saline per hour. b. Give sodium bicarbonate 50 mEq IV push. c. Administer regular insulin 10 U by IV push. d. Start a regular insulin infusion at 0.1 units/kg/hr.

ANS: A The most urgent patient problem is the hypovolemia associated with diabetic ketoacidosis (DKA), and the priority is to infuse IV fluids. The other actions can be done after the infusion of normal saline is initiated

Which information will the nurse include in teaching a female patient who has peripheral arterial disease, type 2 diabetes, and sensory neuropathy of the feet and legs? a. Choose flat-soled leather shoes. b. Set heating pads on a low temperature. c. Use callus remover for corns or calluses. d. Soak feet in warm water for an hour each day.

ANS: A The patient is taught to avoid high heels and that leather shoes are preferred. The feet should be washed, but not soaked, in warm water daily. Heating pad use should be avoided. Commercial callus and corn removers should be avoided. The patient should see a specialist to treat these problems

After receiving change-of-shift report on a heart failure unit, which patient should the nurse assess first? a. A patient who is cool and clammy, with new-onset confusion and restlessness b. A patient who has crackles bilaterally in the lung bases and is receiving oxygen. c. A patient who had dizziness after receiving the first dose of captopril (Capoten) d. A patient who is receiving IV nesiritide (Natrecor) and has a blood pressure of 100/62

ANS: A The patient who has "wet-cold" clinical manifestations of heart failure is perfusing inadequately and needs rapid assessment and changes in management. The other patients also should be assessed as quickly as possible but do not have indications of severe decreases in tissue perfusion.

The nurse is caring for a patient admitted with diabetes insipidus (DI). Which information is most important to report to the health care provider? a. The patient is confused and lethargic. b. The patient reports a recent head injury. c. The patient has a urine output of 400 mL/hr. d. The patient's urine specific gravity is 1.003.

ANS: A The patient's confusion and lethargy may indicate hypernatremia and should be addressed quickly. In addition, patients with DI compensate for fluid losses by drinking copious amounts of fluids, but a patient who is lethargic will be unable to drink enough fluids and will become hypovolemic. A high urine output, low urine specific gravity, and history of a recent head injury are consistent with diabetes insipidus, but they do not require immediate nursing action to avoid life-threatening complications

During a visit to a 78-year-old with chronic heart failure, the home care nurse finds that the patient has ankle edema, a 2-kg weight gain over the past 2 days, and complains of "feeling too tired to get out of bed." Based on these data, the best nursing diagnosis for the patient is a. activity intolerance related to fatigue. b. disturbed body image related to weight gain. c. impaired skin integrity related to ankle edema. d. impaired gas exchange related to dyspnea on exertion.

ANS: A The patient's statement supports the diagnosis of activity intolerance. There are no data to support the other diagnoses, although the nurse will need to assess for other patient problems.

A patient receives aspart (NovoLog) insulin at 8:00 AM. Which time will it be most important for the nurse to monitor for symptoms of hypoglycemia? a. 10:00 AM b. 12:00 AM c. 2:00 PM d. 4:00 PM

ANS: A The rapid-acting insulins peak in 1 to 3 hours. The patient is not at a high risk for hypoglycemia at the other listed times, although hypoglycemia may occur

A female patient is awaiting surgery for acute peritonitis. Which action will the nurse include in the plan of care? a. Position patient with the knees flexed. b. Avoid use of opioids or sedative drugs. c. Offer frequent small sips of clear liquids. d. Assist patient to breathe deeply and cough.

ANS: A There is less peritoneal irritation with the knees flexed, which will help decrease pain. Opioids and sedatives are typically given to control pain and anxiety. Preoperative patients with peritonitis are given IV fluids for hydration. Deep breathing and coughing will increase the patient's discomfort

A patient who was admitted with myxedema coma and diagnosed with hypothyroidism is improving and expected to be discharged in 2 days. Which teaching strategy will be best for the nurse to use? a. Provide written reminders of self-care information. b. Offer multiple options for management of therapies. c. Ensure privacy for teaching by asking visitors to leave. d. Delay teaching until patient discharge date is confirmed.

ANS: A Written instructions will be helpful to the patient because initially the hypothyroid patient may be unable to remember to take medications and other aspects of self-care. Because the treatment regimen is somewhat complex, teaching should be initiated well before discharge. Family members or friends should be included in teaching because the hypothyroid patient is likely to forget some aspects of the treatment plan. A simpler regimen will be easier to understand until the patient is euthyroid

Which prescribed medication should the nurse administer first to a 60-year-old patient admitted to the emergency department in thyroid storm? a. Propranolol (Inderal) b. Propylthiouracil (PTU) c. Methimazole (Tapazole) d. Iodine (Lugol's solution)

ANS: A b-Adrenergic blockers work rapidly to decrease the cardiovascular manifestations of thyroid storm. The other medications take days to weeks to have an impact on thyroid function

Based on the Joint Commission Core Measures for patients with heart failure, which topics should the nurse include in the discharge teaching plan for a patient who has been hospitalized with chronic heart failure (select all that apply)? a. How to take and record daily weight b. Importance of limiting aerobic exercise c. Date and time of follow-up appointment d. Symptoms indicating worsening heart failure e. Actions and side effects of prescribed medications

ANS: A, C, D, E The Joint Commission Core Measures state that patients should be taught about prescribed medications, follow-up appointments, weight monitoring, and actions to take for worsening symptoms. Patients with heart failure are encouraged to begin or continue aerobic exercises such as walking, while self-monitoring to avoid excessive fatigue.

A 45-year-old male patient with suspected acromegaly is seen at the clinic. To assist in making the diagnosis, which question should the nurse ask? a. "Have you had a recent head injury?" b. "Do you have to wear larger shoes now?" c. "Is there a family history of acromegaly?" d. "Are you experiencing tremors or anxiety?"

ANS: B Acromegaly causes an enlargement of the hands and feet. Head injury and family history are not risk factors for acromegaly. Tremors and anxiety are not clinical manifestations of acromegaly

The nurse is planning postoperative care for a patient who is being admitted to the surgical unit form the recovery room after transsphenoidal resection of a pituitary tumor. Which nursing action should be included? a. Palpate extremities for edema. b. Measure urine volume every hour. c. Check hematocrit every 2 hours for 8 hours. d. Monitor continuous pulse oximetry for 24 hours.

ANS: B After pituitary surgery, the patient is at risk for diabetes insipidus caused by cerebral edema. Monitoring of urine output and urine specific gravity is essential. Hemorrhage is not a common problem. There is no need to check the hematocrit hourly. The patient is at risk for dehydration, not volume overload. The patient is not at high risk for problems with oxygenation, and continuous pulse oximetry is not needed

The nurse is admitting a 67-year-old patient with new-onset steatorrhea. Which question is most important for the nurse to ask? a. "How much milk do you usually drink?" b. "Have you noticed a recent weight loss?" c. "What time of day do your bowels move?" d. "Do you eat meat or other animal products?"

ANS: B Although all of the questions provide useful information, it is most important to determine if the patient has an imbalance in nutrition because of the steatorrhea.

A patient being admitted with an acute exacerbation of ulcerative colitis reports crampy abdominal pain and passing 15 or more bloody stools a day. The nurse will plan to a. administer IV metoclopramide (Reglan). b. discontinue the patient's oral food intake. c. administer cobalamin (vitamin B12) injections. d. teach the patient about total colectomy surgery.

ANS: B An initial therapy for an acute exacerbation of inflammatory bowel disease (IBD) is to rest the bowel by making the patient NPO. Metoclopramide increases peristalsis and will worsen symptoms. Cobalamin (vitamin B12) is absorbed in the ileum, which is not affected by ulcerative colitis. Although total colectomy is needed for some patients, there is no indication that this patient is a candidate.

The nurse preparing for the annual physical exam of a 50-year-old man will plan to teach the patient about a. endoscopy. b. colonoscopy. c. computerized tomography screening. d. carcinoembryonic antigen (CEA) testing.

ANS: B At age 50, individuals with an average risk for colorectal cancer (CRC) should begin screening for CRC. Colonoscopy is the gold standard for CRC screening. The other diagnostic tests are not recommended as part of a routine annual physical exam at age 50.

Which nursing action will the nurse include in the plan of care for a 35-year-old male patient admitted with an exacerbation of inflammatory bowel disease (IBD)? a. Restrict oral fluid intake. b. Monitor stools for blood. c. Ambulate four times daily. d. Increase dietary fiber intake.

ANS: B Because anemia or hemorrhage may occur with IBD, stools should be assessed for the presence of blood. The other actions would not be appropriate for the patient with IBD. Because dietary fiber may increase gastrointestinal (GI) motility and exacerbate the diarrhea, severe fatigue is common with IBD exacerbations, and dehydration may occur.

A 34-year-old has a new diagnosis of type 2 diabetes. The nurse will discuss the need to schedule a dilated eye exam a. every 2 years. b. as soon as possible. c. when the patient is 39 years old. d. within the first year after diagnosis.

ANS: B Because many patients have some diabetic retinopathy when they are first diagnosed with type 2 diabetes, a dilated eye exam is recommended at the time of diagnosis and annually thereafter. Patients with type 1 diabetes should have dilated eye exams starting 5 years after they are diagnosed and then annually.

A patient in the emergency department has just been diagnosed with peritonitis caused by a ruptured diverticulum. Which prescribed intervention will the nurse implement first? a. Insert a urinary catheter to drainage. b. Infuse metronidazole (Flagyl) 500 mg IV. c. Send the patient for a computerized tomography scan. d. Place a nasogastric (NG) tube to intermittent low suction.

ANS: B Because peritonitis can be fatal if treatment is delayed, the initial action should be to start antibiotic therapy (after any ordered cultures are obtained). The other actions can be done after antibiotic therapy is initiated.

After change-of-shift report, which patient should the nurse assess first? a. 19-year-old with type 1 diabetes who has a hemoglobin A1C of 12% b. 23-year-old with type 1 diabetes who has a blood glucose of 40 mg/dL c. 40-year-old who is pregnant and whose oral glucose tolerance test is 202 mg/dL d. 50-year-old who uses exenatide (Byetta) and is complaining of acute abdominal pain

ANS: B Because the brain requires glucose to function, untreated hypoglycemia can cause unconsciousness, seizures, and death. The nurse will rapidly assess and treat the patient with low blood glucose. The other patients also have symptoms that require assessments and/or interventions, but they are not at immediate risk for life-threatening complications

The nurse is preparing to teach a 43-year-old man who is newly diagnosed with type 2 diabetes about home management of the disease. Which action should the nurse take first? a. Ask the patient's family to participate in the diabetes education program. b. Assess the patient's perception of what it means to have diabetes mellitus. c. Demonstrate how to check glucose using capillary blood glucose monitoring. d. Discuss the need for the patient to actively participate in diabetes management.

ANS: B Before planning teaching, the nurse should assess the patient's interest in and ability to self-manage the diabetes. After assessing the patient, the other nursing actions may be appropriate, but planning needs to be individualized to each patient.

Which information obtained by the nurse interviewing a 30-year-old male patient is most important to communicate to the health care provider? a. The patient has a history of constipation. b. The patient has noticed blood in the stools. c. The patient had an appendectomy at age 27. d. The patient smokes a pack/day of cigarettes.

ANS: B Blood in the stools is a possible clinical manifestation of colorectal cancer and requires further assessment by the health care provider. The other patient information will also be communicated to the health care provider, but does not indicate an urgent need for further testing or intervention.

A 74-year-old patient preparing to undergo a colon resection for cancer of the colon asks about the elevated carcinoembryonic antigen (CEA) test result. The nurse explains that the test is used to a. identify any metastasis of the cancer. b. monitor the tumor status after surgery. c. confirm the diagnosis of a specific type of cancer. d. determine the need for postoperative chemotherapy.

ANS: B CEA is used to monitor for cancer recurrence after surgery. CEA levels do not help to determine whether there is metastasis of the cancer. Confirmation of the diagnosis is made on the basis of biopsy. Chemotherapy use is based on factors other than CEA.

The nurse is assessing a 41-year-old African American male patient diagnosed with a pituitary tumor causing panhypopituitarism. Assessment findings consistent with panhypopituitarism include a. high blood pressure. b. decreased facial hair. c. elevated blood glucose. d. tachycardia and cardiac palpitations.

ANS: B Changes in male secondary sex characteristics such as decreased facial hair, testicular atrophy, diminished spermatogenesis, loss of libido, impotence, and decreased muscle mass are associated with decreases in follicle stimulating hormone (FSH) and luteinizing hormone (LH). Fasting hypoglycemia and hypotension occur in panhypopituitarism as a result of decreases in adrenocorticotropic hormone (ACTH) and cortisol. Bradycardia is likely due to the decrease in thyroid stimulating hormone (TSH) and thyroid hormones associated with panhypopituitarism

Which patient action indicates good understanding of the nurse's teaching about administration of aspart (NovoLog) insulin? a. The patient avoids injecting the insulin into the upper abdominal area. b. The patient cleans the skin with soap and water before insulin administration. c. The patient stores the insulin in the freezer after administering the prescribed dose. d. The patient pushes the plunger down while removing the syringe from the injection site.

ANS: B Cleaning the skin with soap and water or with alcohol is acceptable. Insulin should not be frozen. The patient should leave the syringe in place for about 5 seconds after injection to be sure that all the insulin has been injected. The upper abdominal area is one of the preferred areas for insulin injection

The nurse is assessing a 31-year-old female patient with abdominal pain. Th nurse,who notes that there is ecchymosis around the area of umbilicus, will document this finding as a. Cullen sign. b. Rovsing sign. c. McBurney sign. d. Grey-Turner's signt.

ANS: B Cullen sign is ecchymosis around the umbilicus. Rovsing sign occurs when palpation of the left lower quadrant causes pain in the right lower quadrant. Deep tenderness at McBurney's point (halfway between the umbilicus and the right iliac crest), known as McBurney's sign, is a sign of acute appendicitis.

The nurse determines that demeclocycline (Declomycin) is effective for a patient with syndrome of inappropriate antidiuretic hormone (SIADH) based on finding that the patient's a. weight has increased. b. urinary output is increased. c. peripheral edema is decreased. d. urine specific gravity is increased.

ANS: B Demeclocycline blocks the action of antidiuretic hormone (ADH) on the renal tubules and increases urine output. An increase in weight or an increase in urine specific gravity indicates that the SIADH is not corrected. Peripheral edema does not occur with SIADH. A sudden weight gain without edema is a common clinical manifestation of this disorder

The cardiac telemetry unit charge nurse receives status reports from other nursing units about four patients who need cardiac monitoring. Which patient should be transferred to the cardiac unit first? a. Patient with Hashimoto's thyroiditis and a heart rate of 102 b. Patient with tetany who has a new order for IV calcium chloride c. Patient with Cushing syndrome and a blood glucose of 140 mg/dL d. Patient with Addison's disease who takes hydrocortisone twice daily

ANS: B Emergency treatment of tetany requires IV administration of calcium; ECG monitoring will be required because cardiac arrest may occur if high calcium levels result from too-rapid administration. The information about the other patients indicates that they are more stable than the patient with tetany

After a total proctocolectomy and permanent ileostomy, the patient tells the nurse, "I cannot manage all these changes. I don't want to look at the stoma." What is the best action by the nurse? a. Reassure the patient that ileostomy care will become easier. b. Ask the patient about the concerns with stoma management. c. Develop a detailed written list of ostomy care tasks for the patient. d. Postpone any teaching until the patient adjusts to the ileostomy.

ANS: B Encouraging the patient to share concerns assists in helping the patient adjust to the body changes. Acknowledgment of the patient's feelings and concerns is important rather than offering false reassurance. Because the patient indicates that the feelings about the ostomy are the reason for the difficulty with the many changes, development of a detailed ostomy care plan will not improve the patient's ability to manage the ostomy. Although detailed ostomy teaching may be postponed, the nurse should offer teaching about some aspects of living with an ostomy.

A 54-year-old critically ill patient with sepsis is frequently incontinent of watery stools. What action by the nurse will prevent complications associated with ongoing incontinence? a. Apply incontinence briefs. b. Use a fecal management system c. Insert a rectal tube with a drainage bag. d. Assist the patient to a commode frequently.

ANS: B Fecal management systems are designed to contain loose stools and can be in place for as long as 4 weeks without causing damage to the rectum or anal sphincters. Although incontinence briefs may be helpful, unless they are changed frequently, they are likely to increase the risk for skin breakdown. Rectal tubes are avoided because of possible damage to the anal sphincter and ulceration of the rectal mucosa. A critically ill patient will not be able to tolerate getting up frequently to use the commode or bathroom.

A 24-year-old woman with Crohn's disease develops a fever and symptoms of a urinary tract infection (UTI) with tan, fecal-smelling urine. What information will the nurse add to a general teaching plan about UTIs in order to individualize the teaching for this patient? a. Bacteria in the perianal area can enter the urethra. b. Fistulas can form between the bowel and bladder. c. Drink adequate fluids to maintain normal hydration. d. Empty the bladder before and after sexual intercourse.

ANS: B Fistulas between the bowel and bladder occur in Crohn's disease and can lead to UTI. Teaching for UTI prevention in general includes good hygiene, adequate fluid intake, and voiding before and after intercourse.

An unresponsive patient with type 2 diabetes is brought to the emergency department and diagnosed with hyperosmolar hyperglycemic syndrome (HHS). The nurse will anticipate the need to a. give a bolus of 50% dextrose. b. insert a large-bore IV catheter. c. initiate oxygen by nasal cannula. d. administer glargine (Lantus) insulin.

ANS: B HHS is initially treated with large volumes of IV fluids to correct hypovolemia. Regular insulin is administered, not a long-acting insulin. There is no indication that the patient requires oxygen. Dextrose solutions will increase the patient's blood glucose and would be contraindicated

Which information will the nurse include in teaching a patient who had a proctocolectomy and ileostomy for ulcerative colitis? a. Restrict fluid intake to prevent constant liquid drainage from the stoma. b. Use care when eating high-fiber foods to avoid obstruction of the ileum. c. Irrigate the ileostomy daily to avoid having to wear a drainage appliance. d. Change the pouch every day to prevent leakage of contents onto the skin.

ANS: B High-fiber foods are introduced gradually and should be well chewed to avoid obstruction of the ileostomy. Patients with ileostomies lose the absorption of water in the colon and need to take in increased amounts of fluid. The pouch should be drained frequently but is changed every 5 to 7 days. The drainage from an ileostomy is liquid and continuous, so control by irrigation is not possible.

Which nursing assessment of a 69-year-old patient is most important to make during initiation of thyroid replacement with levothyroxine (Synthroid)? a. Fluid balance b. Apical pulse rate c. Nutritional intake d. Orientation and alertness

ANS: B In older patients, initiation of levothyroxine therapy can increase myocardial oxygen demand and cause angina or dysrhythmias. The medication also is expected to improve mental status and fluid balance and will increase metabolic rate and nutritional needs, but these changes will not result in potentially life-threatening complications

A 53-year-old patient with Stage D heart failure and type 2 diabetes asks the nurse whether heart transplant is a possible therapy. Which response by the nurse is most appropriate? a. "Because you have diabetes, you would not be a candidate for a heart transplant." b. "The choice of a patient for a heart transplant depends on many different factors." c. "Your heart failure has not reached the stage in which heart transplants are needed." d. "People who have heart transplants are at risk for multiple complications after surgery."

ANS: B Indications for a heart transplant include end-stage heart failure (Stage D), but other factors such as coping skills, family support, and patient motivation to follow the rigorous posttransplant regimen are also considered. Diabetic patients who have well-controlled blood glucose levels may be candidates for heart transplant. Although heart transplants can be associated with many complications, this response does not address the patient's question.

After receiving change-of-shift report about the following four patients, which patient should the nurse assess first? a. A 31-year-old female with Cushing syndrome and a blood glucose level of 244 mg/dL b. A 70-year-old female taking levothyroxine (Synthroid) who has an irregular pulse of 134 c. A 53-year-old male who has Addison's disease and is due for a scheduled dose of hydrocortisone (Solu-Cortef). d. A 22-year-old male admitted with syndrome of inappropriate antidiuretic hormone (SIADH) who has a serum sodium level of 130 mEq/L

ANS: B Initiation of thyroid replacement in older adults may cause angina and cardiac dysrhythmias. The patient's high pulse rate needs rapid investigation by the nurse to assess for and intervene with any cardiac problems. The other patients also require nursing assessment and/or actions but are not at risk for life-threatening complications

Which information will the nurse teach a 23-year-old patient with lactose intolerance? a. Ice cream is relatively low in lactose. b. Live-culture yogurt is usually tolerated. c. Heating milk will break down the lactose. d. Nonfat milk is a better choice than whole milk.

ANS: B Lactose-intolerant individuals can usually eat yogurt without experiencing discomfort. Ice cream, nonfat milk, and milk that has been heated are all high in lactose.

Which question will the nurse in the endocrine clinic ask to help determine a patient's risk factors for goiter? a. "How much milk do you drink?" b. "What medications are you taking?" c. "Are your immunizations up to date?" d. "Have you had any recent neck injuries?"

ANS: B Medications that contain thyroid-inhibiting substances can cause goiter. Milk intake, neck injury, and immunization history are not risk factors for goiter

A patient who had radical neck surgery to remove a malignant tumor developed hypoparathyroidism. The nurse should plan to teach the patient about a. bisphosphonates to reduce bone demineralization. b. calcium supplements to normalize serum calcium levels. c. increasing fluid intake to decrease risk for nephrolithiasis. d. including whole grains in the diet to prevent constipation.

ANS: B Oral calcium supplements are used to maintain the serum calcium in normal range and prevent the complications of hypocalcemia. Whole grain foods decrease calcium absorption and will not be recommended. Bisphosphonates will lower serum calcium levels further by preventing calcium from being reabsorbed from bone. Kidney stones are not a complication of hypoparathyroidism and low calcium levels

After change-of-shift report, which patient should the nurse assess first? a. 40-year-old male with celiac disease who has frequent frothy diarrhea b. 30-year-old female with a femoral hernia who has abdominal pain and vomiting c. 30-year-old male with ulcerative colitis who has severe perianal skin breakdown d. 40-year-old female with a colostomy bag that is pulling away from the adhesive wafer

ANS: B Pain and vomiting with a femoral hernia suggest possible strangulation, which will necessitate emergency surgery. The other patients have less urgent problems.

The nurse is taking a health history from a 29-year-old pregnant patient at the first prenatal visit. The patient reports no personal history of diabetes but has a parent who is diabetic. Which action will the nurse plan to take first? a. Teach the patient about administering regular insulin. b. Schedule the patient for a fasting blood glucose level. c. Discuss an oral glucose tolerance test for the twenty-fourth week of pregnancy. d. Provide teaching about an increased risk for fetal problems with gestational diabetes.

ANS: B Patients at high risk for gestational diabetes should be screened for diabetes on the initial prenatal visit. An oral glucose tolerance test may also be used to check for diabetes, but it would be done before the twenty-fourth week. The other actions may also be needed (depending on whether the patient develops gestational diabetes), but they are not the first actions that the nurse should take

A new 19-year-old male patient has familial adenomatous polyposis (FAP). Which action will the nurse in the gastrointestinal clinic include in the plan of care? a. Obtain blood samples for DNA analysis. b. Schedule the patient for yearly colonoscopy. c. Provide preoperative teaching about total colectomy. d. Discuss lifestyle modifications to decrease cancer risk.

ANS: B Patients with FAP should have annual colonoscopy starting at age 16 and usually have total colectomy by age 25 to avoid developing colorectal cancer. DNA analysis is used to make the diagnosis, but is not needed now for this patient. Lifestyle modifications will not decrease cancer risk for this patient.

Which prescribed intervention for a 61-year-old female patient with chronic short bowel syndrome will the nurse question? a. Ferrous sulfate (Feosol) 325 mg daily b. Senna (Senokot) 1 tablet every day c. Psyllium (Metamucil) 2.1 grams 3 times daily d. Diphenoxylate with atropine (Lomotil) prn loose stools

ANS: B Patients with short bowel syndrome have diarrhea because of decreased nutrient and fluid absorption and would not need stimulant laxatives. Iron supplements are used to prevent iron-deficiency anemia, bulk-forming laxatives help make stools less watery, and opioid antidiarrheal drugs are helpful in slowing intestinal transit time.

Which action should the nurse take after a 36-year-old patient treated with intramuscular glucagon for hypoglycemia regains consciousness? a. Assess the patient for symptoms of hyperglycemia. b. Give the patient a snack of peanut butter and crackers. c. Have the patient drink a glass of orange juice or nonfat milk. d. Administer a continuous infusion of 5% dextrose for 24 hours.

ANS: B Rebound hypoglycemia can occur after glucagon administration, but having a meal containing complex carbohydrates plus protein and fat will help prevent hypoglycemia. Orange juice and nonfat milk will elevate blood glucose rapidly, but the cheese and crackers will stabilize blood glucose. Administration of IV glucose might be used in patients who were unable to take in nutrition orally. The patient should be assessed for symptoms of hypoglycemia after glucagon administration

A 58-year-old man with blunt abdominal trauma from a motor vehicle crash undergoes peritoneal lavage. If the lavage returns brown fecal drainage, which action will the nurse plan to take next? a. Auscultate the bowel sounds. b. Prepare the patient for surgery. c. Check the patient's oral temperature. d. Obtain information about the accident.

ANS: B Return of brown drainage and fecal material suggests perforation of the bowel and the need for immediate surgery. Auscultation of bowel sounds, checking the temperature, and obtaining information about the accident are appropriate actions, but the priority is to prepare to send the patient for emergency surgery.

Which information will the nurse include when teaching a 50-year-old male patient about somatropin (Genotropin)? a. The medication will be needed for 3 to 6 months. b. Inject the medication subcutaneously every day. c. Blood glucose levels may decrease when taking the medication. d. Stop taking the medication if swelling of the hands or feet occurs.

ANS: B Somatropin is injected subcutaneously on a daily basis, preferably in the evening. The patient will need to continue on somatropin for life. If swelling or other common adverse effects occur, the health care provider should be notified. Growth hormone will increase blood glucose levels

Which intervention will the nurse include in the plan of care for a 52-year-old male patient with syndrome of inappropriate antidiuretic hormone (SIADH)? a. Monitor for peripheral edema. b. Offer patient hard candies to suck on. c. Encourage fluids to 2 to 3 liters per day. d. Keep head of bed elevated to 30 degrees.

ANS: B Sucking on hard candies decreases thirst for a patient on fluid restriction. Patients with SIADH are on fluid restrictions of 800 to 1000 mL/day. Peripheral edema is not seen with SIADH. The head of the bed is elevated no more than 10 degrees to increase left atrial filling pressure and decrease antidiuretic hormone (ADH) release

Which patient statement indicates that the nurse's teaching about sulfasalazine (Azulfidine) for ulcerative colitis has been effective? a. "The medication will be tapered if I need surgery." b. "I will need to use a sunscreen when I am outdoors." c. "I will need to avoid contact with people who are sick." d. "The medication will prevent infections that cause the diarrhea."

ANS: B Sulfasalazine may cause photosensitivity in some patients. It is not used to treat infections. Sulfasalazine does not reduce immune function. Unlike corticosteroids, tapering of sulfasalazine is not needed.

The nurse will plan to monitor a patient diagnosed with a pheochromocytoma for a. flushing. b. headache. c. bradycardia. d. hypoglycemia.

ANS: B The classic clinical manifestations of pheochromocytoma are hypertension, tachycardia, severe headache, diaphoresis, and abdominal or chest pain. Elevated blood glucose may also occur because of sympathetic nervous system stimulation. Bradycardia and flushing would not be expected

A 25-year-old male patient calls the clinic complaining of diarrhea for 24 hours. Which action should the nurse take first? a. Inform the patient that laboratory testing of blood and stools will be necessary. b. Ask the patient to describe the character of the stools and any associated symptoms. c. Suggest that the patient drink clear liquid fluids with electrolytes, such as Gatorade or Pedialyte. d. Advise the patient to use over-the-counter loperamide (Imodium) to slow gastrointestinal (GI) motility.

ANS: B The initial response by the nurse should be further assessment of the patient. The other responses may be appropriate, depending on what is learned in the assessment.

The nurse will determine that teaching a 67-year-old man to irrigate his new colostomy has been effective if the patient a. inserts the irrigation tubing 4 to 6 inches into the stoma. b. hangs the irrigating container 18 inches above the stoma. c. stops the irrigation and removes the irrigating cone if cramping occurs. d. fills the irrigating container with 1000 to 2000 mL of lukewarm tap water.

ANS: B The irrigating container should be hung 18 to 24 inches above the stoma. If cramping occurs, the irrigation should be temporarily stopped and the cone left in place. Five hundred to 1000 mL of water should be used for irrigation. An irrigation cone, rather than tubing, should be inserted into the stoma; 4 to 6 inches would be too far for safe insertion.

A 37-year-old patient has just arrived in the postanesthesia recovery unit (PACU) after a thyroidectomy. Which information is most important to communicate to the surgeon? a. The patient reports 7/10 incisional pain. b. The patient has increasing neck swelling. c. The patient is sleepy and difficult to arouse. d. The patient's cardiac rate is 112 beats/minute.

ANS: B The neck swelling may lead to respiratory difficulty, and rapid intervention is needed to prevent airway obstruction. The incisional pain should be treated but is not unusual after surgery. A heart rate of 112 is not unusual in a patient who has been hyperthyroid and has just arrived in the PACU from surgery. Sleepiness in the immediate postoperative period is expected.

A 38-year-old male patient is admitted to the hospital in Addisonian crisis. Which patient statement supports a nursing diagnosis of ineffective self-health management related to lack of knowledge about management of Addison's disease? a. "I frequently eat at restaurants, and my food has a lot of added salt." b. "I had the stomach flu earlier this week, so I couldn't take the hydrocortisone." c. "I always double my dose of hydrocortisone on the days that I go for a long run." d. "I take twice as much hydrocortisone in the morning dose as I do in the afternoon."

ANS: B The need for hydrocortisone replacement is increased with stressors such as illness, and the patient needs to be taught to call the health care provider because medication and IV fluids and electrolytes may need to be given. The other patient statements indicate appropriate management of the Addison's disease.

Which laboratory value reported to the nurse by the unlicensed assistive personnel (UAP) indicates the most urgent need for the nurse's assessment of the patient? a. Bedtime glucose of 140 mg/dL b. Noon blood glucose of 52 mg/dL c. Fasting blood glucose of 130 mg/dL d. 2-hr postprandial glucose of 220 mg/dL

ANS: B The nurse should assess the patient with a blood glucose level of 52 mg/dL for symptoms of hypoglycemia and give the patient a carbohydrate-containing beverage such as orange juice. The other values are within an acceptable range or not immediately dangerous for a diabetic patient

A 71-year-old male patient tells the nurse that growing old causes constipation so he has been using a suppository for constipation every morning. Which action should the nurse take first? a. Encourage the patient to increase oral fluid intake. b. Assess the patient about risk factors for constipation. c. Suggest that the patient increase intake of high-fiber foods. d. Teach the patient that a daily bowel movement is unnecessary.

ANS: B The nurse's initial action should be further assessment of the patient for risk factors for constipation and for his usual bowel pattern. The other actions may be appropriate but will be based on the assessment.

The nurse identifies a need for additional teaching when the patient who is self-monitoring blood glucose a. washes the puncture site using warm water and soap. b. chooses a puncture site in the center of the finger pad. c. hangs the arm down for a minute before puncturing the site. d. says the result of 120 mg indicates good blood sugar control.

ANS: B The patient is taught to choose a puncture site at the side of the finger pad because there are fewer nerve endings along the side of the finger pad. The other patient actions indicate that teaching has been effective

Which nursing action will be included in the plan of care for a 55-year-old patient with Graves' disease who has exophthalmos? a. Place cold packs on the eyes to relieve pain and swelling. b. Elevate the head of the patient's bed to reduce periorbital fluid. c. Apply alternating eye patches to protect the corneas from irritation. d. Teach the patient to blink every few seconds to lubricate the corneas.

ANS: B The patient should sit upright as much as possible to promote fluid drainage from the periorbital area. With exophthalmos, the patient is unable to close the eyes completely to blink. Lubrication of the eyes, rather than eye patches, will protect the eyes from developing corneal scarring. The swelling of the eye is not caused by excessive blood flow to the eye, so cold packs will not be helpful

A 63-year-old patient with primary hyperparathyroidism has a serum phosphorus level of 1.7 mg/dL (0.55 mmol/L) and calcium of 14 mg/dL (3.5 mmol/L). Which nursing action should be included in the plan of care? a. Restrict the patient to bed rest. b. Encourage 4000 mL of fluids daily. c. Institute routine seizure precautions. d. Assess for positive Chvostek's sign.

ANS: B The patient with hypercalcemia is at risk for kidney stones, which may be prevented by a high fluid intake. Seizure precautions and monitoring for Chvostek's or Trousseau's sign are appropriate for hypocalcemic patients. The patient should engage in weight-bearing exercise to decrease calcium loss from bone

A patient who had a subtotal thyroidectomy earlier today develops laryngeal stridor and a cramp in the right hand upon returning to the surgical nursing unit. Which collaborative action will the nurse anticipate next? a. Suction the patient's airway. b. Administer IV calcium gluconate. c. Plan for emergency tracheostomy. d. Prepare for endotracheal intubation.

ANS: B The patient's clinical manifestations of stridor and cramping are consistent with tetany caused by hypocalcemia resulting from damage to the parathyroid glands during surgery. Endotracheal intubation or tracheostomy may be needed if the calcium does not resolve the stridor. Suctioning will not correct the stridor

After receiving change-of-shift report on a heart failure unit, which patient should the nurse assess first? a. Patient who is taking carvedilol (Coreg) and has a heart rate of 58 b. Patient who is taking digoxin and has a potassium level of 3.1 mEq/L c. Patient who is taking isosorbide dinitrate/hydralazine (BiDil) and has a headache d. Patient who is taking captopril (Capoten) and has a frequent nonproductive cough

ANS: B The patient's low potassium level increases the risk for digoxin toxicity and potentially fatal dysrhythmias. The nurse should assess the patient for other signs of digoxin toxicity and then notify the health care provider about the potassium level. The other patients also have side effects of their medications, but their symptoms do not indicate potentially life-threatening complications.

Which information will the nurse include when teaching a 50-year-old patient who has type 2 diabetes about glyburide (Micronase, DiaBeta, Glynase)? a. Glyburide decreases glucagon secretion from the pancreas. b. Glyburide stimulates insulin production and release from the pancreas. c. Glyburide should be taken even if the morning blood glucose level is low. d. Glyburide should not be used for 48 hours after receiving IV contrast media.

ANS: B The sulfonylureas stimulate the production and release of insulin from the pancreas. If the glucose level is low, the patient should contact the health care provider before taking the glyburide, because hypoglycemia can occur with this class of medication. Metformin should be held for 48 hours after administration of IV contrast media, but this is not necessary for glyburide. Glucagon secretion is not affected by glyburide

An active 28-year-old male with type 1 diabetes is being seen in the endocrine clinic. Which finding may indicate the need for a change in therapy? a. Hemoglobin A1C level 6.2% b. Blood pressure 146/88 mmHg c. Heart rate at rest 58 beats/minute d. High density lipoprotein (HDL) level 65 mg/dL

ANS: B To decrease the incidence of macrovascular and microvascular problems in patients with diabetes, the goal blood pressure is usually 130/80. An A1C less than 6.5%, a low resting heart rate (consistent with regular aerobic exercise in a young adult), and an HDL level of 65 mg/dL all indicate that the patient's diabetes and risk factors for vascular disease are well controlled.

A diabetic patient who has reported burning foot pain at night receives a new prescription. Which information should the nurse teach the patient about amitriptyline (Elavil)? a. Amitriptyline decreases the depression caused by your foot pain. b. Amitriptyline helps prevent transmission of pain impulses to the brain. c. Amitriptyline corrects some of the blood vessel changes that cause pain. d. Amitriptyline improves sleep and makes you less aware of nighttime pain.

ANS: B Tricyclic antidepressants decrease the transmission of pain impulses to the spinal cord and brain. Tricyclic antidepressants also improve sleep quality and are used for depression, but that is not the major purpose for their use in diabetic neuropathy. The blood vessel changes that contribute to neuropathy are not affected by tricyclic antidepressants

A 56-year-old patient who is disoriented and reports a headache and muscle cramps is hospitalized with possible syndrome of inappropriate antidiuretic hormone (SIADH). The nurse would expect the initial laboratory results to include a(n) a. elevated hematocrit. b. decreased serum sodium. c. low urine specific gravity. d. increased serum chloride.

ANS: B When water is retained, the serum sodium level will drop below normal, causing the clinical manifestations reported by the patient. The hematocrit will decrease because of the dilution caused by water retention. Urine will be more concentrated with a higher specific gravity. The serum chloride level will usually decrease along with the sodium level

The nurse is providing preoperative teaching for a 61-year-old man scheduled for an abdominal-perineal resection. Which information will the nurse include? a. Another surgery in 8 to 12 weeks will be used to create an ileal-anal reservoir. b. The patient will begin sitting in a chair at the bedside on the first postoperative day. c. The patient will drink polyethylene glycol lavage solution (GoLYTELY) preoperatively. d. IV antibiotics will be started at least 24 hours before surgery to reduce the bowel bacteria.

ANS: C A bowel-cleansing agent is used to empty the bowel before surgery to reduce the risk for infection. A permanent colostomy is created with this surgery. Sitting is contraindicated after an abdominal-perineal resection. Oral antibiotics (rather than IV antibiotics) are given to reduce colonic and rectal bacteria.

A 26-year-old woman is being evaluated for vomiting and abdominal pain. Which question from the nurse will be most useful in determining the cause of the patient's symptoms? a. "What type of foods do you eat?" b. "Is it possible that you are pregnant?" c. "Can you tell me more about the pain?" d. "What is your usual elimination pattern?"

ANS: C A complete description of the pain provides clues about the cause of the problem. Although the nurse should ask whether the patient is pregnant to determine whether the patient might have an ectopic pregnancy and before any radiology studies are done, this information is not the most useful in determining the cause of the pain. The usual diet and elimination patterns are less helpful in determining the reason for the patient's symptoms.

A 29-year-old woman with systemic lupus erythematosus has been prescribed 2 weeks of high-dose prednisone therapy. Which information about the prednisone is most important for the nurse to include? a. "Weigh yourself daily to monitor for weight gain caused by increased appetite." b. "A weight-bearing exercise program will help minimize the risk for osteoporosis." c. "The prednisone dose should be decreased gradually rather than stopped suddenly." d. "Call the health care provider if you experience mood alterations with the prednisone."

ANS: C Acute adrenal insufficiency may occur if exogenous corticosteroids are suddenly stopped. Mood alterations and weight gain are possible adverse effects of corticosteroid use, but these are not life-threatening effects. Osteoporosis occurs when patients take corticosteroids for longer periods.

The nurse is caring for a patient following an adrenalectomy. The highest priority in the immediate postoperative period is to a. protect the patient's skin. b. monitor for signs of infection. c. balance fluids and electrolytes. d. prevent emotional disturbances.

ANS: C After adrenalectomy, the patient is at risk for circulatory instability caused by fluctuating hormone levels, and the focus of care is to assess and maintain fluid and electrolyte status through the use of IV fluids and corticosteroids. The other goals are also important for the patient but are not as immediately life threatening as the circulatory collapse that can occur with fluid and electrolyte disturbances

A patient has just arrived on the unit after a thyroidectomy. Which action should the nurse take first? a. Observe the dressing for bleeding. b. Check the blood pressure and pulse. c. Assess the patient's respiratory effort. d. Support the patient's head with pillows.

ANS: C Airway obstruction is a possible complication after thyroidectomy because of swelling or bleeding at the site or tetany. The priority nursing action is to assess the airway. The other actions are also part of the standard nursing care postthyroidectomy but are not as high of a priority

A patient who is receiving dobutamine (Dobutrex) for the treatment of acute decompensated heart failure (ADHF) has the following nursing interventions included in the plan of care. Which action will be most appropriate for the registered nurse (RN) to delegate to an experienced licensed practical/vocational nurse (LPN/LVN)? a. Assess the IV insertion site for signs of extravasation. b. Teach the patient the reasons for remaining on bed rest. c. Monitor the patient's blood pressure and heart rate every hour. d. Titrate the rate to keep the systolic blood pressure >90 mm Hg.

ANS: C An experienced LPN/LVN would be able to monitor BP and heart rate and would know to report significant changes to the RN. Teaching patients, making adjustments to the drip rate for vasoactive medications, and monitoring for serious complications such as extravasation require RN level education and scope of practice.

After a 22-year-old female patient with a pituitary adenoma has had a hypophysectomy, the nurse will teach about the need for a. sodium restriction to prevent fluid retention. b. insulin to maintain normal blood glucose levels. c. oral corticosteroids to replace endogenous cortisol. d. chemotherapy to prevent malignant tumor recurrence.

ANS: C Antidiuretic hormone (ADH), cortisol, and thyroid hormone replacement will be needed for life after hypophysectomy. Without the effects of adrenocorticotropic hormone (ACTH) and cortisol, the blood glucose and serum sodium will be low unless cortisol is replaced. An adenoma is a benign tumor, and chemotherapy will not be needed

Which diagnostic test will be most useful to the nurse in determining whether a patient admitted with acute shortness of breath has heart failure? a. Serum troponin b. Arterial blood gases c. B-type natriuretic peptide d. 12-lead electrocardiogram

ANS: C B-type natriuretic peptide (BNP) is secreted when ventricular pressures increase, as they do with heart failure. Elevated BNP indicates a probable or very probable diagnosis of heart failure. A twelve-lead electrocardiogram, arterial blood gases, and troponin may also be used in determining the causes or effects of heart failure but are not as clearly diagnostic of heart failure as BNP

Which action will the nurse include in the plan of care for a 42-year-old patient who is being admitted with Clostridium difficile? a. Educate the patient about proper food storage. b. Order a diet with no dairy products for the patient. c. Place the patient in a private room on contact isolation. d. Teach the patient about why antibiotics will not be used.

ANS: C Because C. difficile is highly contagious, the patient should be placed in a private room and contact precautions should be used. There is no need to restrict dairy products for this type of diarrhea. Metronidazole (Flagyl) is frequently used to treat C. difficile. Improper food handling and storage do not cause C. difficile.

A 71-year-old patient had an abdominal-perineal resection for colon cancer. Which nursing action is most important to include in the plan of care for the day after surgery? a. Teach about a low-residue diet. b. Monitor output from the stoma. c. Assess the perineal drainage and incision. d. Encourage acceptance of the colostomy stoma.

ANS: C Because the perineal wound is at high risk for infection, the initial care is focused on assessment and care of this wound. Teaching about diet is best done closer to discharge from the hospital. There will be very little drainage into the colostomy until peristalsis returns. The patient will be encouraged to assist with the colostomy, but this is not the highest priority in the immediate postoperative period.

A 62-year-old patient has had a hemorrhoidectomy at an outpatient surgical center. Which instructions will the nurse include in discharge teaching? a. Maintain a low-residue diet until the surgical area is healed. b. Use ice packs on the perianal area to relieve pain and swelling. c. Take prescribed pain medications before a bowel movement is expected. d. Delay having a bowel movement for several days until healing has occurred.

ANS: C Bowel movements may be very painful, and patients may avoid defecation unless pain medication is taken before the bowel movement. A high-residue diet will increase stool bulk and prevent constipation. Delay of bowel movements is likely to lead to constipation. Warm sitz baths rather than ice packs are used to relieve pain and keep the surgical area clean.

A patient with heart failure has a new order for captopril (Capoten) 12.5 mg PO. After administering the first dose and teaching the patient about the drug, which statement by the patient indicates that teaching has been effective? a. "I will be sure to take the medication with food." b. "I will need to eat more potassium-rich foods in my diet." c. "I will call for help when I need to get up to use the bathroom." d. "I will expect to feel more short of breath for the next few days."

ANS: C Captopril can cause hypotension, especially after the initial dose, so it is important that the patient not get up out of bed without assistance until the nurse has had a chance to evaluate the effect of the first dose. The angiotensin-converting enzyme (ACE) inhibitors are potassium sparing, and the nurse should not teach the patient to purposely increase sources of dietary potassium. Increased shortness of breath is expected with the initiation of -adrenergic blocker therapy for heart failure, not for ACE inhibitor therapy. ACE inhibitors are best absorbed when taken an hour before eating.

Which activity in the care of a 48-year-old female patient with a new colostomy could the nurse delegate to unlicensed assistive personnel (UAP)? a. Document the appearance of the stoma. b. Place a pouching system over the ostomy. c. Drain and measure the output from the ostomy. d. Check the skin around the stoma for breakdown.

ANS: C Draining and measuring the output from the ostomy is included in UAP education and scope of practice. The other actions should be implemented by LPNs or RNs.

Which diet choice by the patient with an acute exacerbation of inflammatory bowel disease (IBD) indicates a need for more teaching? a. Scrambled eggs b. White toast and jam c. Oatmeal with cream d. Pancakes with syrup

ANS: C During acute exacerbations of IBD, the patient should avoid high-fiber foods such as whole grains. High-fat foods also may cause diarrhea in some patients. The other choices are low residue and would be appropriate for this patient.

Which statement by a nurse to a patient newly diagnosed with type 2 diabetes is correct? a. Insulin is not used to control blood glucose in patients with type 2 diabetes. b. Complications of type 2 diabetes are less serious than those of type 1 diabetes. c. Changes in diet and exercise may control blood glucose levels in type 2 diabetes. d. Type 2 diabetes is usually diagnosed when the patient is admitted with a hyperglycemic coma.

ANS: C For some patients with type 2 diabetes, changes in lifestyle are sufficient to achieve blood glucose control. Insulin is frequently used for type 2 diabetes, complications are equally severe as for type 1 diabetes, and type 2 diabetes is usually diagnosed with routine laboratory testing or after a patient develops complications such as frequent yeast infections.

When a patient who takes metformin (Glucophage) to manage type 2 diabetes develops an allergic rash from an unknown cause, the health care provider prescribes prednisone (Deltasone). The nurse will anticipate that the patient may a. need a diet higher in calories while receiving prednisone. b. develop acute hypoglycemia while taking the prednisone. c. require administration of insulin while taking prednisone. d. have rashes caused by metformin-prednisone interactions.

ANS: C Glucose levels increase when patients are taking corticosteroids, and insulin may be required to control blood glucose. Hypoglycemia is not a side effect of prednisone. Rashes are not an adverse effect caused by taking metformin and prednisone simultaneously. The patient may have an increased appetite when taking prednisone, but will not need a diet that is higher in calories

A patient has recently started on digoxin (Lanoxin) in addition to furosemide (Lasix) and captopril (Capoten) for the management of heart failure. Which assessment finding by the home health nurse is a priority to communicate to the health care provider? a. Presence of 1 to 2+ edema in the feet and ankles b. Palpable liver edge 2 cm below the ribs on the right side c. Serum potassium level 3.0 mEq/L after 1 week of therapy d. Weight increase from 120 pounds to 122 pounds over 3 days

ANS: C Hypokalemia can predispose the patient to life-threatening dysrhythmias (e.g., premature ventricular contractions), and potentiate the actions of digoxin and increase the risk for digoxin toxicity, which can also cause life-threatening dysrhythmias. The other data indicate that the patient's heart failure requires more effective therapies, but they do not require nursing action as rapidly as the low serum potassium level.

The health care provider suspects the Somogyi effect in a 50-year-old patient whose 6:00 AM blood glucose is 230 mg/dL. Which action will the nurse teach the patient to take? a. Avoid snacking at bedtime. b. Increase the rapid-acting insulin dose. c. Check the blood glucose during the night d. Administer a larger dose of long-acting insulin.

ANS: C If the Somogyi effect is causing the patient's increased morning glucose level, the patient will experience hypoglycemia between 2:00 and 4:00 AM. The dose of insulin will be reduced, rather than increased. A bedtime snack is used to prevent hypoglycemic episodes during the night

A 26-year-old female with type 1 diabetes develops a sore throat and runny nose after caring for her sick toddler. The patient calls the clinic for advice about her symptoms and a blood glucose level of 210 mg/dL despite taking her usual glargine (Lantus) and lispro (Humalog) insulin. The nurse advises the patient to a. use only the lispro insulin until the symptoms are resolved. b. limit intake of calories until the glucose is less than 120 mg/dL. c. monitor blood glucose every 4 hours and notify the clinic if it continues to rise. d. decrease intake of carbohydrates until glycosylated hemoglobin is less than 7%.

ANS: C Infection and other stressors increase blood glucose levels and the patient will need to test blood glucose frequently, treat elevations appropriately with lispro insulin, and call the health care provider if glucose levels continue to be elevated. Discontinuing the glargine will contribute to hyperglycemia and may lead to diabetic ketoacidosis (DKA). Decreasing carbohydrate or caloric intake is not appropriate because the patient will need more calories when ill. Glycosylated hemoglobin testing is not used to evaluate short-term alterations in blood glucose

When a patient with type 2 diabetes is admitted for a cholecystectomy, which nursing action can the nurse delegate to a licensed practical/vocational nurse (LPN/LVN)? a. Communicate the blood glucose level and insulin dose to the circulating nurse in surgery. b. Discuss the reason for the use of insulin therapy during the immediate postoperative period. c. Administer the prescribed lispro (Humalog) insulin before transporting the patient to surgery. d. Plan strategies to minimize the risk for hypoglycemia or hyperglycemia during the postoperative period.

ANS: C LPN/LVN education and scope of practice includes administration of insulin. Communication about patient status with other departments, planning, and patient teaching are skills that require RN education and scope of practice.

Which information will the nurse teach a 48-year-old patient who has been newly diagnosed with Graves' disease? a. Exercise is contraindicated to avoid increasing metabolic rate. b. Restriction of iodine intake is needed to reduce thyroid activity. c. Antithyroid medications may take several months for full effect. d. Surgery will eventually be required to remove the thyroid gland.

ANS: C Medications used to block the synthesis of thyroid hormones may take 2 to 3 months before the full effect is seen. Large doses of iodine are used to inhibit the synthesis of thyroid hormones. Exercise using large muscle groups is encouraged to decrease the irritability and hyperactivity associated with high levels of thyroid hormones. Radioactive iodine is the most common treatment for Graves' disease although surgery may be used

The nurse determines a need for additional instruction when the patient with newly diagnosed type 1 diabetes says which of the following? a. "I can have an occasional alcoholic drink if I include it in my meal plan." b. "I will need a bedtime snack because I take an evening dose of NPH insulin." c. "I can choose any foods, as long as I use enough insulin to cover the calories." d. "I will eat something at meal times to prevent hypoglycemia, even if I am not hungry."

ANS: C Most patients with type 1 diabetes need to plan diet choices very carefully. Patients who are using intensified insulin therapy have considerable flexibility in diet choices but still should restrict dietary intake of items such as fat, protein, and alcohol. The other patient statements are correct and indicate good understanding of the diet instruction

The nurse plans discharge teaching for a patient with chronic heart failure who has prescriptions for digoxin (Lanoxin) and hydrochlorothiazide (HydroDIURIL). Appropriate instructions for the patient include a. limit dietary sources of potassium. b. take the hydrochlorothiazide before bedtime. c. notify the health care provider if nausea develops. d. skip the digoxin if the pulse is below 60 beats/minute.

ANS: C Nausea is an indication of digoxin toxicity and should be reported so that the provider can assess the patient for toxicity and adjust the digoxin dose, if necessary. The patient will need to include potassium-containing foods in the diet to avoid hypokalemia. Patients should be taught to check their pulse daily before taking the digoxin and if the pulse is less than 60, to call their provider before taking the digoxin. Diuretics should be taken early in the day to avoid sleep disruption.

A patient who has chronic heart failure tells the nurse, "I was fine when I went to bed, but I woke up in the middle of the night feeling like I was suffocating!" The nurse will document this assessment finding as a. orthopnea. b. pulsus alternans. c. paroxysmal nocturnal dyspnea. d. acute bilateral pleural effusion.

ANS: C Paroxysmal nocturnal dyspnea is caused by the reabsorption of fluid from dependent body areas when the patient is sleeping and is characterized by waking up suddenly with the feeling of suffocation. Pulsus alternans is the alternation of strong and weak peripheral pulses during palpation. Orthopnea indicates that the patient is unable to lie flat because of dyspnea. Pleural effusions develop over a longer time period.

A 26-year-old patient with diabetes rides a bicycle to and from work every day. Which site should the nurse teach the patient to administer the morning insulin? a. thigh. b. buttock. c. abdomen. d. upper arm.

ANS: C Patients should be taught not to administer insulin into a site that will be exercised because exercise will increase the rate of absorption. The thigh, buttock, and arm are all exercised by riding a bicycle

A 37-year-old patient is being admitted with a diagnosis of Cushing syndrome. Which findings will the nurse expect during the assessment? a. Chronically low blood pressure b. Bronzed appearance of the skin c. Purplish streaks on the abdomen d. Decreased axillary and pubic hair

ANS: C Purplish-red striae on the abdomen are a common clinical manifestation of Cushing syndrome. Hypotension and bronzed-appearing skin are manifestations of Addison's disease. Decreased axillary and pubic hair occur with androgen deficiency

Four hours after a bowel resection, a 74-year-old male patient with a nasogastric tube to suction complains of nausea and abdominal distention. The first action by the nurse should be to a. auscultate for hypotonic bowel sounds. b. notify the patient's health care provider. c. reposition the tube and check for placement. d. remove the tube and replace it with a new one.

ANS: C Repositioning the tube will frequently facilitate drainage. Because this is a common occurrence, it is not appropriate to notify the health care provider unless other interventions do not resolve the problem. Information about the presence or absence of bowel sounds will not be helpful in improving drainage. Removing the tube and replacing it are unnecessarily traumatic to the patient, so that would only be done if the tube was completely occluded.

After the nurse has finished teaching a patient who has a new prescription for exenatide (Byetta), which patient statement indicates that the teaching has been effective? a. "I may feel hungrier than usual when I take this medicine." b. "I will not need to worry about hypoglycemia with the Byetta." c. "I should take my daily aspirin at least an hour before the Byetta." d. "I will take the pill at the same time I eat breakfast in the morning."

ANS: C Since exenatide slows gastric emptying, oral medications should be taken at least an hour before the exenatide to avoid slowing absorption. Exenatide is injected and increases feelings of satiety. Hypoglycemia can occur with this medication

IV sodium nitroprusside (Nipride) is ordered for a patient with acute pulmonary edema. During the first hours of administration, the nurse will need to titrate the nitroprusside rate if the patient develops a. ventricular ectopy. b. a dry, hacking cough. c. a systolic BP <90 mm Hg. d. a heart rate <50 beats/minute.

ANS: C Sodium nitroprusside is a potent vasodilator, and the major adverse effect is severe hypotension. Coughing and bradycardia are not adverse effects of this medication. Nitroprusside does not cause increased ventricular ectopy.

Which information obtained by the nurse in the endocrine clinic about a patient who has been taking prednisone (Deltasone) 40 mg daily for 3 weeks is most important to report to the health care provider? a. Patient's blood pressure is 148/94 mm Hg. b. Patient has bilateral 2+ pitting ankle edema. c. Patient stopped taking the medication 2 days ago. d. Patient has not been taking the prescribed vitamin D.

ANS: C Sudden cessation of corticosteroids after taking the medication for a week or more can lead to adrenal insufficiency, with problems such as severe hypotension and hypoglycemia. The patient will need immediate evaluation by the health care provider to prevent and/or treat adrenal insufficiency. The other information will also be reported, but does not require rapid treatment

The nurse working on the heart failure unit knows that teaching an older female patient with newly diagnosed heart failure is effective when the patient states that a. she will take furosemide (Lasix) every day at bedtime. b. the nitroglycerin patch is applied when any chest pain develops. c. she will call the clinic if her weight goes from 124 to 128 pounds in a week. d. an additional pillow can help her sleep if she is feeling short of breath at night.

ANS: C Teaching for a patient with heart failure includes information about the need to weigh daily and notify the health care provider about an increase of 3 pounds in 2 days or 3 to 5 pounds in a week. Nitroglycerin patches are used primarily to reduce preload (not to prevent chest pain) in patients with heart failure and should be used daily, not on an "as needed" basis. Diuretics should be taken earlier in the day to avoid nocturia and sleep disturbance. The patient should call the clinic if increased orthopnea develops, rather than just compensating by further elevating the head of the bed.

A patient with chronic heart failure who is taking a diuretic and an angiotensin-converting enzyme (ACE) inhibitor and who is on a low-sodium diet tells the home health nurse about a 5-pound weight gain in the last 3 days. The nurse's priority action will be to a. have the patient recall the dietary intake for the last 3 days. b. ask the patient about the use of the prescribed medications. c. assess the patient for clinical manifestations of acute heart failure. d. teach the patient about the importance of restricting dietary sodium.

ANS: C The 5-pound weight gain over 3 days indicates that the patient's chronic heart failure may be worsening. It is important that the patient be assessed immediately for other clinical manifestations of decompensation, such as lung crackles. A dietary recall to detect hidden sodium in the diet, reinforcement of sodium restrictions, and assessment of medication compliance may be appropriate interventions but are not the first nursing actions indicated.

Which topic will the nurse plan to include in discharge teaching for a patient with systolic heart failure and an ejection fraction of 33%? a. Need to begin an aerobic exercise program several times weekly b. Use of salt substitutes to replace table salt when cooking and at the table c. Benefits and side effects of angiotensin-converting enzyme (ACE) inhibitors d. Importance of making an annual appointment with the primary care provider

ANS: C The core measures for the treatment of heart failure established by The Joint Commission indicate that patients with an ejection fraction (EF) <40% receive an ACE inhibitor to decrease the progression of heart failure. Aerobic exercise may not be appropriate for a patient with this level of heart failure, salt substitutes are not usually recommended because of the risk of hyperkalemia, and the patient will need to see the primary care provider more frequently than annually

While admitting an 82-year-old with acute decompensated heart failure to the hospital, the nurse learns that the patient lives alone and sometimes confuses the "water pill" with the "heart pill." When planning for the patient's discharge the nurse will facilitate a a. consult with a psychologist. b. transfer to a long-term care facility. c. referral to a home health care agency. d. arrangements for around-the-clock care.

ANS: C The data about the patient suggest that assistance in developing a system for taking medications correctly at home is needed. A home health nurse will assess the patient's home situation and help the patient develop a method for taking the two medications as directed. There is no evidence that the patient requires services such as a psychologist consult, long-term care, or around-the-clock home care.

A few weeks after an 82-year-old with a new diagnosis of type 2 diabetes has been placed on metformin (Glucophage) therapy and taught about appropriate diet and exercise, the home health nurse makes a visit. Which finding by the nurse is most important to discuss with the health care provider? a. Hemoglobin A1C level is 7.9%. b. Last eye exam was 18 months ago. c. Glomerular filtration rate is decreased. d. Patient has questions about the prescribed diet.

ANS: C The decrease in renal function may indicate a need to adjust the dose of metformin or change to a different medication. In older patients, the goal for A1C may be higher in order to avoid complications associated with hypoglycemia. The nurse will plan on scheduling the patient for an eye exam and addressing the questions about diet, but the biggest concern is the patient's decreased renal function

Which assessment finding of a 42-year-old patient who had a bilateral adrenalectomy requires the most rapid action by the nurse? a. The blood glucose is 176 mg/dL. b. The lungs have bibasilar crackles. c. The blood pressure (BP) is 88/50 mm Hg. d. The patient reports 5/10 incisional pain.

ANS: C The decreased BP indicates possible adrenal insufficiency. The nurse should immediately notify the health care provider so that corticosteroid medications can be administered. The nurse should also address the elevated glucose, incisional pain, and crackles with appropriate collaborative or nursing actions, but prevention and treatment of acute adrenal insufficiency is the priority after adrenalectomy.

A 76-year-old patient with obstipation has a fecal impaction and is incontinent of liquid stool. Which action should the nurse take first? a. Administer bulk-forming laxatives. b. Assist the patient to sit on the toilet. c. Manually remove the impacted stool. d. Increase the patient's oral fluid intake.

ANS: C The initial action with a fecal impaction is manual disimpaction. The other actions will be used to prevent future constipation and impactions.

A 19-year-old female is brought to the emergency department with a knife handle protruding from the abdomen. During the initial assessment of the patient, the nurse should a. remove the knife and assess the wound. b. determine the presence of Rovsing sign. c. check for circulation and tissue perfusion. d. insert a urinary catheter and assess for hematuria.

ANS: C The initial assessment is focused on determining whether the patient has hypovolemic shock. The knife should not be removed until the patient is in surgery, where bleeding can be controlled. Rovsing sign is assessed in the patient with suspected appendicitis. A patient with a knife in place will be taken to surgery and assessed for bladder trauma there.

While assessing a 68-year-old with ascites, the nurse also notes jugular venous distention (JVD) with the head of the patient's bed elevated 45 degrees. The nurse knows this finding indicates a. decreased fluid volume. b. jugular vein atherosclerosis. c. increased right atrial pressure. d. incompetent jugular vein valves.

ANS: C The jugular veins empty into the superior vena cava and then into the right atrium, so JVD with the patient sitting at a 45-degree angle reflects increased right atrial pressure. JVD is an indicator of excessive fluid volume (increased preload), not decreased fluid volume. JVD is not caused by incompetent jugular vein valves or atherosclerosis.

An outpatient who has chronic heart failure returns to the clinic after 2 weeks of therapy with metoprolol (Toprol XL). Which assessment finding is most important for the nurse to report to the health care provider? a. 2+ pedal edema b. Heart rate of 56 beats/minute c. Blood pressure (BP) of 88/42 mm Hg d. Complaints of fatigue

ANS: C The patient's BP indicates that the dose of metoprolol may need to be decreased because of hypotension. Bradycardia is a frequent adverse effect of -adrenergic blockade, but the rate of 56 is not unusual with â-adrenergic blocker therapy. -Adrenergic blockade initially will worsen symptoms of heart failure in many patients, and patients should be taught that some increase in symptoms, such as fatigue and edema, is expected during the initiation of therapy with this class of drugs

A patient who was admitted with diabetic ketoacidosis secondary to a urinary tract infection has been weaned off an insulin drip 30 minutes ago. The patient reports feeling lightheaded and sweaty. Which action should the nurse take first? a. Infuse dextrose 50% by slow IV push. b. Administer 1 mg glucagon subcutaneously. c. Obtain a glucose reading using a finger stick. d. Have the patient drink 4 ounces of orange juice.

ANS: C The patient's clinical manifestations are consistent with hypoglycemia and the initial action should be to check the patient's glucose with a finger stick or order a stat blood glucose. If the glucose is low, the patient should ingest a rapid-acting carbohydrate, such as orange juice. Glucagon or dextrose 50% might be given if the patient's symptoms become worse or if the patient is unconscious

After change-of-shift report, which patient will the nurse assess first? a. 19-year-old with type 1 diabetes who was admitted with possible dawn phenomenon b. 35-year-old with type 1 diabetes whose most recent blood glucose reading was 230 mg/dL c. 60-year-old with hyperosmolar hyperglycemic syndrome who has poor skin turgor and dry oral mucosa d. 68-year-old with type 2 diabetes who has severe peripheral neuropathy and complains of burning foot pain

ANS: C The patient's diagnosis of HHS and signs of dehydration indicate that the nurse should rapidly assess for signs of shock and determine whether increased fluid infusion is needed. The other patients also need assessment and intervention but do not have life-threatening complications

A 48-year-old male patient screened for diabetes at a clinic has a fasting plasma glucose level of 120 mg/dL (6.7 mmol/L). The nurse will plan to teach the patient about a. self-monitoring of blood glucose. b. using low doses of regular insulin. c. lifestyle changes to lower blood glucose. d. effects of oral hypoglycemic medications.

ANS: C The patient's impaired fasting glucose indicates prediabetes, and the patient should be counseled about lifestyle changes to prevent the development of type 2 diabetes. The patient with prediabetes does not require insulin or oral hypoglycemics for glucose control and does not need to self-monitor blood glucose

A patient develops carpopedal spasms and tingling of the lips following a parathyroidectomy. Which action should the nurse take first? a. Administer the ordered muscle relaxant. b. Give the ordered oral calcium supplement. c. Have the patient rebreathe from a paper bag. d. Start the PRN oxygen at 2 L/min per cannula.

ANS: C The patient's symptoms suggest mild hypocalcemia. The symptoms of hypocalcemia will be temporarily reduced by having the patient breathe into a paper bag, which will raise the PaCO2 and create a more acidic pH. The muscle relaxant will have no impact on the ionized calcium level. Although severe hypocalcemia can cause laryngeal stridor, there is no indication that this patient is experiencing laryngeal stridor or needs oxygen. Calcium supplements will be given to normalize calcium levels quickly, but oral supplements will take time to be absorbed

Which assessment finding for a 33-year-old female patient admitted with Graves' disease requires the most rapid intervention by the nurse? a. Bilateral exophthalmos b. Heart rate 136 beats/minute c. Temperature 103.8° F (40.4° C) d. Blood pressure 166/100 mm Hg

ANS: C The patient's temperature indicates that the patient may have thyrotoxic crisis and that interventions to lower the temperature are needed immediately. The other findings also require intervention but do not indicate potentially life-threatening complications

A 72-year-old male patient with dehydration caused by an exacerbation of ulcerative colitis is receiving 5% dextrose in normal saline at 125 mL/hour. Which assessment finding by the nurse is most important to report to the health care provider? a. Patient has not voided for the last 4 hours. b. Skin is dry with poor turgor on all extremities. c. Crackles are heard halfway up the posterior chest. d. Patient has had 5 loose stools over the last 6 hours.

ANS: C The presence of crackles in an older patient receiving IV fluids at a high rate suggests volume overload and a need to reduce the rate of the IV infusion. The other data will also be reported, but are consistent with the patient's age and diagnosis and do not require a change in the prescribed treatment.

A 62-year-old patient with hyperthyroidism is to be treated with radioactive iodine (RAI). The nurse instructs the patient a. about radioactive precautions to take with all body secretions. b. that symptoms of hyperthyroidism should be relieved in about a week. c. that symptoms of hypothyroidism may occur as the RAI therapy takes effect. d. to discontinue the antithyroid medications taken before the radioactive therapy.

ANS: C There is a high incidence of postradiation hypothyroidism after RAI, and the patient should be monitored for symptoms of hypothyroidism. RAI has a delayed response, with the maximum effect not seen for 2 to 3 months, and the patient will continue to take antithyroid medications during this time. The therapeutic dose of radioactive iodine is low enough that no radiation safety precautions are needed

A patient with a history of chronic heart failure is admitted to the emergency department (ED) with severe dyspnea and a dry, hacking cough. Which action should the nurse do first? a. Auscultate the abdomen. b. Check the capillary refill. c. Auscultate the breath sounds. d. Assess the level of orientation.

ANS: C This patient's severe dyspnea and cough indicate that acute decompensated heart failure (ADHF) is occurring. ADHF usually manifests as pulmonary edema, which should be detected and treated immediately to prevent ongoing hypoxemia and cardiac/respiratory arrest. The other assessments will provide useful data about the patient's volume status and also should be accomplished rapidly, but detection (and treatment) of pulmonary complications is the priority.

A 42-year-old female patient is scheduled for transsphenoidal hypophysectomy to treat a pituitary adenoma. During preoperative teaching, the nurse instructs the patient about the need to a. cough and deep breathe every 2 hours postoperatively. b. remain on bed rest for the first 48 hours after the surgery. c. avoid brushing teeth for at least 10 days after the surgery. d. be positioned flat with sandbags at the head postoperatively.

ANS: C To avoid disruption of the suture line, the patient should avoid brushing the teeth for 10 days after surgery. It is not necessary to remain on bed rest after this surgery. Coughing is discouraged because it may cause leakage of cerebrospinal fluid (CSF) from the suture line. The head of the bed should be elevated 30 degrees to reduce pressure on the sella turcica and decrease the risk for headaches

Which information is most important for the nurse to report to the health care provider before a patient with type 2 diabetes is prepared for a coronary angiogram? a. The patient's most recent HbA1C was 6.5%. b. The patient's admission blood glucose is 128 mg/dL. c. The patient took the prescribed metformin (Glucophage) today. d. The patient took the prescribed captopril (Capoten) this morning.

ANS: C To avoid lactic acidosis, metformin should be discontinued a day or 2 before the coronary arteriogram and should not be used for 48 hours after IV contrast media are administered. The other patient data will also be reported but do not indicate any need to reschedule the procedure

Which care activity for a patient with a paralytic ileus is appropriate for the registered nurse (RN) to delegate to unlicensed assistive personnel (UAP)? a. Auscultation for bowel sounds b. Nasogastric (NG) tube irrigation c. Applying petroleum jelly to the lips d. Assessment of the nares for irritation

ANS: C UAP education and scope of practice include patient hygiene such as oral care. The other actions require education and scope of practice appropriate to the RN.

The nurse is assessing a 22-year-old patient experiencing the onset of symptoms of type 1 diabetes. Which question is most appropriate for the nurse to ask? a. "Are you anorexic?" b. "Is your urine dark colored?" c. "Have you lost weight lately?" d. "Do you crave sugary drinks?"

ANS: C Weight loss occurs because the body is no longer able to absorb glucose and starts to break down protein and fat for energy. The patient is thirsty but does not necessarily crave sugar-containing fluids. Increased appetite is a classic symptom of type 1 diabetes. With the classic symptom of polyuria, urine will be very dilute

A 22-year-old female patient with an exacerbation of ulcerative colitis is having 15 to 20 stools daily and has excoriated perianal skin. Which patient behavior indicates that teaching regarding maintenance of skin integrity has been effective? a. The patient uses incontinence briefs to contain loose stools. b. The patient asks for antidiarrheal medication after each stool. c. The patient uses witch hazel compresses to decrease irritation. d. The patient cleans the perianal area with soap after each stool.

ANS: C Witch hazel compresses are suggested to reduce anal irritation and discomfort. Incontinence briefs may trap diarrhea and increase the incidence of skin breakdown. Antidiarrheal medications are not given 15 to 20 times a day. The perianal area should be washed with plain water after each stool.

An 82-year-old patient in a long-term care facility has several medications prescribed. After the patient is newly diagnosed with hypothyroidism, the nurse will need to consult with the health care provider before administering a. docusate (Colace). b. ibuprofen (Motrin). c. diazepam (Valium). d. cefoxitin (Mefoxin).

ANS: C Worsening of mental status and myxedema coma can be precipitated by the use of sedatives, especially in older adults. The nurse should discuss the use of diazepam with the health care provider before administration. The other medications may be given safely to the patient

A 64-year-old woman who has chronic constipation asks the nurse about the use of psyllium (Metamucil). Which information will the nurse include in the response? a. Absorption of fat-soluble vitamins may be reduced by fiber-containing laxatives. b. Dietary sources of fiber should be eliminated to prevent excessive gas formation. c. Use of this type of laxative to prevent constipation does not cause adverse effects. d. Large amounts of fluid should be taken to prevent impaction or bowel obstruction.

ANS: D A high fluid intake is needed when patients are using bulk-forming laxatives to avoid worsening constipation. Although bulk-forming laxatives are generally safe, the nurse should emphasize the possibility of constipation or obstipation if inadequate fluid intake occurs. Although increased gas formation is likely to occur with increased dietary fiber, the patient should gradually increase dietary fiber and eventually may not need the psyllium. Fat-soluble vitamin absorption is blocked by stool softeners and lubricants, not by bulk-forming laxatives.

Which information is most important for the nurse to communicate rapidly to the health care provider about a patient admitted with possible syndrome of inappropriate antidiuretic hormone (SIADH)? a. The patient has a recent weight gain of 9 lb. b. The patient complains of dyspnea with activity. c. The patient has a urine specific gravity of 1.025. d. The patient has a serum sodium level of 118 mEq/L.

ANS: D A serum sodium of less than 120 mEq/L increases the risk for complications such as seizures and needs rapid correction. The other data are not unusual for a patient with SIADH and do not indicate the need for rapid action

A 23-year-old patient is admitted with diabetes insipidus. Which action will be most appropriate for the registered nurse (RN) to delegate to an experienced licensed practical/vocational nurse (LPN/LVN)? a. Titrate the infusion of 5% dextrose in water. b. Teach the patient how to use desmopressin (DDAVP) nasal spray. c. Assess the patient's hydration status every 8 hours. d. Administer subcutaneous DDAVP.

ANS: D Administration of medications is included in LPN/LVN education and scope of practice. Assessments, patient teaching, and titrating fluid infusions are more complex skills and should be done by the RN.

Which breakfast choice indicates a patient's good understanding of information about a diet for celiac disease? a. Oatmeal with nonfat milk b. Whole wheat toast with butter c. Bagel with low-fat cream cheese d. Corn tortilla with scrambled eggs

ANS: D Avoidance of gluten-containing foods is the only treatment for celiac disease. Corn does not contain gluten, while oatmeal and wheat do.

The nurse is caring for a patient who is receiving IV furosemide (Lasix) and morphine for the treatment of acute decompensated heart failure (ADHF) with severe orthopnea. Which clinical finding is the best indicator that the treatment has been effective? a. Weight loss of 2 pounds in 24 hours b. Hourly urine output greater than 60 mL c. Reduction in patient complaints of chest pain d. Reduced dyspnea with the head of bed at 30 degrees

ANS: D Because the patient's major clinical manifestation of ADHF is orthopnea (caused by the presence of fluid in the alveoli), the best indicator that the medications are effective is a decrease in dyspnea with the head of the bed at 30 degrees. The other assessment data also may indicate that diuresis or improvement in cardiac output has occurred, but are not as specific to evaluating this patient's response.

A hospitalized diabetic patient received 38 U of NPH insulin at 7:00 AM. At 1:00 PM, the patient has been away from the nursing unit for 2 hours, missing the lunch delivery while awaiting a chest x-ray. To prevent hypoglycemia, the best action by the nurse is to a. save the lunch tray for the patient's later return to the unit. b. ask that diagnostic testing area staff to start a 5% dextrose IV. c. send a glass of milk or orange juice to the patient in the diagnostic testing area. d. request that if testing is further delayed, the patient be returned to the unit to eat.

ANS: D Consistency for mealtimes assists with regulation of blood glucose, so the best option is for the patient to have lunch at the usual time. Waiting to eat until after the procedure is likely to cause hypoglycemia. Administration of an IV solution is unnecessarily invasive for the patient. A glass of milk or juice will keep the patient from becoming hypoglycemic but will cause a rapid rise in blood glucose because of the rapid absorption of the simple carbohydrate in these items

The nurse will plan to teach a patient with Crohn's disease who has megaloblastic anemia about the need for a. oral ferrous sulfate tablets. b. regular blood transfusions. c. iron dextran (Imferon) infusions. d. cobalamin (B12) spray or injections.

ANS: D Crohn's disease frequently affects the ileum, where absorption of cobalamin occurs. Cobalamin must be administered regularly by nasal spray or IM to correct the anemia. Iron deficiency does not cause megaloblastic anemia. The patient may need occasional transfusions but not regularly scheduled transfusions.

Which action by a patient indicates that the home health nurse's teaching about glargine and regular insulin has been successful? a. The patient administers the glargine 30 minutes before each meal. b. The patient's family prefills the syringes with the mix of insulins weekly. c. The patient draws up the regular insulin and then the glargine in the same syringe. d. The patient disposes of the open vials of glargine and regular insulin after 4 weeks.

ANS: D Insulin can be stored at room temperature for 4 weeks. Glargine should not be mixed with other insulins or prefilled and stored. Short-acting regular insulin is administered before meals, while glargine is given once daily.

When teaching the patient with newly diagnosed heart failure about a 2000-mg sodium diet, the nurse explains that foods to be restricted include a. canned and frozen fruits. b. fresh or frozen vegetables. c. eggs and other high-protein foods. d. milk, yogurt, and other milk products.

ANS: D Milk and yogurt naturally contain a significant amount of sodium, and intake of these should be limited for patients on a diet that limits sodium to 2000 mg daily. Other milk products, such as processed cheeses, have very high levels of sodium and are not appropriate for a 2000-mg sodium diet. The other foods listed have minimal levels of sodium and can be eaten without restriction.

Which question from the nurse would help determine if a patient's abdominal pain might indicate irritable bowel syndrome? a. "Have you been passing a lot of gas?" b. "What foods affect your bowel patterns?" c. "Do you have any abdominal distention?" d. "How long have you had abdominal pain?"

ANS: D One criterion for the diagnosis of irritable bowel syndrome (IBS) is the presence of abdominal discomfort or pain for at least 3 months. Abdominal distention, flatulence, and food intolerance are also associated with IBS, but are not diagnostic criteria.

The nurse is interviewing a new patient with diabetes who receives rosiglitazone (Avandia) through a restricted access medication program. What is most important for the nurse to report immediately to the health care provider? a. The patient's blood pressure is 154/92. b. The patient has a history of emphysema. c. The patient's blood glucose is 86 mg/dL. d. The patient has chest pressure when walking.

ANS: D Rosiglitazone can cause myocardial ischemia. The nurse should immediately notify the health care provider and expect orders to discontinue the medication. There is no urgent need to discuss the other data with the health care provider

A patient with type 2 diabetes is scheduled for a follow-up visit in the clinic several months from now. Which test will the nurse schedule to evaluate the effectiveness of treatment for the patient? a. Urine dipstick for glucose b. Oral glucose tolerance test c. Fasting blood glucose level d. Glycosylated hemoglobin level

ANS: D The glycosylated hemoglobin (A1C or HbA1C) test shows the overall control of glucose over 90 to 120 days. A fasting blood level indicates only the glucose level at one time. Urine glucose testing is not an accurate reflection of blood glucose level and does not reflect the glucose over a prolonged time. Oral glucose tolerance testing is done to diagnose diabetes, but is not used for monitoring glucose control once diabetes has been diagnosed

A 27-year-old female patient is admitted to the hospital for evaluation of right lower quadrant abdominal pain with nausea and vomiting. Which action should the nurse take? a. Encourage the patient to sip clear liquids. b. Assess the abdomen for rebound tenderness. c. Assist the patient to cough and deep breathe. d. Apply an ice pack to the right lower quadrant.

ANS: D The patient's clinical manifestations are consistent with appendicitis, and application of an ice pack will decrease inflammation at the area. Checking for rebound tenderness frequently is unnecessary and uncomfortable for the patient. The patient should be NPO in case immediate surgery is needed. The patient will need to know how to cough and deep breathe postoperatively, but coughing will increase pain at this time.

After several days of antibiotic therapy, an older hospitalized patient develops watery diarrhea. Which action should the nurse take first? a. Notify the health care provider. b. Obtain a stool specimen for analysis. c. Teach the patient about handwashing. d. Place the patient on contact precautions.

ANS: D The patient's history and new onset diarrhea suggest a C. difficile infection, which requires implementation of contact precautions to prevent spread of the infection to other patients. The other actions are also appropriate but can be accomplished after contact precautions are implemented.

A 45-year-old patient is admitted to the emergency department with severe abdominal pain and rebound tenderness. Vital signs include temperature 102° F (38.3° C), pulse 120, respirations 32, and blood pressure (BP) 82/54. Which prescribed intervention should the nurse implement first? a. Administer IV ketorolac (Toradol) 15 mg. b. Draw blood for a complete blood count (CBC). c. Obtain a computed tomography (CT) scan of the abdomen. d. Infuse 1 liter of lactated Ringer's solution over 30 minutes.

ANS: D The priority for this patient is to treat the patient's hypovolemic shock with fluid infusion. The other actions should be implemented after starting the fluid infusion.

A 47-year-old female patient is transferred from the recovery room to a surgical unit after a transverse colostomy. The nurse observes the stoma to be deep pink with edema and a small amount of sanguineous drainage. The nurse should a. place ice packs around the stoma. b. notify the surgeon about the stoma. c. monitor the stoma every 30 minutes. d. document stoma assessment findings.

ANS: D The stoma appearance indicates good circulation to the stoma. There is no indication that surgical intervention is needed or that frequent stoma monitoring is required. Swelling of the stoma is normal for 2 to 3 weeks after surgery, and an ice pack is not needed.

Surgery, radiation, drug therapy, prevent complications involving other organs. Surgery is the most common form of treatment, after it is removed, the patient needs GH therapy for the rest of their life. If it's removed in women, it can cause infertility. Treat it with sandstatin, helps to reduce the GH. It is a subcutaneous injection that the patient gets a couple times a week.

Acromegaly treatment:

Adrenocortical insufficiency (hypo function of the adrenal cortex) may be from this primary cause or a secondary cause (lack of pituitary ACTH secretion). In this disease, all three classes of adrenal corticosteroids are reduced.

Addison's disease

S: sodium & sugar is low, crave salt T: tired E: electrolyte imbalance R: reproductive issues O: Low blood pressure I: Skin pigmentation D: Diarrhea/ Depression

Addison's disease S/S

The manifestations have a slow (insidious) onset and include anorexia, nausea, progressive weakness, fatigue, weight loss, low BP, salt craving, yellow tan tint to skin, hyperkalemia

Addison's disease: What are the defining characteristics

Decreased cortisol and aldosterone

Addisons

Nursing suggestions for a patient with acute or chronic rhinosinusitis include: a. adequate fluid intake. b. increased humidity. c. local heat applications to promote drainage. d. all of the above.

All of the above

When providing discharge teaching for the patient after a laparoscopic cholecystectomy, what information should the nurse include? A) A lower-fat diet may be better tolerated for several weeks. B) Do not return to work or normal activities for 3 weeks. C) Bile-colored drainage will probably drain from the incision. D) Keep the bandages on and the puncture site dry until it heals.

Although the usual diet can be resumed, a low-fat diet is usually better tolerated for several weeks following surgery. Normal activities can be gradually resumed as the patient tolerates. Bile-colored drainage or pus, redness, swelling, severe pain, and fever may all indicate infection. The bandage may be removed the day after surgery, and the patient can shower.

A 30-year-old man is being admitted to the hospital for elective knee surgery. Which assessment finding is most important to report to the health care provider? a. Tympany on percussion of the abdomen b. Liver edge 3 cm below the costal margin c. Bowel sounds of 20/minute in each quadrant d. Aortic pulsations visible in the epigastric area

B

A 62- year-old man reports chronic constipation. To promote bowel evacuation, the nurse will suggest that the patient attempt defecation a. in the mid-afternoon. b. after eating breakfast. c. right after getting up in the morning. d. immediately before the first daily meal.

B

A 73-yr-old man with dementia has a venous ulcer related to chronic venous insufficiency. The nurse should provide teaching on which type of diet for this patient and his caregiver? A. Low-fat diet B. High-protein diet C. Calorie-restricted diet D. High-carbohydrate diet

B A patient with a venous ulcer should have a balanced diet with adequate protein, calories, and micronutrients; this type of diet is essential for healing. Nutrients most important for healing include protein, vitamins A and C, and zinc. Foods high in protein (e.g., meat, beans, cheese, tofu), vitamin A (green leafy vegetables), vitamin C (citrus fruits, tomatoes, cantaloupe), and zinc (meat, seafood) must be provided. Restricting fat or calories is not helpful for wound healing or in patients of normal weight. For overweight individuals with no active venous ulcer, a weight-loss diet should be considered.

The nurse is reviewing the laboratory test results for a 68-yr-old patient whose warfarin (Coumadin) therapy was terminated during the preoperative period. On postoperative day 2, the international normalized ratio (INR) result is 2.7. Which action by the nurse is most appropriate? A. Hold the daily dose of warfarin. B. Administer the daily dose of warfarin. C. Teach the patient signs and symptoms of bleeding. D. Call the physician to request an increased dose of warfarin.

B The therapeutic range for INR is 2.0 to 3.0 for many clinical diagnoses. To maintain therapeutic values, the nurse will administer the medication as ordered. Holding the medication would lower the INR, which would increase the risk of clot formation. Conversely, the higher the INR is, the more prolonged the clotting time. Calling the health care provider is not indicated. Although teaching is important, administering the medication is a higher priority at this time.

2) The nurse is preparing to administer digoxin to a patient with heart failure. In preparation, laboratory results are reviewed with the following findings: sodium 139 mEq/L, potassium 5.6 mEq/L, chloride 103 mEq/L, and glucose 106 mg/dL. What is the priority action by the nurse? A) Withhold the daily dose until the following day. B) Withhold the dose and report the potassium level. Correct C) Give the digoxin with a salty snack, such as crackers. D)Give the digoxin with extra fluids to dilute the sodium level.

B The normal potassium level is 3.5 to 5.0 mEq/L. The patient is hyperkalemic, which makes the patient more prone to digoxin toxicity. For this reason, the nurse should withhold the dose and wait for the potassium level to normalize. The physician may order the digoxin to be given once the potassium level has been treated and decreases to within normal range.

Physiological Integrity 24. A patient whose heart monitor shows sinus tachycardia, rate 132, is apneic and has no palpable pulses. What is the first action that the nurse should take? a. Perform synchronized cardioversion. b. Start cardiopulmonary resuscitation (CPR). c. Administer atropine per agency dysrhythmia protocol. d. Provide supplemental oxygen via non-rebreather mask.

B The patient's clinical manifestations indicate pulseless electrical activity and the nurse should immediately start CPR. The other actions would not be of benefit to this patient. DIF: Cognitive Level: Apply (application) REF: 800 OBJ: Special Questions: Prioritization TOP: Nursing Process: Implementation MSC:

7. The nurse is preparing to administer a nitroglycerin patch to a patient. When providing instructions regarding the use of the patch, what should the nurse include in the teaching? A. Avoid high-potassium foods B. Avoid drugs to treat erectile dysfunction C. Avoid over-the-counter H2-receptor blockers D. Avoid nonsteroidal antiinflammatory drugs (NSAIDS)

B The use of erectile drugs concurrent with nitrates creates a risk of severe hypotension and possibly death. High-potassium foods, NSAIDs, and H2-receptor blockers do not pose a risk in combination with nitrates.

Safe and Effective Care Environment 13. Which action should the nurse perform when preparing a patient with supraventricular tachycardia for cardioversion who is alert and has a blood pressure of 110/66 mm Hg? a. Turn the synchronizer switch to the "off" position. b. Give a sedative before cardioversion is implemented. c. Set the defibrillator/cardioverter energy to 360 joules. d. Provide assisted ventilations with a bag-valve-mask device.

B When a patient has a nonemergency cardioversion, sedation is used just before the procedure. The synchronizer switch is turned "on" for cardioversion. The initial level of joules for cardioversion is low (e.g., 50). Assisted ventilations are not indicated for this patient. DIF: Cognitive Level: Apply (application) REF: 802 TOP: Nursing Process: Implementation MSC:

Which assessment findings of the left lower extremity will the nurse identify as consistent with arterial occlusion (select all that apply.)? A. Edematous B. Cold and mottled C. Complaints of paresthesia D. Pulse not palpable with Doppler E. Capillary refill less than three seconds F. Erythema and warmer than right lower extremity

B C D Arterial occlusion may result in loss of limb if not timely revascularized. When an artery is occluded, perfusion to the extremity is impaired or absent. On assessment, the nurse would note a cold, mottled extremity with impaired sensation or numbness. The pulse would not be identified, even with a Doppler. In contrast, the nurse would find edema, erythema, and increased warmth in the presence of a venous occlusion (deep vein thrombosis). Capillary refill would be greater than 3 seconds in an arterial occlusion and less than 3 seconds with a venous occlusion.

The nurse would assess a patient with complaints of chest pain for which clinical manifestations associated with a myocardial infarction (MI) (select all that apply.)? A. Flushing B. Ashen skin C. Diaphoresis D. Nausea and vomiting E. S3 or S4 heart sounds

B C D E During the initial phase of an MI, catecholamines are released from the ischemic myocardial cells, causing increased sympathetic nervous system stimulation. This results in the release of glycogen, diaphoresis, and vasoconstriction of peripheral blood vessels. The patient's skin may be ashen, cool, and clammy (not flushed) as a result of this response. Nausea and vomiting may result from reflex stimulation of the vomiting center by severe pain. Ventricular dysfunction resulting from the MI may lead to the presence of the abnormal S3 and S4 heart sounds.

The nurse is caring for a patient treated with intravenous fluid therapy for severe vomiting. As the pt recovers and begins to tolerate oral intake, the N understands that which of the following food choices would be most appropriate? A) Ice tea B) Dry toast C) Warm broth D) Plain hamburger

B) Dry toast (Dry toast or crackers may alleviate the feeling of nausea and prevent further vomiting. Extremely hot or cold liquids and fatty foods are generally not well tolerated.)

The patient with sudden pain in the left upper quadrant radiating to the back and vomiting was diagnosed with acute pancreatitis. What intervention(s) should the nurse expect to include in the patient's plan of care? A) Immediately start enteral feeding to prevent malnutrition. B) Insert an NG and maintain NPO status to allow pancreas to rest. C) Initiate early prophylactic antibiotic therapy to prevent infection. D) Administer acetaminophen (Tylenol) every 4 hours for pain relief.

B) Initial treatment with acute pancreatitis will include an NG tube if there is vomiting and being NPO to decrease pancreatic enzyme stimulation and allow the pancreas to rest and heal. Fluid will be administered to treat or prevent shock. The pain will be treated with IV morphine because of the NPO status. Enteral feedings will only be used for the patient with severe acute pancreatitis in whom oral intake is not resumed. Antibiotic therapy is only needed with acute necrotizing pancreatitis and signs of infection.

A colectomy is scheduled for a 38-year-old woman with ulcerative colitis. The nurse should plan to include what prescribed measure in the preoperative preparation of this patient? A) Instruction on irrigating a colostomy B) Administration of a cleansing enema C) A high-fiber diet the day before surgery D) Administration of IV antibiotics for bowel preparation

B) Preoperative preparation for bowel surgery typically includes bowel cleansing with antibiotics, such as oral neomycin and cleansing enemas, including Fleet enemas. Instructions to irrigate the colostomy will be done postoperatively. Oral antibiotics are given preoperatively, and an IV antibiotic may be used in the OR. A clear liquid diet will be used the day before surgery with the bowel cleansing.

The nurse prepares a discharge teaching plan for a 44-yr-old male patient who has recently been diagnosed with coronary artery disease (CAD). Which risk factor should the nurse plan to focus on during the teaching session? A. Type A personality B. Elevated serum lipids C. Family cardiac history D. Hyperhomocysteinemia

B. Dyslipidemia is one of the four major modifiable risk factors for CAD. The other major modifiable risk factors are hypertension, tobacco use, and physical inactivity. Research findings related to psychologic states (i.e., type A personality) as a risk factor for coronary artery disease have been inconsistent. Family history is a nonmodifiable risk factor. High homocysteine levels have been linked to an increased risk for CAD.

A nurse is caring for a patient with a diagnosis of deep venous thrombosis (DVT). The patient has an order to receive 30 mg enoxaparin (Lovenox). Which injection site should the nurse use to administer this medication safely? A. Buttock, upper outer quadrant B. Abdomen, anterior-lateral aspect C. Back of the arm, 2 inches away from a mole D. Anterolateral thigh, with no scar tissue nearby

B. Enoxaparin (Lovenox) is a low-molecular-weight (LMW) heparin that is given as a deep subcutaneous injection in the right and left anterolateral abdomen. All subcutaneous injections should be given away from scars, lesions, or moles.

Which assessment finding would alert the nurse that a postoperative patient is not receiving the beneficial effects of enoxaparin (Lovenox)? A. Crackles bilaterally in the lung bases B. Pain and swelling in a lower extremity C. Absence of arterial pulse in a lower extremity D. Abdominal pain with decreased bowel sounds

B. Enoxaparin is a low-molecular-weight heparin used to prevent the development of deep vein thromboses (DVTs) in the postoperative period. Pain and swelling in a lower extremity can indicate development of DVT and therefore may signal ineffective medication therapy.

A 67-yr-old man with peripheral artery disease is seen in the primary care clinic. Which symptom reported by the patient would indicate to the nurse that the patient is experiencing intermittent claudication? A. Patient complains of chest pain with strenuous activity. B. Patient says muscle leg pain occurs with continued exercise. C. Patient has numbness and tingling of all his toes and both feet. D. Patient states the feet become red if he puts them in a dependent position.

B. Intermittent claudication is an ischemic muscle ache or pain that is precipitated by a consistent level of exercise, resolves within 10 minutes or less with rest, and is reproducible. Angina is the term used to describe chest pain with exertion. Paresthesia is the term used to describe numbness or tingling in the toes or feet. Reactive hyperemia is the term used to describe redness of the foot; if the limb is in a dependent position the term is dependent rubor.

Which patient is at greatest risk for sudden cardiac death (SCD)? A. A 42-yr-old white woman with hypertension and dyslipidemia B. A 52-yr-old African American man with left ventricular failure C. A 62-yr-old obese man with diabetes mellitus and high cholesterol D. A 72-yr-old Native American woman with a family history of heart disease

B. Patients with left ventricular dysfunction (ejection fraction < 30%) and ventricular dysrhythmias after MI are at greatest risk for sudden cardiac death (SCD). Other risk factors for SCD include (1) male gender (especially African American men), (2) family history of premature atherosclerosis, (3) tobacco use, (4) diabetes mellitus, (5) hypercholesterolemia, (6) hypertension, and (7) cardiomyopathy.

The nurse is caring for a newly admitted patient with vascular insufficiency. The patient has a new order for enoxaparin (Lovenox) 30 mg subcutaneously. What should the nurse do to correctly administer this medication? A. Spread the skin before inserting the needle. B. Leave the air bubble in the prefilled syringe. C. Use the back of the arm as the preferred site. D. Sit the patient at a 30-degree angle before administration.

B. The nurse should not expel the air bubble from the prefilled syringe because it should be injected to clear the needle of medication and avoid leaving medication in the needle track in the tissue.

The nurse is examining the electrocardiogram (ECG) of a patient just admitted with a suspected MI. Which ECG change is most indicative of prolonged or complete coronary occlusion? A. Sinus tachycardia B. Pathologic Q wave C. Fibrillatory P waves D. Prolonged PR interval

B. The presence of a pathologic Q wave, as often accompanies STEMI, is indicative of complete coronary occlusion. Sinus tachycardia, fibrillatory P waves (e.g., atrial fibrillation), or a prolonged PR interval (first-degree heart block) are not direct indicators of extensive occlusion.

The nurse is providing teaching to a patient recovering from a myocardial infarction. How should resumption of sexual activity be discussed? A. Delegated to the primary care provider B. Discussed along with other physical activities C. Avoided because it is embarrassing to the patient D. Accomplished by providing the patient with written material

B. Although some nurses may not feel comfortable discussing sexual activity with patients, it is a necessary component of patient teaching. It is helpful to consider sex as a physical activity and to discuss or explore feelings in this area when other physical activities are discussed. Although providing the patient with written material is appropriate, it should not replace a verbal dialogue that can address the individual patient's questions and concerns.

The nurse admits a 73-yr-old male patient with dementia for treatment of uncontrolled hypertension. The nurse will closely monitor for hypokalemia if the patient receives which medication? A. Clonidine (Catapres) B. Bumetanide (Bumex) C.Amiloride (Midamor) D. Spironolactone (Aldactone)

B. Bumetanide is a loop diuretic. Hypokalemia is a common adverse effect of this medication. Amiloride is a potassium-sparing diuretic. Spironolactone is an aldosterone-receptor blocker. Hyperkalemia is an adverse effect of both amiloride and spironolactone. Clonidine is a central-acting α-adrenergic antagonist and does not cause electrolyte abnormalities.

The home care nurse visits a patient with chronic heart failure. Which clinical manifestations, assessed by the nurse, would indicate acute decompensated heart failure (pulmonary edema)? A. Fatigue, orthopnea, and dependent edema B. Severe dyspnea and blood-streaked, frothy sputum Correct C. Temperature is 100.4oF and pulse is 102 beats/min D. Respirations 26 breaths/min despite oxygen by nasal cannula

B. Clinical manifestations of pulmonary edema include anxiety, pallor, cyanosis, clammy and cold skin, severe dyspnea, use of accessory muscles of respiration, a respiratory rate greater than 30 breaths/min, orthopnea, wheezing, and coughing with the production of frothy, blood-tinged sputum. Auscultation of the lungs may reveal crackles, wheezes, and rhonchi throughout the lungs. The heart rate is rapid, and blood pressure may be elevated or decreased.

The nurse is caring for a patient admitted with a history of hypertension. The patient's medication history includes hydrochlorothiazide daily for the past 10 years. Which parameter would indicate the optimal intended effect of this drug therapy? A. Weight loss of 2 lb B. BP 128/86 mm Hg Correct C. Absence of ankle edema D. Output of 600 mL per 8 hours

B. Hydrochlorothiazide may be used alone as monotherapy to manage hypertension or in combination with other medications if not effective alone. After the first few weeks of therapy, the diuretic effect diminishes, but the antihypertensive effect remains. Because the patient has been taking this medication for 10 years, the most direct measurement of its intended effect would be the blood pressure.

When providing dietary instruction to a patient with hypertension, the nurse would advise the patient to restrict intake of which meat? A. Broiled fish B. Roasted duck Correct C. Roasted turkey D. Baked chicken breast

B. Roasted duck is high in fat, which should be avoided by the patient with hypertension. Weight loss may slow the progress of atherosclerosis and overall cardiovascular disease risk. The other meats are lower in fat and are therefore acceptable in the diet.

The nurse supervises an unlicensed assistant personnel (UAP) who is taking the blood pressure of 58-yr-old obese female patient admitted with heart failure. Which action by the UAP will require the nurse to intervene? A. Waiting 2 minutes after position changes to take orthostatic pressures B. Deflating the blood pressure cuff at a rate of 8 to 10 mm Hg per second C. Taking the blood pressure with the patient's arm at the level of the heart D. Taking a forearm blood pressure because the largest cuff will not fit the patient's upper arm

B. The cuff should be deflated at a rate of 2 to 3 mm Hg per second. The arm should be supported at the level of the heart for accurate blood pressure measurements. Using a cuff that is too small causes a falsely high reading and too large causes a falsely low reading. If the maximum size blood pressure cuff does not fit the upper arm, the forearm may be used. Orthostatic blood pressures should be taken within 1 to 2 minutes of repositioning the patient.

In caring for a patient admitted with poorly controlled hypertension, which laboratory test result should the nurse understand as indicating the presence of target organ damage? A. Serum uric acid of 3.8 mg/dL B. Serum creatinine of 2.6 mg/dL C. Serum potassium of 3.5 mEq/L D. Blood urea nitrogen of 15 mg/dL

B. The normal serum creatinine level is 0.6 to 1.3 mg/dL. This elevated level indicates target organ damage to the kidneys. The other laboratory results are within normal limits.

When teaching a patient about dietary management of stage 1 hypertension, which instruction is most appropriate? A. Increase water intake. B. Restrict sodium intake. C. Increase protein intake. D. Use calcium supplements.

B. The patient should decrease intake of sodium. This will help to control hypertension, which can be aggravated by excessive salt intake, which in turn leads to fluid retention. Protein intake does not affect hypertension. Calcium supplements are not recommended to lower blood pressure.

After teaching a patient with chronic stable angina about nitroglycerin, the nurse recognizes the need for further teaching when the patient makes which statement? A. "I will replace my nitroglycerin supply every 6 months." B. "I can take up to five tablets every 3 minutes for relief of my chest pain." C. "I will take acetaminophen (Tylenol) to treat the headache caused by nitroglycerin." D. "I will take the nitroglycerin 10 minutes before planned activity that usually causes chest pain."

B. The recommended dose of nitroglycerin is one tablet taken sublingually (SL) or one metered spray for symptoms of angina. If symptoms are unchanged or worse after 5 minutes, the patient should be instructed to activate the emergency medical services (EMS) system. If symptoms are improved, repeat the nitroglycerin every 5 minutes for a maximum of three doses and contact EMS if symptoms have not resolved completely.

Your client has a history of being in jail for 3 months and was recently released. He was diagnosed with latent T.B. from exposure while living in a hostel in India 10 years prior. He presents to the ED with bloody sputum, strong and productive cough, fever, and night sweats. As a RN who studied respiratory problems, what are you worried about? A. He has developed respiratory failure B. He now has active T.B. C. He has severe pleurisy D. He has a pulmonary embolism

B. He now has active T.B.

GI manifestations include a sore, red, swollen, shiny, and beefy tongue, anorexia, N/V, and abdominal pain. Neuro manifestations include weakness, paresthesias of the hands and feet, impaired thought processes ranging from confusion to dementia

B12 deficiency anemia (pernicious anemia): Clinical manifestations

A 54-year-old man has just arrived in the recovery area after an upper endoscopy. Which information collected by the nurse is most important to communicate to the health care provider? a. The patient is very drowsy. b. The patient reports a sore throat. c. The oral temperature is 101.6° F. d. The apical pulse is 104 beats/minute.

C

The nurse teaches a 28-yr-old man newly diagnosed with hypertension about lifestyle modifications to reduce his blood pressure. Which patient statement requires reinforcement of teaching? A. "I will avoid adding salt to my food during or after cooking." B."If I lose weight, I might not need to continue taking medications." C. "I can lower my blood pressure by switching to smokeless tobacco." D. "Diet changes can be as effective as taking blood pressure medications.

C Nicotine contained in tobacco products (smoking and chew) cause vasoconstriction and increase blood pressure. Persons with hypertension should restrict sodium to 1500 mg/day by avoiding foods high in sodium and not adding salt in preparation of food or at meals. Weight loss can decrease blood pressure between 5 to 20 mm Hg. Following dietary recommendations (e.g., the DASH diet) lowers blood pressure, and these decreases compare with those achieved with blood pressure-lowering medication.

Physiological Integrity 10. Which information will the nurse include when teaching a patient who is scheduled for a radiofrequency catheter ablation for treatment of atrial flutter? a. The procedure will prevent or minimize the risk for sudden cardiac death. b. The procedure will use cold therapy to stop the formation of the flutter waves. c. The procedure will use electrical energy to destroy areas of the conduction system. d. The procedure will stimulate the growth of new conduction pathways between the atria.

C Radiofrequency catheter ablation therapy uses electrical energy to "burn" or ablate areas of the conduction system as definitive treatment of atrial flutter (i.e., restore normal sinus rhythm) and tachydysrhythmias. All other statements regarding the procedure are incorrect. DIF: Cognitive Level: Apply (application) REF: 805 TOP: Nursing Process: Implementation MSC:

Safe and Effective Care Environment 28. A patient reports dizziness and shortness of breath for several days. During cardiac monitoring in the emergency department (ED), the nurse obtains the following electrocardiographic (ECG) tracing. The nurse interprets this heart rhythm as a. junctional escape rhythm. b. accelerated idioventricular rhythm. c. third-degree atrioventricular (AV) block. d. sinus rhythm with premature atrial contractions (PACs).

C The inconsistency between the atrial and ventricular rates and the variable P-R interval indicate that the rhythm is third-degree AV block. Sinus rhythm with PACs will have a normal rate and consistent P-R intervals with occasional PACs. An accelerated idioventricular rhythm will not have visible P waves. DIF: Cognitive Level: Apply (application) REF: 798 TOP: Nursing Process: Assessment MSC:

Physiological Integrity 29. A patient who is complaining of a "racing" heart and feeling "anxious" comes to the emergency department. The nurse places the patient on a heart monitor and obtains the following electrocardiographic (ECG) tracing. Which action should the nurse take next? a. Prepare to perform electrical cardioversion. b. Have the patient perform the Valsalva maneuver. c. Obtain the patient's vital signs including oxygen saturation. d. Prepare to give a -blocker medication to slow the heart rate.

C The patient has sinus tachycardia, which may have multiple etiologies such as pain, dehydration, anxiety, and myocardial ischemia. Further assessment is needed before determining the treatment. Vagal stimulation or -blockade may be used after further assessment of the patient. Electrical cardioversion is used for some tachydysrhythmias, but would not be used for sinus tachycardia. DIF: Cognitive Level: Analyze (analysis) REF: 793 OBJ: Special Questions: Prioritization TOP: Nursing Process: Implementation MSC:

Physiological Integrity 26. Which nursing action can the registered nurse (RN) delegate to experienced unlicensed assistive personnel (UAP) working as a telemetry technician on the cardiac care unit? a. Decide whether a patient's heart rate of 116 requires urgent treatment. b. Monitor a patient's level of consciousness during synchronized cardioversion. c. Observe cardiac rhythms for multiple patients who have telemetry monitoring. d. Select the best lead for monitoring a patient admitted with acute coronary syndrome.

C UAP serving as telemetry technicians can monitor cardiac rhythms for individuals or groups of patients. Nursing actions such as assessment and choice of the most appropriate lead based on ST segment elevation location require RN-level education and scope of practice. DIF: Cognitive Level: Analyze (analysis) REF: 15 OBJ: Special Questions: Delegation TOP: Nursing Process: Planning MSC:

Which antilipemic medications should the nurse question for a patient with cirrhosis of the liver (select all that apply.)? A. Niacin B. Cholestyramine C. Ezetimibe (Zetia) D. Gemfibrozil (Lopid) E. Atorvastatin (Lipitor)

C D E Ezetimibe (Zetia) should not be used by patients with liver impairment. Adverse effects of atorvastatin (Lipitor), a statin drug, include liver damage and myopathy. Liver enzymes must be monitored frequently and the medication stopped if these enzymes increase. Niacin's side effects subside with time, although decreased liver function may occur with high doses. Cholestyramine is safe for long-term use.

The nurse is teaching a women's group about prevention of hypertension. What information should be included in the teaching for all the women (select all that apply.)? A. Lose weight. B. Limit nuts and seeds. C. Limit sodium and fat intake. D. Increase fruits and vegetables. E. Exercise 30 minutes most days.

C D E Primary prevention of hypertension is to make lifestyle modifications that prevent or delay the increase in BP. Along with exercise for 30 minutes on most days, the DASH eating plan is a healthy way to lower BP by limiting sodium and fat intake, increasing fruits and vegetables, and increasing nutrients that are associated with lowering BP. Nuts and seeds and dried beans are used for protein intake. Weight loss may or may not be necessary for the individual.

The health care team is assessing a male patient for acute pancreatitis after he presented to the emergency department with severe abdominal pain. Which laboratory value is the best diagnostic indicator of acute pancreatitis? A) Gastric pH B) Blood glucose C) Serum amylase D) Serum potassium

C) Elevated serum amylase levels indicate early pancreatic dysfunction and are used to diagnose acute pancreatitis. Serum lipase levels stay elevated longer than serum amylase in acute pancreatitis. Blood glucose, gastric pH, and potassium levels are not direct indicators of acute pancreatic dysfunction.

The nurse determines that a pnt has experienced the beneficial effects of medication therapy with famotidine (Pepcid) when which of the following symptoms is relieved? A) Nausea B) Belching C) Epigastric pain D) Difficulty swallowing

C) Epigastric pain Famotidine is an H2-receptor antagonist that inhibits parietal cell output of HCl acid and minimizes damage to gastric mucosa related to hyperacidity, thus relieving epigastric pain.

The patient is having an esophagoenterostomy with anastomosis of a segment of the colon to replace the resected portion. What initial postoperative care should the nurse expect when this patient returns to the nursing unit? A) Turn, deep breathe, cough, and use spirometer every 4 hours. B) Maintain an upright position for at least 2 hours after eating. C) NG will have bloody drainage, and it should not be repositioned. D) Keep in a supine position to prevent movement of the anastomosis.

C) The patient will have bloody drainage from the NG tube for 8 to 12 hours, and it should not be repositioned or reinserted without contacting the surgeon. Turning and deep breathing will be done every 2 hours, and the spirometer will be used more often than every 4 hours. Coughing would put too much pressure in the area and should not be done. Because the patient will have the NG tube, the patient will not be eating yet. The patient should be kept in a semi-Fowler's or Fowler's position, not supine, to prevent reflux and aspiration of secretions.

A 54-year-old patient admitted with diabetes mellitus, malnutrition, osteomyelitis, and alcohol abuse has a serum amylase level of 280 U/L and a serum lipase level of 310 U/L. To which of the following diagnoses does the nurse attribute these findings? A. Malnutrition B. Osteomyelitis C. Alcohol abuse D. Diabetes mellitus

C) The patient with alcohol abuse could develop pancreatitis as a complication, which would increase the serum amylase (normal 30-122 U/L) and serum lipase (normal 31-186 U/L) levels as shown.

Following bowel resection, a patient has a nasogastric (NG) tube to suction, but complains of nausea and abdominal distention. The nurse irrigates the tube as necessary as ordered, but the irrigating fluid does not return. What should be the priority action by the nurse? A) Notify the physician. B) Auscultate for bowel sounds. C) Reposition the tube and check for placement. D) Remove the tube and replace it with a new one.

C) The tube may be resting against the stomach wall. The first action by the nurse (since this is intestinal surgery and not gastric surgery) is to reposition the tube and check it again for placement. The physician does not need to be notified unless the tube function cannot be restored by the nurse. The patient does not have bowel sounds, which is why the NG tube is in place. The NG tube would not be removed and replaced unless it was no longer in the stomach or the obstruction of the tube could not be relieved.

The patient with chronic gastritis is being put on a combination of medications to eradicate H. pylori. Which drugs does the nurse know will probably be used for this patient? A) Antibiotic(s), antacid, and corticosteroid B) Antibiotic(s), aspirin, and antiulcer/protectant C) Antibiotic(s), proton pump inhibitor, and bismuth D) Antibiotic(s) and nonsteroidal antiinflammatory drugs (NSAIDs)

C) To eradicate H. pylori, a combination of antibiotics, a proton pump inhibitor, and possibly bismuth (for quadruple therapy) will be used. Corticosteroids, aspirin, and NSAIDs are drugs that can cause gastritis and do not affect H. pylori.

A 74-yr-old man with a history of prostate cancer and hypertension is admitted to the emergency department with substernal chest pain. Which action will the nurse complete before administering sublingual nitroglycerin? A. Administer morphine sulfate IV. B. Auscultate heart and lung sounds. C. Obtain a 12-lead electrocardiogram (ECG). Correct D. Assess for coronary artery disease risk factors.

C. If a patient has chest pain, the nurse should institute the following measures: (1) administer supplemental oxygen and position the patient in upright position unless contraindicated, (2) assess vital signs, (3) obtain a 12-lead ECG, (4) provide prompt pain relief first with a nitrate followed by an opioid analgesic if needed, and (5) auscultate heart sounds. Obtaining a 12-lead ECG during chest pain aids in the diagnosis.

When teaching how lisinopril (Zestril) will help lower the patient's blood pressure, which mechanism of action should the nurse explain? A. Blocks β-adrenergic effects B. Relaxes arterial and venous smooth muscle C. Inhibits conversion of angiotensin I to angiotensin II D. Reduces sympathetic outflow from central nervous system

C. Lisinopril is an angiotensin-converting enzyme inhibitor that inhibits the conversion of angiotensin I to angiotensin II, which reduces angiotensin II-mediated vasoconstriction and sodium and water retention. β blockers result in vasodilation and decreased heart rate. Direct vasodilators relax arterial and venous smooth muscle. Central acting α-adrenergic antagonists reduce sympathetic outflow from the central nervous system to produce vasodilation and decreased systemic vascular resistance and blood pressure.

A 32-yr-old woman is prescribed diltiazem (Cardizem) for Raynaud's phenomenon. To evaluate the effectiveness of the medication, which assessment will the nurse perform? A. Improved skin turgor B. Decreased cardiac rate C. Improved finger perfusion D. Decreased mean arterial pressure

C. Raynaud's phenomenon is an episodic vasospastic disorder of small cutaneous arteries, most frequently involving the fingers and toes. Diltiazem (Cardizem) is a calcium channel blocker that relaxes smooth muscles of the arterioles by blocking the influx of calcium into the cells, thus reducing the frequency and severity of vasospastic attacks. Perfusion to the fingertips is improved, and vasospastic attacks are reduced. Diltiazem may decrease heart rate and blood pressure, but that is not the purpose in Raynaud's phenomenon. Skin turgor is most often a reflection of hydration status.

The nurse prepares to administer digoxin 0.125 mg to a patient admitted with influenza and a history of chronic heart failure. What should the nurse assess before giving the medication? A. Prothrombin time B. Urine specific gravity C. Serum potassium level Correct D. Hemoglobin and hematocrit

C. Serum potassium should be monitored because hypokalemia increases the risk for digoxin toxicity. Changes in prothrombin time, urine specific gravity, and hemoglobin or hematocrit would not require holding the digoxin dose.

The nurse assesses the right femoral artery puncture site as soon as the patient arrives after having a stent inserted into a coronary artery. The insertion site is not bleeding or discolored. What should the nurse do next to ensure the femoral artery is intact? A. Palpate the insertion site for induration. B. Assess peripheral pulses in the right leg. C. Inspect the patient's right side and back. D. Compare the color of the left and right legs.

C. The best method to determine that the right femoral artery is intact after inspection of the insertion site is to logroll the patient to inspect the right side and back for retroperitoneal bleeding. The artery can be leaking and blood is drawn into the tissues by gravity. The peripheral pulses, color, and sensation of the right leg will be assessed per agency protocol.

In caring for the patient with angina, the patient said, "While I was having a bowel movement, I started having the worst chest pain ever, like before I was admitted. I called for a nurse, then the pain went away." What further assessment data should the nurse obtain from the patient? A. "What precipitated the pain?" B. "Has the pain changed this time?" C. "In what areas did you feel this pain?" D. "What is your pain level on a 0 to 10 scale?"

C. Using PQRST, the assessment data not volunteered by the patient is the radiation of pain, the area the patient felt the pain, and if it radiated. The precipitating event was going to the bathroom and having a bowel movement. The quality of the pain was "like before I was admitted," although a more specific description may be helpful. Severity of the pain was the "worst chest pain ever," although an actual number may be needed. Timing is supplied by the patient describing when the pain occurred and that he had previously had this pain.

3. What is the priority assessment by the nurse caring for a patient receiving IV nesiritide to treat heart failure? A. Urine output B. Lung sounds C. Blood pressure Correct D. Respiratory rate

C. Although all identified assessments are appropriate for a patient receiving IV nesiritide, the priority assessment would be monitoring for hypotension, the main adverse effect of nesiritide.

The nurse is caring for a patient admitted with chronic obstructive pulmonary disease (COPD), angina, and hypertension. Before administering the prescribed daily dose of atenolol 100 mg PO, the nurse assesses the patient carefully. Which adverse effect is this patient at risk for given the patient's health history? A. Hypocapnia B. Tachycardia C. Bronchospasm D. Nausea and vomiting

C. Atenolol is a cardioselective β1-adrenergic blocker that reduces blood pressure and could affect the β2-receptors in the lungs with larger doses or with drug accumulation. Although the risk of bronchospasm is less with cardioselective β-blockers than nonselective β-blockers, atenolol should be used cautiously in patients with COPD.

A patient with possible disseminated intravascular coagulation (DIC) arrives in the emergency department with a blood pressure of 82/40, temperature of 102° F (38.9° C), and severe back pain. Which prescribed action will the nurse implement first? A. Give acetaminophen (Tylenol) 650 mg. B. Administer morphine sulfate 4 mg IV. C. Infuse normal saline 500 mL over 30 minutes. D. Schedule complete blood count and coagulation studies.

C. Infuse normal saline 500 mL over 30 minutes. The patient's blood pressure indicates hypovolemia caused by blood loss and should be addressed immediately to improve perfusion to vital organs. The other actions are also appropriate and should be rapidly implemented, but improving perfusion is the priority for this patient.

. Streptococcus pneumoniae is the most common organism responsible for which of the following types of pneumonia? a. Hospital-acquired b. Immunocompromised c. Aspiration-specific d. Community-acquired

Community required

Sickled RBCs become rigid and take on an elongated, crescent shape. Sickled cells cannot easily pass through capillaries or other small vessels and can cause this episode, leading to acute or chronic tissue injury.

Complications from a sickling episode

With repeated episodes of sickling, there is gradual involvement of all body systems, especially the spleen, lungs, kidneys, and brain. Organs that have a need for large amounts of oxygen are most often affected and form the basis for many complications. Infection (pneumonia) is a major cause of morbidity and mortality in patients with this. Acute chest syndrome is a term used to describe acute pulmonary complications that is characterized by fever, chest pain, cough, pulmonary infiltrates and dyspnea. The heart may become ischemic and enlarged, leading to HF. Pain: acute abdominal hand/foot syndrome Splenomegaly: congested with blood, infarction Hemolytic crisis: anemia, jaundice Stroke: Cerebral infarction Kidney- Ischemia

Complications of sickle cell anemia

Complications from the sliding hernia

Complications: GERD, esophagitis, hemorrhage from erosion, stenosis (narrowing of the esophagus), ulcers of the herniated portion of the stomach, strangulation of the hernia, and regurgitation with tracheal aspiration

DI intervention:

Conserve water, adjust diet, manage medications. Maintaining fluid balance, giving Desmopressin (DDAVP)

The nurse instructs an obese 22-year-old man with a sedentary job about the health benefits of an exercise program. The nurse evaluates that teaching is effective when the patient makes which statement? a. "The goal is to walk at least 10,000 steps every day of the week." b. "Weekend aerobics for 2 hours is better than exercising every day." c. "Aerobic exercise will increase my appetite and result in weight gain." d. "Exercise causes weight loss by decreasing my resting metabolic rate."

Correct answer: a Rationale: A realistic activity goal is to walk 10,000 steps a day. Increased activity does not promote an increase in appetite or lead to weight gain. Exercise should be done daily, preferably 30 minutes to an hour a day. Exercise increases metabolic rate.

What problem should the nurse assess the patient for if the patient was on prolonged antibiotic therapy? a. Coagulation problems b. Elevated serum ammonia levels c. Impaired absorption of amino acids d. Increased mucus and bicarbonate secretion

Correct answer: a Rationale: Bacteria int he colon (1) synthesize vitamin K, which is needed for the production of prothrombin by the liver and (2) deaminate undigested or non absorbed proteins, producing ammonia, which is converted to urea by the liver. A reduction in normal flora bacteria by antibiotic therapy can lead to decreased vitamin K, resulting in decreased prothrombin and coagulation problems. Bowel bacteria do not influence protein absorption or the secretion of mucus.

Priority Decision: The nurse is caring for a patient receiving 1000 mL of parenteral nutrition solution over 24 hours. When it is time to change the solution, 150 mL remain in the bottle. What is the most appropriate action by the nurse? a. Hang the new solution and discard the unused solution. b. Open the IV line and rapidly infuse the remaining solution. c. Notify the health care provider for instructions regarding the infusion rate. d. Wait to change the solution until the remaining solution infuses at the proscribed rate.

Correct answer: a Rationale: Bacterial growth occurs at room temperature in nutritional solutions. Therefore solutions must not be infused for longer than 24 hours. Remaining solution should be discard. Speeding up the solution may cause hyperglycemia and should not be done. The health care provider does not need to be notified.

A patient is jaundiced and her stools are clay colored (gray). This is most likely related to a. decreased bile flow into the intestine. b. increase production of urobilinogen. c. increased production of cholecystokinin. d. increased bile and bilirubin in the blood.

Correct answer: a Rationale: Bile is produced by the hepatocytes and is stored and concentrated in the gallbladder. When bile is released from the common bile duct, it enters the duodenum. In the intestines, bilirubin is reduced to stercobilinogen and urobilinogen by bacterial action. Stercobilinogen accounts for the brown color of stool. Stools may be clay-colored if bile is not released from the common bile duct into the duodenum. Jaundice may result if the bilirubin level in the blood is elevated.

What characterizes auscultation of the abdomen? a. The presence of borborygmi indicates hyper peristalsis. b. The bell of the stethoscope is used to auscultate high-pitched sounds. c. High-pitched, rushing, and tinkling bowel sounds are heard after eating. d. Absence of bowel sounds for 1 minute in each quadrant is reported as abnormal.

Correct answer: a Rationale: Borborygmi are loud gurgles (stomach growling) that indicate hyper peristalsis. Normal bowel sounds are relatively high-pitched and are heard best with the diaphragm of the stethoscope. High-pitched, tinkling bowel sounds occur when the intestines are under tension, as in bowel obstructions. Absent bowel sounds may be reported when no sounds are heard for 2 to 3 minutes in each quadrant.

What information should be included in the dietary teaching for the patient following a Roux-en-Y gastric bypass? a. Avoid sugary foods and limit fluids to prevent dumping syndrome. b. Gradually increase the amount of food ingested to preoperative levels. c. Maintain a long-term liquid diet to prevent damage to the surgical site. d. Consume foods high in complex carbohydrates, protein, and fiber to add bulk to contents.

Correct answer: a Rationale: Fluids and foods high in carbohydrates tend to promote diarrhea and symptoms of dumping syndrome in patients with gastric bypass surgery. The diet generally should be high in protein and low in carbohydrates, fat, and roughage and consists of six small feedings a day because of the small stomach size. Liquid diets are likely to be used longer for the patient with a gastroplasty.

A patient had a stomach resection for stomach cancer. The nurse should teach the patient about the loss of the hormone that stimulates gastric acid secretion and motility and maintains lower esophageal sphincter tone. Which hormone will be decreased with a gastric resection? a. Gastrin b. Secretin c. Cholecystokinin d. Gastric inhibitory peptide

Correct answer: a Rationale: Gastrin is the hormone activated in the stomach (and duodenal mucosa) by stomach distention that stimulates gastric acid secretion and motility and maintains lower esophageal sphincter tone. Secretin inhibits gastric motility and acid secretion and stimulates pancreatic bicarbonate secretion. Cholecystokinin allows increased flow of bile into the duodenum and release of pancreatic digestive enzymes. Gastric inhibitory peptide inhibits gastric acid secretion and motility.

The nurse is caring for a patient who is 5'6" tall and weighs 186 lb. The nurse has discussed reasonable weight loss goals and a low-calorie diet with the patient. Which statement made by the patient indicates a need for further teaching? a. "I will limit intake to 500 calories a day." b. "I will try to eat very slowly during mealtimes." c. "I'll try to pick foods from all of the basic food groups." d. "It's important for me to begin a regular exercise program."

Correct answer: a Rationale: Limiting intake to 500 calories per day is not indicated for this patient, and the severe calorie energy restriction would place this patient at risk for multiple nutrient deficiencies. Decreasing caloric intake at least 500 to 1000 calories a day is recommended for weight loss of one to two pounds per week. The other options show understanding of the teaching.

Priority Decision: Before selecting a weight reduction plan with an obese patient, what is most important for the nurse to first assess? a. The patient's motivation to lose weight b. The length of time that the patient has been obese c. Whether financial considerations will affect the patient's choices d. The patient's anthropometric measures of height, weight, BMI, waist-to-hip ratio, and skinfold thickness

Correct answer: a Rationale: Motivation is essential. Focus on the reasons for wanting to lose weight. The rest of the options will asset in planning the weight loss if the patient is motivated.

A 50-year-old African American woman has a BMI of 35 kg/m2, type 2 diabetes mellitus, hypercholesterolemia, and irritable bowel syndrome (IBS). She is seeking assistance in losing weight, because, "I have trouble stopping eating when I should, but I do not want to have bariatric surgery." Which drug therapy should the nurse question if it is prescribed for this patient? a. Orlistat (Xenical) b. Locaserin (Belviq) c. Phentermine (Adipex-P) d. Phentermine and topiramate (Qsymia)

Correct answer: a Rationale: Orlistat (Xenical), which blocks fat breakdown and absorption in the intestine, produces some unpleasant GI side effects. This drug would not be appropriate for someone with IBS. Locaserin (Belviq) suppresses the appetite and creates a sense of satiety that may be helpful for this patient. Phentermine (Adipex-P) needs to be used for a limited period of time (3 months or less). Qsymia is a combination of two drugs, phentermine and topiramate. Phentermine is a sympathomimetic agent that suppresses appetite and topiramate induces a sense of satiety.

A patient has been on a 1000-calorie diet with a daily exercise routine. In 2 months, the patient has lost 20 lb (9kg) toward a goal of 50 lb (23 kg) but is now discouraged that no weight has been lost in the last 2 weeks. What should the nurse tell the patient about this? a. Plateaus where no weight is lost normally occur during a weight-loss program. b. A weight considered by the body to most efficient for functioning has been reached. c. A return to former eating habits is the most common cause of not continuing to lose weight. d. A steady weight may be due to water gain from eating foods high in sodium.

Correct answer: a Rationale: Plateau periods during which no weight is lost are normal occurrences during weight reduction and may last for several days to several weeks but weight loss will resume if the prescribed weight reduction plan is continued. Weight loss may stop if former eating habits are resumed but this not the most common cause of plateaus.

A patient asks the nurse about taking phentermine and topiramate (Qsymia) for weight loss. To avoid side effects, it is important for the nurse to determine whether the patient has a history of a. glaucoma. b. hypertension. c. valvular heart disease. d. irritable bowel disease.

Correct answer: a Rationale: Qsymia is a combination of phentermine and topiramate. It must not be used in patients with glaucoma or hyperthyroidism.

When planning nutritional interventions for a healthy 83-year-old man, the nurse recognizes what factor is most likely to affect his nutritional status? a. Living alone on a fixed income b. Changes in cardiovascular function c. An increase in GI motility and absorption d. Snacking between meal, resulting in obesity

Correct answer: a Rationale: Socioeconomic conditions frequently have the greatest effect on the nutritional status of the healthy older adult. Limited income and social isolation can result in the "tea and toast" meals of the older adult. The other options do not interfere with nutritional status.

When considering tube feedings for a patient with severe protein-calorie malnutrition, what is an advantage of a gastrostomy tube versus a nasogastric (NG) tube? a. There is less irritation to the nasal and esophageal mucosa. b. The patient experiences the sights and smells associated with eating. c. Aspiration resulting from reflux of formulas into the esophagus is less common. d. Routine checking for placement is not required because gastrostomy tubes do not become displaced

Correct answer: a Rationale: Standard nasogastric (NG) tubes are used for tube feedings for short-term feeding problems because prolonged therapy can result in irritation and erosion of the mucosa of the upper GI tract. Gastric reflux and the potential for aspiration can occur with both tubes that deliver fluids into the stomach. Both NG and gastrostomy tubes can become displaced and deprive the patient of the sensations associated with eating.

The percentage of daily calories for a healthy individual consists of a. 50% carbohydrates, 25% protein, 25% fat, and <10% of fat from saturated fatty acids. b. 65% carbohydrates, 25% protein, 25% fat, and >10% of fat from saturated fatty acids. c. 50% carbohydrates, 40% protein, 10% fat, and <10% of fat from saturated fatty acids. d. 40% carbohydrates, 30% protein, 30% fat, and >10% of fat from saturated fatty acids.

Correct answer: a Rationale: The 2005 Dietary Guidelines for Americans recommend that 45% to 65% of total calories should come from carbohydrates. Ideally, 10% to 35% of daily caloric needs should come from protein. Individuals should limit their fat intake to 20% to 35% of total calories. Additional recommendations focus on the type of fat consumed because diets high in excess calories, usually in the form of fats, contribute to the development of obesity. Individuals should consume less than 10% of calories from saturated fatty acids, limit intake of fat and oils high in trans fatty acids, and should limit intake of dietary cholesterol to 300 mg/day.

Priority Decision: Following auscultation of the abdomen, what should the nurse's next action be? a. Lightly percuss over all four quadrants b. Have the patient empty his or her bladder c. Inspect perianal and anal areas for color, masses, rashes, and scars d. Perform deep palpation to delineate abdominal organs and masses

Correct answer: a Rationale: The abdomen should be assessed in the following sequence: inspection, auscultation, percussion, palpation. The patient should empty his or her bladder before assessment begins.

During the initial postoperative period following bariatric surgery, the nurse recognizes the importance of monitoring obese patients for respiratory insufficiency based on what knowledge? a. The body stores anesthetics in adipose tissue. b. Postoperative pain may cause a decreased respiratory rate. c. Intubation may be difficult because of extra chin skinfolds. d. The patient's head must remain flat for a minimum of 2 hours postprocedure.

Correct answer: a Rationale: The body stores anesthetics in adipose tissue, placing patients with excess adipose tissue at risk for re-sedation. As adipose cells release anesthetics back into the bloodstream, the patient may become sedated after surgery, increasing the risk of hypoventilation and resultant respiratory insufficiency. Difficult intubation does not cause respiratory insufficiency. Pain usually increases respiratory rate. The patient's head should be elevated after bariatric surgery to decrease abdominal pressure and facilitate respirations.

The nurse evaluates that patient teaching about a high-calorie, high-protien diet has been effective when the patient selects which breakfast option from the hospital menu? a. Two poached eggs, hash brown potatoes, and whole milk b. Two slices of toast with butter and jelly, orange juice, and skim milk c. Three pancakes with butter and syrup, two slices of bacon, and apple juice d. Cream of wheat with 2 tbsp of skim milk powder, one half grapefruit, and a high-protein milkshake

Correct answer: a Rationale: The breakfast with the eggs provides 24 g or protein, compared with 14 g for the protein-fortified cream of wheat and milkshake breakfast. Whole milk instead of skim milk helps to meet the calorie requirements. The toast has 10 g of protein and the pancakes have about 6 g. Bacon is considered a fat rather than a meat serving.

The nurse cares for a 34-year-old woman after bariatric surgery. The nurse determines that discharge teaching related to diet is successful if the patient makes which statement? a. "A high protein diet that is low in carbohydrates and fat will prevent diarrhea." b. "Food should be high in fiber to prevent constipation from the pain medication." c. "Three meals a day with no snacks between meals will provide optimal nutrition." d. "Fluid intake should be at least 2000 mL per day with meals to avoid dehydration."

Correct answer: a Rationale: The diet generally prescribed is high in protein and low in carbohydrates, fat, and roughage and consists of six small feedings daily. Fluids should not be ingested with the meal, and in some cases, fluids should be restricted to less than 1000 mL per day. Fluids and foods high in carbohydrate tend to promote diarrhea and symptoms of the dumping syndrome. Generally, calorically dense foods (foods high in fat) should be avoided to permit more nutritionally sound food to be consumed.

Checking for the return of the gag reflex and monitoring for LUQ pain, nausea and vomiting are necessary nursing actions after which diagnostic procedure? a. ERCP b. Colonoscopy c. Barium swallow d. Esophagogastroduodenoscopy (EGD)

Correct answer: a Rationale: The left upper quadrant (LUQ) pain and nausea and vomiting could occur from perforation. The return of gag reflex is essential to prevent aspiration after an ERCP. The gag reflex is also assessed with an EGD. These are not relevant assessments for the colonoscopy and barium swallow.

The nurse confirms initial placement of a blindly inserted small-bore NG feeding tube by a. x-ray. b. air insufflation. c. observing patient for coughing. d. pH measurement of gastric aspirate.

Correct answer: a Rationale: The nurse should obtain x-ray confirmation to determine whether a blindly placed nasogastric or orogastric tube (small bore or large bore) is properly positioned in the gastrointestinal tract before administering feedings or medications.

A 35-year-old man with a family history of adenomatous polyposis had a colonoscopy with removal of multiple polyps. Which signs and symptoms should the nurse teach the patient to report immediately? a. Fever and abdominal pain b. Flatulence and liquid stool c. Loudly audible bowel sounds d. Sleepiness and abdominal cramps

Correct answer: a Rationale: The patient should be taught to observe for signs of rectal bleeding and peritonitis. Fever, malaise, and abdominal pain and distention could indicate a perforated bowel with peritonitis.

The nurse teaches a 50-year-old woman who has a body mass index (BMI) of 39 kg/m2 about weight loss. Which dietary change would be appropriate for the nurse to recommend to this patient? a. Decrease fat intake and control portion size b. Increase vegetables and decrease fluid intake c. Increase protein intake and avoid carbohydrates d. Decrease complex carbohydrates and limit fiber

Correct answer: a Rationale: The safest dietary guideline for weight loss is to decrease caloric intake by maintaining a balance of nutrients and adequate hydration while controlling portion size and decreasing fat intake.

What contributes to increased protein-calorie needs? a. Surgery b. Vegan diet c. Lowered temperature d. Cultural or religious beliefs

Correct answer: a Rationale: With surgery a patient will recover more rapidly with a balanced nutritional status before the surgery and increased protein is needed for healing after the surgery. Following a vegan diet does not put the patient at risk of low protein intake. A lowered temperature will not cause increased protein need. Following religious and cultural beliefs would not be expected to affect an increased need for protein.

A 90-year-old healthy man is suffering from dysphagia. The nurse explains what age-related change of the GI tract is the most likely cause of his difficulty? a. Xerostomia b. Esophageal cancer c. Decreased taste buds d. Thinner abdominal wall

Correct answer: a Rationale: Xerostomia, decreased volume of saliva, leads to dry oral mucosa and dysphagia. Esophageal cancer is not an age-related change. Decreased taste buds and a thinner abdominal wall do not contribute to difficulty swallowing.

During assessment of the patient with protein-calorie malnutrition, what should the nurse expect to find (select all that apply)? a. Frequent cold symptoms b. Decreased bowel sounds c. Cool, rough, dry, scaly skin d. A flat or concave abdomen e. Prominent bony structures f. Decreased reflexes and lack of attention

Correct answer: a, b, c, e, f Rationale: In malnutrition, metabolic processes are slowed, leading to increased sensitivity to cold, decreased heart rate (HR) and cardiac output (CO), and decreased neurologic function. Because of slowed GI motility and absorption, the abdomen becomes distended and protruding and bowel sounds are decreased. Skin is rough, dry, and scaly whereas bone structures protrude because of muscle loss. Because the immune system is weakened, susceptibility to respiratory infections is increased.

The stable patient has a gastrostomy tube for enteral feeding. Which care could the RN delegate to the LPN (select all that apply)? a. Administer bolus or continuous feedings. b. Evaluate the nutritional status of the patient. c. Administer medications through the gastrostomy tube. d. Monitor for complications related to the tube and enteral feeding. e. Teach the caregiver about feeding via the gastrostomy tube at home.

Correct answer: a, c Rationale: For the stable patient, the LPN can administer bolus or continuous feedings and administer medications through the gastrostomy. The RN must evaluate the nutritional status of the patient, monitor for complications related to the tube and the enteral feeding, and teach the caregiver about feeding via the gastrostomy tube at home.

The nurse is caring for a 45-year-old woman with a herniated lumbar disc. The patient realizes that weight loss is necessary to lessen back strain. The patient is 5'6" tall and weighs 186 lb (84.5 kg) with a body mass index (BMI) of 28 kg/m2. The nurse explains to the patient that this measurement places her in which of the following weight categories? a. Normal weight b. Overweight c. Obese d. Severely obese

Correct answer: b Rationale: A normal BMI is 18.5 to 24.9 kg/m2, whereas a BMI of 25 to 29.9 kg/m2 is considered overweight. A BMI of 30.0-39.9 is considered obese, and a BMI of 40 or greater is severely obese.

A woman is 5 ft, 6 in (166 cm) tall and weighs 200 lb (90.9 kg) with a waist-to-hip ratio of 0.7. The nurse counsels the patient with the knowledge that the patient is at greatest risk for a. heart disease. b. osteoporosis. c. diabetes mellitus. d. endometrial cancer.

Correct answer: b Rationale: A patient who is obese (BMI of 32.2) but has a waist-to-hip ratio of less than 0.8, indicating gynoid obesity, has an increased risk for osteoporosis. The other conditions are risks associated with android obesity.

A 22-year-old female is admitted with anorexia nervosa and a serum potassium level of 2.4 mEq/L. What complication is most important for the nurse to observe for in this patient? a. Muscle weakness b. Cardiac dysrhythmias c. Increased urine output d. Anemia and leukopenia

Correct answer: b Rationale: A serum potassium level less than 2.5 mEq/L indicates severe hypokalemia, which can lead to life-threatening cardiac dysrhythmias (e.g., bradycardia, tachycardia, ventricular dysrhythmias). Other manifestations of potassium deficiency include muscle weakness and renal failure. Patients with anorexia nervosa commonly have iron-deficiency anemia and an elevated blood urea nitrogen level related to intravascular volume depletion and abnormal renal function.

When teaching the older adult about nutritional needs during aging, what does the nurse emphasize? a. Need for all nutrients decreases as one ages. b. Fewer calories, but the same or slightly increased amount of protein, are required as one ages. c. Fats, carbohydrates, and protein should be decreased, but vitamin and mineral intake should be increased. d. High-calorie oral supplements should be taken between meals to ensure that recommended nutrient needs are met.

Correct answer: b Rationale: Although calorie intake should be decreased in the older adult because of decreased activity and basal metabolic rate, the need for specific nutrients, such as proteins and vitamins, does not change.

A patient has an elevated blood level of indirect (unconjugated) bilirubin. One cause of this finding is that a. the gallbladder is unable to contract to release stored bile. b. bilirubin is not being conjugated and excreted into the bile by the liver. c. the Kupffer cells in the liver are unable to remove bilirubin from the blood. d. there is an obstruction in the biliary tract preventing flow of bile into the small intestine.

Correct answer: b Rationale: Bilirubin is a pigment derived from the breakdown of hemoglobin and is insoluble in water. Bilirubin is bound to albumin for transport to the liver and is referred to as unconjugated. An indirect bilirubin determination is a measurement of unconjugated bilirubin, and the level may be elevated in hepatocellular and hemolytic conditions.

A patient who has suffered severe burns in a motor vehicle accident will soon be started on parenteral nutrition (PN). Which principle should guide the nurse's administration of the patient's nutrition? a. Administration of PN requires clean technique. b. Central PN requires rapid dilution in a large volume of blood. c. Peripheral PN delivery is preferred over the use of a central line. d. Only water-soluble medications may be added to the PN by the nurse.

Correct answer: b Rationale: Central PN is hypertonic and requires rapid dilution in a large volume of blood. Because PN is an excellent medium for microbial growth, aseptic technique is necessary during administration. Administration through a central line is preferred over the use of peripheral PN, and the nurse may not add any medications to PN.

A patient is receiving peripheral parenteral nutrition. The parenteral nutrition solution is completed before the new solution arrives on the unit. The nurse administers a. 20% intralipids. b. 5% dextrose solution. c. 0.45% normal saline solution. d. 5% lactated Ringer's solution

Correct answer: b Rationale: If a peripheral parenteral nutrition (PPN) formula bag empties before the next solution is available, a 5% dextrose solution (based on the amount of dextrose in the peripheral PN solution) may be administered to prevent hypoglycemia.

During starvation, the order in which the body obtains substrate for energy is a. visceral protein, skeletal protein, fat, glycogen b. glycogen, skeletal protein, fat stores, visceral protein c. visceral protein, fat stores, glycogen, skeletal protein d. fat stores, skeletal protein, visceral protein, glycogen

Correct answer: b Rationale: Initially, the body selectively uses carbohydrates (e.g., glycogen) rather than fat and protein to meet metabolic needs. These carbohydrate stores, found in the liver and muscles, are minimal and may be totally depleted within 18 hours. After carbohydrate stores are depleted, skeletal protein begins to be converted to glucose for energy. Within 5 to 9 days, body fat is fully mobilized to supply much of the needed energy. In prolonged starvation, up to 97% of calories are provided by fat, and protein is conserved. Depletion of fat stores depends on the amount available, but fat stores typically are used up in 4 to 6 weeks. After fat stores are used, body or visceral proteins, including those in internal organs and plasma, can no longer be spared and rapidly decrease because they are the only remaining body source of energy available.

Priority Decision: When medications are used in the treatment of obesity, what is most important for the nurse to teach the patient? a. Over-the-counter (OTC) diet aids are safer than other agents and con be useful in controlling appetite. b. Drugs should be used only as adjuncts to a diet and exercise program as treatment for a chronic condition. c. All drugs used for weight control are capable of altering central nervous system (CNS) function and should be used with caution. d. The primary effect of the medications is psychologic, controlling the urge to eat in response to stress or feelings of rejection.

Correct answer: b Rationale: Medications are used only as adjuncts to diet and exercise programs in the treatment of obesity. Drugs do not cure obesity; without changes in food intake and physical activity, weight gain will occur when the medications are discontinued. The medications used work in a variety of ways to control appetite but over-the-counter drugs are probably the least effective and most abused of these drugs.

A frail 74-year-old man with recent severe weight loss is instructed to eat a high-protein, high-calorie diet at home. If the man likes all of the items below, which would be the most appropriate for the nurse to suggest? a. Orange juice and dry toast b. Oatmeal, butter, and cream c. Steamed carrots and chicken broth d. Banana and unsweetened applesauce

Correct answer: b Rationale: Oatmeal, butter, and cream are examples of food items that would be appropriate to include for a patient on a high-protein, high-calorie diet.

An 80-year-old man states that, although be adds a lot of salt to his food, it still does not have much taste. The nurse's response is based on the knowledge that the older adult a. should not experience changes in taste. b. has a loss of taste buds, especially for sweet and salty. c. has some loss of taste but no difficulty chewing food. d. loses the sense of taste because the ability to smell is decreased.

Correct answer: b Rationale: Older adults have decreased numbers of taste buds and a decreased sense of smell. These age-related changes diminish the sense of taste (especially of salty and sweet substances).

Priority Decision: The nurse admitting a patient for bariatric surgery obtains the following information from the patient. Which finding should be brought to the surgeon's attention before proceeding with further patient preparation? a. History of hypertension b. History of untreated depression c. History of multiple attempts at weight loss d. History of sleep apnea treated with continuous positive airway pressure (CPAP)

Correct answer: b Rationale: Patients with histories of untreated depression or psychosis are not good candidates for surgery. All other historical information includes medical complications of severe obesity that would help to qualify the patient for the surgery.

An older patient was admitted with a fractured hip after being found on the floor of her home. She was extremely malnourished and started on parenteral nutrition (PN) 3 days ago. Which assessment finding would be of most concern to the nurse? a. Blood glucose level of 125 mg/dL b. Serum phosphate level of 1.9 mg/dL c. White blood cell count of 10,500/µL d. Serum potassium level of 4.6 mEq/L

Correct answer: b Rationale: Refeeding syndrome can occur if a malnourished patient is started on aggressive nutritional support. Hypophosphatemia (serum phosphate level less than 2.4 mg/dL) is the hallmark of refeeding syndrome and could result in cardiac dysrhythmias, respiratory arrest, and neurologic problems. An increase in the blood glucose level is expected during the first few days after PN is started. The goal is to maintain a glucose range of 110 to 150 mg/dL. An elevated white blood cell count (greater than 11,000/µL) could indicate an infection. Normal serum potassium levels are between 3.5 and 5.0 mEq/L.

In preparing a patient for a colonoscopy, the nurse explains that a. a signed permit is not necessary. b. sedation may be used during the procedure. c. only one cleansing enema is necessary for preparation. d. a light meal should be eaten the day before the procedure.

Correct answer: b Rationale: Sedation is induced during a colonoscopy. A signed consent form is necessary for a colonoscopy. A cathartic or enema is administered the night before the procedure, and more than one enema may be necessary. Patients may need to be kept on clear liquids 1 to 2 days before the procedure.

Which patient is at highest risk for complications of obesity? a. A 30-year-old woman who is 5 ft (151 cm) tall, weighs 140 lb (63 kg), and carries weight in her thighs. b. A 56-year-old woman with a BMI of 38 kg/m2, a waist measurement of 38 in (96 cm), and a hip measurement of 36 in (91 cm) c. A 42-year-old man with a waist measurement of 36 in (91 cm) and a hip measurement of 36 in (91 cm) who is 5 ft, 6 in (166 cm) tall and weighs 150 lb (68.2 kg) d. A 68-year-old man with a waist measurement of 38 in (96 cm) and a hip measurement of 42 in (76 cm) who is 5 ft, 11 in (179 cm) tall and weighs 200 lb (90.9 kg)

Correct answer: b Rationale: The 56-year-old woman has a body mass index (BMI) of 38 kg/m2 (obese, Class II) with a waist-to-hip ratio of 1.1 with android obesity and is more at risk (very high) than the other patients. The 30-year-old woman has the least risk with a BMI of 27.3 kg/m2 (overweight) and gynoid shape. The 42-year-old man has a BMI of 24.2 kg/m2 (normal weight) with one risk factor in the waist-to-hip ratio of 1.0 and the 68-year-old man has a BMI of 27.9 kg/m2 (overweight) with a waist-to-hip ratio of 0.9.

A 40-year-old severely obese female patient with type 2 diabetes wants to lose weight. After learning about the surgical procedures, she thinks a combination of restrictive and malabsorptive surgery would be best. Which procedure should the nurse teach her about? a. Lipectomy b. Roux-en-Y gastric bypass c. Adjustable gastric banding d. Vertical sleeve gastrectomy

Correct answer: b Rationale: The Roux-en-Y gastric bypass is a common combination of restrictive (limiting the size of the stomach) and malabsorptive (less food is absorbed) surgery. Lipectomy is used to remove unsightly flabby folds of adipose tissue. Adjustable gastric banding is the most common restrictive procedure. Vertical sleeve gastrectomy is a restrictive procedure that preserves stomach function.

The patient had a car accident and was "scared to death." The patient is now reporting constipation. What affecting the gastrointestinal (GI) tract does the nurse know could be contributing to the constipation? a. The patient is too nervous to eat or drink, so there is no stool. b. The sympathetic nervous system was activated, so the GI tract was slowed. c. The parasympathetic nervous system is now functioning to slow the GI tract. d. The circulation in the GI system has been increased, so less waste is removed.

Correct answer: b Rationale: The constipation is most likely related to the sympathetic nervous system activation from the stress related to the accident. SNS activation can decrease peristalsis. Even without oral intake for a short time, stool will be formed. The parasympathetic system stimulates peristalsis. The circulation to the GI system is decreased with stress.

As gastric contents move into the small intestine, the bowel is normally protected from the acidity of gastric contents by the a. inhibition of secretin release. b. release of bicarbonate by the pancreas. c. release of pancreatic digestive enzymes. d. release of gastrin by the duodenal mucosa.

Correct answer: b Rationale: The hormone secretin stimulates the pancreas to secrete fluid with a high concentration of bicarbonate. This alkaline secretion enters the duodenum and neutralizes acid in the chyme.

A patient who has dysphagia as a consequence of a stroke is receiving enteral feedings through a percutaneous endoscopic gastrostomy (PEG). What intervention should the nurse integrate into this patient's care? a. Flush the tube with 30 mL of normal saline every 4 hours. b. Flush the tube before and after feedings if the patient's feedings are intermittent. c. Flush the PEG with 100 mL of sterile water before and after medication administration. d. To prevent fluid overload, avoid flushing when the patient is receiving continuous feeding.

Correct answer: b Rationale: The nurse should flush feeding tubes with 30 mL of water (not normal saline) every 4 hours and before and after medication administration during continuous feeding or before and after intermittent feeding. Flushes of 100 mL are excessive and may cause fluid overload in the patient.

During an examination of the abdomen the nurse should a. position the patient in the supine position with the head of the bed flat and knees straight. b. listen in the epigastrium and all four quadrants for 2 minutes for bowel sounds. c. use the following order of techniques: inspection, palpation, percussion, auscultation. d. describe bowel sounds as absent if no sound is heard in the lower right quadrant after 2 minutes.

Correct answer: b Rationale: The nurse should listen in the epigastrium and all four quadrants for bowel sounds for at least 2 minutes. The patient should be in the supine position and should slightly flex the knees; the head of the bed should be raised slightly. During examination of the abdomen, the nurse auscultates before performing percussion and palpation because the latter procedures may alter the bowel sounds. Bowel sounds cannot be described as absent until no sound is heard for 5 minutes in each quadrant.

The nurse has completed initial instruction with a patient regarding a weight loss program. The nurse determines that the teaching has been effective when the patient makes which statement? a. "I plan to lose 4 lb a week until I have lost the 60-pound goal." b. "I will keep a diary of weekly weights to illustrate my weight loss." c. "I will restrict my carbohydrate intake to less than 30 g/day to maximize weight loss." d."I should not exercise more than my program requires since increased activity increases the appetite."

Correct answer: b Rationale: The patient should monitor and record weight once per week. This prevents frustration at the normal variations in daily weights and may help the patient to maintain motivation to stay on the prescribed diet. Weight loss should occur at a rate of 1 to 2 lb/week. The diet should be well balanced rather than lacking in specific components that may cause an initial weight loss but is not usually sustainable. Exercise is a necessary component of any successful weight loss program.

A 62-year-old woman patient is scheduled for a percutaneous transhepatic cholangiography to restore biliary drainage. The nurse discusses the patient's health history and is most concerned if the patient makes which statement? a."I am allergic to bee stings." b. "My tongue swells when I eat shrimp." c. "I have had epigastric pain for 2 months." d. "I have a pacemaker because my heart rate was slow."

Correct answer: b Rationale: The percutaneous transhepatic cholangiography procedure will include the use of radiopaque contrast medium. Patients allergic to shellfish and iodine are also allergic to contrast medium. Having a pacemaker will not affect the patient during this procedure. It would be expected that the patient would have some epigastric pain given the patient's condition.

What is a normal finding on physical examination of the abdomen? a. Auscultation of bruits b. Observation of visible pulsations c. Percussion of liver dullness in the left midclavicular line d. Palpation of the spleen 1 to 2 cm below the left costal margin

Correct answer: b Rationale: The pulsation of the aorta in the epigastric area is a normal finding. Bruits indicate that blood flow is abnormal, the liver is percussed in the right midclavicular line, and a normal spleen cannot be palpated.

Which assessment should the nurse prioritize in the care of a patient who has recently begun receiving parenteral nutrition (PN)? a. Skin integrity and bowel sounds b. Electrolyte levels and daily weights c. Auscultation of the chest and tests of blood coagulability d. Peripheral vascular assessment and level of consciousness (LOC)

Correct answer: b Rationale: The use of PN necessitates frequent and thorough assessments. Key focuses of these assessments include daily weights and close monitoring of electrolyte levels. Assessments of bowel sounds, integument, peripheral vascular system, LOC, chest sounds, and blood coagulation may be variously performed, but close monitoring of fluid and electrolyte balance supersedes these in importance.

The patient cannot afford to buy the food she needs for her family, so she makes sure her children eat first, and then she eats. When she comes to the clinic, she reports bleeding gums, loose teeth, and dry, itchy skin. The nurse should know that this patient is most likely lacking which vitamin? a. Folic acid b. Vitamin C c. Vitamin D d. Vitamin K

Correct answer: b Rationale: This patient is lacking Vitamin C as evidenced by the bleeding gums, loose teeth, and dry, itchy skin. Clinical manifestations of folic acid deficiency include megaloblastic anemia, anorexia, fatigue, sore tongue, diarrhea, or forgetfulness. Clinical manifestations of Vitamin D deficiency include muscular weakness, excess sweating, diarrhea, bone pain, rickets, or osteomalacia. Clinical manifestations of Vitamin K deficiency include defective blood coagulation.

Which female patient is most likely to have metabolic syndrome? a. BP 128/78 mm Hg, triglycerides 160 mg/dL, fasting blood glucose 102 mg/dL b. BP 142/90 mm Hg, high-density lipoproteins 45 mg/dL, fasting blood glucose 130 mg/dL c. Waist circumference 36 in, triglycerides 162 mg/dL, high-density lipoproteins 55 mg/dL d. Waist circumference 32 in, high-density lipoproteins 38 mg/dL, fasting blood glucose 122 mg/dL

Correct answer: b Rationale: Three of the following five measures are needed for a woman to be diagnosed with metabolic syndrome: waist circumference >35 in, triglycerides >150 mg/dL, high-density lipoproteins <50 mg/dL, BP >130 mm Hg systolic or >85 mm Hg diastolic, fasting blood glucose >110 mg/dL. Although the other options have some abnormal measures, none has all three measures in the diagnostic ranges. The criteria for metabolic syndrome for both women and men are listed in Table 41-10.

Which statement about obesity is explained by genetics? a. Older obese patients have exacerbated changes of aging. b. Android body shape and weight gain are influenced by genetics. c. White Americans have a higher incidence of obesity than African Americans. d. Men have a harder time losing weight, as they have more muscle mass than women.

Correct answer: b Rationale: Twin studies and studies with adopted children have shown that body shape and weight gain are influenced by genetics but more research is needed. Older obese people do have exacerbated aging problems related to declines in physical function. African Americans and Hispanics have a higher incidence of obesity than whites. Women have a higher incidence of obesity and more difficulty losing weight than men because women have a higher percentage of metabolically less-active fat.

Inspection of an older patient's mouth reveals the presence of white, curd-like lesions on the patient's tongue. What is the most likely etiology for this abnormal assessment finding? a. Herpesvirus b. Candida albicans c. Vitamin deficiency d. Irritation from ill-fitting dentures

Correct answer: b Rationale: White, curd-like lesions surrounded by erythematous mucosa are associated with oral candidiasis. Herpesvirus causes benign vesicular lesions in the mouth. Vitamin deficiencies may cause a reddened, ulcerated, swollen tongue. Irritation from ill-fitting dentures will cause friable, edematous, painful, bleeding gingivae.

Priority Decision: Before administering a bolus of intermittent tube feeding to a patient with a percutaneous endoscopic gastrostomy (PEG), the nurse aspirates 220 mL of gastric contents. How should the nurse respond? a. Return the aspirate to the stomach and recheck the volume of aspirate in an hour. b. Return the aspirate to the stomach and continue with tube feeding as planned. c. Discard the aspirate to prevent over distending the stomach when the new feeding is given. d. Notify the health care provider that the feedings have been scheduled too frequently to allow for stomach emptying.

Correct answer: b Rationale: With intermittent feedings, less than 250 mL residual does not require further action. With continuous feedings and a residual of 250 mL or more after the second residual check, a pro motility agent should be considered.

An 85-year-old woman seen in the primary care provider's office for a well check complains of difficulty swallowing. What common effect of aging should the nurse assess for as a possible cause? a. Anosmia b. Xerostomia c. Hypochlorhydria d. Salivary gland tumor

Correct answer: b Rationale: Xerostomia (decreased saliva production), or dry mouth, affects many older adults and may be associated with difficulty swallowing (dysphagia). Anosmia is loss of sense of smell. Hypochlorhydria, a decrease in stomach acid, does not affect swallowing. Salivary gland tumors are not common.

Which digestive substances are active or activated in the stomach (select all that apply)? a. Bile b. Pepsin c. Gastrin d. Maltase e. Secretin f. Amylase

Correct answer: b, c Rationale: Pepsinogen is changed to pepsin by acidity of the stomach, where it begins to break down proteins. Gastrin stimulates gastric acid secretion and motility and maintains lower esophageal sphincter tone. The stomach also secretes lipase for fat digestion.Bile is secreted by the liver and stored in the gallbladder for emulsifying fats. Maltase is secreted in the small intestine and converts maltose to glucose. Secretin is secreted y the duodenal mucosa and inhibits gastric motility and acid secretion. Amylase is secured in the small intestine and by the pancreas for carbohydrate digestion.

The nurse should recognize that the liver performs which functions (select all that apply) a. Bile storage b. Detoxification c. Protein metabolism d. Steroid metabolism e. Red blood cell (RBC) destruction

Correct answer: b, c, d Rationale: The liver performs multiple major functions that aid in the maintenance of homeostasis. These include metabolism of proteins and steroids as well as detoxification of drugs and metabolic waste products. The Kupffer cells of the liver participate in the breakdown of old RBCs. The liver produces bile, but storage occurs in the gall bladder.

The patient has parenteral nutrition infusing with amino acids and dextrose. In report, the oncoming nurse is told that the tubing, the bag, and the dressing were changed 22 hours ago. What care should the nurse coming on be prepared to do (select all that apply)? a. Give the patient insulin. b. Check amount of feeding left in the bag. c. Check that the next bag has been ordered. d. Check the insertion site and change the tubing. e. Check the label to ensure ingredients and solution are as ordered.

Correct answer: b,c,e Rationale: The nurse should check the amount of feeding left in the bag, and that the next bag has been ordered to be sure the solution will not run out before the next bag is available. Parenteral nutrition solutions are only good for 24 hours and usually take some time for the pharmacy to mix for each patient. The label on the bag should be checked to ensure that the ingredients and solution are what was ordered. The patient would only receive insulin if the patient is experiencing hyperglycemia and was receiving sliding scale insulin or had diabetes mellitus. The insertion site should be checked, but the tubing is only changed every 72 hours unless lipids are being used.

The patient tells the nurse she had a history of abdominal pain, so she had a surgery to make an opening into the common bile duct to remove stones. The nurse knows that this surgery is called a a. colectomy b. cholecystectomy c. choledocholithotomy d. choledochojejunostomy

Correct answer: c Rationale: A choledocholithotomy is an opening into the common bile duct for the removal of stones. A colectomy is the removal of the colon. The cholecystectomy is the removal of the gallbladder. The choledochojejunostomy is an opening between the common bile duct and the jejunum.

In developing a weight reduction program with a 45-year-old female patient who weighs 197 lb, the nurse encourages the patient to set a weight loss goal of how many pounds in 4 weeks? a. 1-2 b. 3-5 c. 4-8 d. 5-10

Correct answer: c Rationale: A realistic weight loss goal for patients is 1 to 2 lb/wk, which prevents the patient from becoming frustrated at not meeting weight loss goals.

What is a normal finding during physical assessment of the mouth? a. A red, slick appearance of the tongue b. Uvular deviation to the side on saying "Ahh" c. A thin, white coating of the dorsum of the tongue d. Scattered red, smooth areas on the dorsum of the tongue

Correct answer: c Rationale: A thin white coating of the dorsum (top) of the tongue is normal. A red, slick appearance is characteristic of cobalamin deficiency and scattered red, smooth areas on the tongue are known as geographic tongue. The uvula should remain in the midline while the patient is saying "Ahh"

The nurse recognizes that the majority of patients' caloric needs should come from which source? a. Fats b. Proteins c. Polysaccharides d. Monosaccharides

Correct answer: c Rationale: Carbohydrates should constitute between 45% and 65% of caloric needs, compared with 20% to 35% from fats and 10% to 35% from proteins. Polysaccharides are the complex carbohydrates that are contained in breads and grains. Monosaccharides are simple sugars.

When assessing a patient's abdomen, what would be most appropriate for the nurse to do? a. Palpate the abdomen before auscultation. b. Percuss the abdomen before auscultation. c. Auscultate the abdomen before palpation. d. Perform deep palpation before light palpation.

Correct answer: c Rationale: During examination of the abdomen, auscultation is done before percussion and palpation because these latter procedures may alter the bowel sounds.

In developing an effective weight reduction plan for an overweight patient who states a willingness to try to lose weight, it is most important for the nurse to first assess which factor? a. The length of time the patient has been obese b. The patient's current level of physical activity c. The patient's social, emotional, and behavioral influences on obesity d. Anthropometric measurements, such as body mass index and skinfold thickness

Correct answer: c Rationale: Eating patterns are established early in life, and eating has many meanings for people. To establish a weight reduction plan that will be successful for the patient, the nurse should first explore the social, emotional, and behavioral influences on the patient's eating patterns. The duration of obesity, current physical activity level, and current anthropometric measurements are not as important for the weight reduction plan.

The health care team is assessing a male patient for acute pancreatitis after he presented to the emergency department with severe abdominal pain. Which laboratory value is the best diagnostic indicator of acute pancreatitis? a. Gastric pH b. Blood glucose c. Serum amylase d. Serum potassium

Correct answer: c Rationale: Elevated serum amylase levels indicate early pancreatic dysfunction and are used to diagnose acute pancreatitis. Serum lipase levels stay elevated longer than serum amylase in acute pancreatitis. Blood glucose, gastric pH, and potassium levels are not direct indicators of acute pancreatic dysfunction.

A patient who is scheduled for surgery with general anesthesia in 1 hour is observed with a moist, but empty water glass in his hand. Which assessment finding may indicate that the patient drank a glass of water? a. Flat abdomen without movement upon inspection b. Tenderness at left upper quadrant upon palpation c. Easily heard, loud gurgling in the right upper quadrant d. High-pitched, hollow sounds in the left upper quadrant

Correct answer: c Rationale: If the patient drank water on an empty stomach, gurgling can be assessed without a stethoscope or assessed with auscultation. High-pitched, hollow sounds are tympanic and indicate an empty cavity. A flat abdomen and tenderness do not indicate that the patient drank a glass of water.

What is an indication for parenteral nutrition that is not an appropriate indication for enteral tube feedings? a. Head and neck cancer b. Hypermetabolic states c. Malabsorption syndrome d. Protein-calorie malnutrition

Correct answer: c Rationale: In malabsorption syndrome, foods that are ingested into the intestinal tract cannot be digested or absorbed and tube feedings infused into the intestinal tract would not be absorbed. All of the other conditions can be treated with enteral or parenteral nutrition, depending on the patient's needs.

The nurse is evaluating the nutritional status of a 55-year-old man who is undergoing radiation treatment for oropharyngeal cancer. Which laboratory test would be the best indicator to determine if the patient has protein-calorie malnutrition? a. Serum transferrin b. C-reactive protein c. Serum prealbumin d. Alanine transaminase (ALT)

Correct answer: c Rationale: In the absence of an inflammatory condition, the best indicator of protein-calorie malnutrition (PCM) is prealbumin; prealbumin is a protein synthesized by the liver and indicates recent or current nutritional status. Decreased albumin and transferrin levels are other indicators that protein is deficient. C-reactive protein (CRP) is elevated during inflammation and is used to determine if prealbumin, albumin, and transferrin are decreased related to protein deficiency or an inflammatory process. Other indicators of protein deficiency include elevated serum potassium levels, low red blood cell counts and hemoglobin levels, decreased total lymphocyte count, elevated liver enzyme levels (ALT), and decreased levels of both fat-soluble and water-soluble vitamins.

What is the main underlying risk factor for metabolic syndrome? a. Age b. Heart disease c. Insulin resistance d. High cholesterol levels

Correct answer: c Rationale: Insulin resistance is the main underlying risk factor for metabolic syndrome. Aging is associated with metabolic syndrome. High cholesterol, hypertension, and increased clotting risk are characteristics of metabolic syndrome.

Which statement accurately describes vitamin deficiencies? a. The two nutrients most often lacking in the diet of a vegan are vitamin B6 and folic acid. b. Vitamin imbalances occur frequently in the United States because of excessive fat intake. c. Surgery on the GI tract may contribute to vitamin deficiencies because of impaired absorption. d. Vitamin deficiencies in adults most commonly are clinically manifested by disorders of the skin.

Correct answer: c Rationale: Patients who have surgery on the GI tract may be at risk for vitamin deficiencies because of inability to absorb or metabolize them. The strict vegan diet most often lacks cobalamin (vitamin B12) and iron. Although the high intake of fat is a major nutritional problem in the United States, vitamin deficiencies are rare in developed countries except in those with eating disorders or chronic alcohol abusers. Some vitamin deficiencies in adults have neurologic manifestations.

The nurse monitors the laboratory results of the patient with protein-calorie malnutrition during treatment. Which result is an indication of improvement in the patient's condition? a. Decreased lymphocytes b. Increased serum potassium c. Increased serum transferrin d. Decreased serum prealbumin

Correct answer: c Rationale: Serum transferrin is a protein that is synthesized by the liver and used for iron transport and decreases when there is protein deficiency. An increase in the protein would indicate a more positive nitrogen balance with amino acids available for synthesis. Decreased lymphocytes and serum prealbumin are indicators of protein depletion and increased serum potassium shows continuing failure of the sodium-potassium pump.

Priority Decision: During care of the severely obese patient, what is most important for the nurse to do? a. Avoid reference to the patient's weight to avoid embarrassing the patient. b. Emphasize to the patient how important it is to lose weight to maintain health. c. Plan for necessary modifications in equipment and nursing techniques before initiating care. d. Recognize that a full assessment of each body system might not be possible because of numerous layers of skinfolds.

Correct answer: c Rationale: Special considerations are needed for the care of the severely obese patient because most hospital units are not prepared with beds, chairs, BP cuffs, and other equipment that will need to be used with the very obese patient. Consideration of all aspects of care should be made before implementing care for the patient, including extra time and perhaps assistance for positioning, physical assessment, and transferring the patient.

When caring for the patient with heart failure, the nurse knows that which gastrointestinal process is most dependent on cardiac output and may affect the patient's nutritional status? a. Ingestion b. Digestion c. Absorption d. Elimination

Correct answer: c Rationale: Substances that interface with the absorptive surfaces of the GI tract (primarily in the small intestine) diffuse across the intestinal membranes into intestinal capillaries and are then carried to other parts of the body for use in energy production. The cardiac output provides the blood flow for this absorption of nutrients to occur.

How will an obstruction at the ampulla of Vater affect the digestion of all nutrients? a. Bile is responsible for emulsification of all nutrients and vitamins. b. Intestinal digestive enzymes are released through the ampulla of Vater. c. Both bile and pancreatic enzymes enter the duodenum at the ampulla of Vater. d. Gastric contents can ply pass to the duodenum when the ampulla of Vater is open.

Correct answer: c Rationale: The ampulla of Vater is the site where the pancreatic duct and common bile duct enter the duodenum and the opening and closing of the ampulla is controlled by the sphincter of Oddi. Because bile from the common bile duct is needed for emulsification of fat to promote digestion and pancreatic enzymes from the pancreas are needed for digestion of all nutrients, a blockage at this point would affect the digestion of all nutrients. Gastric contents pass into the duodenum through the pylorus or pyloric valve.

A patient's serum liver enzyme tests reveal an elevated aspartate aminotransferase (AST). The nurse recognizes what about the elevated AST? a. It eliminates infection as a cause of liver damage. b. It is diagnostic for liver inflammation and damage. c. Tissue damage in organs other than the liver may be identified. d. Nervous system symptoms related to hepatic encephalopathy may be the cause.

Correct answer: c Rationale: The aspartate aminotransferase (AST) level is elevated in liver disease but it is important to note that it is also elevated in damage to the heart and lungs and is not a specific test for liver function. Measurements of most of the transaminases involves nonspecific tests unless isoenzyme fractions are determined. Hepatic encephalopathy is related to elevated ammonia levels.

The nurse is assessing a 50-year-old woman admitted with a possible bowel obstruction. Which assessment finding would be expected in this patient? a. Tympany to abdominal percussion b. Aortic pulsation visible in epigastric region c. High-pitched sounds on abdominal auscultation d. Liver border palpable 1 cm below the right costal margin

Correct answer: c Rationale: The bowel sounds are more high pitched (rushes and tinkling) when the intestines are under tension, as in intestinal obstruction. Bowel sounds may also be diminished or absent with an intestinal obstruction. Normal findings include aortic pulsations on inspection and tympany with percussion, and the liver may be palpable 1 to 2 cm along the right costal margin.

The nurse is caring for a patient admitted to the hospital for asthma who weighs 186 lb (84.5 kg). During dietary counseling, the patient asks the nurse how much protein he should ingest each day. How many grams of protein does the nurse recommend should be included in the diet based on the patient's current weight? a. 24 b. 41 c. 68 d. 93

Correct answer: c Rationale: The daily intake of protein should be between 0.8 and 1 g/kg of body weight. Thus this patient should take in between 68 and 84 g of protein per day in the diet.

A patient receives atropine, an anticholinergic drug, in preparation for surgery. The nurse expects this drug to affect the GI tract by doing what? a. Increasing gastric emptying b. Relaxing pyloric and ileocecal sphincters c. Decreasing secretions and peristaltic action d. Stimulation the nervous system of the GI tract

Correct answer: c Rationale: The parasympathetic nervous system stimulates activity of the gastrointestinal (GI) tract, increasing motility and secretions and relaxing sphincters to promote movement of contents. A drug that blocks this activity decreases secretions and peristalsis, slows gastric emptying, and contracts sphincters. The enteric nervous system of the GI tract is modulated by sympathetic and parasympathetic influence.

What may occur with failure of the sodium-potassium pump during severe protein depletion? a. Ascites b. Anemia c. Hyperkalemia d. Hypoalbuminemia

Correct answer: c Rationale: The sodium-potassium pump uses 20% to 50% of all calories ingested. When energy sources are decreased, the pump fails to function, sodium and water are left in the cell, and potassium remains in extracellular fluids. Hyperkalemia, as well as hyponatremia, can occur.

Priority Decision: When caring for a patient who has had most of the stomach surgically removed, what is important for the nurse to teach the patient? a. Extra iron will need to be taken to prevent anemia. b. Avoid foods with lactose to prevent bloating and diarrhea. c. Lifelong supplementation of cobalamin (vitamin B12) will be needed. d. Because of the absence of digestive enzymes, protein malnutrition is likely.

Correct answer: c Rationale: The stomach secretes intrinsic factor, necessary for cobalamin (vitamin B12) absorption in the intestine. When part or all of the stomach is removed, cobalamin must be supplemented for life. The other options will not be a problem.

The nurse is providing care for a 23-year-old woman who is a strict vegetarian. To prevent the consequences of iron deficiency, what should the nurse recommend? a. Brown rice and kidney beans b. Cauliflower and egg substitutes c. Soybeans and hot breakfast cereal d. Whole-grain bread and citrus fruits

Correct answer: c Rationale: Vegetarians are at a particular risk for iron deficiency, a problem that can be prevented by regularly consuming high-iron foods such as hot cereals and soybeans. The other foods listed are not classified as high sources of iron.

When the nurse is assessing the health perception-health maintenance pattern as related to GI function, an appropriate question to ask is a. "What is your usual bowel elimination pattern?" b. "What percentage of your income is spent on food?" c. "Have you traveled to a foreign country in the last year?" d. "Do you have diarrhea when you are under a lot of stress?"

Correct answer: c Rationale: When assessing gastrointestinal function in relation to the health perception-health management pattern, the nurse should ask the patient about recent foreign travel with possible exposure to hepatitis, parasitic infestation, or bacterial infection.

Which explanation about weight reduction should be included when teaching the obese patient and her obese husband? a. Weight gain is caused by psychologic factors. b. Daily weighing is recommended to monitor weight loss. c. Fat is not burned until the glycogen-water pool is depleted. d. Men lose weight less quickly than women because they have a higher percentage of metabolically less-active fat.

Correct answer: c Rationale: With reducing diets that severely restrict carbohydrates, the body's glycogen stores become depleted within a few days. The glycogen normally binds to water in fat cells and it is this water loss that causes weight loss in the first few days. Fat is not burned until the glycogen-water pool is depleted. Although psychosocial components (i.e., using food for comfort or reward and inability to buy high-nutritional quality food) may have an influence on weight gain, these factors along with lack of physical exercise, underestimation of portion size, and genetics contribute to weight gain. Weekly weighing is recommended as a more reliable indicator of weight loss because daily weighing shows frequent fluctuation from retained water (including urine) and elimination of feces. Men are able to lose weight more quickly than women because women have a higher percentage of metabolically less-active fat.

Which nursing actions are indicated for a liver biopsy (select all that apply)? a. Observe for white stools b. Monitor for rectal bleeding c. Monitor for internal bleeding d. Position to right side after test e. Ensure bowel preparation was done f. Check coagulation status before test

Correct answer: c, d, f Rationale: Because the liver is a vascular organ, vital signs are monitored to assess for internal bleeding. Prevention of bleeding is the reason for positioning on the right side for at least 2 hours and for splinting the puncture site. Again, because of the vasculature of the liver, coagulation status is checked before the biopsy is done. White stools occur with upper gastrointestinal (UGI) or barium swallow tests. No smoking is to be done after midnight before the study with an UGI. The bowel must be cleared before a lower GI or barium enema, a virtual colonoscopy, or a colonoscopy. Rectal bleeding may occur with a sigmoidoscopy or colonoscopy. A perforation may occur with an esophagogastroduodenoscopy (EGD), ERCP, or peritoneoscopy.

When the nurse identifies an individual at risk for malnutrition with nutritional screening, what is the next step for the nurse to take? a. Supply supplements between meals. b. Encourage eating meals with others. c. Have family bring in food from home. d. Complete a full nutritional assessment.

Correct answer: d Rationale: A full nutritional assessment includes history and physical examination and laboratory data. The nutritional assessment will need to be done to provide the basis for nutrition intervention. The interventions may include supplements if ordered, family bringing food from home, and socializing with meals.

A complete nutritional assessment including anthropometric measurements is important for the patient who a. has a BMI of 25.5 kg/m2. b. complains of frequent nocturia. c. reports a 5-year history of constipation. d. reports an unintentional weight loss of 10 lb in 2 months.

Correct answer: d Rationale: A loss of more than 5% of usual body weight over 6 months, whether intentional or unintentional, is a critical indicator for further assessment.

Which patient is at highest risk for developing metabolic syndrome? a. A 62-year-old white man who has coronary artery disease with chronic stable angina b. A 54-year-old Hispanic woman who is sedentary and has nephrogenic diabetes insipidus c. A 27-year-old Asian American woman who has preeclampsia and gestational diabetes mellitus d. A 38-year-old Native American man who has diabetes mellitus and elevated hemoglobin A1C

Correct answer: d Rationale: African Americans, Hispanics, Native Americans, and Asians are at an increased risk for development of metabolic syndrome. Other risk factors include individuals who have diabetes that cannot maintain a normal glucose level, have hypertension, and secrete a large amount of insulin, or who have survived a heart attack and have hyperinsulinemia.

In the immediate postoperative period a nurse cares for a severely obese 72-year-old man who had surgery for repair of a lower leg fracture. Which assessment would be most important for the nurse to make? a. Cardiac rhythm b. Surgical dressing c. Postoperative pain d. Oxygen saturation

Correct answer: d Rationale: After surgery an older and/or severely obese patient should be closely monitored for oxygen desaturation. The body stores anesthetics in adipose tissue, placing patients with excess adipose tissue (e.g., obesity, older) at risk for resedation. As adipose cells release anesthetic back into the bloodstream, the patient may become sedated after surgery. This may depress the respiratory rate and result in a drop in oxygen saturation.

The nurse is performing a focused abdominal assessment of a patient who has been recently admitted. In order to palpate the patient's liver, where should the nurse palpate the patient's abdomen? a. Left lower quadrant b. Left upper quadrant c. Right lower quadrant d. Right upper quadrant

Correct answer: d Rationale: Although the left lobe of the liver is located in the left upper quadrant of the abdomen, the bulk of the liver is located in the right upper quadrant.

At the first visit to the clinic, the female patient with a BMI of 29 kg/m2 tells the nurse that she does not want to become obese. Which question used for assessing weight issues is the most important question for the nurse to ask? a. "What factors contributed to your current body weight?" b. "How is your overall health affected by your body weight?" c. "What is your history of gaining weight and losing weight?" d. "In what ways are you interested in managing your weight differently?"

Correct answer: d Rationale: Asking the patient about her desire to manage her weight in a different manner helps the nurse determine the patient's readiness for learning, degree of motivation, and willingness to change lifestyle habits. The nurse can help the patient set realistic goals. This question will also lead to discussing the patient's history of gaining and losing weight and factors that have contributed to the patient's current weight. The patient may be unaware of the overall health effects of her body weight, so this question is not helpful at this time.

After eating, a patient with an inflamed gallbladder experiences pain caused by contraction of the gallbladder. What is the mechanism responsible for this action? a. Production of bile by the liver b. Production of secretin by the duodenum c. Release of gastrin from the stomach antrum d. Production of cholecystokinin by the duodenum

Correct answer: d Rationale: Cholecystokinin is secreted by the duodenal mucosa when fats and amino acids enter the duodenum and stimulate the gallbladder to release bile to emulsify the fats for digestion. The bile is produced by the liver but stored in the gallbladder. Secretin is responsible for stimulating pancreatic bicarbonate secretion and gastrin increases gastric motility and acid secretion.

A severely obese patient has undergone Roux-en-Y gastric bypass surgery. In planning postoperative care, the nurse anticipates that the patient a. may have severe diarrhea early in the postoperative period. b. will not be allowed to ambulate for 1 to 2 days postoperatively. c. will require nasogastric suction until the incision heals. d. may have only liquids orally, and in very limited amounts, during the postoperative period.

Correct answer: d Rationale: During the immediate postoperative period, water and sugar-free clear liquids are given (30 mL every 2 hours while the patient is awake).

What is the most common cause of secondary protein-calorie malnutrition in the United States? a. The unavailability of foods high in protein b. A lack of knowledge about nutritional needs c. A lack of money to purchase high-protien foods d. An alteration in ingestion, digestion, absorption, or metabolism

Correct answer: d Rationale: In the United States, where rote in intake is high and of good quality, protein-calorie malnutrition most commonly results from problems of the GI system. In developing countries, adequate food sources might not exist, the inhabitants may not be well educated about nutritional needs, and economic conditions can prevent purchase of balanced diets.

The obesity classification that is most often associated with cardiovascular health problems is a. primary obesity. b. secondary obesity. c. gynoid fat distribution. d. android fat distribution.

Correct answer: d Rationale: Individuals with fat located primarily in the abdominal area (i.e., whose body is apple-shaped) are at greater risk for obesity-related complications (e.g., heart disease) than are those whose fat is primarily located in the upper legs (i.e., whose body is pear-shaped). Individuals whose fat is distributed over the abdomen and upper body (i.e., neck, arms, and shoulders) are classified as having android obesity.

A patient received a small-bore nasogastric (NG) tube after a laryngectomy. What should be the nurse's priority intervention before starting the enteral feeding? a. Aspiration b. Auscultation of air c. Set head of bed at 40 degrees. d. Verify NG tube placement on x-ray.

Correct answer: d Rationale: It is imperative to ensure that an NG tube is situated in the GI tract rather than the patient's lungs. When an NG tube has been recently inserted, it is important to confirm this placement with an x-ray that will identify the tube's radiopaque tip. Aspiration and air auscultation may not differentiate between gastric and respiratory placement of the tube. The head of bed elevated at least 30 degrees is to prevent aspiration. To determine the maintenance of the feeding tube's proper position, the exit site of the tube is marked at the time of the x-ray and the external portion measured to allow for assessment of a change position with a change in the length of the tube.

Which patient has the highest risk for poor nutritional balance related to decreased ingestion? a. Tuberculosis infection b. Malabsorption syndrome c. Draining decubitus ulcers d. Severe anorexia resulting from radiation therapy

Correct answer: d Rationale: Malnutrition that results form a deceased intake of food is most common in individuals with severe anorexia where there is a decreased desire to eat. Infections created a hypermetabolic state that increases nutritional demand, malabsorption causes loss of nutrients that are ingested, and draining decubitus ulcers are examples of disorders that cause both loss of protein and hypermetabolic states.

The nurse has completed initial instruction with a patient regarding a weight-loss program. Which patient comment indicates to the nurse that the teaching has been effective? a. "I will keep a diary of daily weight to illustrate my weight loss." b. "I plan to lose 4 lb a week until I have lost the 60 lb I want to lose." c. "I should not exercise more than what is required so I don't increase my appetite." d. "I plan to join a behavior modification group to help establish long-term behavior changes."

Correct answer: d Rationale: People who have undergone behavior therapy are more successful in maintaining weight losses over time because most programs deemphasize the diet, focus on how and when the person eats and education, and provide support from others. Weighing daily is not recommended and plateaus may not allow for consistent weight loss. A goal for weight loss must be set and 1 to 2 pounds a week is realistic. A more rapid loss often causes skin and underlying tissue to lose elasticity and become flabby folds of tissue. Exercising more often depresses appetite and exercise need not be limited.

A patient is admitted to the hospital with a diagnosis of diarrhea with dehydration. The nurse recognizes that increased peristalsis resulting in diarrhea can be related to a. sympathetic inhibition. b. mixing and propulsion. c. sympathetic stimulation. d. parasympathetic stimulation.

Correct answer: d Rationale: Peristalsis is increased by parasympathetic stimulation.

The nurse is reviewing the laboratory test results for a 71-year-old patient with metastatic lung cancer. The patient was admitted with a diagnosis of malnutrition. The serum albumin level is 4.0 g/dL, and prealbumin is 10 mg/dL. What should this indicate to the nurse? a. The albumin level is normal, and therefore the patient does not have protein malnutrition. b. The albumin level is increased, which is a common finding in patients with cancer who have malnutrition. c. Both the serum albumin and prealbumin levels are reduced, consistent with the admitting diagnosis of malnutrition. d. Although the serum albumin level is normal, the prealbumin level more accurately reflects the patient's nutritional status.

Correct answer: d Rationale: Prealbumin has a half-life of 2 days and is a better indicator of recent or current nutritional status. Serum albumin has a half-life of approximately 20 to 22 days. The serum level may lag behind actual protein changes by more than 2 weeks and is therefore not a good indicator of acute changes in nutritional status.

A patient with anorexia nervosa shows signs of malnutrition. During initial referring, the nurse carefully assesses the patient for a. hyperkalemia. b. hypoglycemia. c. hypercalcemia. d. hypophosphatemia.

Correct answer: d Rationale: Refeeding syndrome is characterized by fluid retention, electrolyte imbalances (e.g., hypophosphatemia, hypokalemia, hypomagnesemia), and hyperglycemia. Conditions that predispose patients to refeeding syndrome include long-standing malnutrition states such as those induced by chronic alcoholism, vomiting and diarrhea, chemotherapy, and major surgery. Refeeding syndrome can occur any time a malnourished patient is started on aggressive nutritional support. Hypophosphatemia is the hallmark of refeeding syndrome, and it is associated with serious outcomes, including cardiac dysrhythmias, respiratory arrest, and neurologic disturbances (e.g., paresthesias).

The best nutritional therapy plan for a person who is obese is a. the Zone diet. b. the Atkins diet. c. Sugar Busters. d. foods from the basic food groups.

Correct answer: d Rationale: Restricted food intake is a cornerstone for any weight loss or maintenance program. A good weight loss plan should include foods from the basic food groups.

The severely obese patient has elected to have the Roux-en-Y gastric bypass (RYGB) procedure. The nurse will know the patient understands the preoperative teaching when the patient makes which statement? a. "This surgery will preserve the function of my stomach." b. "This surgery will remove the fat cells from my abdomen." c. "This surgery can be modified whenever I need it to be changed." d. "This surgery decreases how much I can eat and how many calories I can absorb."

Correct answer: d Rationale: The RYGB decreases the size of the stomach to a gastric pouch and attaches it directly to the small intestine so food bypasses 90% of the stomach, the duodenum, and a small segment of the jejunum. The vertical sleeve gastrectomy removes 85% of the stomach, but preserves the function of the stomach. Lipectomy and liposuction remove fat tissue from the abdomen or other areas. Adjustable gastric banding can be modified or reversed at a later date.

Which patient has the highest morbidity risk? a. Male 6 ft. 1 in. tall, BMI 29 kg/m2 b. Female 5 ft. 6 in. tall, weight 150 lb. c. Male with waist circumference 46 in. d. Female 5 ft. 10 in. tall, obesity Class III

Correct answer: d Rationale: The patient in Class III obesity has the highest risk for disease because Class III denotes severe obesity or a BMI greater than 40 kg/m2. The patient with waist circumference 46 in. has a high risk for disease, but without the BMI or obesity class, a more precise determination cannot be made. The female who is 5 ft. 6 in. tall has a normal weight for her height. The male patient who is over 6 ft. tall is overweight, which increases his risk of disease, but a more precise determination cannot be made without the waist circumference.

The patient being admitted has been diagnosed with anorexia nervosa. What clinical manifestations should the nurse expect to see on admission assessment? a. Tan skin, blonde hair, and diarrhea b. Sensitivity to heat, fatigue, and polycythemia c. Dysmenorrhea, gastric ulcer pain, and hunger d. Hair loss; dry, yellowish skin; and constipation

Correct answer: d Rationale: The patient with anorexia nervosa, along with abnormal weight loss, is likely to have hair loss; dry, yellow skin; constipation; sensitivity to cold, and absent or irregular menstruation. Other signs of malnutrition are also noted during physical examination.

Priority Decision: An 18-year-old female patient with anorexia nervosa is admitted to the hospital for treatment. On admission she weighs 82 lb (37 kg) and is 5 ft. 3 in (134.6 cm). Her laboratory test results include the following: K+ 2.8 mEq/L (2.8 mmol/L), Hgb 8.9 g/dL (89 g/L), and BUN 64 mg/dL (22.8 mmol/L). In planning care for the patient, the nurse gives the highest priority to which of the following nursing diagnoses? a. Risk for injury related to dizziness and weakness resulting from anemia b. Imbalanced nutrition: less than body requirements related to inadequate food intake c. Risk for impaired urinary elimination related to elevated BUN resulting from renal failure d. Risk for decreased cardiac output (CO) related to dysrhythmias resulting from hypokalemia

Correct answer: d Rationale: The potential life-threatening cardiac complications related to the hypokalemia are the most important immediate considerations in the patient's care. The other nursing diagnoses are important for the patient's care but do not pose immediate risk that the hypokalemia does.

What is a postoperative nursing intervention for the obese patient who has undergone bariatric surgery? a. Irrigating and repositioning the nasogastric (NG) tube as needed b. Delaying ambulation until the patient has enough strength to support self c. Keeping the patient positioned on the side to facilitate respiratory function d. Providing adequate support to the incision during coughing, deep breathing, and turning

Correct answer: d Rationale: Turning, coughing, and deep breathing are essential to prevent postoperative complications. Protecting the incision from strain is important since wound dehiscence is a problem for obese patients. If a nasogastric (NG) tube that is present following gastric surgery for severe obesity becomes blocked or needs repositioning, the health care provider should be notified. Ambulation is usually started on the evening of surgery and addition help will be needed to support the patient. Respiratory function is promoted by keeping the head of the bed elevated at an angle of 35 to 40 degrees.

A patient who is unable to swallow because of progressive amyotrophic lateral sclerosis is prescribed enteral nutrition through a newly placed gastrostomy tube. Which task is appropriate for the nurse to delegate to unlicensed assistive personnel (UAP)? a. Irrigate the tube between feedings. b. Provide wound care at the gastrostomy site. c. Administer prescribed liquid medications through the tube. d.Position the patient with a 45-degree head of bed elevation.

Correct answer: d Rationale: Unlicensed assistive personnel (UAP) may position the patient receiving enteral feedings with the head of bed elevated. A licensed practical nurse/licensed vocational nurse (LPN/LVN) or an RN could perform the other activities.

A 68-year-old patient is in the office for a physical. She notes that she no longer has regular bowel movements. Which suggestion by the nurse would be most helpful to the patient? a. Take an additional laxative to stimulate defecation. b. Eat less acidic foods to enable the gastrointestinal system to increase peristalsis. c. Eat less food at each meal to prevent feces from backing up related to slowed peristalsis. d. Attempt defecation after breakfast because gastrocolic reflexes increase colon peristalsis at that time.

Correct answer: d Rationale: When food inters the stomach and duodenum, the gastrocolic and duodenocolic reflexes are initiated and are more active after the first daily meal. Additional laxatives or laxative abuse contribute to constipation in older adults. Decreasing food intake is not recommended, as many older adults have a decreased appetite. Fibre and fluids should be increased.

The ED nurse has inspected, auscultated, and palpated the abdomen with no obvious abnormalities, except pain. When the nurse palpates the abdomen for rebound tenderness, there is severe pain. The nurse should know that this could indicate what problem? a. Hepatic cirrhosis b. Hypersplenomegaly c. Gall bladder distention d. Peritoneal inflammation

Correct answer: d Rationale: When palpating for rebound tenderness, the problem area of the abdomen will produce pain and severe muscle spasm when there is peritoneal inflammation. Hepatic cirrhosis, hypersplenomegaly, and gall bladder distention do not manifest with rebound tenderness.

This bariatric surgical procedure involves creating a stoma and gastric pouch that is reversible, and no malabsorption occurs. What surgical procedure is this? a. Vertical gastric banding b. Biliopancreatic diversion c. Roux-en-Y gastric bypass d. Adjustable gastric banding

Correct answer: d Rationale: With adjustable gastric banding (AGB), the stomach size is limited by an inflatable band placed around the fundus of the stomach. The band is connected to a subcutaneous port and can be inflated or deflated to change the stoma size to meet the patient's needs as weight is lost. The procedure is performed laparoscopically and, if necessary, can be modified or reversed after the initial procedure.

Normally, which hormones and peptides affect appetite (select all that apply)? a. Leptin b. Insulin c. Ghrelin d. Peptide YY e. Neuropeptide Y f. Cholecystokinin

Correct answers: a, b, c, d, e, f Rationale: Normally ghrelin and neuropeptide Y stimulate appetite. Leptin suppresses appetite and hunger. Insulin decreases appetite. Peptide YY and cholecystokinin inhibit appetite by slowing gastric emptying and sending satiety signals to the hypothalamus.

A normal physical assessment finding of the GI system is/are (select all that apply) a. nonpalpable liver and spleen. b. borborygmi in upper right quadrant. c. tympany on percussion of the abdomen. d. liver edge 2 to 4 cm below the costal margin. e. finding of a firm, nodular edge on the rectal examination.

Correct answers: a, c Rationale: Normal assessment findings for the gastrointestinal system include a nonpalpable liver and spleen and generalized tympany on percussion. Normally, bowel sounds are high pitched and gurgling; loud gurgles indicate hyperperistalsis and are called borborygmi (stomach growling). If the patient has chronic obstructive pulmonary disease, large lungs, or a low-set diaphragm, the liver may be palpated 0.4 to 0.8 inch (1 to 2 cm) below the right costal margin. On palpation, the rectal wall should be soft and smooth and should have no nodules.

Priority Decision: The nurse is teaching a moderately obese woman interventions for the management of obesity. Initially, which strategies will support restricting dietary intake to below energy requirements (select all that apply)? a. Limit alcohol b. Rest when fatigued c. Determine portion sizes d. 1800- to 2200-calorie diet e. Attend Overeaters Anonymous

Correct answers: a, c Rationale: To restrict dietary intake so that it is below energy requirements, the moderately obese woman should limit or avoid alcohol intake because it increases caloric intake and has low nutritional value. Portion sizes have increased over the years and are larger than they should be. Teach the patient to determine portion sizes by weight or learn equivalencies such as that a serving of fruit is the size of a baseball. A progressive exercise program will increase energy requirements and a diet with an initial 800- to 1200-calorie limit would decrease calorie intake. Overeaters Anonymous would not restrict dietary intake below energy requirements, although it may offer support for the patient.

Health risks associated with obesity include (select all that apply) a. colorectal cancer. b. rheumatoid arthritis. c. polycystic ovary syndrome. d. nonalcoholic steatohepatitis. e. systemic lupus erythematosus.

Correct answers: a, c, d Rationale: Health risks associated with obesity include cardiovascular disease (related to increased low-density lipoprotein levels, increased triglyceride levels, and decreased high-density lipoprotein levels), hypertension, sleep apnea, obesity hypoventilation syndrome, reduced chest wall compliance, increased work of breathing, decreased total lung capacity and functional residual capacity, type 2 diabetes mellitus (i.e., hyperinsulinemia and insulin resistance), osteoarthritis, hyperuricemia, gout, gastroesophageal reflux disease, gallstones, nonalcoholic steatohepatitis, fatty liver and cirrhosis, cancer (mainly breast, endometrial, kidney, colorectal, pancreas, esophagus, and gallbladder), psychosocial problems (employment, education, and health care), low self-esteem, withdrawal from social interactions, and major depression.

A community health nurse is conducting an initial assessment of a new patient. Which assessments should the nurse include when screening the patient for metabolic syndrome (select all that apply)? a. Blood pressure b. Resting heart rate c. Physical endurance d. Waist circumference e. Fasting blood glucose

Correct answers: a, d, e Rationale: The diagnostic criteria for metabolic syndrome include elevated blood pressure, fasting blood glucose, waist circumference, triglycerides, and HDL cholesterol. Resting heart rate and physical endurance are not part of the diagnostic criteria.

What characteristics describe adjustable gastric banding (select all that apply)? a. 85% of the stomach is removed. b. Stomach restriction can be reversed. c. Eliminates hormones that stimulate hunger. d. Malabsorption of fat-soluble vitamins occurs. e. Inflatable band allows for modification of gastric stoma size. f. Stomach with a gastric pouch surgically anastomosed to the jejunum.

Correct answers: b, e Rationale: The adjustable gastric banding procedure is reversible and allows a change in gastric stoma size by inflation or deflation of the band around the funds of the stomach. The vertical sleeve gastrectomy removes 85% of the stomach and eliminates the hormones produced in the stomach that stimulate hunger. The biliopancreatic diversion is a maladaptive surgery that prevents absorption of nutrients, including fat-soluble vitamins. The Roux-en-Y gastric bypass reduces the stomach size with a gastric pouch anastomosed to the small intestine, so it is both restrictive and malabsorptive.

In preparing to care for the obese patient with cancer, what physiologic problems is this patient at a greater risk for having (select all that apply)? a. Tinnitus b. Fractures c. Sleep apnea d. Trousseau's sign e. Type 2 diabetes mellitus f. Gastroesophageal reflux disease (GERD)

Correct answers: c, e, f Rationale: Obese patients are at a higher risk for cancer, sleep apnea and sleep deprivation, type 2 diabetes mellitus, gastroesophageal reflux disease (GERD), nonalcoholic steatohepatits, osteoarthritis, and cardiovascular problems. The other options are not related to obesity.

What is the preferred diagnostic procedure in appendicitis?

Ct scan, but ultrasound is also used

Conjunctivitis: Collaborative Management

Culture Medications Patient Education

A clinical condition that results from chronic exposure to excess corticosteroids, particularly glucocorticoids.

Cushing syndrome

Excess corticosteroids, High blood sugar, increase Na+ and decrease in K+, Fluid retention, adipose tissue, buffalo hump, poor wound healing, protein wasting (muscle weakness), loss of collagen (thin skin), virilization in women & feminization in men

Cushing syndrome. What are the defining characteristics

Too much cortisol

Cushing's syndrome

The nurse receives the following information about a 51-year-old woman who is scheduled for a colonoscopy. Which information should be communicated to the health care provider before sending the patient for the procedure? a. The patient has a permanent pacemaker to prevent bradycardia. b. The patient is worried about discomfort during the examination. c. The patient has had an allergic reaction to shellfish and iodine in the past. d. The patient refused to drink the ordered polyethylene glycol (GoLYTELY).

D

Physiological Integrity 6. A patient has a normal cardiac rhythm and a heart rate of 72 beats/minute. The nurse determines that the P-R interval is 0.24 seconds. The most appropriate intervention by the nurse would be to a. notify the health care provider immediately. b. give atropine per agency dysrhythmia protocol. c. prepare the patient for temporary pacemaker insertion. d. document the finding and continue to monitor the patient.

D First-degree atrioventricular (AV) block is asymptomatic and requires ongoing monitoring because it may progress to more serious forms of heart block. The rate is normal, so there is no indication that atropine is needed. Immediate notification of the health care provider about an asymptomatic rhythm is not necessary. DIF: Cognitive Level: Apply (application) REF: 798 TOP: Nursing Process: Implementation MSC:

Physiological Integrity 17. Which laboratory result for a patient with multifocal premature ventricular contractions (PVCs) is most important for the nurse to communicate to the health care provider? a. Blood glucose 243 mg/dL b. Serum chloride 92 mEq/L c. Serum sodium 134 mEq/L d. Serum potassium 2.9 mEq/L

D Hypokalemia increases the risk for ventricular dysrhythmias such as PVCs, ventricular tachycardia, and ventricular fibrillation. The health care provider will need to prescribe a potassium infusion to correct this abnormality. Although the other laboratory values also are abnormal, they are not likely to be the etiology of the patient's PVCs and do not require immediate correction. DIF: Cognitive Level: Apply (application) REF: 799 OBJ: Special Questions: Prioritization TOP: Nursing Process: Assessment MSC:

Physiological Integrity 4. The nurse obtains a rhythm strip on a patient who has had a myocardial infarction and makes the following analysis: no visible P waves, P-R interval not measurable, ventricular rate 162, R-R interval regular, and QRS complex wide and distorted, QRS duration 0.18 second. The nurse interprets the patient's cardiac rhythm as a. atrial flutter. b. sinus tachycardia. c. ventricular fibrillation. d. ventricular tachycardia.

D The absence of P waves, wide QRS, rate >150 beats/minute, and the regularity of the rhythm indicate ventricular tachycardia. Atrial flutter is usually regular, has a narrow QRS configuration, and has flutter waves present representing atrial activity. Sinus tachycardia has P waves. Ventricular fibrillation is irregular and does not have a consistent QRS duration. DIF: Cognitive Level: Apply (application) REF: 794 TOP: Nursing Process: Assessment MSC:

Physiological Integrity 7. A patient who was admitted with a myocardial infarction experiences a 45-second episode of ventricular tachycardia, then converts to sinus rhythm with a heart rate of 98 beats/minute. Which of the following actions should the nurse take next? a. Immediately notify the health care provider. b. Document the rhythm and continue to monitor the patient. c. Perform synchronized cardioversion per agency dysrhythmia protocol. d. Prepare to give IV amiodarone (Cordarone) per agency dysrhythmia protocol.

D The burst of sustained ventricular tachycardia indicates that the patient has significant ventricular irritability, and antidysrhythmic medication administration is needed to prevent further episodes. The nurse should notify the health care provider after the medication is started. Defibrillation is not indicated given that the patient is currently in a sinus rhythm. Documentation and continued monitoring are not adequate responses to this situation. DIF: Cognitive Level: Apply (application) REF: 799 TOP: Nursing Process: Implementation MSC:

Physiological Integrity 22. The nurse has received change-of-shift report about the following patients on the progressive care unit. Which patient should the nurse see first? a. A patient who is in a sinus rhythm, rate 98, after having electrical cardioversion 2 hours ago b. A patient with new onset atrial fibrillation, rate 88, who has a first dose of warfarin (Coumadin) due c. A patient with second-degree atrioventricular (AV) block, type 1, rate 60, who is dizzy when ambulating d. A patient whose implantable cardioverter-defibrillator (ICD) fired two times today who has a dose of amiodarone (Cordarone) due

D The frequent firing of the ICD indicates that the patient's ventricles are very irritable, and the priority is to assess the patient and administer the amiodarone. The other patients may be seen after the amiodarone is administered. DIF: Cognitive Level: Analyze (analysis) REF: 803 OBJ: Special Questions: Prioritization; Multiple Patients TOP: Nursing Process: Implementation MSC:

Physiological Integrity 19. A patient's cardiac monitor shows sinus rhythm, rate 64. The P-R interval is 0.18 seconds at 1:00 AM, 0.22 seconds at 2:30 PM, and 0.28 seconds at 4:00 PM. Which action should the nurse take next? a. Place the transcutaneous pacemaker pads on the patient. b. Administer atropine sulfate 1 mg IV per agency dysrhythmia protocol. c. Document the patient's rhythm and assess the patient's response to the rhythm. d. Call the health care provider before giving the next dose of metoprolol (Lopressor).

D The patient has progressive first-degree atrioventricular (AV) block, and the -blocker should be held until discussing the medication with the health care provider. Documentation and assessment are appropriate but not fully adequate responses. The patient with first-degree AV block usually is asymptomatic, and a pacemaker is not indicated. Atropine is sometimes used for symptomatic bradycardia, but there is no indication that this patient is symptomatic. DIF: Cognitive Level: Apply (application) REF: 798 OBJ: Special Questions: Prioritization TOP: Nursing Process: Implementation MSC:

Physiological Integrity 20. A patient develops sinus bradycardia at a rate of 32 beats/minute, has a blood pressure (BP) of 80/42 mm Hg, and is complaining of feeling faint. Which actions should the nurse take next? a. Recheck the heart rhythm and BP in 5 minutes. b. Have the patient perform the Valsalva maneuver. c. Give the scheduled dose of diltiazem (Cardizem). d. Apply the transcutaneous pacemaker (TCP) pads.

D The patient is experiencing symptomatic bradycardia, and treatment with TCP is appropriate. Continued monitoring of the rhythm and BP is an inadequate response. Calcium channel blockers will further decrease the heart rate, and the diltiazem should be held. The Valsalva maneuver will further decrease the rate. DIF: Cognitive Level: Apply (application) REF: 804 OBJ: Special Questions: Prioritization TOP: Nursing Process: Implementation MSC:

Psychosocial Integrity 11. After providing a patient with discharge instructions on the management of a new permanent pacemaker, the nurse knows that teaching has been effective when the patient states a. "I will avoid cooking with a microwave oven or being near one in use." b. "It will be 1 month before I can take a bath or return to my usual activities." c. "I will notify the airlines when I make a reservation that I have a pacemaker." d. "I won't lift the arm on the pacemaker side up very high until I see the doctor."

D The patient is instructed to avoid lifting the arm on the pacemaker side above the shoulder to avoid displacing the pacemaker leads. The patient should notify airport security about the presence of a pacemaker before going through the metal detector, but there is no need to notify the airlines when making a reservation. Microwave oven use does not affect the pacemaker. The insertion procedure involves minor surgery that will have a short recovery period. DIF: Cognitive Level: Apply (application) REF: 805 TOP: Nursing Process: Evaluation MSC:

Physiological Integrity 2. The nurse needs to quickly estimate the heart rate for a patient with a regular heart rhythm. Which method will be best to use? a. Count the number of large squares in the R-R interval and divide by 300. b. Print a 1-minute electrocardiogram (ECG) strip and count the number of QRS complexes. c. Calculate the number of small squares between one QRS complex and the next and divide into 1500. d. Use the 3-second markers to count the number of QRS complexes in 6 seconds and multiply by 10.

D This is the quickest way to determine the ventricular rate for a patient with a regular rhythm. All the other methods are accurate, but take longer. DIF: Cognitive Level: Apply (application) REF: 789-790 TOP: Nursing Process: Assessment MSC:

When teaching the patient about the diet for diverticular disease, which foods should the nurse recommend? A) White bread, cheese, and green beans B) Fresh tomatoes, pears, and corn flakes C) Oranges, baked potatoes, and raw carrots D) Dried beans, All Bran (100%) cereal, and raspberries

D) A high fiber diet is recommended for diverticular disease. Dried beans, All Bran (100%) cereal, and raspberries all have higher amounts of fiber than white bread, cheese, green beans, fresh tomatoes, pears, corn flakes, oranges, baked potatoes, and raw carrots.

When planning care for a patient with cirrhosis, the nurse will give highest priority to which of the following nursing diagnoses? A. Imbalanced nutrition: less than body requirements B. Impaired skin integrity related to edema, ascites, and pruritus C. Excess fluid volume related to portal hypertension and hyperaldosteronism D. Ineffective breathing pattern related to pressure on diaphragm and reduced lung volume

D) Although all of these nursing diagnoses are appropriate and important in the care of a patient with cirrhosis, airway and breathing are always the highest priorities.

The nurse is performing a focused abdominal assessment of a patient who has been recently admitted. In order to palpate the patient's liver, where should the nurse palpate the patient's abdomen? A) Left lower quadrant B) Left upper quadrant C) Right lower quadrant D) Right upper quadrant

D) Although the left lobe of the liver is located in the left upper quadrant of the abdomen, the bulk of the liver is located in the right upper quadrant.

The nurse is preparing to administer a dose of bisacodyl (Dulcolax). In explaining the medication to the patient, the nurse would state that it acts in which of the following ways? A) Increases bulk in the stool B) Lubricates the intestinal tract to soften feces C) Increases fluid retention in the intestinal tract D) Increases peristalsis by stimulating nerves in the colon wall

D) Bisacodyl is a stimulant laxative that aids in producing a bowel movement by irritating the colon wall and stimulating enteric nerves. It is available in oral and suppository forms. Fiber and bulk forming drugs increase bulk in the stool; water and stool softeners soften feces, and saline and osmotic solutions cause fluid retention in the intestinal tract.

The nurse is preparing to administer a scheduled dose of docusate sodium (Colace) when the patient complains of an episode of loose stool and does not want to take the medication. Which of the following is the appropriate action by the nurse? A) Write an incident report about this untoward event. B) Attempt to have the family convince the patient to take the ordered dose. C) Withhold the medication at this time and try to administer it later in the day. D) Chart the dose as not given on the medical record and explain in the nursing progress notes.

D) Bisacodyl is a stimulant laxative that aids in producing a bowel movement by irritating the colon wall and stimulating enteric nerves. It is available in oral and suppository forms. Fiber and bulk forming drugs increase bulk in the stool; water and stool softeners soften feces, and saline and osmotic solutions cause fluid retention in the intestinal tract.

The patient with a history of lung cancer and hepatitis C has developed liver failure and is considering liver transplantation. After the comprehensive evaluation, the nurse knows that which factor discovered may be a contraindication for liver transplantation? A) Has completed a college education B) Has been able to stop smoking cigarettes C) Has well-controlled type 1 diabetes mellitus D) The chest x-ray showed another lung cancer lesion.

D) Contraindications for liver transplant include severe extrahepatic disease, advanced hepatocellular carcinoma or other cancer, ongoing drug and/or alcohol abuse, and the inability to comprehend or comply with the rigorous post-transplant course.

What should the nurse instruct the patient to do to best enhance the effectiveness of a daily dose of docusate sodium (Colace)? A) Take a dose of mineral oil at the same time. B) Add extra salt to food on at least one meal tray. C) Ensure dietary intake of 10 g of fiber each day. D) Take each dose with a full glass of water or other liquid.

D) Docusate lowers the surface tension of stool, permitting water and fats to penetrate and soften the stool for easier passage. The patient should take the dose with a full glass of water and should increase overall fluid intake, if able, to enhance effectiveness of the medication. Dietary fiber intake should be a minimum of 20 g daily to prevent constipation. Mineral oil and extra salt are not recommended.

The nurse is caring for a 68-year-old patient admitted with abdominal pain, nausea, and vomiting. The patient has an abdominal mass, and a bowel obstruction is suspected. The nurse auscultating the abdomen listens for which type of bowel sounds that are consistent with the patient's clinical picture? A) Low-pitched and rumbling above the area of obstruction B) High-pitched and hypoactive below the area of obstruction C) Low-pitched and hyperactive below the area of obstruction D) High-pitched and hyperactive above the area of obstruction

D) Early in intestinal obstruction, the patient's bowel sounds are hyperactive and high-pitched, sometimes referred to as "tinkling" above the level of the obstruction. This occurs because peristaltic action increases to "push past" the area of obstruction. As the obstruction becomes complete, bowel sounds decrease and finally become absent.

The nurse is caring for a postoperative patient with a colostomy. The nurse is preparing to administer a dose of famotidine (Pepcid) when the patient asks why the medication was ordered since the patient does not have a history of heartburn or gastroesophageal reflux disease (GERD). What response by the nurse would be the most appropriate? A) "This will prevent air from accumulating in the stomach, causing gas pains." B) "This will prevent the heartburn that occurs as a side effect of general anesthesia." C) "The stress of surgery is likely to cause stomach bleeding if you do not receive it." D) "This will reduce the amount of HCl in the stomach until the nasogastric tube is removed and you can eat a regular diet again."

D) Famotidine is an H2-receptor antagonist that inhibits gastric HCl secretion and thus minimizes damage to gastric mucosa while the patient is not eating a regular diet after surgery. Famotidine does not prevent air from accumulating in the stomach or stop the stomach from bleeding. Heartburn is not a side effect of general anesthesia.

The health care provider orders lactulose for a patient with hepatic encephalopathy. The nurse will monitor for effectiveness of this medication for this patient by assessing which of the following? A. Relief of constipation B. Relief of abdominal pain C. Decreased liver enzymes D. Decreased ammonia levels

D) Hepatic encephalopathy is a complication of liver disease and is associated with elevated serum ammonia levels. Lactulose traps ammonia in the intestinal tract. Its laxative effect then expels the ammonia from the colon, resulting in decreased serum ammonia levels and correction of hepatic encephalopathy.

The family of a patient newly diagnosed with hepatitis A asks the nurse what they can do to prevent becoming ill themselves. Which of the following responses by the nurse is most appropriate? A. "The hepatitis vaccine will provide immunity from this exposure and future exposures." B. "I am afraid there is nothing you can do since the patient was infectious before admission." C. "You will need to be tested first to make sure you don't have the virus before we can treat you." D. "An injection of immunoglobulin will need to be given to prevent or minimize the effects from this exposure."

D) Immunoglobulin provides temporary (1-2 months) passive immunity and is effective for preventing hepatitis A if given within 2 weeks after exposure. It may not prevent infection in all persons, but it will at least modify the illness to a subclinical infection. The hepatitis vaccine is only used for preexposure prophylaxis

The patient with suspected pancreatic cancer is having many diagnostic studies done. Which one can be used to establish the diagnosis of pancreatic adenocarcinoma and for monitoring the response to treatment? A) Spiral CT scan B) A PET/CT scan C) Abdominal ultrasound D) Cancer-associated antigen 19-9

D) The cancer-associated antigen 19-9 (CA 19-9) is the tumor marker used for the diagnosis of pancreatic adenocarcinoma and for monitoring the response to treatment. Although a spiral CT scan may be the initial study done and provides information on metastasis and vascular involvement, this test and the PET/CT scan or abdominal ultrasound do not provide additional information.

When teaching the patient with acute hepatitis C (HCV), the patient demonstrates understanding when the patient makes which statement? A) "I will use care when kissing my wife to prevent giving it to her." B) "I will need to take adofevir (Hepsera) to prevent chronic HCV." C) "Now that I have had HCV, I will have immunity and not get it again." D) "I will need to be checked for chronic HCV and other liver problems."

D) The majority of patients who acquire HCV usually develop chronic infection, which may lead to cirrhosis or liver cancer. HCV is not transmitted via saliva, but percutaneously and via high-risk sexual activity exposure. The treatment for acute viral hepatitis focuses on resting the body and adequate nutrition for liver regeneration. Adofevir (Hepsera) is taken for severe hepatitis B (HBV) with liver failure. Chronic HCV is treated with pegylated interferon with ribavirin. Immunity with HCV does not occur as it does with HAV and HBV, so the patient may be reinfected with another type of HCV.

The nurse asks a 68-year-old patient scheduled for colectomy to sign the operative permit as directed in the physician's preoperative orders. The patient states that the physician has not really explained very well what is involved in the surgical procedure. What is the most appropriate action by the nurse? A) Ask family members whether they have discussed the surgical procedure with the physician. B) Have the patient sign the form and state the physician will visit to explain the procedure before surgery. C) Explain the planned surgical procedure as well as possible and have the patient sign the consent form. D) Delay the patient's signature on the consent and notify the physician about the conversation with the patient.

D) The patient should not be asked to sign a consent form unless the procedure has been explained to the satisfaction of the patient. The nurse should notify the physician, who has the responsibility for obtaining consent.

The nurse is preparing to administer a scheduled dose of docusate sodium (Colace) when the patient reports an episode of loose stool and does not want to take the medication. What is the appropriate action by the nurse? A) Write an incident report about this untoward event. B) Attempt to have the family convince the patient to take the ordered dose. C) Withhold the medication at this time and try to administer it later in the day. D) Chart the dose as not given on the medical record and explain in the nursing progress notes.

D) Whenever a patient refuses medication, the dose should be charted as not given with an explanation of the reason documented in the nursing progress notes. In this instance, the refusal indicates good judgment by the patient, and the patient should not be encouraged to take it today.

The patient is being dismissed from the hospital after acute coronary syndrome and will be attending rehabilitation. What information would be taught in the early recovery phase of rehabilitation? A. Therapeutic lifestyle changes should become lifelong habits. B. Physical activity is always started in the hospital and continued at home. C. Attention will focus on management of chest pain, anxiety, dysrhythmias, and other complications. D. Activity level is gradually increased under cardiac rehabilitation team supervision and with electrocardiographic (ECG) monitoring.

D. In the early recovery phase after the patient is dismissed from the hospital, the activity level is gradually increased under supervision and with ECG monitoring. The late recovery phase includes therapeutic lifestyle changes that become lifelong habits. In the first phase of recovery, activity is dependent on the severity of the angina or myocardial infarction, and attention is focused on the management of chest pain, anxiety, dysrhythmias, and other complications. With early recovery phase, the cardiac rehabilitation team may suggest that physical activity be initiated at home, but this is not always done.

For which problem is percutaneous coronary intervention (PCI) most clearly indicated? A. Chronic stable angina B. Left-sided heart failure C. Coronary artery disease D. Acute myocardial infarction

D. PCI is indicated to restore coronary perfusion in cases of myocardial infarction. Chronic stable angina and coronary artery disease are normally treated with more conservative measures initially. PCI is not relevant to the pathophysiology of heart failure.

The patient reports tenderness when she touches her leg over a vein. The nurse assesses warmth and a palpable cord in the area. The nurse knows the patient needs treatment to prevent which sequela? A. Pulmonary embolism B. Pulmonary hypertension C. Post-thrombotic syndrome D. Venous thromboembolism

D. The clinical manifestations are characteristic of a superficial vein thrombosis. If untreated, the clot may extend to deeper veins, and venous thromboembolism may occur. Pulmonary embolism, pulmonary hypertension, and postthrombotic syndrome are the sequelae of venous thromboembolism.

The nurse is preparing to administer a scheduled dose of enoxaparin (Lovenox) 30 mg subcutaneously. What should the nurse do to administer this medication correctly? A. Remove the air bubble in the prefilled syringe. B. Aspirate before injection to prevent IV administration. C. Rub the injection site after administration to enhance absorption. D. Pinch the skin between the thumb and forefinger before inserting the needle.

D. The nurse should gather together or "bunch up" the skin between the thumb and the forefinger before inserting the needle into the subcutaneous tissue. The nurse should not remove the air bubble in the prefilled syringe, aspirate, nor rub the site after injection.

A 39-yr-old woman with a history of smoking and oral contraceptive use is admitted with a venous thromboembolism (VTE) and prescribed unfractionated heparin. What laboratory test should the nurse review to evaluate the expected effect of the heparin? A. Platelet count B. Activated clotting time (ACT) C. International normalized ratio (INR) D. Activated partial thromboplastin time (APTT)

D. Unfractionated heparin can be given by continuous IV for VTE treatment. When given IV, heparin requires frequent laboratory monitoring of clotting status as measured by activated partial thromboplastin time (aPTT). Platelet counts can decrease as an adverse reaction to heparin.

After having a myocardial infarction (MI), the nurse notes the patient has jugular venous distention, gained weight, developed peripheral edema, and has a heart rate of 108 beats/min. What should the nurse suspect is happening? A. ADHF B. Chronic HF C. Left-sided HF D. Right-sided HF Correct

D. An MI is a primary cause of heart failure. The jugular venous distention, weight gain, peripheral edema, and increased heart rate are manifestations of right-sided heart failure.

A 44-yr-old man is diagnosed with hypertension and receives a prescription for benazepril (Lotensin). After providing instruction, which statement by the patient indicates correct understanding? A. "If I take this medication, I will not need to follow a special diet." B. "It is normal to have some swelling in my face while taking this medication." C. "I will need to eat foods such as bananas and potatoes that are high in potassium." D. "If I develop a dry cough while taking this medication, I should notify my doctor."

D. Benazepril is an angiotensin-converting enzyme inhibitor. The medication inhibits breakdown of bradykinin, which may cause a dry, hacking cough. Other adverse effects include hyperkalemia. Swelling in the face could indicate angioedema and should be reported immediately to the prescriber. Patients taking drug therapy for hypertension should also attempt lifestyle modifications to lower blood pressure such as a reduced-sodium diet.

A patient is scheduled for a heart transplant. Beyond the first year after a heart transplant, the nurse knows that what is a major cause of death? A. Infection B. Acute rejection C. Immunosuppression D. Cardiac vasculopathy

D. Beyond the first year after a heart transplant, malignancy (especially lymphoma) and cardiac vasculopathy (accelerated coronary artery disease) are the major causes of death. During the first year after transplant, infection and acute rejection are the major causes of death. Immunosuppressive therapy will be used for posttransplant management to prevent rejection and increases the patient's risk of an infection.

When evaluating a patient's knowledge regarding a low-sodium, low-fat cardiac diet, the nurse recognizes additional teaching is needed when the patient selects which food? A. Baked flounder B. Angel food cake C. Baked potato with margarine D. Canned chicken noodle soup

D. Canned soups are very high in sodium content. Patients need to be taught to read food labels for sodium and fat content.

A patient admitted to the emergency department 24 hours ago with complaints of chest pain was diagnosed with a ST-segment-elevation myocardial infarction (STEMI). What complication of myocardial infarction should the nurse anticipate? A. Unstable angina B. Cardiac tamponade C. Sudden cardiac death D. Cardiac dysrhythmias

D. Dysrhythmias are present in 80% to 90% of patients after myocardial infarction (MI). Unstable angina is considered a precursor to MI rather than a complication. Cardiac tamponade is a rare event, and sudden cardiac death is defined as an unexpected death from cardiac causes. Cardiac dysfunction in the period following an MI would not be characterized as sudden cardiac death.

A 67-yr-old woman with hypertension is admitted to the emergency department with a blood pressure of 234/148 mm Hg and was started on nitroprusside (Nitropress). After one hour of treatment, the mean arterial blood pressure (MAP) is 55 mm Hg. Which nursing action is a priority? A. Start an infusion of 0.9% normal saline at 100 mL/hr. B. Maintain the current administration rate of the nitroprusside. C. Request insertion of an arterial line for accurate blood pressure monitoring. D. Stop the nitroprusside infusion and assess the patient for potential complications.

D. Nitroprusside is a potent vasodilator medication. A blood pressure of 234/118 mm Hg would have a calculated MAP of 177 mm Hg. Subtracting 25% (or 44 mm Hg) = 133 mm Hg. The initial treatment goal is to decrease MAP by no more than 25% within minutes to 1 hour. For this patient, the goal MAP would be approximately 133 mm Hg. Minimal MAP required to perfuse organs is around 60 to 65 mm Hg. Lowering the blood pressure too rapidly may decrease cerebral, coronary, or renal perfusion and could precipitate a stroke, myocardial infarction, or renal failure. The priority is to stop the nitroprusside infusion and then use fluids only if necessary to support restoration of MAP.

An asymptomatic patient with acute decompensated heart failure (ADHF) suddenly becomes dyspneic. Before dangling the patient on the bedside, what should the nurse assess first? A. Urine output B. Heart rhythm C. Breath sounds D. Blood pressure

D. The nurse should evaluate the blood pressure before dangling the patient on the bedside because the blood pressure can decrease as blood pools in the periphery and preload decreases. If the patient's blood pressure is low or marginal, the nurse should put the patient in the semi-Fowler's position and use other measures to improve gas exchange.

The nurse instructs a 68-yr-old woman with hypercholesterolemia about natural lipid-lowering therapies. The nurse determines further teaching is necessary if the patient makes which statement? A. "Omega-3 fatty acids are helpful in reducing triglyceride levels." B. "I should check with my physician before I start taking any herbal products." C. "Herbal products do not go through as extensive testing as prescription drugs do." D. "I will take garlic instead of my prescription medication to reduce my cholesterol."

D. Current evidence does not support using garlic in the treatment of elevated cholesterol. Strong evidence supports the use of omega-3 fatty acids for reduction of triglyceride levels. Many herbal products are not standardized and effects are not predictable. Patients should consult with their health care provider before starting herbal or natural therapies.

The nurse teaches a client diagnosed with neutropenia about the condition. Which client statement indicates to the nurse an understanding of the instructions? A. "I will use a soft toothbrush" B. "I have to wear a mask at all times" C. "My grandchildren may get an infection from me" D. "I will call my health provider if I develop a fever"

D. "I will call my health provider if I develop a fever" 1) INCORRECT - A soft toothbrush is indicated for the client with thrombocytopenia, but it is not necessary for the client with neutropenia. 2) INCORRECT - There is no clinical indication for the client with neutropenia to wear a mask at all times. A mask is only necessary if the client is going to be in crowded places. It is best for the client to avoid crowded places altogether. 3) INCORRECT - The client is not at increased risk of giving an infection to someone. The client is at risk of acquiring an infection from other persons, raw fruits and vegetables, and unclean surfaces. The client must wash hands frequently. 4) CORRECT - With neutropenia, the client has an increased risk for acquiring infections. Therefore, the client should notify the health care provider immediately if an increased temperature or chills occur.

Long term tx depends on the cause of the injury, could be a head injury or a tumor. Immediate tx is fluid and electrolyte imbalance, avoid caffeine and other natural diuretics Skin tugor is very slow due to dehydration, encourage fluids, restrict foods that promote natural diuresis (caffeine and lemons) Medication Stimate: ADH replacement medication. If you give too much of this it can cause SIADH Desmopression (DDAVP) or Vasmopressin ->Synthetic pituitary hormone, Increases reabsorption of water by increasing permeability of collecting ducts in kidney

DI: treatment

Treating the underlying cause, and managing blood loss or blood clots are important for the nurse. The nurse should assess for signs of both internal and external bleeding Prevent further bleeding or thrombosis development, fresh frozen plasma and platelet transfusion, Heparin IV Stabilize quickly and increase the volume: get oxygen on, get fluids hanging, and hang blood. If we know the source of the injury, then we want to stop it

DIC: priority nursing interventions?

This is a life-threatening clotting disorder. This is always caused by an underlying disease process or event, which causes the body to have an abnormal clotting response. At first, accelerated clotting occurs (thrombosis) but then all the clotting factors are used up and the blood loses its ability to clot, and massive hemorrage can occur. It is important to diagnose DIC quickly, stabilize the patient if needed (e.g., oxygenation, volume replacement), institute therapy that will resolve the underlying causative disease or problem, and provide supportive care for the manifestations resulting from the pathology of DIC itself.

DIC: treatment

Is caused by a profound deficiency of insulin and is characterized by hyperglycemia, ketosis, acidosis, and dehydration It is most likely to occur in people with type 1 diabetes by may be seen in people with type 2 diabetes in conditions of severe illness stress when the pancreas cannot meet the extra demand for insulin.

DKA

When the nurse is evaluating the fluid balance for a patient admitted for hypovolemia associated with multiple draining wounds, the most accurate assessment to include is 1. skin turgor. 2. presence of edema. 3. daily weight. 4. hourly urine output.

Daily weight

• Nicotine • Fatty foods • Morphine • Alcohol • Valium • Beta Blockers • Caffeine • Ca+Channel Blockers • Nitrates

Decreased Lower Esophageal Sphincter Pressure (LES) Risks of GERD:

Type 2 differs from type 1 primarily in that:

Decreased insulin secretion and cell resistance to insulin

Thin, bulging eyes, thin hair, weight loss, tachycardia, nervous tremors, labile (extra) emotions

Defining characteristics of Hyperthyroidism:

Slow metabolism, cold, weight gain, sleepy, lethargy, mental change

Defining characteristics of Hypothyroidism:

A higher than normal hematocrit can indicate:

Dehydration A disorder, such as polycythemia vera, that causes your body to produce too many red blood cells

Tx for hyponatremia

Depends on cause of low sodium. If the patient is dehydrated, slow replacement of sodium with adequate fluid intake is the easiest method. Slow treatment is necessary as rapid correction of serum sodium can precipitate severe neurological complications. If it is due to the body retaining water, treatment would be restricting the intake of water If it is due to fluid overload, then diuretics will be used to remove excess water.

Which factor should be considered when caring for a woman with suspected coronary artery disease? A. Fatigue may be the first symptom. B. Classic signs and symptoms are expected. D. Increased risk is present before menopause. E. Women are more likely to develop collateral circulation.

Fatigue, rather than pain or shortness of breath, may be the first symptom of impaired cardiac circulation. Women may not exhibit the classic signs and symptoms of ischemia such as chest pain which radiates down the left arm. Neck, throat, or back pain may be symptoms experienced by women. Risk for coronary artery disease increases four times after menopause. Men are more likely to develop collateral circulation.

Vincent's Infection etiology

Fusiform bacteria. Vincent spirochetes; predisposing factors of stress, excessive fatigue, poor oral hygiene, nutritional deficiencies (B and C vitamins)

Complications that may occurs with hiatal hernia include

GERD, esophagitis, hemorrhage from erosion, stenosis (narrowing of the esophagus), ulcerations of the herniated portion of the stomach, strangulation of the hernia, and reguritation with tracheal aspiration

Glucose is stored in the liver, but how does it get there? What makes that possible?

Glucose is a large molecule that cannot diffuse freely across the plasma membrane; it requires a process termed facilitated diffusion, which is an insulin-supported process that occurs at the cell's plasma membrane. -Insulin is produced by beta cells in the islets of Langerhans; specialized tissue within the pancreas. -Glucose absorbed into bloodstream through the intestines -Rise in blood glucose causes pancreas to release insulin -Insulin binds to cell surface membrane receptors, main cells permeable to glucose, causing rapid uptake, storage, and use of glucose by cells of body, particularly the liver. -Energy production, stored as glycogen, or converted into fat -Liver and muscle cells store the largest amount of glycogen

A patient who has been hospitalized for 2 days has been receiving normal saline IV at 100 ml/hr, has a nasogastric tube to low suction, and is NPO. Which assessment finding by the nurse is the priority to report to the health care provider? 1. Serum sodium level of 138 mEq/L (138 mmol/L) 2. Weight gain of 2 pounds (1 kg) above the admission weight 3. Gradually decreasing level of consciousness (LOC) 4. Oral temperature of 100.1° F with bibasilar lung crackles

Gradually decreasing level of consciousness (LOC)

Positioning for Enteral nutrition:

HOB needs to be placed at 30-45 degrees when feeding to prevent aspiration

What is the most common clinical manifestation of GERD?

Heartburn (pyrosis)

Myeloblasts

Hematopoietic cell found in bone marrow that granulocytes develop from.

is the term used for anemia that results from the body destroying RBC's at a faster rate than they can be produced.

Hemolytic anemia

Fatigue, weakness, pale or yellowish skin, facial bone deformities, slow growth, abdominal swelling, and dark urine

Hemolytic anemia: clinical manifestations

The primary organ responsible for destruction of old, defective, or damaged RBC's is the spleen. The patient will often present with jaundice due to an elevation of bilirubin, and enlarged spleen and liver from overuse of cleaning up the destroyed RBC.

Hemolytic anemia: effected organs

Protecting the kidneys is a major part of treatment as they must filter out all the extra byproducts. During a hemolytic crisis aggressive hydration will be needed, and it may even be necessary to remove the spleen, a procedure called a splenectomy.

Hemolytic anemia: treatments.

High iodine foods because the thyroid uses the iodine(salt) to produce the thyroid hormone, encourage a low calorie, low cholesterol, high fiber diet

Hypothyroidism: any specific foods?

Five types of shock are:

Hypovolemic, cardiogenic, neurogenic, anaphylactic, and septic

A nurse notes that the FEV1/FVC ratio is less than 70% for a patient with COPD. The nurse documents that the patient is in stage: a. 0. b. I. c. II. d. III

I

What causes anemia

Impaired erythrocyte production, acute or chronic blood loss, increased erythrocyte destruction or a combination of all of these

The most common nutritional disorder in the world. Those most susceptible to this anemia are the very young, those on poor diets, and women in their reproductive years.

Iron deficiency anemia

Three types of nutritional anemias:

Iron deficiency anemia, vitamin B12 deficiency, and folic acid anemia

Common causes of chronic abdominal pain include

Irritable bowel syndrome (IBS), peptic ulcer disease, chronic pancreatitis, hepatitis, pelvic inflammatory disease, and vascular insufficiency

GERD (gastroesophageal reflux disease)

Is a chronic symptom of mucosal damage caused by reflux of acidic gastric contents into the esophagus. Gastric HCl acid and pepsin secretions in refluxate cause esophageal irritation and inflammation.

Irritable bowel syndrome

Is a disorder characterized by chronic abdominal pain or discomfort and alteration of bowel patterns. Diarrhea or constipation may predominate, or they may alternate. As a functional GI disorder, this has no known organic cause. NOT inflammatory like IBD

BMI

Is a scale used to measure obesity

Hyponatremia

Is a sodium serum level of less that 135. When dehydration occurs because the body has lost sodium and fluid together, it is known as hypovolemic hyponatremia, its cause is either renal or nonrenal. It can also occur in the presence of excess water.

Constipation

Is a syndrome defined by difficult or infrequent stools; hard, dry stools that are difficult to pass; or a feeling of incomplete evacuation

HGb level of 6.5 reveals the patient as:

Lethargic pale and irritable

In teaching a patient with hypertension about controlling the illness, the nurse recognizes that

Lifestyle modifications are indicated for all persons with elevated BP Lifestyle modifications are indicated for all patients with prehypertension and hypertension.

Herpes Simplex clinical manifestation

Lip lesions, mouth lesions, vesicle formation shallow, painful ulcers

You are caring for an older patient who is receiving IV fluids postoperatively. During the 8:00 AM assessment of this patient, you note that the IV solution, which was ordered to infuse at 125 mL/hr, has infused 950 mL since it was hung at 4:00 AM. What is the priority nursing intervention?

Listen to the patient's lung sounds and assess respiratory status.

Epitaxis (Nose Bleeds):Nursing & Collaborative Care- second treatment

Local vasoconstrictive agent (Cocaine) Cauterization or anterior packing Posterior Packing Pain management Respiratory & cardiac assessment Prophylactic antibiotics

Glargine (Lantus) Determir (Levermir) Degludec (Tresiba)

Long acting insulin

Long term use of corticosteroids

Long-term use of corticosteroids can cause adrenal gland atrophy, and decreased secretion of natural cortisol. When patients are administered prolonged corticosteroid treatment beyond 4 to 5 weeks, there is a negative feedback suppression of CRH and ACTH. As a result, the adrenal gland can down-regulate its receptors and undergo glandular atrophy. With atrophy, there is a decreased ability to secrete natural cortisol. The patient will then develop symptoms of adrenal insufficiency or potentially adrenal crisis. Symptoms are loss of weight, weakness, severe hypotension, crisis: life-threatening, severe abdominal pain, fever, and shock.

The nurse is caring for a 76-year-old woman admitted to the clinical unit with hypernatremia and dehydration after prolonged fever. Which beverage would be safest for the nurse to offer the patient?

Malted milk Orange juice (Correct) Tomato juice Hot chocolate Orange juice has the least amount of sodium (approximately 2 mg in 8 ounces). Hot chocolate has approximately 75 mg sodium in 8 ounces. Tomato juice has approximately 650 mg sodium in 8 ounces. Malted milk has approximately 625 mg sodium in 8 ounces.

Pancytopenia

Marked decrease in RBCs, WBCs & platelets.

Clotting factors in aging

May be elevated

MCHC changes with aging

May be slightly decreased

MCV changes with aging

May be slightly increased

Reticulocyte count

Measurement of immature RBCs, a reflection of bone marrow activity in producing RBCs. WNL - 0.5 to 1.5%

A patient with newly discovered high BP has an average reading of 158/98 mmHg after 3 months of exercise and diet modifications. Which management strategy will be a priority for this patient?

Medication will be required because the BP is still not at goal The patient has hypertension, stage 1. Lifestyle modifications will continue, but drug initiation of therapy is a priority. Reduction of BP can help to prevent serious complications related to hypertension.

Cerumen Impaction: How to relieve? What is not a safe way to clear ear of excess cerumen?

Medications Irrigation not safe= Candle waxing

When assessing a patient with increased extracellular fluid (ECF) osmolality, the priority assessment for the nurse to obtain is 1. capillary refill. 2. mental status. 3. heart sounds. 4. skin turgor.

Mental status

You are admitting a patient with complaints of abdominal pain, nausea, and vomiting. A bowel obstruction is suspected. You assess this patient for which anticipated primary acid-base imbalance if the obstruction is high in the intestine?

Metabolic alkalosis Because gastric secretions are rich in hydrochloric acid, the patient who is vomiting will lose a significant amount of gastric acid and be at an increased risk for metabolic alkalosis.

-Increased waist circumference (Obesity) -Elevated triglycerides -Decreased HDL cholesterol -Elevated Blood Pressure (HTN) -Elevated Fasting blood glucose (plasma glucose)

Metabolic syndrome Criteria:

Oral candidiasis treatment

Miconazole buccal tablets (oravig), nystatin or amphoterician B as oral suspension or buccal tablets. Good oral hygiene

The auscultatory area in the left midclavicular line at the level of the fifth ICS is the best location to hear sounds from which heart valve?

Mitral The mitral valve can be assessed by auscultation at the left midclavicular line at the fifth intercostal space (ICS).

Which nursing intervention is most appropriate when caring for a patient with dehydration?

Monitor daily weight and intake and output.

A key aspect of teaching for the patient on anticoagulant therapy includes which instructions?

Monitor for and report any signs of bleeding Patients taking anticoagulants should be taught to monitor and report any signs of bleeding, which can be a serious complication. Other important patient teaching includes maintenance of a consistent intake of foods containing vitamin K and avoidance of supplements that contain vitamin K. If a patient is taking warfarin, routine coagulation laboratory studies are necessary, although frequency is patient dependent, not necessarily twice a week.

Monitoring of PN:

Monitoring electrolytes, because these commercially prepared solutions have dextrose in them: we need to monitor blood glucose levels, urea nitrogen (BUN), CMP, CBC, BMP, and hepatic enzymes a minimum of three times a week

Respiratory acidosis, respiratory depression, hypotension, and hypothermia

Myxedema Coma S/S

Uncontrolled hypothyroidism can lead to this. It is characterized by subnormal temperature, hypotension, and hypoventilation. Cardiovascular collapse can result from hypoventilation, hyponatremia, hypoglycemia, and lactic acidosis.

Myxedema Coma: Indications of progression to life-threatening disease.

The mental sluggishness, drowsiness, and lethargy of hypothyroidism may progress gradually or suddenly to a notable impairment of consciousness or coma. This situation is a medical emergency that can be precipitated by infection, drugs, exposure to cold, and trauma.

Myxedma Coma

What can inhibit the synthesis of prostaglandins that are protective to the gastric mucosa?

NSAIDS, including aspirin, and corticosteroids, this makes the mucosa more susceptible to injury

These types of anemia result in a decreased production of RBC.

Nutritional anemia

What is SaO2

O2 saturation of hemoglobin

Two key risk factors that contribute to type 2 diabetes:

Obesity and sedentary lifestyle

When discussing the administration of nasal spray with a patient. What information is most important to include. a. Finish the bottle of nasal spray to clear the infection effectively b. Nasal spray can be shared between family members c. Administer the nasal spray in a prone position d. Overuse of nasal spray may cause rebound congestion.

Overuse of nasal spray may cause rebound congestion.

Parathyroid can be damaged in the procedure and can result in hypocalcemia (monitor phosphorus too, in that reciprocal relationship), which will cause tetany, Chvostek sign, claw (Trousseau), numbers and tingling. Replace calcium, monitoring calcium levels very closely. Sometimes put on a calcium supplement right away, or administering IV calcium gluconate if needed to replace those calcium levels.

Potential effects on parathyroid from a thyroidectomy & post-op nursing management

SaO2 is measured by what?

Pulse ox

Clinical manifestations of Cushing Syndrome

Purpura, muscle weakness, osteoporosis, amenorrhea, cardiac hypertrophy, hypertension, thin, wrinkled skin, fat pads on shoulders (buffalo hump)

Epitaxis (Nose Bleeds):Nursing & Collaborative Care- first treatment

Quiet Upright Positon Pressure 10-15 min Ice

MCHC - Mean Corpuscular hemoglobin

RBC saturation to Hgb. (Hgbx100)/Hct. Decreased = hypochromia Increased = scherocytosis

Hypochromia

RBCs have less color than normal when examined under a microscope

Assessment: Upper Respiratory Tract- Physical Assessment

RR, SOB (working hard), cyanosis, mouth, nose, larynx

Lispro (Humalog) Aspart (Novolog) Glulisine (Apidra)

Rapid acting insulin

Macrocytosis (macrocythemia)

Red blood cells that are larger than normal.

Only insulin drip used IV:

Regular Insulin

Thyroid—what does it do?

Regulates our body temperature, brain function, and heart rate Metabolism Growth development Burns calories Cell replacement How fast food digests Stimulates SNS

Type 1 with sepsis related to UTI has altered awareness and dehydration:

Related to HHS

Pharyngitis: Chronic

Related to exposure Symptomatic treatment Tonsillectomy optional

tx of DIC

Removing stimulus and to maintain hemodynamic status

The release of ADH despite normal or low plasma osmolarity. ADH increase the permeability of the renal distal tubule and collecting duct, which leads to the reabsorption of water into the circulation. Thus the disorder is characterized by fluid retention, serum hypoosmolality, dilution hyponatremia, hypochloremia, and concentrated urine in the presence of normal or increased intravascular volume.

SIADH

S/S of a hypoxic state, hypoxia r/t high altitude, lung disease, cardiopulmonary disease, defective oxygen transport, erythropoietin production stimulated

Secondary Polycythemia: clinical manifestations

Look for underlying diseases that are going on to treat for, why are the oxygen deprived. The blood volume increases and becomes more viscous, so removal of300-500mL off blood (phlebotomy) until normal Hct, hydration therapy, myelosuppressive agents, adequate oxygen and comfort measures are used for treatment.

Secondary Polycythemia: treatment.

A patient with advanced lung cancer is admitted to the emergency department with urinary retention caused by renal calculi. Which of these laboratory values will require the most immediate action by the nurse? 1. Serum calcium is 18 mEq/L. 2. Arterial blood pH is 7.32. 3. Arterial oxygen saturation is 91%. 4. Serum potassium is 5.1 mEq/L.

Serum calcium is 18 mEq/L.

Herpes Simplex treatment

Spirits of camphor, corticosteroid cream, mild antiseptic mouthwash, viscous lidocaine; removal or control of preidsposing factors, antiviral agents

A 50-year-old woman with hypertension has a serum potassium level that has acutely risen to 6.2 mEq/L. Which type of order, if written by the health care provider, should be questioned by the nurse?

Spironolactone (Aldactone) daily Spironolactone (Aldactone) is a potassium-sparing diuretic that inhibits the exchange of sodium for potassium in the distal renal tubule and helps to prevent potassium loss.

The recommended treatment for an initial VTE in an otherwise healthy person with no significant co-morbidities would include

Subcutaneous low-molecular-weight heparin as an outpatient Patients with confirmed VTE should receive initial treatment with low-molecular-weight heparin (LMWH), unfractionated heparin (UH), fondaparinux, or rivaroxaban, followed by warfarin for 3 months to maintain the international normalized ratio (INR) between 2.0 and 3.0 for 24 hours. Patients with multiple comorbid conditions, complex medical issues, or a very large VTE usually are hospitalized for treatment and typically receive intravenous UH. LMWH only for 3 months is another option for patients with acute VTE. Depending on the clinical presentation, patients often can be managed safely and effectively as outpatients.

The nurse is caring for a patient who has just returned to the unit after abdominal surgery. The nurse understands which of the following interventions should be included in the immediate post-operative plan of care? Early ambulation Oral analgesics Supplemental Oxygen Resumption of pre-operative diet

Supplemental O2

4.ID: 14478139862 A patient with a long-standing history of heart failure recently qualified for hospice care. What measure should the nurse now prioritize when providing care for this patient? A. Taper the patient off his current medications. B. Continue education for the patient and his family. C. Pursue experimental therapies or surgical options. D. Choose interventions to promote comfort and prevent suffering.

The central focus of hospice care is the promotion of comfort and the prevention of suffering. Patient education should continue, but providing comfort is paramount. Medications should be continued unless they are not tolerated. Experimental therapies and surgeries are not commonly used in the care of hospice patients.

What are the four diagnostic criteria for diagnosing diabetes (1124)?

The diagnosis of DM is made using one of the four following methods: 1. Fasting plasma glucose (FBS) > 126 mg/dl 2. 2 hr postprandial plasma glucose > 200 mg/dl 3. Random plasma glucose (RPG, RBS) > 200 mg/dl 4. Glycosylated hemoglobin (Hgb A1c) > 6.5%

You are caring for a patient admitted with diabetes mellitus, malnutrition, and massive GI bleed. In analyzing the morning lab results, the nurse understands that a potassium level of 5.5 mEq/L could be caused by which factors in this patient

The potassium level may be increased if the patient has renal nephropathy. The potassium level may be increased as a result of dehydration that accompanies high blood glucose levels. There may be excess potassium being released into the blood as a result of massive transfusion of stored hemolyzed blood.

Increase in fever, HR, BP and hypermetabolism. Can be so severe that we can see seizures and delirium

Thyroid Storm: Clinical Manifestations

B- bleeding O- open airway W- whisper T- trach set I- incision E- emergency

Thyroidectomy post-op nursing management

Immediate post-op priority is airway because it can be affected by swelling or bleeding. Monitor for patent airway, if the airway is swelling and narrowing, you can hear the stridor sound from this. Make sure a hematoma isn't developing under dressing. Use pillows to support the head, so the patient isn't hyperextending the incision and causing bleeding that will impact the airway. Prevent bleeding, hypocalcemia, place them in semi fowler's position. Calcium imbalance after: treat it with frequent turns and calcium gluconate

Thyroidectomy post-op nursing management

Laryngeal damage Hemorrhage Tissue swelling Hypocalcemia

Thyroidectomy. Complications from the surgery

Fluid restriction, hypertonic IV solution (3%) only with diuretic, neuro checks, medication: Declamyocin- inhibits ADH Monitoring sodium levels, sodium supplements, monitoring for fluid overload, and crackles in the lungs. Administering diuretics to help get some of the fluid off.

Tx of SIADH:

For the patient to survive this, vital functions must be supported and IV thyroid hormone replacement administered. Use Beta blockers to lower HR, and antipyretics to lower the fever, cooling blankets and cold fluids Managing respiratory distress, fever reduction, fluid replacement, elimination of initial stressor.

Tx of thyroid storm:

Auto immune process body's T cells attack and destroy pancreatic B-cells; could be caused by an autoimmune disease, virus, or due to genetics

Type 1 DM

No endogenous insulin is produced

Type 1 DM

Occurs rapidly

Type 1 DM

Patient will develop DKA w/o insulin which can lead to metabolic acidosis

Type 1 DM

formally known as juvenile diabetes or insulin dependent diabetes is an autoimmune disorder that eventually results in the pancreas not producing insulin.

Type 1 DM

Accounts for 90% of all cases, pancreas produces some insulin but it is either insufficient or poorly used by the tissues

Type 2 DM

Endogenous insulin is produced

Type 2 DM

All clinical manifestations relate to bleeding and bleeding events can be life-threatening. Replacement of deficient clotting factors is the primary treatment, and this is especially important before any surgical or dental procedure as a prophylactic measure. The patient with this disorder is usually managed in the home setting with careful teaching to recognize disease-related problems and learn which problems can be resolved at home and which require hospitalization.

Von Willebrand disease and Hemophilia: How do we prevent bleeding, or manage bleeding for these patients?

The primary symptom is pain. The pain severity can range from trivial to excruciating. During a crisis the pain is severe because of ischemia of tissue. The episodes can affect any area of the body or several sites simultaneously, with back, chest, extremities, and abdomen being most commonly affected. They are manifested by fever, swelling, tenderness, tachypnea, HTN, and N/V.

What is often their sickle cell anemia patient's chief complaint?

Elderly patients with heart failure, cerebral edema, hypertension, and coronary artery disease. -Risk factors for constipation are postpartum, opioid use, old and young age, diet lacking in fiber, sedentary lifestyle, not enough fluid intake

What patients are at risk for complication of constipation:

What maybe used for replacement of volume in massive hemorrhage?

Whole blood, packed RBCs and fresh frozen plasma

Nutrition Education for newly diagnosed DM

Whole grains/ DASH/ MyPlate diet Limit saturated and trans fats Exercise increases insulin sensitivity-> lowering glucose levels and use insulin better Carb counting

A hospitalized patient with a history of chronic stable angina tells the nurse that she is having chest pain. The nurse bases his actions on the knowledge that ischemia

Will be relieved by rest, nitroglycerin, or both Chronic stable angina is chest pain that occurs intermittently over a long period with the same pattern of onset, duration, and intensity of symptoms. The chest pain is relieved by rest or by rest and medication (e.g., nitroglycerin). The ischemia is transient and does not cause myocardial damage.

A patient with chronic HF and atrial fibrillation is treated with a digitalis glycoside and a loop diuretic. To prevent possible complications of this combination of drugs, what does the nurse need to do (select all that apply)? a. Monitor serum potassium levels b. teach the patient how to take a pulse rate. c. keep an accurate measure of intake and output d. Teach the patient about dietary restriction or potassium e. Withhold digitalis and notify health care provider if heart rate is irregular

a & b Rationale: Hypokalemia, which can be caused by the use of potassium-depleting diuretics (e.g., thiazides, loop diuretics), is one of the most common causes of digitalis toxicity. Low serum levels of potassium enhance the action of digitalis, causing a therapeutic dose to achieve toxic levels. Hypokalemia can also precipitate dysrhythmias. Monitoring the serum potassium levels of patients receiving digitalis preparations and potassium-depleting diuretics is essential. Patients taking digitalis preparations should be taught how to measure their pulse rate because bradycardia and atrioventricular blocks are late signs of digitalis toxicity. In addition, patients should know what pulse rate would necessitate a call to the health care provider.

Gastritis occurs as the result of

a breakdown in the normal gastric mucosal barrier

Inflammatory bowel disease (IBD)

a chronic inflammation of the GI tract. characterized by periods of remission interspersed with periods of exacerbation. exact cause is unknown, no cure

Gastroesophageal reflux disease (GERD)

a chronic symptom of mucosal damaged caused by reflux of stomach acid into the lower esophagus, an upper GI problem, no on single cause

metabolic syndrome is characterized by

a cluster of health problems, including obesity, hypertension, abnormal lipid levels, and high blood glucose

Hemoptysis, a symptom of cardiopulmonary disorders, is characterized by all of the following except: a. a coffee ground appearance. b. an alkaline pH. c. a sudden onset. d. bright red bleeding mixed with sputum.

a coffee ground appearance

Metabolic syndrome

a collection of risk factors that increase an individual's chance of developing cardiovascular disease, stroke, and diabetes mellitus

Qsymia

a combination of two drugs, phentermine and topiramate

Peptic ulcer disease (PUD)

a condition characterized by erosion of the GI mucosa resulting from the digestive action of HCl acid and pepsin

Cathartic colon syndrome

a condition in which the colon becomes dilated and atonic (lacking muscle tone). ultimatey, the person cannot defecate without a laxative

Von Willebrand disease: Which clotting factors are affected?

a deficiency in von Willebrand factor

Malnutrition

a deficit, excess, or imbalance of essential nutrients

Nausea

a feeling of discomfort in epigastrium with a conscious desire to vomit

projectile vomiting

a forceful expulsion of stomach contents without nausea and is characteristic of CNS (brain and spinal cord) tumors

EN may be ordered for the patient who has

a functioning GI tract but is unable to take any or enough oral nourishment, or when it is unsafe to do so

Ghrelin

a gut hormone, regulates appetite through inhibition of leptin

For patients with esophageal cancer what kind of diet is recommended?

a high-calorie, high protein, some patients require a liquid form of this diet. Some patients may need IV fluid replacement or parenteral nutrition

Angioedema as a risk factor that leads to laryngeal obstruction is usually caused by: a. heavy alcohol use. b. a history of airway problems. c. the presence of foreign body. d. the use of ACE inhibitors.

a history of airway problems.

The patient receiving PN must be able to tolerate

a large volume of fluid

Body mass index (BMI)

a measure of weight for height -a bmi of less than 18.5 kg/m is considered underweight -a bmi between 18.5-24.9 kg/m is normal weight -a bmi between 25-29 kg/m is over weight -a bmi 30 kg/m or greater is obese

Patients who have had Roux-en-Y- need to take

a multivitamin with iron and calcium supplements

Tardive dyskinesia

a neurologic condition characterized by involuntary and repetitive movements of the body (e.g. extremity movement, lip smacking)

as a consequence of a total gastrectomy

a patient experiences the symptoms of dumping syndrome.

Who should you consider at high risk of an upper GI bleed?

a patient with a history of chronic gastritis or PUD, patients with cirrhosis and GI bleeding from varices

Individuals with fat located primarily in the upper legs

a pear-shaped body, having gynoid obesity

A good candidate for liposuction is

a person who has achieved weight reduction but who has excess fat under the chin, along the jaw line, in the nasolabial folds, over the abdomen, or around the waist and upper thighs

People with EE frequently have

a personal or family history of other allergic disease

When large muscles are involved in the exercise program,

a primary benefit is cardiovascular conditioning

Transferrin,

a protein synthesized by the liver and used to transport iron, decreases when protein is deficient

Prealbumin

a protein synthesized by the liver, is a better indicator of recent of current nutritional status.

To prevent aspiration, put the patient who cannot adequately manage self-care in

a semi-flower's or side-lying position

The restrictive effect of the band creates

a sense of fullness as the upper portion of the stomach now accommodates less than the average stomach. The band then causes a delay in stomach emptying, providing patients with further satiety

Enteral feedings can be started within 24 to 48 hours after

a surgically placed gastrostomy or jejunostomy tube without waiting for flatus or a bowel movement

Before a patient can have oral fluids what is done?

a swallowing study

phentermine

a sympathomimetic agent already approved for short term management of obesity, it suppresses appetite

Postoperative abdominal distention seems to be directly related to: a. a temporary loss of peristalsis and gas accumulation in the intestines. b. beginning food intake in the immediate postoperative period. c. improper body positioning during the recovery period. d. the type of anesthetic administered

a temporary loss of peristalsis and gas accumulation in the intestines.

When the lesion is located in the fundus,

a total gastrectomy with esophagojejunostomy is performed

What is the initial form of treatment for EE?

a trial of avoidance of the foods to which the person has positive allergy tests

A physician's admitting note lists a wound as healing by second intention. The nurse expects to see: a. a deep, open wound that was previously sutured. b. a sutured incision with a little tissue reaction. c. a wound with a deep, wide scar that was previously resutured. d. a wound in which the edges were not approximated.

a wound in which the edges were not approximated.

What are the characteristics of neutrophils (select all that apply)? a. Also known as "segs" b. Band is immature cell c. First WBC at injury site d. Arises from megakaryocyte e. Increased in individuals with allergies f. 50% to 70% of white blood cells (WBCs)

a, b, c, f. These characteristics are evident with neutrophils. Platelets arise from megakaryocytes and are stored in the spleen. Eosinophils are increased in individuals with allergies and make up 2% to 4% of WBCs.

A patient admitted with heart failure appears very anxious and complains of shortness of breath. Which nursing actions would be appropriate to alleviate this patient's anxiety (select all that apply)? a. Administer ordered morphine sulfate. b. Position patient in a semi-Fowler's position. c. Position patient on left side with head of bed flat. d. Instruct patient on the use of relaxation techniques. e. Use a calm, reassuring approach while talking to patient.

a, b, d, & e Morphine sulfate reduces anxiety and may assist in reducing dyspnea. The patient should be positioned in semi-Fowler's position to improve ventilation that will reduce anxiety. Relaxation techniques and a calm reassuring approach will also serve to reduce anxiety.

3. Which cells are classified as granulocytes (select all that apply)? a. Basophil b. Monocyte c. Eosinophil d. Neutrophil e. Lymphocyte

a, c, d. Basophils, eosinophils, and neutrophils are the granulocytic leukocytes. Lymphocytes are the agranular leukocytes that form the basis of the cellular and humoral immune responses. Monocytes are agranulocytes that are potent phagocytic cells.

The patient has heart failure (HF) with an ejection fraction of less than 40%. What core measures should the nurse expect to include in the plan of care for this patient (select all that apply)? a. Left ventricular function is documented. b. Controlling dysrhythmias will eliminate HF. c. Prescription for digoxin (Lanoxin) at discharge d. Prescription for angiotensin-converting enzyme (ACE) inhibitor at discharge e. Education materials about activity, medications, weight monitoring, and what to do if symptoms worsen

a, d, & e The Joint Commission has identified these three core measures for heart failure patients. Although controlling dysrhythmias will improve CO and workload, it will not eliminate HF. Prescribing digoxin for all HF patients is no longer done because there are newer effective drugs and digoxin toxicity occurs easily related to electrolyte levels and the therapeutic range must be maintained.

The nurse determines a need for additional instruction when the patient with newly diagnosed type 1 diabetes says which of the following? A. "I can choose any foods, as long as I use enough insulin to cover the calories" B. "I will need a bedtime snack because I take an evening dose of NPH insulin" C. "I will eat something at meal times to prevent hypoglycemia, even if I am not hungry" D. "I can have an occasional beverage with alcohol if I include it in my meal plan"

a. "I can choose any foods, as long as I use enough insulin to cover the calories" Most patients with type 1 diabetes need to plan diet choices very carefully. Patients who are using intensified insulin therapy have considerable flexibility in diet choices but still should limit (but not complete eliminate) dietary intake of items such as fat, protein, and alcohol. The other patient statements are correct and indicate good understanding of the diet instruction.

10. Isotonic expansion of the ECF caused by abnormal retention of water and sodium

a. Hypervolemia/ fluid-volume excess

The patient with chronic heart failure is being discharged from the hospital. What information should the nurse emphasize in the patient's discharge teaching to prevent progression of the disease to ADHF? a. Take medications as prescribed. b. Use oxygen when feeling short of breath. c. Only ask the physician's office questions. d. Encourage most activity in the morning when rested.

a. Take medications as prescribed. The goal for the patient with chronic HF is to avoid exacerbations and hospitalization. Taking the medications as prescribed along with nondrug therapies such as alternating activity with rest will help the patient meet this goal. If the patient needs to use oxygen at home, it will probably be used all the time or with activity to prevent respiratory acidosis. Many HF patients are monitored by a care manager or in a transitional program to assess the patient for medication effectiveness and monitor for patient deterioration and encourage the patient. This nurse manager can be asked questions or can contact the health care provider if there is evidence of worsening HF.

Chronic abdominal pain may originate from

abdominal structures or may be referred from a site with the same or a similar nerve supply. pain is often described as dull, aching, or diffuse

The characteristic sign of a paralytic ileus is: a. abdominal tightness. b. abdominal distention. c. absence of peristalsis. d. increased abdominal girth.

absence of peristalsis.

What can be used in combination for treatment of EE?

acid suppression, corticosteroids, and endoscopic dilation

Bright red blood indicates

active bleeding -this could be due to a Mallory-Weiss Tear, esophageal varcies, gastric or duondenal ulcer, or neoplasm

Leptin

acts in the hypothalamus to suppress appetite, increase physical activity, and increase fat metabolism. A genetic deficiency of leptin causes extreme obesity

What can be effective in reducing postoperative nausea and vomiting?

acupressure or acupuncture, herbs such as ginger and peppermint oil

The most prevalent lung carcinoma that is peripherally located and frequently metastasizes is: a. adenocarcinoma. b. bronchioalveolar. c. large cell. d. squamous cell.

adenocarcinoma

In general it is best to recommend a diet that includes

adequate amounts of fruits and vegetables, provides enough bulk to prevent constipation, and meets daily vitamin A and Vitamin C requirements, lean meat, fish, and eggs provide sufficient protein and the B complex vitamin

Test regiment of PUD consists of

adequate rest, drug therapy, elimination of smoking, dietary modifications and long term follow up care. treatment is aimed to decrease gastric acidity and enhance mucosal defense mechanisms. aspirin and NSAID are discontinued for 4 to 6 weeks, as must co-administration of PPI , h2 receptor blocker or misoprostol,

The body stores anesthetics in

adipose tissue, thus placing patients with excess adipose tissue at risk for re-sedation

Antacids are used as

adjunct therapy for PUD . neutralizing the HCl acid, taken after meals the effects last longer

(ch 22 q1) Nursing measures associated with the uncomplicated common cold include all of the following except: a. administering prescribed antibiotics to decrease the severity of the viral infection. b. informing the patient about the symptoms of secondary infection, the major complication of a cold. c. suggesting adequate fluid intake and rest. d. teaching people that the virus is contagious for 2 days before symptoms appear and during the first part of the symptomatic phase

administering prescribed antibiotics to decrease the severity of the viral infection

An early sign of cancer of the larynx in the glottic area (66% of cases) is: a. affected voice sounds. b. burning of the throat when hot liquids are ingested. c. enlarged cervical nodes. d. dysphagia.

affected voice sounds

Stimulus Control

aimed at separating events that trigger eating from the act of eating

A complication of acute pharyngitis can be: a. mastoiditis. b. otitis media. c. peritonsillar abscess. d. all of the above.

all of the above

Nursing assessment for a patient with chest pain includes: a. determining whether there is a relationship between the pain and the patient's posture. b. evaluating the effect of the phases of respiration on the pain. c. looking for factors that precipitate the pain. d. all of the above

all of the above

Nursing assessment of a patient with bronchospasm associated with COPD would include assessment for: a. compromised gas exchange. b. decreased airflow. c. wheezes. d. all of the above.

all of the above

Nursing management for a person diagnosed as having acute tracheobronchitis includes: a. increasing fluid intake to remove secretions. b. encouraging the patient to remain in bed. c. using cool-vapor therapy to relieve laryngeal and tracheal irritation. d. all of the above.

all of the above

Nursing measures to prevent thrombophlebitis include: a. assisting the patient with leg exercises. b. encouraging early ambulation. c. avoiding placement of pillows or blanket rolls under the patient's knees. d. all of the above.

all of the above

Preoperatively, an anesthesiologist is responsible for: a. assessing pulmonary status. b. inquiring about preexisting pulmonary infections. c. knowing the patient's history of smoking. d. all of the above

all of the above

Uneven perfusion of the lung is primarily due to: a. pulmonary artery pressure. b. gravity. c. alveolar pressure. d. all of the above.

all of the above

When taking a respiratory history, the nurse should assess: a. the previous history of lung disease in the patient or family. b. occupational and environmental influences. c. smoking and exposure to allergies. d. all of the above.

all of the above

To assess for an upper respiratory tract infection, the nurse should palpate: a. the frontal and maxillary sinuses. b. the trachea. c. the neck lymph nodes. d. all of the above areas

all of the above areas

A nurse knows that perioperative risks increase with age because: a. ciliary action decreases, reducing the cough reflex. b. fatty tissue increases, prolonging the effects of anesthesia. c. liver size decreases, reducing the metabolism of anesthetics. d. all of the above biologic changes exist

all of the above biologic changes exist

Nursing management for a patient with acute pharyngitis includes: a. applying an ice collar for symptomatic relief of a severe sore throat. b. encouraging bed rest during the febrile stage of the illness. c. suggesting a liquid or soft diet during the acute stage of the disease. d. all of the above measures.

all of the above measures

The pathophysiology of ARF is directly related to: a. decreased respiratory drive. b. chest wall abnormalities. c. dysfunction of lung parenchyma. d. all of the above mechanisms

all of the above mechanisms

Nursing instructions for a patient who is scheduled for a perfusion lung scan should include informing the patient that: a. a mask will be placed over his or her nose and mouth during the test. b. he or she will be expected to lie under the camera. c. the imaging time will amount to 20 to 40 minutes. d. all of the above will occur.

all of the above will occur

Potential complications of enlarged adenoids include all of the following except: a. bronchitis. b. nasal obstruction. c. allergies. d. acute otitis media.

allergies

In severe acute gastritis

an NG tube may be used 1) monitor for bleeding 2) to lavage the precipitating agent from the stomach, or 3) to keep the stomach empty and free of noxious stimuli -clear liquids are resumed when symptoms have subsided with graduation reintroduction of solids

What is the most common diagnostic method for abdominal trauma?

an abdominal ct scan and focused abdominal sonography

Stomach Cancer

an adenocarcinoma of the stomach

What can be used to slow intestinal transit?

an antidiarrheal agent such as loperamide (imodium)

Individuals with fat located primarily in the abdominal area

an apple-shaped body, have android obesity

Regurgitation

an effortless process in which partially digested food slowly comes up from the stomach

People with long standing diarrhea can develop

anemia from and iron and folate deficiencies

Cobalamin can be obtained only from

animal protein, special supplements, or foods that have been fortified with the vitamin

In acute gastritis the symptoms include

anorexia, nausea and vomiting, epigastric tenderness, and a feeling of fullness

Barrett's Esophagus

another complication of GERD, considered a precancerous lesion that increases the patients risk for esophageal adenocarcinoma

Waist circumference

another way to assess and classify a person's weight

Unlike angina GERD related chest pains is relieved with

antacids

Treatment of peritonitis consists of

antibiotics, NG suction, analgesics, and IV fluid administration

parotitis treatment

antibiotics, mouthwashes, warm compresses; preventive measures such as chewing gum, sucking on hard candy, adequate fluid intake

Crohn's disease can involve

any segment of the Gi tract from the mouth to the anus

The hypothalamus is a mjaor site for regulating

appetite

Neuropeptide Y, produced in the hypothalamus, is a powerful

appetite stimulant, when imbalanced it can lead to overeating and obesity

Linaclotide (linzess)

approved for the treatment of IBS with constipation in men and women

Lubiprostone (amitiza)

approved for the treatment of IBS with constipation in women

Patient who injury to the esophageal mucosa (e.g. from occupational exposure)

are at greater risk for esophageal cancer

Esophageal varcies

are dilated, tortuous veins occurring in the lower portion of the esophagus as a result of portal hypertension

The serotonin (5-HT3) receptor antagnoists

are effective in reducing cancer chemotherapy-induced vomiting caused by delayed gastric emptying and also the nausea and vomiting related to migraine headache and anxiety

Acute disease-or injury-related malnutrition

associated with acute disease or injury states with marked inflammatory response (e.g. major infection, burns, trauma closed head injury)

The nurse should be alert for a complication of bronchiectasis that results from a combination of retained secretions and obstruction that leads to the collapse of alveoli. This complication is known as: a. atelectasis. b. emphysema. c. pleurisy. d. pneumonia

atelectasis

gastropexy

attachment of the stomach subdiaphrahmatically to prevent reherniation

borboygmi

audible abdominal sounds produced by hyperactive intestinal peristalsis

Scherocytosis

auto-hemolytic anemia characterized by the production of spherocytes - red blood cells (RBCs), or erythrocytes, that are sphere-shaped, rather than bi-concave disk shaped.

Patients with a heart transplantation are at risk for which complications in the first year after transplantation (select all that apply)? a. cancer b. infection c. rejection d. vasculopathy e. sudden cardiac death

b, c, & e Rationale: A variety of complications can occur after heart transplantation. In the first year after transplantation, the major causes of death are acute rejection and infection. Heart transplant recipients also are at risk for sudden cardiac death. Later, malignancy (especially lymphoma) and cardiac vasculopathy (accelerated CAD) are major causes of death.

19. If a patient with blood type O Rh+ is given AB Rh- blood, what would the nurse expect to happen? a. The patient's Rh factor will react with the RBCs of the donor blood. b. The anti-A and anti-B antibodies in the patient's blood will hemolyze the donor blood. c. The anti-A and anti-B antibodies in the donor blood will hemolyze the patient's blood. d. No adverse reaction is expected because the patient has no antibodies against the donor blood.

b. A patient with type O Rh+ blood has no A or B antigens on the RBC but does have anti-A and anti-B antibodies in the blood and has an Rh antigen. Type AB Rh− blood has both A and B antigens on the RBC but no Rh antigen and no anti-A or anti-B antibodies. If the type AB Rh− blood is given to the patient with type O Rh+ blood, the antibodies in the patient's blood will react with the antigens in the donor blood, causing hemolysis of the donor cells. There will be no Rh reaction because the donor blood has no Rh antigen.

9. Laboratory test results indicate increased fibrin split products (FSPs). An appropriate nursing action is to monitor the patient for a. fever. b. bleeding. c. faintness. d. thrombotic episodes

b. During fibrinolysis by plasmin, the fibrin clot is split into smaller molecules known as FSPs or FDPs. Increased FSPs impair platelet aggregation, reduce prothrombin, and prevent fibrin stabilization and lead to bleeding.

What are a major cause of upper GI bleeding?

bleeding peptic ulcers and drugs

Single-agent chemotherapeutic agents include

bleomycin, mitomycin (mutamycin), methotrextate, paclitaxel, docetaxel, and irinotecan (camptosar)

Primary peritonitis occurs when

blood borne organisms enter the peritoneal cavity

Diagnostic peritoneal lavage can be used to detect

blood, bile, intestinal contents, and urine in the peritoneal cavity, but is generally used only for unstable patients to identify blood in the peritoneum

what is used for the determination of h. pylori infection?

breath, urine, serum, stool, and gastric tissue biopsy test

aspirate is usually

bright red at first, with a gradual darkening within the first 24 hours after surgery. normally changes to yellow-green within 36 to 48 hours

Choose the initial part of the respiratory tract that is not considered part of the gas-exchange airways. a. Bronchioles b. Respiratory bronchioles c. Alveolar duct d. Alveolar sacs

bronchioles

Heartburn is a

burning, tight sensation felt intermittently beneath the lower sternum and spreading upward to the throat or jaw

You are caring for a patient with ADHF who is receiving IV dobutamine (Dobutrex). You know that this drug is ordered because it (select all that apply): a. incerases SVR b. produces diuresis c. improves contractility d. dilates renal blood vessels e. works on the B1-receptors in the heart.

c & e Rationale: Dobutamine (Dobutrex) has a positive chronotropic effect and increases heart rate and improves contractility. It is a selective β-adrenergic agonist and works primarily on the β1-adrenergic receptors in the heart. It is frequently used in the short-term management of acute decompensated heart failure (ADHF).

The nurse teaches a client newly diagnosed with type 2 diabetes mellitus (DM) about exercise. Which statement is appropriate for the nurse to include in the teaching session? a. "You should exercise 2 days per week" b. "It is important for you to avoid resistance training" c. "You need to wear appropriate footwear when exercising" d. "Avoid exercise since you also have peripheral neuropathy"

c. "You need to wear appropriate footwear when exercising" a) INCORRECT - A client with type 2 DM should be encouraged to exercise at least 3 times/week, with no more than 2 consecutive days without exercise. b) INCORRECT - Adults with type 2 DM should perform resistance training at least 2 times/week, unless contraindicated. c) CORRECT- Clients with type 2 DM should wear proper footwear and inspect their feet after exercise to detect lesions early. d) INCORRECT - Studies show that moderate-intensity walking does not increase the risk for foot injury in clients with peripheral neuropathy who wear proper footwear. Exercise should not be avoided in these individuals.

17. A patient is being treated with chemotherapy. The nurse revises the patient's care plan based on which result? a. WBC count 4000/μL b. RBC count 3.8 × 106/μL c. Platelets 50,000/μL d. Hematocrit (Hct) 39%

c. Any platelet count <150,000/μL is considered thrombocytopenia and could place the patient at risk for bleeding, necessitating special consideration in nursing care. Chemotherapy may cause bone marrow suppression and a depletion of all blood cells. The other factors are all within normal range.

What is the priority assessment by the nurse caring for a patient receiving IV nesiritide (Natrecor) to treat heart failure? a. Urine output b. Lung sounds c. Blood pressure d. Respiratory rate

c. Blood pressure Although all identified assessments are appropriate for a patient receiving IV nesiritide, the priority assessment would be monitoring for hypotension, the main adverse effect of nesiritide.

secondary obesity

can result from various congenital anomalies, chromosomal anomalies, metabolic problems, or central nervous system lesions and disorders

Calories in PN are supplied primarily by

carbohydrates in the form of dextrose and fat in the form of fat emulsion ( should provide up to 30% of total calories)

The major components of the basic food groups are

carbohydrates, fats, proteins, vitamins, minerals, and water

Android obesity is the best predictor of

cardiovascular disease and is linked with increased LDLs, high triglycerides, and decreased HDLs

People who have visceral fat with truncal obesity are at an increased risk for

cardiovascular disease and metabolic syndrome

Postoperative nursing assessment for a patient who has received a depolarizing neuromuscular blocking agent includes careful monitoring of the: a. cardiovascular system. b. endocrine system. c. gastrointestinal system. d. genitourinary system.

cardiovascular system

Central PN maybe given through a

central venous catheter that originates at the subclavian or jugular vein and whose tip lies in the superior vena cava

Refeeding syndrome

characterized by fluid retention and electrolyte imbalances (hypophosphatemia, hypokalemia, hypomagnesemia)

Eosinophilic esophagitis (EE)

characterized by swelling of the esophagus caused by an infiltration of esoinophils

The underlying pathophysiology of COPD is: a. inflamed airways that obstruct airflow. b. mucus secretions that block airways. c. overinflated alveoli that impair gas exchange. d. characterized by variations of all of the above.

characterized by variations of all of the above.

Saliva production can be increased by

chewing gum and oral lozenges, which may help patients with mild symptoms of GERD

Teach patients to avoid foods that decrease LES pressure, such as

chocolate, peppermint, tomatoes, fatty foods, coffee, and tea, which predispose them to reflux

The most common cause of esophageal strictures (or narrowing) is

chronic GERD, as well as ingestion of strong acids or alkalis, external beam radiation, and surgical anastomosis

Two diseases common to the etiology of COPD are: a. asthma and atelectasis. b. chronic bronchitis and emphysema. c. pneumonia and pleurisy. d. tuberculosis and pleural effusions.

chronic bronchitis and emphysema.

Irritable bowel syndrome

chronic function disorder, meaning that no organic cause is currently known

Indications for parenteral nutrition

chronic severe diarrhea and vomiting, complicated surgery or trauma, GI obstruction, Intractable diarrhea, severe anorexia nervosa, severe malabsorption, short bowel syndrome, GI tract anormalies and fistulae

More than 80% of all lung cancers are primarily caused by: a. cigarette smoking. b. fibrosis. c. inhalation of environmental carcinogens. d. tuberculosis

cigarette smoking

herniorrhaphy

closure of the hiatal defect

The goal of endoscopic hemostasis is to

coagulate or thrombose the bleeding vessel.

The primary deficiency for a strict vegan is lack of

cobalamin

Crackles, noncontiguous breath sounds, would be assessed for a patient with: a. asthma. b. bronchospasm. c. collapsed aleveoli. d. pulmonary fibrosis

collapsed alveoli

A factor involved in postspinal anesthesia headaches is the: a. degree of patient hydration. b. leakage of spinal fluid from the subarachnoid space. c. size of the spinal needle used. d. combination of the above mechanisms

combination of the above mechanisms

Blunt trauma

commonly occurs with motor vehicle accidents, beatings, and falls and may not be obvious because it does not leave an open wound. Both compression (e.g. direct blow to the abdomen) and shearing injuries

Diabetic retinopathy refers to the process of microvascular damage in the retina because of chronic hyperglycemia. There are two types: proliferative and nonproliferative. Because the earliest and most treatable stages produce no vision changes, teach persons with diabetes to have an annual dilated eye examination. •Diabetic nephropathy is a microvascular complication associated with damage to the small blood vessels that supply the glomeruli of the kidneys. Teach patients to get screened annually for albuminuria. A measurement of albumin-creatinine ratio from a urine specimen may also be done to assess renal function. •Diabetic neuropathy is nerve damage that occurs because of the metabolic derangements associated with diabetes mellitus. The two major categories of diabetic neuropathy are sensory neuropathy, which affects the peripheral nervous system, and autonomic

complications of DM

End-organ disease from damage to blood vessels from chronic hyperglycemia. These are divided into two categories: macrovascular complications and microvascular complications. ♣ Macrovascular complications are diseases of the large and medium-sized blood vessels (heart, peripheral vascular system, and brain) that occur with greater frequency and with an earlier onset in people with diabetes. ♣Microvascular complications effect the tiny vessels of the eyes, kidneys, and nerves. They result from several pathways, all, in response to chronic hyperglycemia.

complications of DM

Common delivery options of EN are

continuous infusion by pump, intermittent infusion by gravity, intermittent bolus by syringe, and cyclic feedings by infusion pump

The circulating nurse's responsibilities, in contrast to the scrub nurse's responsibilities, include: a. assisting the surgeon. b. coordinating the surgical team. c. setting up the sterile tables. d. all of the above functions.

coordinating the surgical team.

Respiratory complications of GERD include

cough, bronchospasm, laryngospasm, and cricopharyngeal spasm

The most frequent symptom of lung cancer is: a. copious sputum production. b. coughing. c. dyspnea. d. severe pain.

coughing

A nursing measure for evisceration is to: a. apply an abdominal binder snugly so that the intestines can be slowly pushed back into the abdominal cavity. b. approximate the wound edges with adhesive tape so that the intestines can be gently pushed back into the abdomen. c. carefully push the exposed intestines back into the abdominal cavity. d. cover the protruding coils of intestines with sterile dressings moistened with sterile saline solution.

cover the protruding coils of intestines with sterile dressings moistened with sterile saline solution.

Recent research has indicated that inadvertent hypothermia in gerontologic patients can be effectively and inexpensively prevented by: a. placing the patient on a hyperthermia blanket. b. maintaining environmental temperature at 37ºC. c. covering the top of the patient's head with an ordinary plastic shower cap during anesthesia. d. frequent massage of th

covering the top of the patient's head with an ordinary plastic shower cap during anesthesia.

. Auscultation can be used to diagnose the presence of pulmonary edema when the following adventitious breath sounds are present: a. crackles in the lung bases. b. low-pitched rhonchi during expiration. c. pleural friction rub. d. sibilant wheezes

crackles at the base of the lung

Continuous infusion is most often used with

critically ill patients

IBD is classified as either

crohn's disease or ulcerative colitis

A commonly prescribed mast cell stabilizer used for asthma is: a. albuterol. b. budesonide. c. cromolyn sodium. d. theophylline.

cromolyn sodium

The nurse is administering a dose of digoxin (Lanoxin) to a patient with heart failure (HF). The nurse would become concerned with the possibility of digitalis toxicity if the patient reported which symptom(s)? a. Muscle aches b. Constipation c. Pounding headache d. Anorexia and nausea

d. Anorexia and nausea Anorexia, nausea, vomiting, blurred or yellow vision, and cardiac dysrhythmias are all signs of digitalis toxicity. The nurse would become concerned and notify the health care provider if the patient exhibited any of these symptoms.

Beyond the first year after a heart transplant, the nurse knows that what is a major cause of death? a. Infection b. Acute rejection c. Immunosuppression d. Cardiac vasculopathy

d. Cardiac vasculopathy Beyond the first year after a heart transplant, malignancy (especially lymphoma) and cardiac vasculopathy (accelerated CAD) are the major causes of death. During the first year after transplant, infection and acute rejection are the major causes of death. Immunosuppressive therapy will be used for posttransplant management to prevent rejection and increases the patient's risk of an infection.

A male patient with a long-standing history of heart failure has recently qualified for hospice care. What measure should the nurse now prioritize when providing care for this patient? a. Taper the patient off his current medications. b. Continue education for the patient and his family. c. Pursue experimental therapies or surgical options. d. Choose interventions to promote comfort and prevent suffering.

d. Choose interventions to promote comfort and prevent suffering. The central focus of hospice care is the promotion of comfort and the prevention of suffering. Patient education should continue, but providing comfort is paramount. Medications should be continued unless they are not tolerated. Experimental therapies and surgeries are not commonly used in the care of hospice patients.

Patients remain in the recovery room or postanesthesia care unit (PACU) until they are fully recovered from anesthesia. This is evidenced by: a. a patient airway. b. a reasonable degree of consciousness. c. a stable blood pressure. d. indication that all of the above have occurred

d. all of the above

Unless contraindicated, any unconscious patient should be positioned: a. flat on his or her back, without elevation of the head, to facilitate frequent turning and minimize pulmonary complications. b. in semi-Fowler's position, to promote respiratory function and reduce the incidence of orthostatic hypotension when the patient can eventually stand. c. in Fowler's position, which most closely simulates a sitting position, thus facilitating respiratory as well as gastrointestinal functioning. d. on his or her side with a pillow at the patient's back and his or her chin extended, to minimize the dangers of aspiration.

d. on his or her side with a pillow at the patient's back and his or her chin extended, to minimize the dangers of aspiration.

In teaching a newly diagnosed type 1 diabetic "survival skills", the nurse includes information about: a. weight loss measures b. elimination of sugar from the diet c. need to reduce physical activity d. self-monitoring of blood glucose

d. self-monitoring of blood glucose

When vomiting occurs postoperatively, the most important nursing intervention is to: a. measure the amount of vomitus to estimate fluid loss, in order to accurately monitor fluid balance. b. offer tepid water and juices to replace lost fluids and electrolytes. c. support the wound area so that unnecessary strain will not disrupt the integrity of the incision. d. turn the patient's head completely to one side to prevent aspiration of vomitus into the lungs.

d. turn the patient's head completely to one side to prevent aspiration of vomitus into the lungs.

Causes of acute abdominal pain include

damage to organs in the abdomen and pelvis, which leads to inflammation, infection, obstruction, bleeding, and perforation

The aim of therapy for obstruction is to

decompress the stomach, correct any existing fluid and electrolyte imbalances, and improve the patient's general state of health

Serum iron in aging

decreased

Long term use of PPIs has been associated with

decreased bone density, chronic hypochlorhydria, and increased risk of C diff in hospitalized patients, as well as pneumonia

A recently admitted patient has a small cell carcinoma of the lung, which is causing the syndrome of inappropriate antidiuretic hormone (SIADH). The nurse will monitor carefully for 1. decreased serum sodium level. 2. rapid and unexpected weight loss. 3. increased total urinary output. 4. elevation of serum hematocrit.

decreased serum sodium level.

In gastric bypass surgery, Ghrelin is

decreased significantly, which helps suppress appetite

Airway clearance in a patient with an upper airway infection is facilitated by all of the following except: a. decreasing systemic hydration. b. humidifying inspired room air. c. positional drainage of the affected area. d. administering p

decreasing the systemic hydration

Drug therapy for GERD focuses on

decreasing volume and acidity of reflux, improving LES function, increasing esophageal clearance, and protecting the esophageal mucosa

Enteral Nutrition (EN), also known as tube feeding

defined as nutrition provided through the GI tract via a tube, catheter, or stomach that deliver nutrients distal to the oral cavity

The treatment of esophageal cancer

depends on the location of the tumor and whether invasion or metastasis has occurred. Has a poor prognosis, usually not diagnosised until advanced

Undernutrition

describes a state of poor nourishment as a result of inadequate diet or diseases that interfere with normal appetite and assimilation of ingested food.

Obstruction of the airway in the patient with asthma is caused by all of the following except: a. thick mucus. b. swelling of bronchial membranes. c. destruction of the alveolar wall. d. contraction of muscles surrounding the bronchi.

destruction of the alveolar wall.

Psychologic stessors (e.g. depression, anxiety, sexual abuse, ptsd) are associated with

development and exacerbation of IBS

Angiography is used in

diagnosing upper GI bleeding when endoscopy cannot be done or when bleeding persists after endoscopic therapy, an invasive procedure

To correctly perform the Heimlich maneuver, a person should forcefully apply pressure against the victim's: a. abdomen. b. diaphragm. c. lungs. d. trachea.

diaphragm

Fluids and foods high in carbohydrates tend to promote

diarrhea and symptoms of the dumping syndrome

The exchange of oxygen and carbon dioxide from the alveoli into the blood occurs by: a. active transport. b. diffusion. c. osmosis. d. pinocytosis.

diffusion

What drugs can have a direct irritating effect on the gastric mucosa?

digitalis (digoxin) and alendronate (fosamax)

Clinical manifestations directly related to cor pulmonale include all of the following except: a. dyspnea and cough. b. diminished peripheral pulses. c. distended neck veins. d. edema of the feet and legs.

diminished peripheral pulses

with obstruction the patient reports

discomfort or pain that is worse toward the end of the day as the stomach fills and dilates. relief maybe obtained by belching or by self-inducing vomiting, often projectile

Mallory-Weiss tear

disruption of the mucosal lining near the esophagogastric junction

Health teaching for viral rhinitis (common cold) includes advising the patient to: a. blow his or her nose gently to prevent spread of the infection. b. blow through both nostrils to equalize the pressure. c. rest, to promote overall comfort. d. do all of the above

do all of the above

Low carbohydrate diets

do produce a rapid weight loss, but reduce the opportunity to get adequate amounts of fiber, vitamins, and minerals. Make it difficult to maintain long term

Epinephrine is often used in combination with a local infiltration anesthetic, because it: a. causes vasoconstriction. b. prevents rapid absorption of the anesthetic drug. c. prolongs the local action of the anesthetic agent. d. does all of the above

does all of the above

Patient education for a laryngectomy includes: a. advising that large amounts of mucus can be coughed up through the stoma. b. cautioning about preventing water from entering the stoma. c. telling the patient to expect a diminished sense of taste and smell. d. doing all of the above

doing all of the above

To assess for a positive Homans' sign, the nurse should: a. dorsiflex the foot while the leg is elevated to check for calf pain. b. elevate the patient's legs for 20 minutes and then lower them slowly while checking for areas of inadequate blood return. c. extend the leg, plantar flex the foot, and check for the patency of the dorsalis pedis pulse. d. lower the patient's legs and massage the calf muscles to note any areas of tenderness.

dorsiflex the foot while the leg is elevated to check for calf pain.

A complication of the RYGB is

dumping syndrome

The primary presenting symptom of emphysema is: a. chronic cough. b. dyspnea. c. tachypnea. d. wheezing.

dyspena

Typical symptoms of esophageal diverticula include

dysphagia, regurgitaiton, chronic cough, aspiration, and weight loss. patient frequently complains of of tasting sour food and smelling a foul odor

A nurse knows to assess a patient with pulmonary arterial hypertension for the primary symptom of: a. ascites. b. dyspnea. c. hypertension. d. syncope.

dyspnea

To facilitate early interventions in upper GI bleeding, the physical examination should focus on

early identification of signs and symptoms of shock such as tachycardia, weak pulse, hypotension, cool extremities, prolonged capillary refill, and apprehension. monitor vital signs every 15 to 30 minutes. IV lines, preferably two, with a 16 or 18 gauge needle are placed for fluid and blood replacement

Patients who have had Roux-en-Y are discouraged from

eating sugary foods after surgery to avoid dumping syndrome

Obstruction in the distal stomach and duodenum is the result

edema, inflammation, or pylorospasm and fibrous scar tissue formation

The new definition of COPD leaves only one disorder under its classification. That is: a. asthma. b. bronchiectasis. c. cystic fibrosis. d. emphysema

emphysema

What may decrease the amount of bleeding and the need for endoscopic therapy in an upper GI bleed?

empiric PPI therapy with high-dose IV bolus and subsequent infusion

Nursing measures to assist in the prevention of pulmonary embolism in a hospitalized patient include all of the following except: a. a liberal fluid intake. b. assisting the patient to do leg elevations above the level of the heart. c. encouraging the patient to dangle his or her legs over the side of the bed for 30 minutes, four times a day. d. the use of elastic stockings, especially when decreased mobility would promote venous stasis.

encouraging the patient to dangle his or her legs over the side of the bed for 30 minutes, four times a day.

Strictures can be dilated

endoscopically using mechanical bougies or balloons

What can be used to determine the presence and location of an ulcer?

endoscopy

The first line management of upper GI bleeding is

endoscopy ane endotherapy

What can be used to establish a diagnosis of esophageal diverticula?

endoscopy or barium studies

Fats are a major source of

energy and provide a feeling of satiety after eating

Nutrition is important for

energy, growth, and maintenance and repair of body tissues

Injection therapy of what during an endoscopy is effective for acute hemostasis?

epinephrine

Diagnostic confirmation of a lung abscess is made by: a. chest x-ray. b. bronchoscopy. c. sputum culture. d. evaluating all of the above studies`

evaluating all of the above studies

The majority of obese persons have primary obesity which is

excess calorie intake over energy expenditure for the body's metabolic demands

stomatitis clinical manifestation

excessive salivation, halitosis, sore mouth

Non-GI symptoms may include

fatigue and sleep disturbances

Steatorrhea

fatty stools

As a cause of death among men and women in the United States, lung cancer ranks: a. first. b. second. c. third. d. fourth

first

Decreased LES pressure can be due to certain

foods (e.g caffeine, chocolate, peppermint) and drugs (e.g. anticholinergics.

A gastrostomy tube may be used

for a patient who requires EN over an extended time -can be placed surgically, radiologically, or endoscopically

1. As a cause of death in the United States, chronic obstructive pulmonary disease (COPD) ranks: a. second. b. third. c. fourth. d. fifth.

fourth

Monosaccharides (e.g. glucose , fructose) are found in

fruits and honey

The symptoms of duodenal ulcers occur when

gastric acid comes in contact with the ulcers. With meal ingestion, food is present to help buffer the acid. symptoms general occurs 2 to 5 hours after a meal. described as burning or cramplike. located in the midepigastric region beneath the xiphoid process. can produce back pain

Vomitus with a "coffee ground" appearance is related to

gastric bleeding,w here blood changes to dark brown as a result of its interaction with HCl acid

Dumping syndrome

gastric contents empty too rapidly into the small bowel intestine, overwhelming its ability to digest nutrients.

Emesis containing partially digested food several hours after a meal is indicative of

gastric outlet obstruction or delayed gastric emptying

In dumping syndrome patients describe feelings of

generalized weakness, sweating, palpitations, and dizziness; due to sudden decrease in plasma volume. the patient complains of abdominal cramps, borboygmi, and urge to defecate

Qsymia must not be used in patients with

glaucoma or hyperthyroidism

A common cause of GERD is a

hiatal hernia

The discomfort generally associated with gastric ulcers is located

high in the epigastrium and occurs about 1 to 2 hours after meals. described as burning or gaseous. if the ulcer has eroded through the gastric mucosa, food tends to aggravate rather than alleviate the pain

The pulmonary circulation is considered a: a. high-pressure, high-resistance system. b. low-pressure, low-resistance system. c. high-pressure, low-resistance system. d. low-pressure, high-resistance system.

high-pressure, high-resistance system.

Food thats commonly cause gastric irritation include

hot, spicy foods; pepper; carbonated beverages; and broth (meat extract)

When fluids are permitted, water (30 to 60mL) is given

hourly, with gradual progression to small frequent bland meals

What is the hallmark of refeeding syndrome?

hypophosphatemia, and is associated with serious outcomes including cardiac dysrhythmias, respiratory arrest, and neurologic disturbances

Hormones and peptides produced in the gut and adipocyte cells affect the

hypothalamus and have a critical role in appetite and energy balance

complications of peritonitis include

hypovolemic shock, sepsis, intraabdominal abscess formation, paralytic ileus, and acute respiratory distress syndrome

What does hypoxemia lead to?

hypoxia

a nurse is caring for a patient who has just returned to the unit after thoracic surgery. Upon assessment the patient is lethargic, has pale and cool nail beds, and a heart rate of 102. The nurse recognizes the patient is at risk for developing what? a.Prolonged sedation b. Hypoxemia c.Hypothermia d. Hypoxia

hypoxia

Appendicitis

inflammation of the appendix

Esophagitis

inflammation of the esophagus and a common complication of GERD

Antidiarrheal agents are used cautiously in

inflammatory bowel disease because of the danger of toxic megacolon

Endoscopic visualization of the lower esophagus provides

information on the degree of mucosal inflammation or other abnormalities

Heart may occur after

ingestion of food or drugs that decrease the LES pressure or directly irritate the esophageal mucosa

The hypothalamus, gut, and adipose tissue synthesize hormones and peptides that

inhibit or stimulate the appetite

The main underlyng risk factor for metabolic syndrome is

insulin resistance related to excessive visceral fat

The presence of fecal odor and bile after prolonged vomiting suggests

intestinal obstruction below the level of the pulorus

Adjustable gastric banding (AGB)

involves limiting the stomach size with an inflatable band placed around the fundus of the stomach. Done using a lap-band or realize band.

The nurse is aware that the most common surgical procedure for a small, apparently curable tumor of the lung is a: a. lobectomy. b. pneumonectomy. c. segmentectomy. d. wedge resection.

lobectomy

in peritonitis surgery is indicated to

locate the cause of the inflammation, drain purulent fluid, and repair any damage (e.g. perforated organs)

Conditions that predispose patients to refeeding syndrome include

long standing malnutrition states such as chronic alcoholism, vomiting and diarrhea, chemotherapy, and majory surgery

What is hypoxemia?

low O2- arterial in blood

A patient is receiving 3% NaCl solution for correction of hyponatremia. During administration of the solution, the most important assessment for the nurse to monitor is 1. lung sounds. 2. peripheral pulses. 3. peripheral edema. 4. urinary output.

lung sounds.

imbalances with what occur with malnutrition?

macronutrients (carbohydrates, proteins, fat) or micronutrients (electrolytes, minerals, vitamins)

A nutritional assessment is a comprehensive approach that includes

medical, nutritional, and medication histories; physical examination; antrhopometric measurements; and laboratory data

Some patients may benefit from appetite stimulants such as

megestrol acetate (megace) or dronabinol (marinol ) to improve nutritional intake

What is essential for patients with esophageal cancer?

meticulous oral care. Cleanse the mouth thoroughly, including the tongue, gingivae, and teeth or dentures. milk of magnesia with mineral oil helps remove crust formation

Because of the small diameter of some tubes they are

more easily clogged when feedings are thick, and the tubes are more difficult to use for checking residual volumes. They are particularly prone to obstruction when oral drugs are not thoroughly crushed and dissolved in water before administration

PPIs are

more effective than h2 receptor blockers in reducing gastric acid secretion and promoting ulcer healing. also used in combination with antibiotics to treat ulcers caused by h. pylori

A health care provider should evaluate heartburn that occurs

more than twice a week, is rated as severe, occurs at night and wakes a person from sleep, or is associated with dysphagia

oral cavity cancer starts in the

mouth

Esophagram (barium swallow) may show

narrowing of the esophagus at the tumor site

A patient with a total laryngectomy would no longer have: a. natural vocalization. b. protection of the lower airway from foreign particles. c. a normal effective cough. d. all of the above mechanisms.

natural vocalization

Gingivitis etiology

neglected oral hygiene, malocclusion, missing or irregular teeth, faulty denistry, eating of soft rather than fibrous foods

Clinical manifestations of vitamin imbalances are most commonly

neurologic signs

Nutritional Therapy PUD

no specific recommendation. Caffeine-containing beverages and food can increase symptom distress. eliminate alcohol-can delay would healing

The use of NSAIDs is responsible for the majority of

non-h. pylori peptic ulcers. -patients on corticosteroids, anticoagulants, and selective serotonin reuptake inhibitors are also at increased risk of ulcer development

Oral cancer patients may report

nonspecific symptoms such as chronic sore throat, sore mouth, and voice changes

In restrictive bariatric surgery digestion is

not altered so the risk of anemia or cobalamin deficiency is low

Starvation -related malnutrition, or primary PCM, occurs when

nutritional needs are not met.

High calorie, high protein diet

o Breads and cereals: -Hot cereals (oatmeal or cream of wheat) prepared with milk, added fat like butter and sugar -Potatoes with added fats like butter and whole milk -Granola and other cereal with dried fruit -Croissants, biscuits, banana and zucchini bread, muffins o Vegetables -Fried and prepared with margarine/butter oFruits -Canned in heavy syrup -Dried fruit oMeat -Fried meats -Meats covered in cream sauce or gravy -Casseroles o Milk and Milk Product -Milkshakes -Whole milk products (yogurt, ice cream, cheese) -Whipping or heavy cream -Whole milk

Metabolic syndrome

o Characterized by a cluster of health problems that increase a patient's chance of developing cardiovascular disease and DM; including obesity, hypertension, abnormal lipid levels, and high blood glucose. This diagnosed if an individual has three or more of the criteria

PUD diet changes:

o There is no specific recommended dietary modifications for this. o Patients should eat and drink foods and fluids that do not cause any distressing symptoms o Foods that commonly cause gastric irritation include pepper, carbonated beverages, broth (meat extract), and hot, spicy foods. o Caffeine-containing beverages and foods can increase symptoms in some patients. o Teach the patient to avoid alcohol use because it can delay healing o Smoking cessation

Gastroesophageal reflux disease (GERD) and gallstones are more prevalent in

obese people

Factors that increase intraabdominal pressure include

obesity, pregnancy, ascites, tumors, intense physical exertion, and heavy lifting on a continual basis, may predispose patients to develop hiatal hernias

A common cause of appendicitis is

obstruction of the lumen by a fecalith (accumulated feces) -the obstruction results in distention; venous engorgement; and the accumulation of mucus and bacteria which can lead to gangrene, perforation, and peritonitis

Penetrating injuries

occur when a gunshot or stabbing produces an obvious, open wound into the abdomen

When pain begins to develop in esophageal cancer what can it be described as?

occurring in the substernal, epigastric, or back areas and usually increases with swallowing. the pain may radiate to the neck, jaw, ears, and shoulders

one type of polyunsatured fat

omega 3 fatty acids, and can be beneficial to your heart

The treatment of chronic gastritis focuses

on evaluating and eliminating the specific cause (e.g. cessation of alcohol intake, abstinence from drugs, h. pylori eradication)

If an operating room nurse is to assist a patient to the Trendelenburg position, he or she would place him: a. flat on his back with his arms next to his sides. b. on his back with his head lowered so that the plane of his body meets the horizontal on an angle. c. on his back with his legs and thighs flexed at right angles. d. on his side with his uppermost leg adducted and flexed at the knee.

on his back with his head lowered so that the plane of his body meets the horizontal on an angle.

When is surgery performed in PUD?

on patients with complications that are unresponsive to medical management or concerns about stomach cancer

Antiemetics

ondansetron (zofran), trimethobenzamide (tigan), and prochlorperazine (compazine)

a chronic ulcer

one of long duration, eroding through the muscular wall with the formation of fibrous tissue. present for many months or intermittently throughout the person's lifetime

Surgery remain the most effective treatment for

oral cancer

Patients with autoimmunte atrophic gastrics often have

other autoimmune disorders, associated with an increased risk of stomach cancer

If the feedings are necessary for an extended time,

other tubes are placed in the stomach or small bowel by surgical, endoscopic, or fluoroscopic procedures

Nursing intervention for a patient with a fractured nose includes all of the following except: a. applying cold compresses to decrease swelling and control bleeding. b. assessing respirations to detect any interference with breathing. c. observing for any clear fluid drainage from either nostril. d. packing each nostril with a cotton pledget to minimize bleeding and help maintain the shape of the nose during fracture setting

packing each nostril with a cotton pledget to minimize bleeding and help maintain the shape of the nose during fracture setting

Dyspepsia

pain or discomfort centered in the upper abdomen (mainly in or around the midline as opposed to the right or left hypochondrium)

Concurrent radiation and chemotherapy are administered in esophageal cancer for

palliation of symptoms, especially dysphagia, and to increase survival

adverse effects of nonamphetamines include

palpitations, tachycardia, overstimulation, restlessness, dizziness, insomnia, weakness and fatigue

After esophageal surgery, what fluids are given?

parenteral

Surgical procedures include

partial gastrectomy, vagotomy, and pyloroplasty

Nasoduodenal and nasojejunal tubes are used when

pathophysiologic conditions such as risk of aspiration warrant feeding the patient below the pyloric sphincter

Postoperative care for esophageal cancer

patient usually has an NG tube and may have bloody drainage for 8 to 12 hours, drainage gradually changes to greenish yellow. because of the location of surgery emphasize prevention of respiratory complications. Turn and deep breath every 2 hours. Pt in semi-fowlers or fowlers position to prevent reflux and aspiration of gastric secretions

incontience associated dermatitis (IAD)

patients who are critically ill and have fecal incontinence are at risk for, characterized by erythema and skin denudement of the perianal or genital areas, buttocks, or upper thighs. it is caused by chemical irritants in feces

An example of a stable and safe nondepolarizing muscle relaxant is: a. anectine (succinylcholine chloride). b. norcuron (vercuronium bromide). c. pavulon (pancuronium bromide). d. syncurine (decamethonium)

pavulon (pancuronium bromide)

Oral candidiasis clinical manifestation

pearly, bluish white "milk curd" membranous lesions on mucosa of mouth and larynx. sore mouth, yeasty halitosis

An intravenous anesthetic that has a powerful respiratory depressant effect sufficient to cause apnea and cardiovascular depression is: a. amidate. b. ketalar. c. pentothal. d. versed.

pentothal.

The fetal posture is common with

peritoneal irritation (e.g. appendicitis), a supine posture with outstretched legs is seen with visceral pain, and restlessness and a seated posture commonly occur with bowel obstruction and obstructions from kidney stones and gallstones

Loss of bowel sounds occurs with

peritonitis, if present bowel sounds are heard in the chest when the diaphragm ruptures.

Typically appendicitis begins with

periumbilical pain, followed by anorexia, nausea and vomiting. pain is persistent and continuous, eventually shifting to the right lower quadrant and localizing at mcburney's point

What is a long term complication of total gastrectomy and may occur after partial gastrectomy?

pernicious anemia, due to the loss of intrinsic factor

Because it is essential for the growth and maturation of RBCs the lack of cobalamin results in

pernivious anemia and neurologic complication

An NG tube connected to suction may be needed for the patient with

persistent vomiting or when a bowel obstruction or paralytic ileus is suspected

Indications for EN may include

persons with anorexia, orofacial fractures, head and neck cancer, neurologic or psychiatric conditions that prevent oral intake, extensive burns, or critical illness (especially if mechanical ventilation is required), and those who are receiving chemotherapy or radiation therapy.

nonamphetamines include

phentermine (adipex-P, fastin, ionamin), diethylpropion (tenuate), phendimetrazine ( bontril), and benzphetamine (didrex)

What helps in the healing of an ulcer?

physical and emotional rest, the patients immediate environment should be quiet and restful. the use of mild sedatives or tranquilizer has beneficial effects when the patient is anxious and apprehensive

A nurse documents the presence of granulation tissue in a healing wound. She describes the tissue as: a. necrotic and hard. b. pale yet able to blanch with digital pressure. c. pink to red and soft, noting that it bleeds easily. d. white with long, thin areas of scar tissue

pink to red and soft, noting that it bleeds easily.

Chest pain described as knifelike on inspiration would most likely be diagnostic of: a. bacterial pneumonia. b. bronchogenic carcinoma. c. lung infarction. d. pleurisy.

pleurisy

In the United States, the most common causes of death from infectious diseases are influenza and: a. atelectasis. b. pulmonary embolus. c. pneumonia. d. tracheobronchitis

pneumonia

One of the most common postoperative respiratory complications in elderly patients is: a. pleurisy. b. pneumonia. c. hypoxemia. d. pulmonary edema

pneumonia

Endoscopic approach using photodynamic

porfimer sodium (photofrin) injected IV, The light (activator) is transmitted via a fiber passed through the endoscope. Warn patients to avoid direct sunlight for up to 4 weeks after the procedure

Misoprostol

prescribed for the prevention of gastric ulcers caused by NSAIDs and aspirin

Gingivitis treatment

prevention through health teaching, dental care, gingival massage, professional cleaning of teeth, fibrous foods, conscientious brushing habits with flossing

Bleeding from an arterial source is

profuse and the blood is bright red

What is the most common symptom of esophageal cancer?

progressive dysphagia and may be described as a substernal feeling as if food is not passing. pain develops late.

Prokinetic drugs (metoclopramide {reglan})

promote gastric emptying and reduce the risk of gastric acid reflux but are not used as primary therapies for GERD

H2 receptor blockers

promote ulcer healing, includes famotidine, ranitidine, and cimetidine, given orally or iv. Nizatidine is only available orally

Treatment for IBS is directed at

psychologic and dietary factors and drugs to regulate stool output

Erythroplasia

red velvety patch on the mouth or tongue, is also a precancerous lesion

Teach the patient hiatal hernias to

reduce intraabdominal pressure by eliminating constricting garments and avoiding lifting and straining

The goal of surgical interventions is to

reduce reflux by enhancing the integrity of the LES

Restrictive bariatric surgery

reduces either the size of the stomach, which causes the patient to feel full more quickly, or the amount allowed to enter the stomach

Surgical approaches to hiatal hernias can include

reduction of the herniated stomach into the abdomen, herniotomy, herniorrhaphy, an antireflux procedure, and gastropexy

Parenteral nutrition (PN)

refers to the administration of nutrients by a route other than the GI tract (e.g. the bloodstream)

overnutrition

refers to the ingestion of more food than is required for body needs, as in obesity

When esophageal stenosis (narrowing) is severe,

regurgitation of blood-flecked esophageal contents is common

What may be a common manifestation of GERD?

regurgitation, as well as hypersalivation

hemiglossectomy

removal of half of the tongue

Mandibulectomy

removal of the mandible

glossectomy

removal of the tongue

stomatits treatment

removal of treatment of cause, oral hygiene with soothing solutions, topical medicals. soft, bland diet

A total gastectomy requires

resection of the lower esophagus, removal of the entire stomach, and anastomosis of the sophagus to the jejunum.

Antireflux surgery

reserved for patients with complications of reflux, including esophagitis, intolerance of medications, stricture, Barrett's metaplasia, and persistence of severe symptoms

Gas exchange between the lungs and blood and between the blood and tissues is called: a. active transport. b. respiration. c. ventilation. d. cellular respiration.

respiration

The most serious and most frequent postoperative complications involve the ___________________ system.

respiratory

Pathophysiology of emphysema directly related to airway obstruction. The end result of deterioration is: a. diminished alveolar surface area. b. hypercapnia resulting from decreased carbon dioxide elimination. c. hypoxemia secondary to impaired oxygen diffusion. d. respiratory acidosis due to airway obstruction

respiratory acidosis due to airway obstruction

The nurse understands that a patient with status asthmaticus will likely initially evidence symptoms of: a. metabolic acidosis. b. metabolic alkalosis. c. respiratory acidosis. d. respiratory alkalosis.

respiratory alkalosis

1.A patient who has required prolonged mechanical ventilation has the following arterial blood gas results: pH 7.48, PaO2 85 mm Hg, PaCO2 32 mm Hg, and HCO3 25 mEq/L. The nurse interprets these results as 1. metabolic alkalosis. 2. respiratory alkalosis. 3. respiratory acidosis. 4. metabolic acidosis.

respiratory alkalosis.

The primary nursing objective during the immediate postoperative assessment is to maintain ___________________________ and prevent _________________________.

respiratory function (ventilation); hypoxemia and hypercapnia

The oral cavity is a potential reservoir for

respiratory pathogens. and oral pathogens have been associated with diabetes and heart disease

If vomiting accompanies acute gastritis

rest, NPO status, and IV fluids may be prescribed

postprandial hypoglycemia

results of uncontrolled gastric emptying of a bolus of fluid high in carbohydrate into the small intestine. results in hyperglycemia and the release of excessive amounts of insulin into the circulation-then reflex hypoglycemia

Ecchymosis around the umbilicus or flanks may indicate

retroperitoneal hemorrhage

Esophageal diverticula

saclike outpouchings of one or more layers of the esophagus

Repeated esophagitis may lead to

scar tissue formation, stricture, and ultimately dysphagia

Small perforation

seal themselves resulting in a cessation of symptoms

Obstipation

severe constipation when no gas or stool is expelled

Clinical manifestations of EE include

severe heartburn, difficulty swallowing, food impaction in the esophagus, nausea, vomiting, and weight loss

Sucralfate is used for

short term treatment of ulcers. provides cytoprotection of the esophagus, stomach, and duodenum. does not have acid-neutralizing capabilities. most effective at a low pH and should be given at least 30 minutes before or after an antacid.

If oxygen supply or demand changes, the cell becomes stressed and pt goes into crisis

sickle cell crisis

in Chronic gastritis the manifestations

similar to those for acute gastritis, can be asymptomatic, can have a cobalamin deficiency

Measurements used with the obese person may include

skinfold thickness, waist circumference, height (without shoes), weight, and BMI

Melena (black, tarry stools), indicates

slow bleeding from an upper GI source

Stomach cancer has been associated with diets containing

smoked foods, salted fish and meat, and pickled vegetables

The most important environmental risk factor for emphysema is: a. air pollution. b. allergens. c. infectious agents. d. cigarette smoking

smoking

Assessment: Upper Respiratory Tract- Health History

social history, family history, prior surgery, lifestyle

For a patient with chronic bronchitis, the nurse expects to see the major clinical symptoms of: a. chest pain during respiration. b. sputum and a productive cough. c. fever, chills, and diaphoresis. d. tachypnea and tachycardia.

sputum and a productive cough.

The most diagnostic clinical symptom of pleurisy is: a. dullness or flatness on percussion over areas of collected fluid. b. dyspnea and coughing. c. fever and chills. d. stabbing pain during respiratory movement

stabbing pain during respiratory movement

Endoscopic ultrasonography (EUS) is an important tool to

stage esophageal cancerl

Complex carbohydrates or polysaccharides include

starches, such as cereal grains, potatoes, and legumes

Biopsy and cytologic specimens can differentiate

stomach or esophageal carcinoma from Barrett's esophagus

Signs and symptoms of toxicity due to O2 therepy

substernal discomfort, restlessness, fatigue

Clinical manifestations of perforation

sudden and dramatic in onset. - initial phase 0 to 2 hours the patient experiences sudden, severe upper abdominal pain that quickly spreads throughout the abdomen. pain radiates to back and not relieved by food or antacids. abdomen appears rigid and boardlike.respirations becomes shallow and rapid. tachycardic and pulse is weak. bowel sounds usually absent. n&v

acute gastroenteritis is defined as

sudden diarrhea accompanied by nausea, vomiting, and abdominal cramping

An initial characteristic symptom of a simple pneumothorax is: a. ARDS. b. severe respiratory distress. c. sudden onset of chest pain. d. tachypnea and hypoxemia.

sudden onset of chest pain

The sympathomimetic amines

suppress appetite ; the amphetamines and nonamphetamines

There are four stages to general anesthesia. An unconscious patient with normal pulse and respirations is considered to be in the stage known as: a. beginning anesthesia. b. excitement. c. surgical anesthesia. d. medullary depression

surgical anesthesia.

Damage to the anal sphincter may require

surgical repair

Proton pump inhibitors (PPIs) and Histamine (H2) receptor blocks are common and effective treatments for

symptomatic GERD

GERD is usually Diagnosed on the basis of

symptoms and the patient's response to behavioral and drug therapies

Loperamide

synthetic opioid that slows intestinal transit, may be used to treat diarrhea when it occurs

Disaccharides (e.g. sucrose, maltose)are found in

table sugar, malted cereal, and milk

Bile in the emesis may suggest obstruction below

the ampulla of Vater

ulcerative colitis is usually limited to

the colon

Preoperative management of patients with stomach cancer focuses on

the correction of nutritional deficits and treatment of anemia, preparations also include a low-residue diet, enemas to cleanse the bowel and antibiotics to reduce the intestinal bacteria

GERD results when

the defenses of the esophagus are overwhelmed by the reflux of acidic gastric contents into the esophagus.

Environmental allergns such as pollens, molds, cat, dog, and dust mite, may be involved in

the development of EE

Complications of GERD are due to

the direct local effects of gastric acid on the sophageal mucosa

Dumping syndrome

the direct result of surgical removal of a large portion of the stomach and the pyloric sphincter -a large bolus of hypertonic fluid enters the intestine and results in fluid being drawn into the bowel lumen. occurs within 15 to 30 minutes after eating.

Metrondiazole

the first line treatment in mild disease of C. difficile because of concerns about vancomycin resistant enterococcus

Vomiting

the forceful ejection of partially digested food and secretions (emesis) from the upper GI tract

Malabsorption syndrome

the impaired absorption of nutrients from the GI tract

Fecal incontienence

the involuntary passage of stool -problems with motor function and/or sensory function can result

Sliding

the junction of the stomach and the esophagus is above the diaphgram, and a part of the stomach slides through the hiatal opening in the diaphgram. This occurs when the patient is supine, and the hernia usually goes back into the abdominal cavity when the patient is standing upright.

malabsorptive procedures

the length of the small intestine is decreased (less food absorbed)

Most oral malignant lesions occur on

the lower lip

The nurse should know that, postoperatively, a general anesthetic is primarily eliminated by: a. the kidneys. b. the lungs. c. the skin. d. all of these routes.

the lungs

To assess for an upper respiratory tract infection, the nurse should inspect: a. the nasal mucosa. b. the frontal sinuses. c. the tracheal mucosa. d. all of the above

the nasal mucosa

oropharyngeal cancer develops in

the part of the throat just behind the mouth

Intermittent feeding may be preferred as

the patient improves or is receiving EN at home

Diagnosis of acute gastritis is most often based on

the patient's history of drug and alcohol use. endoscopic examination with biopsy may be warranted to obtain a definitive diagnosis.

Following a thyroidectomy, a patient complains of "a tingling feeling around my mouth." The nurse will immediately check for 1. the presence of Chvostek's sign. 2. an elevated serum potassium level. 3. bleeding on the patient's dressing. 4. a decreased thyroid hormone level.

the presence of Chvostek's sign.

An esophagram (barium swallow) may show

the protrusion of gastric mucosa throught he esophageal hiatus

In restrictive procedures

the stomach is reduced in size (less food eaten)

The diagnosis of Esoinophilic esophagitis is based on

the symptoms and biopsy findings of eosinophils infiltrating the esophageal tissue obtained from endoscopy

When the stomach is empty of gastric secretions,

the ulcer pain diminishes and ulcer healing begins

A commonly prescribed methylxanthine used as a bronchodilator is: a. albuteral. b. levalbuteral. c. theophylline. d. terbutaline.

theophylline

Chemotherapy and radiation therapy are used together for oral cancer when

there are positive margains, bone erosin, or positive lymph nodes

Starvation-related malnutrition's clinical state is in which

there is chronic starvation without inflammation (e.g. anorexia)

Bacterial pneumonia can be indicated by the presence of all of the following except: a. green, purulent sputum. b. thick, yellow sputum. c. thin, mucoid sputum. d. rusty sputum.

thin, mucoid sputum

The key to accurate diagnosis of IBS is a

thorough history and physical examination. Diagnostic tests are selectively used to rule out other disorders

proteins are essential for

tissue growth, repair, and maintenance; body regulatory functions; and energy production

What is the immediate focus of management of a patient with a perforation

to stop the spillage of gastric or duodenal contents into the peritoneal cavity and restore blood volume. An NG tube is inserted into the stomach to provide continuous aspiration and gastric decompression to stop spillage through the perforation. Circulating blood volume is replaced with LR and albumin solutions

Brachytherapy with implantations of radioactive seeds may be used

to treat early-stage oral cancer

Because the body stores excess fat-soluble vitamins, there is the potential for

toxicity when too much is consumed

What maybe needed if nutrition is disrupted?

treatment via an endoscopic or open approach.

Any portion that comes into contact with gastric secretions is susceptible to

ulcer development

aphthous stomatitis clinical manifestation

ulcers of mouth and lips, causing extreme pain.

Currently, the most commonly used volatile liquid anesthetic agent is: a. ethrane. b. florane. c. nitrous oxide. d. ultrane.

ultrane

The clinical manifestations of stomach cancer can include

unexplained weight loss, early satiety, indigestion, abdominal discomfort or pain, and s/s of anemia

A key characteristic feature of adult respiratory distress syndrome (ARDS) is: a. unresponsive arterial hypoxemia. b. diminished alveolar dilation. c. tachypnea. d. increased PaO2.

unresponsive arterial hypoxemia

What is the best diagnostic tool for stomach cancer?

upper GI endoscopy

Alosteron (lotronex)

used for IBS patients with severe symptoms of pain and diarrhea. restricted access program for women who have not responded to other IBS therapies

The LINX Reflux Management System

used for patients who continue to have symptoms despite maximum medical management. titanium beads with a magnetic core are strung together and implanted into the LES.

Aprepitant (Emend)

used for the prevention of chemotherapy induced and postoperative nausea and vomiting

Dexamethasone (Decadron)

used in the management of both acute and and delayed cancer chemotherap-induced emsis, usually in combination with other antiemetics such as ondansetron (zofran).

Parotitis (inflammaiton of parotid gland, surgical mumps) etiology

usually staphylococcus species

A nurse caring for a patient with a pulmonary embolism understands that a high ventilation-perfusion ratio may exist. This means that: a. perfusion exceeds ventilation. b. there is an absence of perfusion and ventilation. c. ventilation exceeds perfusion. d. ventilation matches perfusion.

ventilation exceeds perfusion

Conjunctivitis what are the two types?

viral and bacterial

For the patient complaining of acute abdominal pain, take

vital signs immediately and again at frequent intervals. Increased pulse and decreasing blood pressure indicate impending shock. an elevated temp suggests an inflammatory or infectious process. altered mental status indicates poor cerebral perfusion

fat soluble vitamins

vitamin a, d, e, and k

Water soluble vitamins

vitamin c and b complex vitamins

Symptoms of dumping syndrome include

vomiting, nausea, weakness, sweating, faintness, and on occasion, diarrhea

The first step in the treatment of obesity is to determine

whether any physical conditions are present that may be causing or contributing to obesity

Radial neck dissection includes

wide excision of the primary lesion with removal of the regional lymph nodes, the deep cervical lymph nodes, and their lymphatic channels -a tracheostomy is commonly performed along with

You are caring for a patient admitted with an exacerbation of asthma. After several treatments, the ABG results are pH 7.40, PaCO2 40 mm Hg, HCO3 24 mEq/L, PaO2 92 mm Hg, and O2 saturation of 99%. You interpret these results as

within normal limits. The normal pH is 7.35 to 7.45. Normal PaCO2 levels are 35 to 45 mm Hg, and HCO3 is 22 to 26 mEq/L. Normal PaO2 is >80 mm Hg. Normal oxygen saturation is >95%. Since the patient's results all fall within these normal ranges, the nurse can conclude that the patient's blood gas results are within normal limits.

Orlistat (xenical)

works by blocking fat breakdown and absorption in the intestine. Some fat soluble vitamin levels may also decrease and may need to be supplemented

In the surgical procedures the fundus of the stomach is

wrapped around the lower portion of the esophagus to reinforce and repair the defective barrier

Retinal detachment is emergent or not?

yes! needs surgical management

Duodenal Ulcer S/S:

• Account for about 80% of all peptic ulcers • Eating decreases pain, because food is present to help buffer the acid, symptoms occur when gastric acid comes in contact with the ulcers, generally 2-5 hours after a • Pain located in the mid-epigastric region beneath the xiphoid process. • There is increased gastric secretions. Associated with weight gain because no pain when eating • The pain is "burning or cramp-like". • They can also produce back pain that can radiate to the shoulder • Well nourished • Antacids alone or in combination with an H2 receptor blocker, as well as food, neutralize the acid to provide relief. • The symptoms are continuous for a few weeks or months and then disappear for a time, only to recur some time later.

Roux-en-Y Gastric Bypass (RYGB) Benefits:

• Better weight loss results than with restrictive procedures • Lower incidence of malnutrition and diarrhea • Rapids improvement of weight-related comorbidities • Good long-term results

Roux-en-Y Gastric Bypass (RYGB):

• Considered as combination of restrictive and malabsorptive surgery. • It is the most common bariatric procedure performed in the United States It is considered the gold standard among bariatric procedures. • Overall, it has low complication rates, has excellent patient tolerance, and sustains long-term weight loss. • Outcomes include improved glucose control with improvement or reversal of diabetes, normalization of BP, decreased total cholesterol, and triglycerides, decreased GERD, and decreased sleep apnea.

Adjustable Gastric Banding (AGB) Benefits:

• Food digestion occurs through normal process • Band can be adjusted to increase or decrease restriction • Surgery can be reversed • Absence of dumping syndrome • Lack of malabsorption • Low complication rates

Adjustable Gastric Banding (AGB) Description:

• Inflatable band encircles stomach • Creation of gastric pouch with 30 mL (1 oz) capacity • Later stretches to 60-90 mL (2-3oz) • Upper gastric pouch connected by very narrow channel to lower section of stomach • Less invasive

Adjustable Gastric Banding (AGB):

• It is the most common restrictive surgery done. • This procedure can be either modified or reversed at a later date if necessary. • It is the preferred option for patients who are surgical risks, because it is a less invasive approach.

Adjustable Gastric Banding (AGB) Disadvantges:

• Some cause N/V initially when eating too much too quickly • Problems with adjustment device • Band may slip or erode into stomach wall • Gastric perforation • Weight loss may be more limited than with other types of surgery

The nurse recognizes that primary manifestations of systolic failure include

↓ EF and ↑ PAWP Systolic heart failure results in systolic failure in the left ventricle (LV). The LV loses its ability to generate enough pressure to eject blood forward through the aorta. This results in increased pulmonary artery wedge pressure (PAWP). The hallmark of systolic failure is a decrease in the left ventricular ejection fraction (EF).

Obese BMI

≥ 30

Morbidly obese BMI:

≥ 40

After teaching about ways to decrease risk factors for CAD, the nurse recognizes that additional instruction is needed when the patient says

"I would like to add weight lifting to my exercise program" Risk factors for coronary artery disease include elevated serum levels of lipids, elevated BP, tobacco use, physical inactivity, obesity, diabetes, metabolic syndrome, certain psychologic states, and elevated homocysteine levels. Weight lifting is not a cardioprotective exercise. An example of health-promoting regular physical activity is brisk walking (3 to 4 miles/hr) for at least 30 minutes five or more times each week.

Leukoplakia

"smoker's patch" a white patch on the mouth mucosa or tongue -results from chronic irritation, especially smoking

A patient has a severe blockage in his right coronary artery. Which heart structures are most likely to be affected by this blockage? (select all that apply)

- AV node - Left ventricle - Right ventricle The right coronary artery (RCA) supplies blood to the right atrium, the right ventricle, and a portion of the posterior wall of the left ventricle. In 90% of people, the RCA supplies blood to the atrioventricular (AV) node, bundle of His, and part of the cardiac conduction system.

Patients with a heart transplantation are at risk for which complications in the first year after transplantation? (select all that apply)

- Infection - Rejection - Sudden cardiac death A variety of complications can occur after heart transplantation. In the first year after transplantation, the major causes of death are acute rejection and infection. Heart transplant recipients also are at risk for sudden cardiac death. Later, malignancy (especially lymphoma) and cardiac vasculopathy (accelerated CAD) are major causes of death.

Examples of Antacids:

-Calcium Carbonate (Tums) -Sodium Bicarbonate (Alka-Seltzer)

Drug alert: Metoclopramide (reglan)

-chronic use or high doses carry the risk of tardive dyskinesia -with discontinuation the tardive dyskinesia persists

Patient position with Entertal feeding

-proper position decreases the risk of aspiration -elevate head of bed @ least 30 degrees, preferably 45 degrees -if intermittent delivery is used, the head should remain elevated for 30 to 60 minutes after feeding

Drug Alert : Promethazine Injection

-should not be administered into an artery or under the skin -when administered IV, it can leach out from the vein and cause serious damage to surround tissue -deep muscle injection is the preferred route

esophageal diverticula occur in three main areas:

1) above the upper esophageal sphincter (zenker's diverticulum), most common 2) near the esophageal midpoint (traction diverticulum) and 3) above the LES (epiphrenic diverticulum)

The most common long term postoperative complications from PUD surgery are

1) dumping syndrome, 2) postprandial hypoglycemia, and 3) bile reflux gastritis

Clinical manifestations of abdominal trauma are

1) guarding and splinting of the abdominal wall (indicating peritonitis) 2) a hard distended abdomen (indicating intraabdominal bleeding) 3) decreased or absent bowel sounds 4) contusions, abrasions, or bruising over the abdomen 5) abdominal pain 6) pain over the scapula caused by irritation of the phrenic nerve by free blood in the abdomen 7) hematemesis or hematuria and 8) signs of hypovolemic shock

The types of surgical procedures done for esophageal cancer are

1) removal of part or all of the esophagus (esophagectomy) with the use of a Dacron graft to replace the resected part 2) resection of a portion of the esophagus and anatomosis of the remaining portion of the stomach (esophagogastrostomy) and 3) resection of a portion of the esophagus and anastomosis of a segment of colon to the remaining portion ( esophagoenterostomy

Various behavioral techniques for patients engaged in a weight loss program include:

1) self monitoring, 2) stimulus control, and 3) rewards

In patients receiving gastrostomy or jejunostomy feeding be alert to two possible problems:

1) ski irritation and 2) dislodgment of the tube

Hiatal hernias are classified into the following two types:

1) sliding and 2) paraesophageal, or rolling

The nurse teaches a client diagnosed with Cushing syndrome about the disease process. Which client statements indicate to the nurse that teaching is effective? (Select all that apply.) 1. "My diagnosis helps to explain why my bones are weak." 2. "I need to increase my daily caloric intake 3. "My health care provider may prescribe a diuretic for me." 4. "I need to avoid people who has infections." 5. "I may have to take potassium supplements" 6. "I feel weak because the syndrome makes my blood glucose low"

1, 3, 4, 5 1) CORRECT — Cushing syndrome results from chronic exposure to excess corticosteroids. Excess corticosteroids adversely affect the bone structure, leading to weakening. 2) INCORRECT - Weight gain is a common symptom of Cushing syndrome. The client should decrease caloric intake. 3) CORRECT— Edema of the lower extremities is common in Cushing syndrome. A potassium-sparing diuretic may be prescribed. 4) CORRECT— The client is at risk for an infection related to lowered resistance to stress and suppression of immune system caused by excessive corticosteroids. 5) CORRECT— Hypokalemia commonly occurs in Cushing syndrome. Potassium supplements are often prescribed. 6) INCORRECT - While weakness is a common symptom, clients diagnosed with Cushing syndrome have hyperglycemia. The weakness is not being caused by hypoglycemia.

The nurse assesses a client with Addison disease. Which finding will the nurse expect the client to exhibit? 1. Muscle cramps, fatigue, and hypotension 2. Shortness of breath, pallor, and hirsutism 3. Rales, maculopapular rash, and weight loss. 4. Hypertension, peripheral edema, and petechiae.

1. Muscle cramps, fatigue, and hypotension 1) CORRECT- The client with Addison disease will demonstrate symptoms of hyponatremia because the disease is characterized by hyposecretion of the adrenal hormones. 2) INCORRECT- Shortness of breath, pallor, and hirutism are not associated with Addison disease. Symptoms of Addison disease include fatigue, weakness, dehydration, decreased blood pressure, "eternal tan, " decreased resistance to physical stress, and alopecia. 3) INCORRECT- Clients with Addison disease show symptoms of dehydration and an "eternal tan. " Rales, maculopapular rash and weight loss are not symptoms of this disorder. 4) INCORRECT- Hypertension and edema are symptoms of Cushing disease, which is caused by hypersecretion of the adrenal hormones.

What are the ranges for fasting glucose for a prediabetic person?

101-125 mg/dL

Hemoglobin WNL - female

11.7 to 16.0

A PEG tube feeding may be started within

2 hours of insertions, although institutionally policies may vary

A 154-lb, 60-year-old woman is being treated for ARDS. The nurse knows that the minimum daily caloric requirement to meet normal requirements is: a. 1,200 to 1,800 calories. b. 1,800 to 2,200 calories. c. 2,000 to 2,400 calories. d. 2,500 to 3,000 calories

2,500 to 3,000 calories

2. An increase in which blood cell indicates an increased rate of erythropoiesis? a. Basophil b. Monocyte c. Reticulocyte d. Lymphocyte

2. c. Increased reticulocytes, or immature RBCs, indicate an increased rate of erythropoiesis or stimulation of erythrocyte (RBC) production by the bone marrow. Basophils release granules that increase allergic and inflammatory responses and are stimulated by granulocyte colony-stimulating factor. Monocytes may become tissue macrophages. Lymphocytes are primarily responsible for the immune response.

Anticoagulant therapy with Heparin is administered in attempt to maintain INR a. 0.5 to 1.0. b. 1.5 to 2.5. c. 2.0 to 2.5. d. 3.0 to 3.5.

2.0-2.5

A pleural effusion results when fluid accumulation in the pleural space is greater than: a. 5 mL. b. 10 mL. c. 15 mL. d. 20 mL.

20 mL

When planning the care of a patient with dehydration, what would the nurse instruct the unlicensed assistive personnel (UAP) to report?

20 mL urine output for 2 consecutive hours The minimal urine output necessary to maintain kidney function is 30 mL/hr. If the output is less than this for 2 consecutive hours, the nurse should be notified so that additional fluid volume replacement therapy can be instituted.

Rule of thumb for calories in losing weight

20 to 25 cal/kg

An average adult requires an estimated how many calories?

20 to 35 cal/kg of body weight per day

With the PACU room scoring guide, a nurse would give a patient an admission cardiovascular score of 2 if the patient's blood pressure is what percentage of his or her preanesthetic level? a. 20% b. 30% to 40% c. 40% to 50% d. Greater than 50%

20%

20. Priority Decision: A patient is undergoing a contrast computed tomography (CT) of the spleen. What is most important for the nurse to ask the patient about before the test? a. Iodine sensitivity b. Prior blood transfusions c. Phobia of confined spaces d. Internal metal implants or appliances

20. a. A contrast CT scan involves the use of an iodine-based dye that could cause a reaction if the patient is sensitive to iodine. Metal implants or internal, appliances and claustrophobia should be determined before magnetic resonance imaging (MRI). Prior blood transfusions are not a factor in this diagnostic test.

What are the ranges for postprandial (after eating) glucose for a diabetic person?

200 mg/dL and above

22. A lymph node biopsy is most often performed to diagnose a. leukemia. b. cause of lymphedema. c. hemorrhagic tendencies. d. neoplastic cells in lymph nodes.

22. d. Lymph node biopsy is usually done to determine whether malignant cells are present in lymph nodes and can be used to diagnose lymphomas as well as metastatic spread from any malignant tumor in the body. Leukemias may infiltrate lymph nodes but biopsy of the nodes is more commonly used to detect any type of neoplastic cells.

23. The patient's laboratory results show a marked decrease in RBCs, WBCs, and platelets. What term should the nurse use when reporting the results to the physician? a. Hemolysis b. Leukopenia c. Pancytopenia d. Thrombocytosis

23. c. Pancytopenia is decreased RBCs, WBCs, and platelets. Hemolysis is RBC destruction. Leukopenia is WBC <4000/μL. Thrombocytosis is increased platelets and thrombocytopenia is decreased platelets.

Rule of thumb for calories for maintaining weight

25 to 30 cal/kg

Overweight BMI:

25.0 - 29.9

A local infiltration anesthetic can last for up to: a. 1 hour. b. 3 hours. c. 5 hours. d. 7 hours

3 hours

The nurse recognizes that a clean-contaminated wound has a relative probability of infection of: a. 1% to 3%. b. 7% to 16%. c. 3% to 7%. d. more than 16%.

3% to 7%

Surgical reduction of nasal fractures is usually performed how long after the fracture? a. Within 24 hours b. 3 to 7 days c. 2 to 3 weeks d. 2 months

3-7 days

Which serum potassium result best supports the rationale for administering a stat dose of potassium chloride 20 mEq in 250 mL of normal saline over 2 hours?

3.1 mEq/L Correct 3.9 mEq/L 4.6 mEq/L 5.3 mEq/L The normal range for serum potassium is 3.5 to 5.0 mEq/L. This IV order provides a substantial amount of potassium. Thus the patient's potassium level must be low. The only low value shown is 3.1 mEq/L.

Electrolyte Review - Potassium - important in cardiac function

3.5 to 5.0

RBCs WNL - female

3.8 to 5.1

Rule of thumb for calories to gain weight

30 to 35 cal/kg

Hematocrit WNL - female

35 to 47%

It is estimated that Mycobacterium tuberculosis infects about what percentage of the world's population? a. 10% b. 25% c. 35% d. 50%

35%

Hematocrit WNL - male

39-50%

4. After a woman had a right breast mastectomy, her right arm became severely swollen. What hematologic problem caused this? a. Lymphedema b. Right-sided heart failure c. Wound on her right hand d. Refusal to use her right arm

4. a. Lymphedema is the obstruction of lymph flow that results in accumulation of lymph fluid for the patient in the right arm following a right-sided breast mastectomy. The other options are not hematologic problems that would cause extreme swelling.

RBCs WNL - male

4.3 to 5.7

Electrolyte Review - Calcium - important in cardiac function

4.5 to 5.5

Health risk increase if the waist circumference is greater than

40 inches in men and greater than 35 inches in women

. A nurse understands that a safe but low level of oxygen saturation provides for adequate tissue saturation but allows no reserve for situations that threaten ventilation. A safe but low oxygen saturation level is: a. 40 mm Hg. b. 75 mm Hg. c. 80 mm Hg. d. 95 mm Hg.

40 mm Hg

White Blood Cells WNL

4000 to 11,000

Chemotherapeutic agents used include

5-fluorouracil (5-FU), methotrexate, cisplatin (platinol), carboplatin (paraplatin), paclitaxel (taxol), docetaxel (taxotere), cetuximab (erbitux), and bleomycin (clenoxane)

5. Which nutrients are essential for red blood cell production (select all that apply)? a. Iron b. Folic acid c. Vitamin C e. Cobalamin aka: Vitamin B12 f. Carbohydrates d. Vitamin D

5. a, b, e. Although all of the listed nutrients are helpful, iron, folic acid, and cobalamin (vitamin B12) are essential for erythropoiesis.

A patient exhibits cyanosis when how much hemoglobin is unoxygenated? a. 0.77 g/dL b. 2.3 g/dL c. 15.0 g/dL d. 5.0 g/dL

5.0 g/dL

A patient is being scheduled for endoscopic retrograde cholangiopancreatography (ERCP) as soon as possible. Which actions from the agency policy for ERCP should the nurse take first? a. Place the patient on NPO status. b. Administer sedative medications. c. Ensure the consent form is signed. d. Teach the patient about the procedure.

A

The patient has chronic venous insufficiency and a venous ulcer. The unlicensed assistive personnel (UAP) decides to apply compression stockings because that is what patients 'always' have ordered. Which assessment finding would indicate the application of compression stockings could harm the patient? A. Rest pain B. High blood pressure C. Elevated blood sugar D. Dry, itchy, flaky skin

A Rest pain occurs as peripheral artery disease (PAD) progresses and involves multiple arterial segments. Compression stockings should not be used on patients with PAD. Elevated blood glucose, possibly indicating uncontrolled diabetes mellitus, and hypertension may or may not indicate arterial problems. Dry, itchy, flaky skin indicates venous insufficiency. The RN should be the one to obtain the order and instruct the UAP to apply compression stockings if they are ordered.

Physiological Integrity 8. After the nurse gives IV atropine to a patient with symptomatic type 1, second-degree atrioventricular (AV) block, which finding indicates that the medication has been effective? a. Increase in the patient's heart rate b. Increase in strength of peripheral pulses c. Decrease in premature atrial contractions d. Decrease in premature ventricular contractions

A Atropine will increase the heart rate and conduction through the AV node. Because the medication increases electrical conduction, not cardiac contractility, the quality of the peripheral pulses is not used to evaluate the drug effectiveness. The patient does not have premature atrial or ventricular contractions. DIF: Cognitive Level: Apply (application) REF: 798 TOP: Nursing Process: Evaluation MSC:

1. To determine whether there is a delay in impulse conduction through the atria, the nurse will measure the duration of the patient's a. P wave. b. Q wave. c. P-R interval. d. QRS complex.

A The P wave represents the depolarization of the atria. The P-R interval represents depolarization of the atria, atrioventricular (AV) node, bundle of His, bundle branches, and the Purkinje fibers. The QRS represents ventricular depolarization. The Q wave is the first negative deflection following the P wave and should be narrow and short. DIF: Cognitive Level: Understand (comprehension) REF: 791 TOP: Nursing Process: Assessment MSC:

SIADH

A condition in which high levels of a hormone cause the body to retain water. SIADH secretion is a common condition in patients who sustain brain injury or those who undergo neurosurgery for brain disorders, it is characterized by hyponatremia and hypo-osmolality of the blood that result from excessive secretion or action of ADH. ADH stimulates reabsorption of water at the collecting duct of the nephron. SIADH causes excess water reabsorption into the bloodstream. This excess water creates hypervolemia, dilutionaly hyponatremia and highly concentrated urine.

A 72-year-old patient was admitted with epigastric pain due to a gastric ulcer. Which patient assessment warrants an urgent change in the nursing plan of care? A) Chest pain relieved with eating or drinking water B) Back pain 3 or 4 hours after eating a meal C) Burning epigastric pain 90 minutes after breakfast D) Rigid abdomen and vomiting following indigestion

A rigid abdomen with vomiting in a patient who has a gastric ulcer indicates a perforation of the ulcer, especially if the manifestations of perforation appear suddenly. Midepigastric pain is relieved by eating, drinking water, or antacids with duodenal ulcers, not gastric ulcers. Back pain 3-4 hours after a meal is more likely to occur with a duodenal ulcer. Burning epigastric pain 1-2 hours after a meal is an expected manifestation of a gastric ulcer related to increased gastric secretions and does not cause an urgent change in the nursing plan of care.

A pt with a history of peptic ulcer disease has presented to the emergency department with complaints of severe abdominal pain and a rigid, boardlike abdomen, prompting the health care team to suspect a perforated ulcer. Which of the following actions should the nurse anticipate? A) Providing IV fluids and inserting a nasogastric tube B) Administering oral bicarbonate and testing the patient's gastric pH level C) Performing a fecal occult blood test and administering IV calcium gluconate D) Starting parenteral nutrition and placing the patient in a high-Fowler's position

A) Providing IV fluids and inserting a nasogastric tube A perforated peptic ulcer requires IV replacement of fluid losses and continued gastric aspiration by NG tube. Nothing is given by mouth and gastric pH testing is not a priority. Calcium gluconate is not a medication directly relevant to the patient's suspected diagnosis and parenteral nutrition is not a priority in the short term.

The nurse is preparing to insert a nasogastric (NG) tube into a 68-year-old female patient who is nauseated and vomiting. She has an abdominal mass and suspected small intestinal obstruction. The patient asks the nurse why this procedure is necessary. What response by the nurse is most appropriate? A) "The tube will help to drain the stomach contents and prevent further vomiting." B) "The tube will push past the area that is blocked and thus help to stop the vomiting." C) "The tube is just a standard procedure before many types of surgery to the abdomen." D) "The tube will let us measure your stomach contents so that we can plan what type of IV fluid replacement would be best."

A) The NG tube is used to decompress the stomach by draining stomach contents and thereby prevent further vomiting. The NG tube will not push past the blocked area. Potential surgery is not currently indicated. The location of the obstruction will determine the type of fluid to use, not measure the amount of stomach contents.

When evaluating the patient's understanding about the care of the ileostomy, what statement by the patient indicates the patient needs more teaching? A) "I will be able to regulate when I have stools." B) "I will be able to wear the pouch until it leaks." C) "Dried fruit and popcorn must be chewed very well." D) "The drainage from my stoma can damage my skin."

A) The ileostomy is in the ileum and drains liquid stool frequently, unlike the colostomy which has more formed stool the further distal the ostomy is in the colon. The ileostomy pouch is usually worn 4-7 days or until it leaks. It must be changed immediately if it leaks because the drainage is very irritating to the skin. To avoid obstruction, popcorn, dried fruit, coconut, mushrooms, olives, stringy vegetables, food with skin, and meats with casings must be chewed extremely well before swallowing because of the narrow diameter of the ileostomy lumen.

When administered a dose of metoclopramide (Reglan), a patient complains of nausea. The nurse would teach the patient to report which of the following potential adverse effects? A) Tremors B) Constipation C) Double vision D) Numbness in the fingers and dose

A) Tremors (Extrapyramidal side effects, including tremors and dyskinesias, may occur as a result of metoclopramide (Reglan) administration.)

A patient with chronic HF and atrial fibrillation is treated with a digitalis glycoside and a loop diuretic. To prevent possible complications of this combination of drugs, what does the nurse need to do (select all that apply)? a. Monitor serum potassium levels. b. Teach the patient how to take a pulse rate. c. Keep an accurate measure of intake and output. d. Teach the patient about dietary restriction of potassium. e. Withhold digitalis and notify health care provider if heart rate is irregular.

A, B

Physiological Integrity OTHER 1. When preparing to defibrillate a patient. In which order will the nurse perform the following steps? (Put a comma and a space between each answer choice [A, B, C, D, E].) a. Turn the defibrillator on. b. Deliver the electrical charge. c. Select the appropriate energy level. d. Place the paddles on the patient's chest. e. Check the location of other staff and call out "all clear."

A, C, D, E, B This order will result in rapid defibrillation without endangering hospital staff. DIF: Cognitive Level: Analyze (analysis) REF: 802 TOP: Nursing Process: Implementation MSC:

When caring for a patient with a biliary obstruction, the nurse will anticipate administering which of the following vitamin supplements (select all that apply)? A. Vitamin A B. Vitamin D C. Vitamin E D. Vitamin K E. Vitamin B

A,B,C,D) Biliary obstruction prevents bile from entering the small intestine and thus prevents the absorption of fat-soluble vitamins. Vitamins A, D, E, and K are all fat-soluble and thus would need to be supplemented in a patient with biliary obstruction.

A patient has just arrived on the unit after a thyroidectomy. Which action should the nurse take first? A. Assess the patient's respiratory effort B. Support the patient's head with pillows C. Observe the dressing for bleeding D. Check the blood pressure and pulse

A. Assess the patient's respiratory effort Airway obstruction is a possible complication after thyroidectomy because of swelling or bleeding at the site or tetany. The priority nursing action is to assess the airway. The other actions are also part of the standard nursing care postthyroidectomy but are not as high of a priority.

The client with a right-sided pneumothorax had chest tubes inserted 2 hours ago. There is no fluctuation in the water-seal chamber of the Pleurovac. Which intervention should the nurse implement first? A. Check for any kinks in the tubing. B. Place the patient on oxygen therapy C. Ask the client to take deep breaths. D. Turn the client from side to side.

A. Check for any kinks in the tubing. Rationale: There is no indication that the patient needs oxygen therapy at this time. The nurse should first determine why there is no tidaling in the water-seal chamber. Since the client just had the chest tubes inserted, it is probably a kink or a dependent loop, or the client is lying on the tubing. The nurse should first check the tubing prior to taking any other action. The nurse should encourage the client to take deep breaths and cough, which may push a clot through the tubing, but should not do so before checking for a kink.

What is the priority action that must be taken for this client? A. Place him in T.B. isolation B. Start a heparin drip STAT C. Administer midazolam (Versed) to reduce his anxiety D. Administer Morphine to reduce his pain

A. Place him in T.B. isolation

An expected patient problem for a patient admitted to the hospital with symptoms of diabetes insipidus is: A. Sleep pattern disturbance related to frequent waking to void B. Excess fluid volume related to intake greater than output C. Risk for impaired skin integrity related to generalized edema D. Impaired gas exchange related to fluid retention in lungs

A. Sleep pattern disturbance related to frequent waking to void

Which of the following is an immediate life-threatening lung condition? A.Tension pneumothorax B. Pneumonia C. Bronchitis D. Pleural effusion

A.Tension pneumothorax

Importance of Hgb A1C? What does it tell us?

A1C measure the amount of glycosylated hemoglobin as a percentage of total hemoglobin (ex: A1C of 6.5% means that 6.5% of the total hemoglobin has glucose attached to it), The amount of hemoglobin that is glycosylated depends on the blood glucose level. It provides a measurement of blood glucose levels over the previous 2 to 3 months, with increases in the A1C reflecting elevated blood glucose levels. It has several advantages over the FPG, including greater convenience, since fasting is not required. For individuals with prediabetes, monitoring the A1C can detect overt diabetes and provide feedback on efforts to prevent diabetes.

A patient admitted with heart failure is anxious and reports shortness of breath. Which nursing actions would be appropriate to alleviate this patient's anxiety (select all that apply.)? A. Administer ordered morphine sulfate. B. Position patient in a semi-Fowler's position. C. Position patient on left side with head of bed flat. D. Instruct patient on the use of relaxation techniques. E. Use a calm, reassuring approach while talking to patient.

ABDE Morphine sulfate reduces anxiety and may assist in reducing dyspnea. The patient should be positioned in semi-Fowler's position to improve ventilation that will reduce anxiety. Relaxation techniques and a calm reassuring approach will also serve to reduce anxiety.

The patient has heart failure (HF) with an ejection fraction of less than 40%. What core measures should the nurse expect to include in the plan of care for this patient (select all that apply.)? A. Left ventricular function is documented. B. Controlling dysrhythmias will eliminate HF. C. Prescription for digoxin (Lanoxin) at discharge D. Prescription for angiotensin-converting enzyme inhibitor at discharge E. Education materials about activity, medications, weight monitoring, and what to do if symptoms worsen

ADE The Joint Commission has identified these three core measures for heart failure patients. Although controlling dysrhythmias will improve CO and workload, it will not eliminate HF. Prescribing digoxin for all HF patients is no longer done because there are newer effective drugs and digoxin toxicity occurs easily related to electrolyte levels and the therapeutic range must be maintained.

Which assessment finding in a patient admitted with acute decompensated heart failure (ADHF) requires the most immediate action by the nurse? a. Oxygen saturation of 88% b. Weight gain of 1 kg (2.2 lb) c. Heart rate of 106 beats/minute d. Urine output of 50 mL over 2 hours

ANS: A A decrease in oxygen saturation to less than 92% indicates hypoxemia. The nurse should administer supplemental oxygen immediately to the patient. An increase in apical pulse rate, 1-kg weight gain, and decreases in urine output also indicate worsening heart failure and require nursing actions, but the low oxygen saturation rate requires the most immediate nursing action.

The nurse determines that additional instruction is needed for a 60-year-old patient with chronic syndrome of inappropriate antidiuretic hormone (SIADH) when the patient says which of the following? a. "I need to shop for foods low in sodium and avoid adding salt to food." b. "I should weigh myself daily and report any sudden weight loss or gain." c. "I need to limit my fluid intake to no more than 1 quart of liquids a day." d. "I will eat foods high in potassium because diuretics cause potassium loss."

ANS: A Patients with SIADH are at risk for hyponatremia, and a sodium supplement may be prescribed. The other patient statements are correct and indicate successful teaching has occurred

To monitor for complications in a patient with type 2 diabetes, which tests will the nurse in the diabetic clinic schedule at least annually (select all that apply)? a. Chest x-ray b. Blood pressure c. Serum creatinine d. Urine for microalbuminuria e. Complete blood count (CBC) f. Monofilament testing of the foot

ANS: B, C, D, F Blood pressure, serum creatinine, urine testing for microalbuminuria, and monofilament testing of the foot are recommended at least annually to screen for possible microvascular and macrovascular complications of diabetes. Chest x-ray and CBC might be ordered if the diabetic patient presents with symptoms of respiratory or infectious problems but are not routinely included in screening

A 42-year-old male patient has had a herniorrhaphy to repair an incarcerated inguinal hernia. Which patient teaching will the nurse provide before discharge? a. Soak in sitz baths several times each day. b. Cough 5 times each hour for the next 48 hours. c. Avoid use of acetaminophen (Tylenol) for pain. d. Apply a scrotal support and ice to reduce swelling.

ANS: D A scrotal support and ice are used to reduce edema and pain. Coughing will increase pressure on the incision. Sitz baths will not relieve pain and would not be of use after this surgery. Acetaminophen can be used for postoperative pain.

Following an acute myocardial infarction, a previously healthy 63-year-old develops clinical manifestations of heart failure. The nurse anticipates discharge teaching will include information about a. digitalis preparations. b. -adrenergic blockers. c. calcium channel blockers. d. angiotensin-converting enzyme (ACE) inhibitors.

ANS: D ACE inhibitor therapy is currently recommended to prevent the development of heart failure in patients who have had a myocardial infarction and as a first-line therapy for patients with chronic heart failure. Digoxin therapy for heart failure is no longer considered a first-line measure, and digoxin is added to the treatment protocol when therapy with other medications such as ACE-inhibitors, diuretics, and -adrenergic blockers is insufficient. Calcium channel blockers are not generally used in the treatment of heart failure. The -adrenergic blockers are not used as initial therapy for new onset heart failure.

A 73-year-old patient with diverticulosis has a large bowel obstruction. The nurse will monitor for a. referred back pain. b. metabolic alkalosis. c. projectile vomiting. d. abdominal distention.

ANS: D Abdominal distention is seen in lower intestinal obstruction. Referred back pain is not a common clinical manifestation of intestinal obstruction. Metabolic alkalosis is common in high intestinal obstruction because of the loss of HCl acid from vomiting. Projectile vomiting is associated with higher intestinal obstruction.

A patient who has just been admitted with pulmonary edema is scheduled to receive the following medications. Which medication should the nurse question before giving? a. Furosemide (Lasix) 60 mg b. Captopril (Capoten) 25 mg c. Digoxin (Lanoxin) 0.125 mg d. Carvedilol (Coreg) 3.125 mg

ANS: D Although carvedilol is appropriate for the treatment of chronic heart failure, it is not used for patients with acute decompensated heart failure (ADHF) because of the risk of worsening the heart failure. The other medications are appropriate for the patient with ADHF.

Which finding by the nurse when assessing a patient with a large pituitary adenoma is most important to report to the health care provider? a. Changes in visual field b. Milk leaking from breasts c. Blood glucose 150 mg/dL d. Nausea and projectile vomiting

ANS: D Nausea and projectile vomiting may indicate increased intracranial pressure, which will require rapid actions for diagnosis and treatment. Changes in the visual field, elevated blood glucose, and galactorrhea are common with pituitary adenoma, but these do not require rapid action to prevent life-threatening complications

Which finding indicates a need to contact the health care provider before the nurse administers metformin (Glucophage)? a. The patient's blood glucose level is 174 mg/dL. b. The patient has gained 2 lb (0.9 kg) since yesterday. c. The patient is scheduled for a chest x-ray in an hour. d. The patient's blood urea nitrogen (BUN) level is 52 mg/dL.

ANS: D The BUN indicates possible renal failure, and metformin should not be used in patients with renal failure. The other findings are not contraindications to the use of metformin

A 44-year-old female patient with Cushing syndrome is admitted for adrenalectomy. Which intervention by the nurse will be most helpful for a nursing diagnosis of disturbed body image related to changes in appearance? a. Reassure the patient that the physical changes are very common in patients with Cushing syndrome. b. Discuss the use of diet and exercise in controlling the weight gain associated with Cushing syndrome. c. Teach the patient that the metabolic impact of Cushing syndrome is of more importance than appearance. d. Remind the patient that most of the physical changes caused by Cushing syndrome will resolve after surgery.

ANS: D The most reassuring communication to the patient is that the physical and emotional changes caused by the Cushing syndrome will resolve after hormone levels return to normal postoperatively. Reassurance that the physical changes are expected or that there are more serious physiologic problems associated with Cushing syndrome are not therapeutic responses. The patient's physiological changes are caused by the high hormone levels, not by the patient's diet or exercise choices

The nurse has been teaching a patient with type 2 diabetes about managing blood glucose levels and taking glipizide (Glucotrol). Which patient statement indicates a need for additional teaching? a. "If I overeat at a meal, I will still take the usual dose of medication." b. "Other medications besides the Glucotrol may affect my blood sugar." c. "When I am ill, I may have to take insulin to control my blood sugar." d. "My diabetes won't cause complications because I don't need insulin."

ANS: D The patient should understand that type 2 diabetes places the patient at risk for many complications and that good glucose control is as important when taking oral agents as when using insulin. The other statements are accurate and indicate good understanding of the use of glipizide

Which patient should the nurse assess first after receiving change-of-shift report? a. 60-year-old patient whose new ileostomy has drained 800 mL over the previous 8 hours b. 50-year-old patient with familial adenomatous polyposis who has occult blood in the stool c. 40-year-old patient with ulcerative colitis who has had six liquid stools in the previous 4 hours d. 30-year-old patient who has abdominal distention and an apical heart rate of 136 beats/minute

ANS: D The patient's abdominal distention and tachycardia suggest hypovolemic shock caused by problems such as peritonitis or intestinal obstruction, which will require rapid intervention. The other patients should also be assessed as quickly as possible, but the data do not indicate any life-threatening complications associated with their diagnoses.

A 28-year-old male patient with type 1 diabetes reports how he manages his exercise and glucose control. Which behavior indicates that the nurse should implement additional teaching? a. The patient always carries hard candies when engaging in exercise. b. The patient goes for a vigorous walk when his glucose is 200 mg/dL. c. The patient has a peanut butter sandwich before going for a bicycle ride. d. The patient increases daily exercise when ketones are present in the urine.

ANS: D When the patient is ketotic, exercise may result in an increase in blood glucose level. Type 1 diabetic patients should be taught to avoid exercise when ketosis is present. The other statements are correct

DIC

Acquired when clotting and hemorrhage occur simultaneously, result of increased protease activity caused by unregulated release of thrombin with subsequent fibrin formation and accelerated fibrinolysis Primary initiator is endothelial damage

A rare condition characterized by an overproduction of growth hormone. It often occurs because of a benign growth hormone- secreting pituitary adenoma. The excess GH results in an overgrowth of soft tissues and bones in the hands, feet, and face. Because the problem develops after epiphyseal closure, the bones of the arms and legs do not grow longer.

Acromegaly

Insidious, goes unnoticed, thickening/ enlargement of face, feet, head, muscle weakness, joint pain, tongue enlargement

Acromegaly. Signs and symptoms.

What are the priority nursing interventions after an abdominal aortic aneurysm repair?

Administration of IV fluids and monitoring of kidney function Postoperative priorities include administration of IV fluids and maintenance of renal perfusion. An adequate blood pressure is important for maintaining graft patency, and administration of IV fluids and blood components (as indicated) is essential for adequate blood flow. The nurse should evaluate renal function by measuring hourly urine output and monitoring daily blood urea nitrogen (BUN) and serum creatinine levels. Irreversible renal failure may occur after aortic surgery, particularly in individuals at high risk.

You are the clinic nurse doing triage when a patient complaining of frequent nose bleeds comes in. During your intake assessment, you ask the patient what medications she is taking. Which medication would put the patient at a higher risk for recurrent epistaxis? Afrin Beconase Sinustop Pro Singulair

Afrin (?)

The patient is having a gastroduodenostomy (Billroth I operation) for stomach cancer. What long-term complication is occurring when the patient reports generalized weakness, sweating, palpitations, and dizziness 15 to 30 minutes after eating? A) Malnutrition B) Bile reflux gastritis C) Dumping syndrome D) Postprandial hypoglycemia

After a Billroth I operation, dumping syndrome may occur 15 to 30 minutes after eating because of the hypertonic fluid going to the intestine and additional fluid being drawn into the bowel. Malnutrition may occur but does not cause these symptoms. Bile reflux gastritis cannot happen when the stomach has been removed. Postprandial hypoglycemia occurs with similar symptoms, but 2 hours after eating.

DKA: Treatment and care for these patients.

Airway - Oxygen IV Normal Saline 1L/hr until BP stabilizes and urine output 60 ml/hr Continuous IV Regular Insulin Monitor & treat K+ imbalances IV 5-10% D/NS & Insulin SQ or IV when blood glucose < 250 Identify & treat underlying cause Correct the metabolic acidosis, so we can give bicarb to correct it

Tonsillitis and Adenoiditis: treatment

Airway maintenance Needle aspiration What post-procedure self-care will you teach?

About 60% of cases of acute rhinosinusitis are caused by bacterial organisms. The antibiotic of choice is: a. Augmentin. b. Amoxil. c. erythromycin. d. septra.

Amoxil

Postoperative care after major abdominal surgery

An NG tube is used to decompress stomach to decrease pressure on the suture line and to allow for resolution of edema and inflammation -observe the gastric aspirate for color, amount, and odor -observe for decreased peristalsis ( abdominal distention and lower abdominal discomfort)

A lower than normal hematocrit can indicate:

An insufficient supply of healthy red blood cells (anemia) A large number of white blood cells due to long-term illness, infection or a white blood cell disorder such as leukemia or lymphoma Vitamin or mineral deficiencies Recent or long-term blood loss

S/S for hypoglycemia:

An irregular heart rhythm Fatigue Pale skin Shakiness Anxiety Sweating Hunger Irritability Confusion Visual disturbances Loss of consciousness Seizures

1) The nurse is administering a dose of digoxin to a patient with heart failure (HF). The nurse would become concerned with the possibility of digitalis toxicity if the patient reported which symptom? A. Muscle aches B. Constipation C. Pounding headache D. Anorexia and nausea

Anorexia, nausea, vomiting, blurred or yellow vision, and cardiac dysrhythmias are all signs of digitalis toxicity. The nurse would become concerned and notify the health care provider if the patient exhibited any of these symptoms.

Otitis Media: Clinical Therepy- Antibiotics

Antibiotic < 6 years = 10 days > 6 years = 5-7 days

You are caring for a patient admitted with heart failure. The morning laboratory results reveal a serum potassium level of 2.9 mEq/L. Which classification of medications should you withhold until consulting with the physician?

Antibiotics Loop diuretics (Correct) Bronchodilators Antihypertensives Loop diuretics are contraindicated during episodes of hypokalemia because these medications cause the kidneys to excrete sodium and potassium.

The nurse is caring for a 4-year-old child who had a tonsillectomy earlier in the day. What is an appropriate beverage to give the child? Chocolate ice cream Orange juice Fruit punch Apple juice

Apple Juice

In planning care and patient teaching for the patient with venous leg ulcers, the nurse recognizes that the most important intervention in healing and control of this condition is

Applying graduated compression stockings Compression is essential for treating chronic venous insufficiency (CVI), healing venous ulcers, and preventing ulcer recurrence. Use of custom-fitted graduated compression stockings is one option for compression therapy.

The nurse is planning patient teaching for a patient who is scheduled for an open hemicolectomy. The nurse intends to address the topics of incision splinting and leg exercises during this teaching session. When is the best time for the nurse to provide teaching? A. Upon the patient's admission to the postanesthesia care unit (PACU) B. When the patient returns from the PACU C. During the intraoperative period D. As soon as possible before the surgical procedure

As soon as possible before the surgical procedure

The nurse knows that a sputum culture is necessary to identify the causative organism for acute tracheobronchitis care. If the culture identifies a fungal agent the nurse knows that it would most likely be: a) Aspergillus. b)Haemophilus. c)Mycoplasma pneumoniae d)Streptococcus pneumoniae.

Aspergillus

Activated Partial Thromboplastin Time (aPTT)

Assessment of intrinsic coagulation by measuring the clotting factors 25-35 seconds

Complications of Diabetes

Autonomic neuropathy: dizziness, syncope Microangiopathy: cerebral infarcts, hemorrhage Hypertension Neuropathy: glomerulosclerosis, chronic kidney disease Atherosclerosis: peripheral vascular disease, myocardial infarct, ischemic heart disease Somatic neuropathy: abnormal sensory/motor function, foot ulcers Gangrene infections GI tract: bladder stasis, infections, erectile dysfunction Delayed gastric emptying, diarrhea, constipation Eye: retinopathy, cataracts, glaucoma

MCH - Mean Corpuscular Hemoglobin

Average weight of Hemoglobin to RBC's. (Hgbx10)/(RBCx10 to the 6th power) WNL - 27 to 34 Decreased = microcytosis Increased = macrocytosis

The nurse is assessing an alert and independent 78-year-old woman for malnutrition risk. The most appropriate initial question is which of the following? a. "How do you get to the store to buy your food?" b. "Can you tell me the food that you ate yesterday?" c. "Do you have any difficulty in preparing or eating food?" d. "Are you taking any medications that alter your taste for food?"

B

The nurse will plan to monitor a patient with an obstructed common bile duct for a. melena. b. steatorrhea. c. decreased serum cholesterol levels. d. increased serum indirect bilirubin levels.

B

Which area of the abdomen will the nurse palpate to assess for splenomegaly? a. Right Upper Quadrant b. Left Upper Quadrant c. Right Lower Quadrant d. Left Lower Quadrant

B

Which information about an 80-year-old man at the senior center is of most concern to the nurse? a. Decreased appetite b. Unintended weight loss c. Difficulty chewing food d. Complaints of indigestion

B

A patient with critical limb ischemia had peripheral artery bypass surgery to improve circulation. What nursing care should be provided on postoperative day 1? A. Keep patient on bed rest. B. Assist patient to walk several times. C. Have patient sit in the chair several times. D. Place patient on their side with knees flexed.

B To avoid blockage of the graft or stent, the patient should walk several times on postoperative day 1 and subsequent days. Having the patient's knees flexed for sitting in a chair or in bed increase the risk of venous thrombosis and may place stress on the suture lines.

The nurse is caring for a patient with hypertension who is scheduled to receive a dose of metoprolol (Lopressor). The nurse should withhold the dose and consult the prescribing physician for which vital sign taken just before administration? A. O2 saturation 93% B. Pulse 48 beats/min C. Respirations 24 breaths/min D.Blood pressure 118/74 mm Hg

B Because metoprolol is a β1-adrenergic blocking agent, it can cause hypotension and bradycardia as adverse effects. The nurse should withhold the dose and consult with the health care provider for parameters regarding pulse rate limits.

A 62-yr-old Hispanic male patient with diabetes mellitus has been diagnosed with peripheral artery disease (PAD). The patient is a smoker with a history of gout. To prevent complications, which factor is priority in patient teaching? A. Gender B. Smoking C. Ethnicity D. Comorbidities

B Smoking is the most significant factor for this patient. PAD is a marker of advanced systemic atherosclerosis. Therefore, tobacco cessation is essential to reduce PAD progression, CVD events, and mortality. Diabetes mellitus and hyperuricemia are also risk factors. Being male or Hispanic are not risk factors for PAD.

A patient with varicose veins has been prescribed compression stockings. Which nursing instruction would be most appropriate? A. "Try to keep your stockings on 24 hours a day, as much as possible." B. "While you're still lying in bed in the morning, put on your stockings." C. "Dangle your feet at your bedside for 5 minutes before putting on your stockings." D. "Your stockings will be most effective if you can remove them for a few minutes several times a day."

B The patient with varicose veins should apply stockings in bed before rising in the morning. Stockings should not be worn continuously and should not be removed several times daily. Dangling at the bedside before application is likely to decrease their effectiveness.

The nurse is caring for a patient with a recent history of deep vein thrombosis (DVT) who is scheduled for an emergency appendectomy. Vitamin K is ordered for immediate administration. The international normalized ratio (INR) value is 1.0. Which nursing action is most appropriate? A. Administer the medication as ordered. B. Hold the medication and record in the electronic medical record. C. Hold the medication until the lab result is repeated to verify results. D. Administer the medication and seek an increased dose from the health care provider.

B Vitamin K is the antidote to warfarin (Coumadin), which the patient has most likely been taking before admission for treatment of DVT. Warfarin is an anticoagulant that impairs the ability of the blood to clot. Therefore, it may be necessary to give vitamin K before surgery to reduce the risk of hemorrhage. However, the INR value is normal, and vitamin K is not required, so the medication would be held and recorded in the electronic medical record.

5.What should the nurse recognize as an indication for the use of dopamine in the care of a patient with heart failure? A) Acute anxiety B) Hypotension and tachycardia C) Peripheral edema and weight gain D) Paroxysmal nocturnal dyspnea (PND)

B Dopamine is a β-adrenergic agonist whose inotropic action is used for treatment of severe heart failure accompanied by hemodynamic instability. Such a state may be indicated by tachycardia accompanied by hypotension. PND, anxiety, edema, and weight gain are common signs and symptoms of heart failure, but these do not necessarily warrant the use of dopamine.

Physiological Integrity 21. A 19-year-old student comes to the student health center at the end of the semester complaining that, "My heart is skipping beats." An electrocardiogram (ECG) shows occasional premature ventricular contractions (PVCs). What action should the nurse take next? a. Start supplemental O2 at 2 to 3 L/min via nasal cannula. b. Ask the patient about current stress level and caffeine use. c. Ask the patient about any history of coronary artery disease. d. Have the patient taken to the hospital emergency department (ED).

B In a patient with a normal heart, occasional PVCs are a benign finding. The timing of the PVCs suggests stress or caffeine as possible etiologic factors. It is unlikely that the patient has coronary artery disease, and this should not be the first question the nurse asks. The patient is hemodynamically stable, so there is no indication that the patient needs to be seen in the ED or that oxygen needs to be administered. DIF: Cognitive Level: Apply (application) REF: 799 OBJ: Special Questions: Prioritization TOP: Nursing Process: Implementation MSC:

Physiological Integrity 16. A patient has ST segment changes that support an acute inferior wall myocardial infarction. Which lead would be best for monitoring the patient? a. I b. II c. V2 d. V6

B Leads II, III, and AVF reflect the inferior area of the heart and the ST segment changes. Lead II will best capture any electrocardiographic (ECG) changes that indicate further damage to the myocardium. The other leads do not reflect the inferior part of the myocardial wall and will not provide data about further ischemic changes in that area. DIF: Cognitive Level: Analyze (analysis) REF: 806 TOP: Nursing Process: Implementation MSC:

Physiological Integrity 5. The nurse notes that a patient's cardiac monitor shows that every other beat is earlier than expected, has no visible P wave, and has a QRS complex that is wide and bizarre in shape. How will the nurse document the rhythm? a. Ventricular couplets b. Ventricular bigeminy c. Ventricular R-on-T phenomenon d. Multifocal premature ventricular contractions

B Ventricular bigeminy describes a rhythm in which every other QRS complex is wide and bizarre looking. Pairs of wide QRS complexes are described as ventricular couplets. There is no indication that the premature ventricular contractions (PVCs) are multifocal or that the R-on-T phenomenon is occurring. DIF: Cognitive Level: Apply (application) REF: 799 TOP: Nursing Process: Assessment MSC:

The results of a patients recent endoscopy indicate the presence of peptic ulcer disease (PUD). Which of the following teaching points should the nurse provide to the pt in light of his new diagnosis? A) "You'll need to drink at least two to three glasses of milk daily." B) "It would likely be beneficial for you to eliminate drinking alcohol." C) "Many people find that a minced or pureed diet eases their sxs of PUD." D) "Your medications should allow you to maintain your present diet while minimizing symptoms."

B) "It would likely be beneficial for you to eliminate drinking alcohol." Although there is no specific recommended dietary modification for PUD, most patients find it necessary to make some sort of dietary modifications to minimize symptoms. Milk may exacerbate PUD and alcohol is best avoided because it can delay healing.

The nurse is planning care for a 68-year-old patient with an abdominal mass and suspected bowel obstruction. Which factor in the patient's history increases the patient's risk for colorectal cancer? A) Osteoarthritis B) History of colorectal polyps C) History of lactose intolerance D) Use of herbs as dietary supplements

B) A history of colorectal polyps places this patient at risk for colorectal cancer. This tissue can degenerate over time and become malignant. Osteoarthritis, lactose intolerance, and the use of herbs do not pose additional risk to the patient.

A 61-year-old patient with suspected bowel obstruction had a nasogastric tube inserted at 4:00 AM. The nurse shares in the morning report that the day shift staff should check the tube for patency at what times? A) 7:00 AM, 10:00 AM, and 1:00 PM B) 8:00 AM, 12:00 PM, and 4:00 PM C) 9:00 AM and 3:00 PM D) 9:00 AM, 12:00 PM, and 3:00 PM

B) A nasogastric tube should be checked for patency routinely at 4-hour intervals. Thus if the tube were inserted at 4:00 AM, it would be due to be checked at 8:00 AM, 12:00 PM, and 4:00 PM.

A patient is seeking emergency care after choking on a piece of steak. The nursing assessment reveals a history of alcoholism, cigarette smoking, and hemoptysis. Which diagnostic study is most likely to be performed on this patient? A) Barium swallow B) Endoscopic biopsy C) Capsule endoscopy D) Endoscopic ultrasonography

B) Because of this patient's history of excessive alcohol intake, smoking, hemoptysis, and the current choking episode, cancer may be present. A biopsy is necessary to make a definitive diagnosis of carcinoma, so an endoscope will be used to obtain a biopsy and observe other abnormalities as well. A barium swallow may show narrowing of the esophagus, but it is more diagnostic for achalasia. An endoscopic ultrasonography may be used to stage esophageal cancer. Capsule endoscopy can show alterations in the esophagus but is more often used for small intestine problems. A barium swallow, capsule endoscopy, and endoscopic ultrasonography cannot provide a definitive diagnosis for cancer when it is suspected.

A patient who is given a bisacodyl (Dulcolax) suppository asks the nurse how long it will take to work. The nurse replies that the patient will probably need to use the bedpan or commode within which time frame after administration? A) 2-5 minutes B) 15-60 minutes C) 2-4 hours D) 6-8 hours

B) Bisacodyl suppositories usually are effective within 15 to 60 minutes of administration, so the nurse should plan accordingly to assist the patient to use the bedpan or commode.

6. A patient with a recent diagnosis of heart failure has been prescribed furosemide. What outcome does the nurse anticipate will occur that demonstrates medication effectiveness? A. Promote vasodilation. B. Reduction of preload. C. Decrease in afterload. D. Increase in contractility.

B) Diuretics such as furosemide are used in the treatment of heart failure to mobilize edematous fluid, reduce pulmonary venous pressure, and reduce preload. They do not directly influence afterload, contractility, or vessel tone.

The pt who is admitted with a diagnosis of diverticulitis and a history of irritable bowel disease and gastroesophageal reflux disease (GERD) has received a dose of Mylanta 30 ml PO. The nurse would evaluate its effectiveness by questioning the patient as to whether which of the following sxs has been resolved? A) Diarrhea B) Heartburn C) Constipation D) Lower abdominal pain

B) Heartburn (Mylanta is an antacid that contains both aluminum and magnesium. It is indicated for the relief of GI discomfort, such as with heartburn associated with GERD.)

The nurse should administer a prn dose of magnesium hydroxide (MOM) after noting which of the following while reviewing a patient's medical record? A) Abdominal pain and bloating B) No bowel movement for 3 days C) A decrease in appetite by 50% over 24 hours D) Muscle tremors and other signs of hypomagnesemia

B) MOM is an osmotic laxative that produces a soft, semisolid stool usually within 15 minutes to 3 hours. This medication would benefit the patient who has not had a bowel movement for 3 days. MOM would not be given for abdominal pain and bloating, decreased appetite, or signs of hypomagnesemia.

The patient with right upper quadrant abdominal pain has an abdominal ultrasound that reveals cholelithiasis. What should the nurse expect to do for this patient? A) Prevent all oral intake. B) Control abdominal pain. C) Provide enteral feedings. D) Avoid dietary cholesterol.

B) Patients with cholelithiasis can have severe pain, so controlling pain is important until the problem can be treated. NPO status may be needed if the patient will have surgery but will not be used for all patients with cholelithiasis. Enteral feedings should not be needed, and avoiding dietary cholesterol is not used to treat cholelithiasis.

The patient had a car accident and was "scared to death." The patient is now reporting constipation. What affecting the gastrointestinal (GI) tract does the nurse know could be contributing to the constipation? A) The patient is too nervous to eat or drink, so there is no stool. B) The sympathetic nervous system was activated, so the GI tract was slowed. C) The parasympathetic nervous system is now functioning to slow the GI tract. D) The circulation in the GI system has been increased, so less waste is removed.

B) The constipation is most likely related to the sympathetic nervous system activation from the stress related to the accident. SNS activation can decrease peristalsis. Even without oral intake for a short time, stool will be formed. The parasympathetic system stimulates peristalsis. The circulation to the GI system is decreased with stress.

Inspection of an older patient's mouth reveals the presence of white, curd-like lesions on the patient's tongue. What is the most likely etiology for this abnormal assessment finding? A) Herpesvirus B) Candida albicans C) Vitamin deficiency D) Irritation from ill-fitting dentures

B) White, curd-like lesions surrounded by erythematous mucosa are associated with oral candidiasis. Herpesvirus causes benign vesicular lesions in the mouth. Vitamin deficiencies may cause a reddened, ulcerated, swollen tongue. Irritation from ill-fitting dentures will cause friable, edematous, painful, bleeding gingivae.

The nurse should recognize that the liver performs which functions (select all that apply)? A) Bile storage B) Detoxification C) Protein metabolism D) Steroid metabolism E) Red blood cell (RBC) destruction

B, C, D) The liver performs multiple major functions that aid in the maintenance of homeostasis. These include metabolism of proteins and steroids as well as detoxification of drugs and metabolic waste products. The Kupffer cells of the liver participate in the breakdown of old RBCs. The liver produces bile, but storage occurs in the gall bladder.

Patients with a heart transplantation are at risk for which complications in the first year after transplantation (select all that apply)? a. Cancer b. Infection c. Rejection d. Vasculopathy e. Sudden cardiac deat

B, C, E

The nurse is providing teaching to a patient recovering from a myocardial infarction. How should resumption of sexual activity be discussed? A. Delegated to the primary care provider B. Discussed along with other physical activities C. Avoided because it is embarrassing to the patient D. Accomplished by providing the patient with written material

B. Although some nurses may not feel comfortable discussing sexual activity with patients, it is a necessary component of patient teaching. It is helpful to consider sex as a physical activity and to discuss or explore feelings in this area when other physical activities are discussed. Although providing the patient with written material is appropriate, it should not replace a verbal dialogue that can address the individual patient's questions and concerns.

After receiving change-of-shift report for several patients with neutropenia, which patient should the nurse assess first? A. A 66-yr-old who has white pharyngeal lesions B. A 33-yr-old with a fever of 100.8 degrees F C. A 23-yr-old who is complaining of severe fatigue D. A 56-yr-old with frequent explosive diarrhea

B. A 33-yr-old with a fever of 100.8 degrees F Any fever in a neutropenic patient indicates infection and can quickly lead to sepsis and septic shock. Rapid assessment and (if prescribed) initiation of antibiotic therapy within 1 hour are needed. The other patients also need to be assessed but do not exhibit symptoms of potentially life-threatening problems.

A patient who had a subtotal thyroidectomy earlier today develops laryngeal stridor and a cramp in the right hand upon returning to the surgical nursing unit. Which collaborative action will the nurse anticipate next? A. Suction the patient's airway. B. Administer IV calcium gluconate. C. Plan for emergency tracheostomy. D. Prepare for endotracheal intubation.

B. Administer IV calcium gluconate The patient's clinical manifestations of stridor and cramping are consistent with tetany caused by hypocalcemia resulting from damage to the parathyroid glands during surgery. Endotracheal intubation or tracheostomy may be needed if the calcium does not resolve the stridor.

The anesthetist is coming to the surgical admissions unit to see a patient prior to surgery scheduled for tomorrow morning. Which of the following is the priority information that the nurse should provide to the anesthetist during the visit? A. Last bowel movement B. Latex allergy C. Number of pregnancies D. Difficulty falling asleep

B. Latex allergy

A nurse is providing preoperative teaching to a patient who will soon undergo a cardiac bypass. The nurse's teaching plan includes exercises of the extremities. What is the purpose of teaching a patient leg exercises prior to surgery? A. Leg exercises increase the patient's muscle mass postoperatively. B. Leg exercises improve circulation and prevent venous thrombosis. C. Leg exercises help to prevent pressure sores to the sacrum and heels. D. Leg exercise help increase the patient's level of consciousness after surgery.

B. Leg exercises improve circulation and prevent venous thrombosis.

The cardiac telemetry unit charge nurse receives status reports from other nursing units about four patients who need cardiac monitoring. Which patient should be transferred to the cardiac unit first? A. Patient with Hashimoto's thyroiditis and a heart rate of 102 B. Patient with tetany who has a new order for IV calcium chloride C. Patient with Cushing syndrome and a blood glucose of 140 mg/dL D. Patient with Addison's disease who takes hydrocortisone twice daily

B. Patient with tetany who has a new order for IV calcium chloride Emergency treatment of tetany requires IV administration of calcium; electrocardiographic monitoring will be required because cardiac arrest may occur if high calcium levels result from too-rapid administration. The information about the other patients indicates that they are more stable than the patient with tetany.

This is the most common cause of cobalamin deficiency. It is caused by an absence of intrinsic factor. The gastric mucosa is not secreting IF because of either gastric mucosal atrophy or autoimmune destruction of parietal cells. This is a disease of insidious onset that begins in middle age or later, with 60 years being the most common age at diagnosis.

B12 deficiency anemia (pernicious anemia)

Tonsillitis and Adenoiditis: Acute

Bacterial: Penicillin or Cephalosporin Viral: Symptomatic support

When a person's blood pressure rises, the homeostatic mechanism to compensate for an elevation involves stimulation of

Baroreceptors that inhibit the sympathetic nervous system, causing vasodilation Baroreceptors in the aortic arch and carotid sinus are sensitive to stretch or pressure within the arterial system. Stimulation of these receptors sends information to the vasomotor center in the brainstem. This results in temporary inhibition of the sympathetic nervous system and enhancement of the parasympathetic influence, which cause a decrease in heart rate and peripheral vasodilation.

Several important risk factors of esophageal cancer include

Barrett's metaplasia, smoking, excessive alcohol intake, and obesity

A patient is recovering from an uncomplicated MI. Which rehabilitation guideline is a priority to include in the teaching plan?

Begin an exercise program that aims for at least five 30-minute sessions per week Physical activity should be regular, rhythmic, and repetitive, with the use of large muscles to build up endurance (e.g., walking, cycling, swimming, rowing). Physical activity sessions should be at least 30 minutes long. Instruct the patient to begin slowly at personal tolerance (perhaps only 5 to 10 minutes) and build up to 30 minutes.

s/s of DIC

Bleeding from venipuncture sites and arterial lines Purpura, petechiae, hematosis Symmetric cyanosis of fingers and toes

RANDI: 1. use electric Razors 2. no Aspirin 3. small gauge Needles 4. Decrease needle sticks 4. protect from Injury

Bleeding precautions for hemophilia and von Willibrand disease

Preop teaching for transspeoidal hypophysectomy for pituitary adenoma the nurse instrust patient to NOT:

Brush teeth 10 days after surgery

What lab value should you ass during upper Gi bleeding?

Bun levels. during a significant hemorrhage, blood proteins are broken down by GI tract bacteria, resulting in elevate BUN levels

A compensatory mechanism involved in HF that leads to inappropriate fluid retention and additional workload of the heart is a. ventricular dilation. b. ventricular hypertrophy. c. neurohormonal response. d. sympathetic nervous system activation.

C

After assisting with a needle biopsy of the liver at a patient's bedside, the nurse should a. put pressure on the biopsy site using a sandbag. b. elevate the head of the bed to facilitate breathing. c. place the patient on the right side with the bed flat. d. check the patient's postbiopsy coagulation studies.

C

Which finding by the nurse during abdominal auscultation indicates a need for a focused abdominal assessment? a. Loud gurgles b. High-pitched gurgles c. Absent bowel sounds d. Frequent clicking sounds

C

Which statement to the nurse from a patient with jaundice indicates a need for teaching? a. "I used cough syrup several times a day last week." b. "I take a baby aspirin every day to prevent strokes." c. "I use acetaminophen (Tylenol) every 4 hours for back pain." d. "I need to take an antacid for indigestion several times a week"

C

While interviewing a 30-year-old man, the nurse learns that the patient has a family history of familial adenomatous polyposis (FAP). The nurse will plan to assess the patient's knowledge about a. preventing noninfectious hepatitis. b. treating inflammatory bowel disease. c. risk for developing colorectal cancer. d. using antacids and proton pump inhibitors.

C

The nurse is caring for a patient who has been receiving warfarin (Coumadin) and digoxin (Lanoxin) as treatment for atrial fibrillation. Because the warfarin has been discontinued before surgery, the nurse should diligently assess the patient for which complication early in the postoperative period until the medication is resumed? A. Decreased cardiac output B. Increased blood pressure C. Cerebral or pulmonary emboli D. Excessive bleeding from incision or IV sites

C Warfarin is an anticoagulant that is used to prevent thrombi from forming on the walls of the atria during atrial fibrillation. When the medication is terminated, thrombi could again form. If one or more thrombi detach from the atrial wall, they could travel as cerebral emboli from the left atrium or pulmonary emboli from the right atrium.

Safe and Effective Care Environment 23. A patient who is on the progressive care unit develops atrial flutter, rate 150, with associated dyspnea and chest pain. Which action that is included in the hospital dysrhythmia protocol should the nurse do first? a. Obtain a 12-lead electrocardiogram (ECG). b. Notify the health care provider of the change in rhythm. c. Give supplemental O2 at 2 to 3 L/min via nasal cannula. d. Assess the patient's vital signs including oxygen saturation.

C Because this patient has dyspnea and chest pain in association with the new rhythm, the nurse's initial actions should be to address the patient's airway, breathing, and circulation (ABC) by starting with oxygen administration. The other actions also are important and should be implemented rapidly. DIF: Cognitive Level: Apply (application) REF: 793 OBJ: Special Questions: Prioritization TOP: Nursing Process: Implementation MSC:

Physiological Integrity 3. A patient has a junctional escape rhythm on the monitor. The nurse will expect the patient to have a heart rate of _____ beats/minute. a. 15 to 20 b. 20 to 40 c. 40 to 60 d. 60 to 100

C If the sinoatrial (SA) node fails to discharge, the atrioventricular (AV) node will automatically discharge at the normal rate of 40 to 60 beats/minute. The slower rates are typical of the bundle of His and the Purkinje system and may be seen with failure of both the SA and AV node to discharge. The normal SA node rate is 60 to 100 beats/minute. DIF: Cognitive Level: Understand (comprehension) REF: 797 TOP: Nursing Process: Assessment MSC:

The patient tells the nurse she had a history of abdominal pain, so she had a surgery to make an opening into the common bile duct to remove stones. The nurse knows that this surgery is called a A) colectomy B) cholecystectomy C) choledocholithotomy D) choledochojejunostomy

C) A choledocholithotomy is an opening into the common bile duct for the removal of stones. A colectomy is the removal of the colon. The cholecystectomy is the removal of the gallbladder. The choledochojejunostomy is an opening between the common bile duct and the jejunum.

A patient with cholelithiasis needs to have the gallbladder removed. Which patient assessment is a contraindication for a cholecystectomy? A) Low-grade fever of 100° F and dehydration B) Abscess in the right upper quadrant of the abdomen C) Activated partial thromboplastin time (aPTT) of 54 seconds D) Multiple obstructions in the cystic and common bile duct

C) An aPTT of 54 seconds is above normal and indicates insufficient clotting ability. If the patient had surgery, significant bleeding complications postoperatively are very likely. Fluids can be given to eliminate the dehydration; the abscess can be assessed, and the obstructions in the cystic and common bile duct would be relieved with the cholecystectomy.

When assessing a patient's abdomen, what would be most appropriate for the nurse to do? A) Palpate the abdomen before auscultation. B) Percuss the abdomen before auscultation. C) Auscultate the abdomen before palpation. D) Perform deep palpation before light palpation.

C) During examination of the abdomen, auscultation is done before percussion and palpation because these latter procedures may alter the bowel sounds.

A female patient has a sliding hiatal hernia. What nursing interventions will prevent the symptoms of heartburn and dyspepsia that she is experiencing? A) Keep the patient NPO. B) Put the bed in the Trendelenberg position. C) Have the patient eat 4 to 6 smaller meals each day. D) Give various antacids to determine which one works for the patient.

C) Eating smaller meals during the day will decrease the gastric pressure and the symptoms of hiatal hernia. Keeping the patient NPO or in a Trendelenberg position are not safe or realistic for a long period of time for any patient. Varying antacids will only be done with the care provider's prescription, so this is not a nursing intervention.

The condition of a patient who has cirrhosis of the liver has deteriorated. Which diagnostic study would help determine if the patient has developed liver cancer? A) Serum α-fetoprotein level B) Ventilation/perfusion scan C) Hepatic structure ultrasound D) Abdominal girth measurement

C) Hepatic structure ultrasound, CT, and MRI are used to screen and diagnose liver cancer. Serum α-fetoprotein level may be elevated with liver cancer or other liver problems. Ventilation/perfusion scans do not diagnose liver cancer. Abdominal girth measurement would not differentiate between cirrhosis and liver cancer

A patient who is scheduled for surgery with general anesthesia in 1 hour is observed with a moist, but empty water glass in his hand. Which assessment finding may indicate that the patient drank a glass of water? A) Flat abdomen without movement upon inspection B) Tenderness at left upper quadrant upon palpation C) Easily heard, loud gurgling in the right upper quadrant D) High-pitched, hollow sounds in the left upper quadrant

C) If the patient drank water on an empty stomach, gurgling can be assessed without a stethoscope or assessed with auscultation. High-pitched, hollow sounds are tympanic and indicate an empty cavity. A flat abdomen and tenderness do not indicate that the patient drank a glass of water.

Following administration of a dose of metoclopramide (Reglan) to the patient, the nurse determines that the medication has been effective when what is noted? A) Decreased blood pressure B) Absence of muscle tremors C) Relief of nausea and vomiting D) No further episodes of diarrhea

C) Metoclopramide is classified as a prokinetic and antiemetic medication. If it is effective, the patient's nausea and vomiting should resolve. Metoclopramide does not affect blood pressure, muscle tremors, or diarrhea.

The patient receiving chemotherapy rings the call bell and reports an onset of nausea. The nurse should prepare a prn dose of which of the following medications? A) Morphine sulfate B) Zolpidem (Ambien) C) Ondansetron (Zofran) D) Dexamethasone (Decadron)

C) Ondansetron (Zofran) (Ondansetron is a 5-HT3 receptor antagonist antiemetic that is especially effective in reducing cancer chemotherapy-induced nausea and vomiting.)

When caring for the patient with heart failure, the nurse knows that which gastrointestinal process is most dependent on cardiac output and may affect the patient's nutritional status? A) Ingestion B) Digestion C) Absorption D) Elimination

C) Substances that interface with the absorptive surfaces of the GI tract (primarily in the small intestine) diffuse across the intestinal membranes into intestinal capillaries and are then carried to other parts of the body for use in energy production. The cardiac output provides the blood flow for this absorption of nutrients to occur.

A patient who had bladder surgery 2 days ago develops acute decompensated heart failure (ADHF) with severe dyspnea. Which action by the nurse would be indicated first? A. Perform a bladder scan to assess for urinary retention. B. Restrict the patient's oral fluid intake to 500 mL per day. C. Assist the patient to a sitting position with arms on the overbed table. D. Instruct the patient to use pursed-lip breathing until the dyspnea subsides.

C, dyspnea subsides. The nurse should place the patient with ADHF in a high Fowler's position with the feet horizontal in the bed or dangling at the bedside. This position helps decrease venous return because of the pooling of blood in the extremities. This position also increases the thoracic capacity, allowing for improved ventilation. Pursed-lip breathing helps with obstructive air trapping but not with acute pulmonary edema. Restricting fluids takes considerable time to have an effect.

Which clinical manifestations of inflammatory bowel disease are common to both patients with ulcerative colitis (UC) and Crohn's disease (select all that apply)? A) Restricted to rectum B) Strictures are common. C) Bloody, diarrhea stools D) Cramping abdominal pain E) Lesions penetrate intestine.

C, D) Clinical manifestations of UC and Crohn's disease include bloody diarrhea, cramping abdominal pain, and nutritional disorders. Intestinal lesions associated with UC are usually restricted to the rectum before moving into the colon. Lesions that penetrate the intestine or cause strictures are characteristic of Crohn's disease.

You are caring for a patient with ADHF who is receiving IV dobutamine (Dobutrex). You know that this drug is ordered because it (select all that apply) a. increases SVR. b. produces diuresis. c. improves contractility. d. dilates renal blood vessels. e. works on the β1 -receptors in the heart.

C, E

A postoperative patient asks the nurse why the physician ordered daily administration of enoxaparin (Lovenox). Which reply by the nurse is most appropriate? A. "This medication will help prevent breathing problems after surgery, such as pneumonia." B. "This medication will help lower your blood pressure to a safer level, which is very important after surgery." C. "This medication will help prevent blood clots from forming in your legs until your level of activity, such as walking, returns to normal." D. "This medication is a narcotic pain medication that will help take away any muscle aches caused by positioning on the operating room table."

C. Enoxaparin is an anticoagulant that is used to prevent DVTs postoperatively. All other options do not describe the action or purpose of enoxaparin.

Postoperative care of a patient undergoing coronary artery bypass graft (CABG) surgery includes monitoring for which common complication? A. Dehydration B. Paralytic ileus C. Atrial dysrhythmias D. Acute respiratory distress syndrome

C. Postoperative dysrhythmias, specifically atrial dysrhythmias, are common in the first 3 days after CABG surgery. Although the other complications could occur, they are not common complications.

The UAP is taking orthostatic vital signs. In the supine position, the blood pressure (BP) is 130/80 mm Hg, and the heart rate (HR) is 80 beats/min. In the sitting position, the BP is 140/80, and the HR is 90 beats/min. Which action should the nurse instruct the UAP to take next? A. Repeat BP and HR in this position. B. Record the BP and HR measurements. C. Take BP and HR with patient standing. D. Return the patient to the supine position

C. The vital signs taken do not reflect orthostatic changes, so the UAP will continue with the measurements while the patient is standing. There is no need to repeat or delay the readings. The patient does not need to return to the supine positon. When assessing for orthostatic changes, the UAP will take the BP and pulse in the supine position, then place the patient in a sitting position for 1 to 2 minutes and repeat the readings, and then reposition to the standing position for 1 to 2 minutes and repeat the readings. Results consistent with orthostatic changes would have a decrease of 20 mm Hg or more in systolic BP, a decrease of 10 mm Hg or more in diastolic BP, and/or an increase in HR of greater than or equal to 20 beats/min with position changes.

Postoperative care of a patient undergoing coronary artery bypass graft (CABG) surgery includes monitoring for which common complication? A. Dehydration B. Paralytic ileus C. Atrial dysrhythmias D. Acute respiratory distress syndrome

C. Postoperative dysrhythmias, specifically atrial dysrhythmias, are common in the first 3 days after CABG surgery. Although the other complications could occur, they are not common complications.

The most common source of a pulmonary embolism is A. The heart. B. The lungs C. Deep leg veins D. Carotid artery

C. Deep leg veins

Which patient statement to the nurse indicates a need for additional instruction about taking oral ferrous sulfate? A. I should take the iron with orange juice about an hour before eating B. I will take a stool softener if I feel constipated occasionally C. I will call my health care provider if my stools turn black D. I should increase my fluid and fiber intake while I am taking iron tablets.

C. I will call my health care provider if my stools turn black It is normal for the stools to appear black when a patient is taking iron, and the patient should not call the health care provider about this. The other patient statements are correct.

A patient with diabetic ketoacidosis is brought to the emergency department. Which prescribed action should the nurse implement first? A. Give sodiuim bicarbonate 50 mEq IV push B. Administer regular insulin 10 U by IV push C. Infuse 1L of normal saline per hour D. Start a regular insulin infusion at 0.1 units/kg/hr.

C. Infuse 1L of normal saline per hour The most urgent patient problem is the hypovolemia associated with diabetic ketoacidosis (DKA), and the priority is to infuse IV fluids. The other actions can be done after the infusion of normal saline is initiated.

A patient screened for diabetes at a clinic has a fasting plasma glucose level of 120 mg/dL (6.7 mmol/L). The nurse will plan to teach the patient about A. Effects of oral hypoglycemic medications B. Using low doses of regular insulin C. Lifestyle changes to lower blood glucose D. Self-monitoring of blood glucose

C. Lifestyle changes to lower blood glucose The patient's impaired fasting glucose indicates prediabetes (diabetes would be indicated by multiple fasting glucose of 126 mg/dL or greater), and the patient should be counseled about lifestyle changes to prevent the development of type 2 diabetes. The patient with prediabetes does not require insulin or oral hypoglycemics for glucose control and does not need to self-monitor blood glucose.

Which laboratory value reported to the nurse by the unlicensed assistive personnel (UAP) indicates an urgent need for the nurse's assessment of the patient? A. Bedtime glucose of 140 mg/dL B. 2-hr postprandial glucose of 220 mg/dL C. Noon blood glucose of 52 mg/dL D. Fasting blood glucose of 130 mg/dL

C. Noon blood glucose of 52 mg/dL The nurse should assess the patient with a blood glucose level of 52 mg/dL for symptoms of hypoglycemia and give the patient a carbohydrate-containing beverage such as orange juice. The other values are within an acceptable range or not immediately dangerous for a patient with diabetes.

Your client presents to the ED with shortness of breath, tachycardia, fever of 103, WBC 18, and oxygen saturation of 86% on room air. A chest xray is taken which shows patchy infiltrates/consolidation. Which of the following conditions most likely defines the condition this client is experiencing? A. Pneumothorax B. Bronchitis C. Pneumonia D. Pulmonary embolism

C. Pneumonia

Which action by a patient indicates that the home health nurse's teaching about glargine and regular insulin has been successful? A. The patient draws up the regular insulin and then the glargine in the same syringe B. The patient administers the glargine 30 minutes before each meal C. The patient discards the open vials of glargine and regular insulin after 4 weeks (30 days) D. The patient's family prefills the syringes with the mix of insulins weekly

C. The patient discards the open vials of glargine and regular insulin after 4 weeks (30 days) Insulin can be stored at room temperature for 4 weeks. Glargine should not be mixed with other insulins or prefilled and stored. Short-acting regular insulin or is administered 30 min before meals, and glargine is given once daily.

Which information is most important for the nurse to communicate rapidly to the health care provider about a patient admitted with possible syndrome of inappropriate antidiuretic hormone (SIADH)? A. The patient has a recent weight gain of 9 lb B. The patient has a urine specific gravity of 1.025 C. The patient has a serum sodium level of 118 mEq/L D. The patient complains of dyspnea with activity

C. The patient has a serum sodium level of 118 mEq/L A serum sodium of less than 120 mEq/L increases the risk for complications such as seizures and needs rapid correction. The other data are not unusual for a patient with SIADH and do not indicate the need for rapid action.

Your client has had a chest tube for 3 days. You walk into the room and notice that there is continuous bubbling in the water-seal chamber of the system. As a nurse who has learned about chest tubes, what has occurred? A. Fluid drained out of the chest tube at that moment B. There is a kink in the tubing C. There is an air leak in the system D. That is normal fluctuation of the chest tube

C. There is an air leak in the system

PGJ Tube:

Can be used long term, placed into the jejunum (more used for those with reflux so it bypasses it and avoids aspiration), placed surgically and is on the outside of body

Oral candidiasis etiology

Candida albicans (a yeastlike fungus), delilitation, prlonged high dose antibiotic or corticosteroid therapy

While eating dinner at a restaurant the nurse notices a person puts his hand to his throat. After administering the Heimlich maneuver the person falls to the floor, the nurse should: Check for the pulse Assess oral cavity for object Attempt to ventilate Administer five abdominal thrusts

Check for pulse

Residual volume in Enteral feeding

Check gastric residual volumes every 4 hours during the first 48 hours for gastrically fed patients. After goal is achieved, gastric residual monitoring may be decreased to every 6 to 8 hours in non-crically ill patients or continued every 4 hours in crically ill patients

is most commonly caused by pernicious anemia, which results in poor cobalamin absorption through the gastrointestinal (GI) tract. Parenteral or intranasal administration of cobalamin is the treatment of choice.

Cobalamin (vitamin B12) deficiency

UTIs, pneumonia, sepsis, acute illness, newly diagnosed type 2 diabetes r/t impaired thirst and inability to replace fluids

Common cause of HHS:

Neutropenia

Condition in which Neutrophils are less than 1000

The first priority of interprofessional care of a patient with a suspected acute aortic dissection is to

Control BP The initial goals of therapy for acute aortic dissection without complications are blood pressure (BP) control and pain management. BP control reduces stress on the aortic wall by reducing systolic BP and myocardial contractility.

When preparing a patient for a capsule endoscopy study, what should the nurse do? a. Ensure the patient understands the required bowel preparation. b. Have the patient return to the procedure room for removal of the capsule. c. Teach the patient to maintain a clear liquid diet throughout the procedure. d. Explain to the patient that conscious sedation will be used during placement of the capsule.

Correct answer: a Rationale: A capsule endoscopy study involves the patient performing a bowel prep to cleanse the bowel before swallowing the capsule. The patient will be on a clear liquid diet for 1 to 2 days before the procedure and will remain NPO for 4 to 6 hours after swallowing the capsule. The capsule is disposable and will pass naturally with the bowel movement, although the monitoring device will need to be removed.

The nurse is teaching a female patient with type 1 diabetes mellitus about nutrition before discharge. She had surgery to revise a lower leg stump with a skin graft. What food should the nurse teach the patient to eat to best facilitate healing? a. Non-fat milk b. Chicken breast c. Fortified oatmeal d. Olive oil and nuts

Correct answer: b Rationale: High quality protein such as chicken breast is important for tissue repair. Although the non-fat milk, nuts, and fortified oatmeal have some protein, they do not have as much as the chicken breast.

A patient is admitted to the hospital with left upper quadrant (LUQ) pain. What may be a possible source of the pain? a. Liver b. Pancreas c. Appendix d. Gallbladder

Correct answer: b Rationale: The pancreas is located in the left upper quadrant, the liver is in the right upper quadrant, the appendix is in the right lower quadrant, and the gallbladder is in the right upper quadrant.

Which statement best describes the etiology of obesity? a. Obesity primarily results from a genetic predisposition. b. Psychosocial factors can override the effects of genetics in the etiology of obesity. c. Obesity is the result of complex interactions between genetic and environmental factors. d. Genetic factors are more important than environmental factors in the etiology of obesity.

Correct answer: c Rationale: The cause of obesity involves significant genetic and biologic susceptibility factors that are highly influenced by environmental and psychosocial factors.

When teaching a patient about weight reduction diets, the nurse teaches the patient that an appropriate single serving of a food is a. a 6-inch bagel. b. 1 cup of chopped vegetables. c. a piece of cheese the size of three dice. d. a chicken breast the size of a deck of cards.

Correct answer: d Rationale: A chicken breast the size of a deck of cards is about 3 oz, a recommended portion size of meat. Other normal portions include a 3-inch bagel, 1/2 cup of chopped vegetables, and a piece of cheese the size of six dice.

To evaluate the effect of nutritional interventions for a patient with protein-calorie malnutrition, what is the best indicator for the nurse to use? a. Height and weight b. Body mass index (BMI) c. Weight in relation to ideal body weight d. Mid-upper arm circumference and triceps skinfold

Correct answer: d Rationale: Anthropometric measurements, including mid-upper arm circumference and triceps skinfold measurements, are good indicators of lean body mass and skeletal protein reserves and are valuable in evaluating persons who may have been or are being treated for acute protein malnutrition. The other measurements do not specifically address muscle mass.

The nurse is reviewing the home medication list for a 44-year-old man admitted with suspected hepatic failure. Which medication could cause hepatotoxicity? a. Nitroglycerin b. Digoxin (Lanoxin) c. Ciprofloxacin (Cipro) d. Acetaminophen (Tylenol)

Correct answer: d Rationale: Many chemicals and drugs are potentially hepatotoxic (see Table 39-6) and result in significant patient harm unless monitored closely. For example, chronic high doses of acetaminophen and nonsteroidal antiinflammatory drugs (NSAIDs) may be hepatotoxic.

What is a clinical manifestation of age-related changes in the GI system that the nurse may find in an older patient? a. Gastric hyperacidity b. Intolerance to fatty foods c. Yellowish tinge to the skin d. Reflux of gastric contents into the esophagus

Correct answer: d Rationale: There is decreased tone of the lower esophageal sphincter with again and regurgitation of gastric contents back into the esophagus occurs, causing heartburn and belching. There is a decrease in hydrochloric acid secretion with aging. Jaundice and intolerance to fatty foods are symptoms of liver or gallbladder disease and are not normal age-related findings.

Wich of the following criteria must be met for a diagnosis of metabolic syndrome (select all that apply)? a. Hypertension b. Elevated triglycerides c. Elevated plasma glucose d. Increased waist circumference e. Decreased low-density lipoproteins

Correct answers: a, b, c, d Rationale: Three of the following five criteria must be met for a diagnosis of metabolic syndrome: • Waist circumference of 40 inches or more in men and 35 inches or more in women • Triglyceride levels higher than 150 mg/dL, or need for drug treatment for elevated triglyceride levels • High-density lipoprotein (HDL) cholesterol levels lower than 40 mg/dL in men and lower than 50 mg/dL in women, or need for drug treatment for reduced HDL cholesterol levels • Blood pressure: 130 mm Hg or higher systolic or 85 mm Hg or higher diastolic, or need for drug treatment for hypertension • Fasting blood glucose level of 110 mg/dL or higher, or need for drug treatment for elevated glucose levels

Which teaching points are important when providing information to a patient with metabolic syndrome (select all that apply)? a. Stop smoking. b. Monitor weight daily. c. Increase level of activity. d. Decrease saturated fat intake. e. Reduce weight and maintain lower weight. f. Check blood glucose each morning prior to eating.

Correct answers: a, c, d, e Rationale: Patients with metabolic syndrome need to lower their risk factors by reducing and maintaining weight, increasing physical activity, establishing healthy diet habits, and smoking cessation. Some patients with metabolic syndrome are diabetic and would need to monitor glucose levels frequently. When monitoring weight reduction, it is recommended to check weight weekly, not daily.

Aphthous stomatitis treatment

Corticosteroids tetracycline oral suspension

S: skin is fragile T: truncal obesity R: round face/ moon face E: ecymosis S: striae S: sugar is high E: excess of body hair D: deformity/depression/ buffalo hump

Cushings syndrome

A 42-year-old woman is admitted to the outpatient testing area for an ultrasound of the gallbladder. Which information obtained by the nurse indicates that the ultrasound may need to be rescheduled? a. The patient took a laxative the previous evening. b. The patient had a high-fat meal the previous evening. c. The patient has a permanent gastrostomy tube in place. d. The patient ate a low-fat bagel 4 hours ago for breakfast.

D

When caring for a patient with a history of a total gastrectomy, the nurse will monitor for a. constipation. b. dehydration. c. elevated total serum cholesterol. d. cobalamin (vitamin B12) deficiency.

D

Physiological Integrity 15. When analyzing the rhythm of a patient's electrocardiogram (ECG), the nurse will need to investigate further upon finding a(n) a. isoelectric ST segment. b. P-R interval of 0.18 second. c. Q-T interval of 0.38 second. d. QRS interval of 0.14 second.

D Because the normal QRS interval is 0.04 to 0.10 seconds, the patient's QRS interval of 0.14 seconds indicates that the conduction through the ventricular conduction system is prolonged. The P-R interval and Q-T interval are within normal range, and ST segment should be isoelectric (flat). DIF: Cognitive Level: Apply (application) REF: 791 TOP: Nursing Process: Assessment MSC:

The nurse would question the use of which cathartic agent in a patient with renal insufficiency? A) Bisacodyl (Dulcolax) B) Lubiprostone (Amitiza) C) Cascara sagrada (Senekot) D) Magnesium hydroxide (Milk of Magnesia)

D) Milk of Magnesia may cause hypermagnesemia in patients with renal insufficiency. The nurse should question this order with the health care provider. Bisacodyl, lubiprostone, and cascara sagrada are safe to use in patients with renal insufficiency as long as the patient is not currently dehydrated.

A pt reports having dry mouth and asks for some liquid to drink. The nurse reasons that this symptom can most likely be attributed to a common adverse effect of which of the following medications? A) Digoxin (Lanoxin) B) Cefotetan (Cefotan) C) Famotidine (Pepcid) D) Promethazine (Phenergan)

D) Promethazine (Phenergan) A common adverse effect of promethazine, an antihistamine antiemetic agent, is dry mouth; another is blurred vision.

A patient who has hepatitis B surface antigen (HBsAg) in the serum is being discharged with pain medication after knee surgery. Which medication order should the nurse question because it is most likely to cause hepatic complications? A) Tramadol (Ultram) B) Hydromorphone (Dilaudid) C) Oxycodone with aspirin (Percodan) D) Hydrocodone with acetaminophen (Vicodin)

D) The analgesic with acetaminophen should be questioned because this patient is a chronic carrier of hepatitis B and is likely to have impaired liver function. Acetaminophen is not suitable for this patient because it is converted to a toxic metabolite in the liver after absorption, increasing the risk of hepatocellular damage.

The nurse is teaching a group of high school students about the prevention of food poisoning. Which comment by the student shows understanding of foodborne illness protection? A) "We like to mix up the ingredients so the flavors will melt before we cook our beef stew." B) "For a snack, I like to eat raw cookie dough from the package instead of baking the cookies." C) "We only have one cutting board, so we cut up our chicken and salad vegetables at the same time." D) "When they gave me a pink hamburger I sent it back and asked for a new bun and clean plate."

D) The student who did not accept the pink hamburger and asked for a new bun and clean plate understood that the pink meat may not have reached 160° and could be contaminated with bacteria. Mixing ingredients and leaving them long enough for the flavors to melt, eating raw cookie dough from a refrigerated package, and only using one cutting board without washing it with hot soapy water between the chicken and salad vegetables could all lead to food poisoning from contamination.

The ED nurse has inspected, auscultated, and palpated the abdomen with no obvious abnormalities, except pain. When the nurse palpates the abdomen for rebound tenderness, there is severe pain. The nurse should know that this could indicate what problem? A) Hepatic cirrhosis B) Hypersplenomegaly C) Gall bladder distention D) Peritoneal inflammation

D) When palpating for rebound tenderness, the problem area of the abdomen will produce pain and severe muscle spasm when there is peritoneal inflammation. Hepatic cirrhosis, hypersplenomegaly, and gall bladder distention do not manifest with rebound tenderness.

A patient experienced sudden cardiac death (SCD) and survived. Which preventive treatment should the nurse expect to be implemented? A. External pacemaker B. An electrophysiologic study (EPS) C. Medications to prevent dysrhythmias D. Implantable cardioverter-defibrillator (ICD)

D. An ICD is the most common approach to preventing recurrence of SCD. An external pacemaker may be used in the hospital but will not be used for the patient living daily life at home. An EPS may be done to determine if a recurrence is likely and determine the most effective medication treatment. Medications to prevent dysrhythmias are used but are not the best prevention of SCD.

A male patient who has coronary artery disease (CAD) has serum lipid values of low-density lipoprotein (LDL) cholesterol of 98 mg/dL and high-density lipoprotein (HDL) cholesterol of 47 mg/dL. What should the nurse include in patient teaching? A. Consume a diet low in fats. B. Reduce total caloric intake. C. Increase intake of olive oil. D. The lipid levels are normal.

D. For men, the recommended LDL is less than 100 mg/dL, and the recommended level for HDL is greater than 40mg/dL. His normal lipid levels should be included in the patient teaching and encourage him to continue taking care of himself. Assessing his need for teaching related to diet should also be done.

The nurse knows that all except which of the following is a chest tube rule: A. Mark drainage amounts each shift or more frequently as needed. B. Keep below level of client's chest C. Keep tubing straight D. Aggressively strip tubing every hour

D. Aggressively strip tubing every hour

The recovery room nurse is admitting a patient from the OR following the patient's successful splenectomy. What is the first assessment that the nurse should perform on this newly admitted patient? A. Heart rate and rhythm B. Skin integrity C. Core body temperature D. Airway patency

D. Airway patency

Which intervention will the nurse include in the plan of care for a patient with syndrome of inappropriate antidiuretic hormone (SIADH)? A. Monitor for increasing peripheral edema B. Keep head of bed elevated to 30 degrees C. Encourage fluids to 2 to 3 L/day D. Offer the patient hard candies to suck on

D. Offer the patient hard candies to suck on Sucking on hard candies decreases thirst for a patient on fluid restriction. Patients with SIADH are on fluid restrictions of 800 to 1000 mL/day. Peripheral edema is not seen with SIADH. The head of the bed is elevated no more than 10 degrees to increase left atrial filling pressure and decrease antidiuretic hormone (ADH) release.

Which prescribed medication should the nurse expect will have rapid effects on a patient admitted to the emergency department in thyroid storm? A. Iodine B. Methimazole C. Propylthiouracil D. Propranolol (Inderal)

D. Propranolol (Inderal) β-Adrenergic blockers work rapidly to decrease the cardiovascular manifestations of thyroid storm. The other medications take days to weeks to have an impact on thyroid function.

Which problem reported by a patient with hemophilia is most important for the nurse to communicate to the health care provider? A. Leg bruises B. Bleeding gums C. Skin abrasions D. Tarry stools

D. Tarry stools Melena is a sign of gastrointestinal bleeding and requires collaborative actions such as checking hemoglobin and hematocrit and administration of coagulation factors. The other problems indicate a need for patient teaching about how to avoid injury but are not indicators of possible serious blood loss.

Is caused by a deficiency of production or secretion of ADH or a decreased renal response to ADH. The decrease in ADH results in fluid and electrolyte imbalances caused by increased urine output and increased plasma osmolality. Depending on the cause, this could be transient or a chronic, lifelong condition

DI

Decrease in Sp. Gr, increase in serum osmolality, polyuria, polydipsia. Lower blood pressure, could be related to drugs, patients urinate a lot; anywhere from 4 L- 24L a day; end up being dehydrated and crave water, become hypernatremic

DI: What are defining characteristics

at dawn hyperglycemia r/t release of growth hormone cortisol; starts normal levels until 3 am then levels begin to rise. Treatment: increase/ adjust insulin doses

Dawn Phenomenon and the treatment:

Rest pain is a manifestation of PAD that occurs due to a chronic

Decrease in arterial blood flow to the nerves of the feet Rest pain most often occurs in the forefoot or toes and is aggravated by limb elevation. Rest pain occurs when blood flow is insufficient to meet basic metabolic requirements of the distal tissues. Rest pain occurs more often at night because cardiac output tends to drop during sleep and the limbs are at the level of the heart. Patients often try to achieve partial pain relief by dangling the leg over the side of the bed or sleeping in a chair to allow gravity to maximize blood flow.

Partial Thromboplastin Time in aging

Decreased

MCV - Mean corpuscular Volume

Determine relative size of RBCs (Hctx10)/(RBCx10 to the 6th power) WNL - 80-100 Decreased = microcytosis Increased - macrocytosis

When assessing the cardiovascular system of a 79-year-old patient, you may expect to find

Difficulty in isolating the apical pulse Myocardial hypertrophy and the downward displacement of the heart in an older adult may cause difficulty in isolating the apical pulse.

WBC count in aging

Diminished response to infections

Dumping Syndrome:

Direct result of surgical removal of a large portion of the stomach and pyloric sphincter

Hypokalemia

Diuretic therapy is the most common cause NPO, alcoholics, bariatric surgery, eating disorders are at risk N/V, sluggish bowel, arrhythmias, leg cramps, decreased deep tendon reflexes Increased risk for digitalis toxicity related to open binding sites in heart, and when there is a decrease in potassium levels, the digitalis binds to the sites Drink oj, eat banana's, dried fruits, meats, oral potassium, IV (SLOW...can cause arrest)

What should type 1 Diabetics do/not do with their insulin regimen if they are ill?

Don't stop taking their insulin because their BS will go up and can could a hyperglycemic event (DKA), have a high carbohydrate drink and don't exercise

A 46-year-old woman with a subclavian triple-lumen catheter is transferred from a critical care unit after an extended stay for respiratory failure. Which action is important for the nurse to take?

During removal of the catheter, have the patient perform the Valsalva maneuver. Deep breath in and push as if having a bowel movement.

When assessing a patient, you note a pulse deficit of 23 beats. This finding may be caused by

Dysrhythmias A pulse deficit occurs if there is a difference between the apical and radial beats per minute. A pulse deficit indicates cardiac dysrhythmias.

Combination regimens include

ECF (epirubicin ellence), cisplatin, 5-FU, and taxane and irinotecan based combinations

Lethargy and weakness, dehydration, poor skin tugor, dry mucous membranes, tachycardia, abdominal pain, kussmaul's breathing (rapid, deep breathing), orthostatic hypotension, fruity breath, and ketones in urine that put the body into acidosis

Early S/S of DKA:

Symptoms of Von Willebrand Disease

Easy bruising, excessive menstrual blood loss, and bleeding from the nose, mouth and GI tract.

What is needed to make a definitive diagnosis of carcinoma by identification of malignant cells?

Endoscopic biopsy

What is the primary tool for diagnosing the source of upper GI bleeding?

Endoscopy

The portion of the vascular system responsible for hemostasis is the

Endothelial layer of the arteries The innermost lining of the arteries is the endothelium. The endothelium maintains hemostasis, promotes blood flow, and under normal conditions, inhibits blood coagulation.

While obtaining subjective assessment data from a patient with hypertension, the nurse recognizes that a modifiable risk factor for the development of hypertension is

Excessive alcohol intake Alcohol intake is a modifiable risk factor for hypertension. Excessive alcohol intake is strongly associated with hypertension. Males with hypertension should limit their daily intake of alcohol to 2 drinks per day, and 1 drink per day for females with hypertension.

Newly diagnosed type 1 teaching of non pharm measures

Exercise lowers need for insulin

Dry loose skin with sunken eyes

Extreme S/S of DKA:

Paradoxical chest movement is associated with which chest disorder? a. Pneumothorax b. Flail chest c. ARDS d. Tension pneumothorax

Flail chest

can lead to megaloblastic anemia. This acid is required for DNA synthesis leading to RBC formation and maturation. This deficiency is treated by replacement therapy.

Folic acid deficiency

What are major complications of DM?

Foot ulcers, retina detachments, peripheral neuropathy, artery damage, nephropathy, neuropathy (and why it's important to control BS)

Contraindications for enteral nutrition include

GI obstruction, prolonged ileus, severe diarrhea or vomitting, and enterocutaneous fistual

Acute: ↑ HR & RR Skin pale, cool, clammy Shock Chronic: Pallor ↑ HR & RR Tissue hypoxia (angina, fatigue, dyspnea) Night cramps Bone pain Headache, dizziness, dim vision Heart failure

General anemia clinical manifestations.

Clinical manifestations are determined by the severity. Mild: (Hgb 10 to 12): may exist without causing symptoms. If symptoms develop, it is because the patient has an underlying disease or is experiencing a compensatory response to heavy exercise. Symptoms include palpitations, dyspnea, and mild fatigue Moderate: (Hgb 6 to 10): the above cardiopulmonary symptoms are increased. The patient may experience the while resting, as well as with activity Severe (Hgb less than 6): the patient has many manifestations involving multiple body

General anemia clinical manifestations.

Complications are caused by the body's response to tissue hypoxia. Hemoglobin (Hgb) levels are often used to determine the severity of this.

General anemia complications

New type 2: what general guideline

Get most cals from carbs (50-60%)

Glaucoma: Collaborative Management

Goal: prevent optic nerve damage Medications - table 63-5 Surgery Self-care KEY!

Acute pharyngitis of a bacterial nature is most commonly caused by: a. group A, beta-hemolytic streptococci. b. gram-negative Klebsiella. c. Pseudomonas. d. Staphylococcus aureus.

Group A, beta-hemolytic streptococci

An important cause of chronic gastritis is

H. pylori infection

Involves extremely high blood sugar (glucose) level without the presence of ketones.

HHS

Difference between HHS and DKA:

HHS has enough circulating insulin that ketoacidosis doesn't occur

Monitor IV fluids for hydration, electrolyte replacement, usually requires large volumes of fluid replacement that should be accomplished slowly and carefully. Rapid administration of IV NS or ½ NS IV Regular Insulin IV 5-10% D/NS & Insulin SQ or IV when blood glucose < 250 Monitor & treat electrolyte imbalances

HHS: Treatment and care for these patients.

Nursing management for the patient with acute symptoms of GERD

Head of bed is elevated to approximately 30 degrees. for 2 to 3 hours after a meal the patient should not be supine. avoid food and activities that cause reflux (e.g. late night eating.) Patients on PPI need to take meds before the first meal of the day.

Hemophilia: which clotting factors are affected?

Hemophila A (classic hemophilia, factor VIII (8) deficiency) Hemophilia B (Christmas disease, factor IX (4) deficiency) The type of hemophilia (A or B) indicates which clotting factors are missing. Hemophilia A is the most common form, accounting for about 80-85% of all hemophilia cases.

This is an X-linked recessive genetic disorder caused by a defective or deficient coagulation factor, passed on from the mother.

Hemophilia

Herpes Simplex etiology

Herpes simplex virus (type 1 and 2) predisposing factors of upper respiratory tract infections, excessive exposure to sunlight, food allergies, emotional tension, onset of menstruation

Nursing priority for client with sickle cell

Hydration and pain management

A life threatening syndrome that can occur in the patient with diabetes who is able to produce enough insulin to prevent DKA, but not enough to prevent severe hyperglycemia, osmotic diuresis, and extracellular fluid depletion.

Hyperosmolar hyperglycemic syndrom (HHS)

Clinical manifestations of Addison's disease

Hyperpigmentation, hypoglycemia, weight loss, hypotension, adrenal atrophy, vomiting diarrhea

Syndrome of inappropriate antidiuretic hormone secretion (SIADH)

Hypersecretion of ADH causes enhanced renal water retention, hyponatremia, and hypoosmolality

Epistaxis (Nose Bleeds): Etiology

Hypertension Prolonged bleeding times or altered platelet counts Trauma

Increase in T3 & T4, decrease in TSH

Hyperthyroidism

AVOID high iodine food: shrimp, seafood, leafy greens, organic foods.

Hyperthyroidism (Graves Disease): any specific foods?

Clinical manifestations of polycythemia

Hypervolemia and hyperviscosity

Patient with DKA is getting infused for rehydration what complication

Hypokalemia

Decrease in T3 & T4, increase in TSH

Hypothyroidism

Oxygen Therapy: Indications for need

Hypoxemia or Hypoxia SaO2 vs PaO2 Weaning

Treatment for a hypoglycemic event (conscious & unconscious)

If conscious: 1/2-3/4 cup fruit juice, 6-8oz of soda, 4 tsp jelly, 2 packets of sugar or honey, dextrose If unconscious: glucagon 1mg SQ or IM, IV 50% dextrose bolus

Priority nursing intervention for Hemarthrosis resulting from hemophilia

Immobilize and elevate

Where are most esophageal tumors located?

In the middle and lower portions of the esophagus

•Obesity • Pregnancy • Ascites • Intense physical exertion

Increased Intra-abdominal pressure risks of GERD:

In an obese patient the intraabdominal pressure is

Increased which can exacerbate GERD

Gingivitis clinical manifestation

Inflamed gingivae and interdental papillae, bleeding during tooth brushing, development of pus, formation of abscess with loosening of teeth

Intravenous potassium chloride (KCl) 60 mEq is prescribed for treatment of a patient with severe hypokalemia. Which action should the nurse take? 1. Give the KCl only through a central venous line. 2. Administer the KCl as a rapid IV bolus. 3. Add no more than 40 mEq/L to a liter of IV fluid. 4. Infuse the KCl at a rate of 20 mEq/hour.

Infuse the KCl at a rate of 20 mEq/hour.

Diabetes Insipidus

Insufficiency of ADH Partial or total inability to concentrate urine, polyuria, polydipsia

Storage for insulin: patients should store unopened vials in the refridgorator, but once they're opened; they can stay at room temperature (between 32-85 degrees) for no more than 4 weeks (30 days). Can prefill syringes, but if were mixing pre-filled syringes we should only keep them at room temp for one week. Don't mix or prefill long acting insulins in syringes We mix NPH, rapid acting and short acting The syringe should be kept so the needle point is up, so that the insulin doesn't pool in the needle.

Insulin storage for different types of insulin (1127)

NPH (Humulin N, Novolin N)

Intermediate acting insulin

Interventions may include blood or blood product transfusions, drug therapy, volume replacement, oxygen therapy, dietary modifications, and lifestyle changes. Correcting the cause is ultimately the goal of therapy.

Interventions for general anemia

A 22-year-old male is admitted to the emergency department with a stab wound to the abdomen. The patient's vital signs are blood pressure 82/56 mm Hg, pulse 132 beats/minute, respirations 28 breaths/minute, and temperature 97.9° F (36.6° C). Which fluid, if ordered by the health care provider, would be used?

Intravenous solutions used for volume expansion for hypovolemic shock include lactated Ringer's solution and 0.9% saline

This supplement is hard on the stomach but is best taken on empty stomach. It should be taken about an hour before meals. Taking it with vitamin C or orange juice, contains ascorbic acid and enhances this iron absorption. Undiluted liquid supplement may stain the patient teeth, therefore it should be diluted and ingested through a straw. Patients on an this supplement should be educated to expect dark black stools

Iron Deficiency Anemia: Patient teaching for iron supplements.

Goals of PN

Keep the patient free of infections, maintain good nutrition, improve condition, live a normal life. Goal is to provide complete nutritional support

Used for type 2 and not for type 1 because most of them require some working insulin cells in the body for their effectiveness. Sulfonlyureas is the most common agent that can cause hypoglycemia because it stimulates the pancreas to make more insulin Biguanide (Metformin) is the OA that can't be given with IV contrast dye because it can cause renal toxicity

Key points for Oral Agents for Type 2 diabetes:

Endoscopy is useful in assessing the

LES competence and the degree of inflammation, potential scarring, and strictures.

The most common finding in individuals at risk for sudden cardiac death is

Left ventricular dysfunction Left ventricular dysfunction (ejection fraction less than 30%) and ventricular dysrhythmias after myocardial infarction are the strongest predictors of sudden cardiac death (SCD).

A WHR measurements

Less than 0.8 is optimal, and a WHR greater than 0.8 indicates more truncal fat, which puts the individual at a greater risk for health complications

Foot Care teaching in newly diagnosed DM

Loss of sensation in feet, poor would healing

MOA for H2 Blockers:

MOA: Block the action of histamine on the H2 receptors to decrease HCl secretion, decrease conversion of pepsinogen to pepsin, and decrease irritation of the esophageal and gastric mucosa

MOA of PPIs:

MOA: HCl acid secretion by inhibiting the proton pump responsible for the secretion of H+, and decreases the irritation of the esophageal and gastric mucosa

Hx of type 1 has DKA: whats the priority

Maintain and monitor fluid balance

Fasting glucose of 120: teach patient

Maintain healthy weight

Nasal Obstruction:Treatment

Nasal corticosteroids Oral corticosteroids Rhinoplasty Elevate HOB Avoid blowing nose

A compensatory mechanism involved in HF that leads to inappropriate fluid retention and additional workload of the heart is

Neurohormonal response The following mechanisms in heart failure lead to inappropriate fluid retention and additional workload of the heart: activation of the renin-angiotensin-aldosterone system (RAAS) cascade and release of antidiuretic hormone from the posterior pituitary gland in response to low cerebral perfusion pressure that results from low cardiac output.

What kind of treatment is there for esophageal diverticula?

No specific treatment. The diet may need to be limited to foods that pass mroe radily

An expected nursing diagnosis for a 30-year-old patient admitted to the hospital with symptoms of diabetes insipidus is a. excess fluid volume related to intake greater than output. b. impaired gas exchange related to fluid retention in lungs. c. sleep pattern disturbance related to frequent waking to void. d. risk for impaired skin integrity related to generalized edema.

Nocturia occurs as a result of the polyuria caused by diabetes insipidus. Edema, excess fluid volume, and fluid retention are not expected

Discharge teaching for newly diagnosed DM?

Normal BS level Eye exams Hydrations Administering insulin Monitor BS before every meal and take appropriate insulin Exercise improves circulation and increase muscle tones

What do you do for cushings:

Notify provider if you run a fever

A patient with infective endocarditis develops sudden left leg pain with pallor, paresthesia, and a loss of peripheral pulses. The nurse's initial action should be to

Notify the HCP of the change in peripheral perfusion The patient has potentially developed acute arterial ischemia (sudden interruption in the arterial blood supply to the extremity), caused by an embolism from a cardiac thrombus that occurred as a complication of infective endocarditis. Clinical manifestations of acute arterial ischemia include any or all of the six Ps : pain, pallor, paralysis, pulselessness, paresthesia, and poikilothermia. Without immediate intervention, ischemia may progress quickly to tissue necrosis and gangrene within a few hours. If the nurse detects these signs, the HCP should be notified immediately.

Teach the patient to perform daily oral hygiene without causing trauma. Understanding how to prevent injuries is another consideration. Advise the patient to participate only in non contact sports (golf) and to wear gloves when doing household chores to prevent cuts or abrasions from knives, hammers and other tools. The patient should wear a Medic Alert tag to ensure the HCPs know about the hemophilia in case of an accident.

Nursing interventions for patients with hemophilia and von Willibrand disease

The nurse is caring for a patient who is 2 days post MI. The patient reports that she is experiencing chest pain. She states, "It hurts when I take a deep breath." Which action would be a priority?

Obtain vital signs and auscultate for a pericardial friction rub Acute pericarditis is inflammation of the visceral and/or parietal pericardium. It often occurs 2 to 3 days after an acute myocardial infarction. Chest pain may vary from mild to severe and is aggravated by inspiration, coughing, and movement of the upper body. Sitting in a forward position often relieves the pain. The pain is usually different from pain associated with a myocardial infarction. Assessment of the patient with pericarditis may reveal a friction rub over the pericardium.

What is available as prescription and OTC?

Omeprazole, and prilosec otc and zegrid

Insulin onset, peak, duration for long acting

Onset: 0.8-4 hours Peak: less defined or no pronounced peak Duration: 16-24 hours

Insulin onset, peak, duration for rapid acting

Onset: 10-30 mins Peak: 30 mins- 3 hrs Duration: 3-5 hours

Insulin onset, peak, duration for short acting

Onset: 30 mins- 1 hours Peak: 2-5 hours Duration: 5-8 hours

What are cataracts

Opacity covering the lens

The arterial blood gas measurement that best reflects the adequacy of alveolar ventilation is the: a. PaO2. b. PaCO2. c. pH. d. SaO2.

PaCO2

Parotitis clinical manifestation

Pain in area of gland and ear, absence of salivation, purulent exudate from gland, erythema, ulcers

Rest, O2, pain control, vigorous hydration Avoid precipitating factors, administration of Heparin or anticoagulants to present DVTs

Pain management considerations for sickle cell disease.

Vincent's Infection Clinical Manifestation

Painful, bleeding gingivae. Eroding necrotic lesions of interdental papillae, ulcerations that bleed, increased saliva with metallic taste, fetid mouth odor, anorexia, fever, general malaise

You are caring for a patient admitted with a diagnosis of chronic obstructive pulmonary disease (COPD) who has the following arterial blood gas results: pH 7.33, PaO2 47 mm Hg, PaCO2 60 mm Hg, HCO3 32 mEq/L, and O2 saturation of 92%. What is the correct interpretation of these results?

Partially compensated respiratory acidosis The elevated HCO3 indicates a partial compensation for the elevated CO2.

A patient has just returned from surgery following the placement of a tracheostomy. Which assessment by the nurse should receive highest priority during the first 24 hours postoperatively? Lung sounds and respiratory rate Patency of the tracheostomy tube Surgical dressing site Pulse and blood pressure

Patency of the tracheostomy tube

What is different about INR when a patient has an artificial heart valve?

Patient's INR is maintained at a ratio of 2.5 to 3.5 to reduce risk of thrombus formation on artificial valve.

What is aPTT measured with

Patients using heparin. (Therapeutic range is longer and calculated using prescribed range.)

Tonsillitis and Adenoiditis: Complications

Peritonsillar Abscess Can be life-threatening Airway obstruction Spread

1. Crackles in the lungs and a productive cough noted on a pt. with influenza

Pneumonia

This is the production and presence of increased RBC and includes two different types

Polycythemia; PRIMARY and SECONDARY

Three P's are? For which type of Diabetes are these presenting symptoms?

Polydipsia, polyuria and polyphagia, type 1

When assessing a patient admitted with nausea and vomiting, which finding supports the nursing diagnosis of deficient fluid volume?

Polyuria Decreased pulse Difficulty breathing General restlessness (Correct)

Distinguish among the three classifications of hemorrhage (primary, intermediary, and secondary) and include the defining characteristics of each. Primary: Intermediary: Secondary:

Primary: Hemorrhage occurs at the time of the operation Intermediary: Hemorrhage occurs within the first few hours after surgery when a return of blood pressure to its normal level dislodges insecure clots Secondary: Hemorrhage occurs some time after the operation as a result of the slipping of a ligature, which may happen because of infection, insecure tying or erosion of a vessel by a drainage tube

Nasal Obstruction

Prone to chronic infections Sleep deprivation

One of the most effective nursing procedures for reducing nosocomial infections is: a. administration of prophylactic antibiotics. b. aseptic wound care. c. control of upper respiratory tract infections. d. proper hand-washing techniques.

Proper hand washing techniques

Microcytosis (microcythemia)

Red blood cells are unusually small.

Vincent's Infection Treatment

Rest, avoidance of smoking and alcoholic beverages. Soft, nutritious diet. Correct oral hygiene habits. Topical applications of antibiotics . Mouth irrigrations with chlorhexidine and saline solutions

Peritonitis

Results from a localized or generalized inflammatory process of the peritoneum

A patient with a tricuspid valve disorder will have impaired blood flow between the

Right atrium and left ventricle The tricuspid valve is located between the right atrium and right ventricle.

o Tobacco and Alcohol consumption o Men > women o > 40 years of age o African Americans o Outdoor occupations o Prolonged sun exposure o Irritation from the pipe stem resting on the lip is a factor in pipe smokers o HPV o Multiple sexual partners

Risk factors for Oral Cancer:

What does TIRED stand for:

S/S of Hypoglycemia: T- tremors, tachycardia I- Irritability R- Restless E- Excessive hunger D- Diaphoresis, depression

Can affect the heart, confusion or memory loss. muscle spasms. numbness and tingling in the hands, feet, and face.

S/S of hypocalcemia

A P wave on an ECG represents an impulse arising at the

SA node and depolarizing the atria The first wave, P, begins with the firing of the sinoatrial (SA) node and represents depolarization of the fibers of the atria.

increase in water retention, symptoms of fluid overload; weight gain, decreased urine output (look at urine specific gravity is really high, very concentrated), sodium level goes down in relation to being diluted by excess water in the body Decrease in Na and Sr. osmolality Increase in Sp. Gr.

SIADH: What are defining characteristics

The hypoxia driven state causes the body to think that if it makes more RBCs it can compensate for this hypoxic state that it's in where the tissues aren't getting enough oxygen Hypoxia may stimulate an increased production of RBC. This hypoxia could be a result of high altitude, pulmonary disease, cardiovascular disease, or defective oxygen transport. A tumor or cyst is another reason for increased production of RBCs.

Secondary Polycythemia: Causes

Erythrocyte sedimentation rate (ESR) WNL

Settling of RBCs in 1 hour. < 30 mm/hr. (some gender variations)

What is "shift to the left"?

Severe infections where high numbers of granulocytes are released, resulting in many young, immature neutrophils (bands) released. Less mature forms appear on the left side of the written report from the lab.

Regular (Humulin R, Novolin R)

Short acting insulin

Nasogastric (NG) tube:

Short term (4 weeks or less), placed through the nose into the stomach

Is a severe, painful, acute exacerbation of RBC sickling, causing this crisis. As blood flow is impaired by sickled cells, vasospasm occurs, further restricting blood flow. Severe capillary hypoxia causes changes in membranes permeability, leading to plasma loss, hemoconcentration, clot, and further circulatory stagnation. Tissue ischemia, infarction, and necrosis eventually occur from lack of oxygen. Shock is a possibly life-threatening consequence of this crisis because of severe oxygen depletion of the tissues and a reduction of the circulation fluid volume.

Sickle Cell disease: What is a vaso-occlusive crisis?

Ischemia & pain, RBCs live about 20 days Increase in RBC destruction and spleen is working more to clean the RBCs out Are at increased risk for infection because of the over-working of the spleen

Sickle cell anemia

Two types of hemolytic anemias:

Sickle cell anemia and thalassemia

This can begin suddenly and persist for days to weeks. The frequency, extent, and severity of these episodes are highly variable and unpredictable but largely depend on the percentage of Hgb S present. Individuals with this anemia have the embolism most severe form because the erythrocytes contain a high percentage of Hgb S

Sickle cell crisis

Hypernatremia

Sodium Imbalance (greater than 145 mEq/L) Cause by water loss or salt overload Cells become dehydrated and shrink After 1 hour, intracellular solutes are generated in attempt to avoid structural damage to the cell Must treat patient slowly to allow for excretion of the extra solutes; can lead to cerebral edema Weight gain, hypertension, irritable, tachycardia, flushed skin

Hyponatremia

Sodium Imbalance (less than 135 mEq/L) Adrenal insufficiency, osmotic diuresis, diarrhea, vomiting, sweating, CF Thirst, dry mouth, low BP, tachycardia, low urine output Seizures, coma Slow treatment related to pulmonary edema

Too much insulin at night, leads to hypoglycemia in the middle of the night, which causes counter-regulating hormones causing hyperglycemia in the morning. Tx: less insulin at night

Somogyi effect: What is the treatment?

International normalization ratio

Standardized system of reporting PT based on a reference calibration model and calculated by comparing the PT with a control value - WNL while receiving anticoagulants - 2-3

Lack of knowledge of management of Addisonian crisis

Stopped hydrocortisone on flu

A patient is admitted to the hospital with a diagnosis of abdominal aortic aneurysm. Which signs and symptoms would suggest that his aneurysm has ruptured?

Sudden, severe low back pain and bruising along his flank The clinical manifestations of a ruptured abdominal aortic aneurysm include severe back pain, back or flank ecchymosis (Grey Turner's sign), and hypovolemic shock (tachycardia, hypotension, pale clammy skin, decreased urine output, altered level of consciousness, and abdominal tenderness).

One sentence that contains information about something you would teach a patient who has Tuberculosis:

Teach the patient about the importance of hand hygiene and covering their mouth when they cough or sneeze because TB is an airborne spread disease and to be compliant with their medication because treatment can last up to 24 months or longer.

Most important intervention to assess for presence of infection of neutropenia

Temperature

What white blood cells are granulocytes?

The Phil's - eosinophils, basophils & neutrophils

PN

The administration of nutrients directly into the bloodstream.

Paraesophageal, or rolling

The esophagogastic junction remains in the normal position, but the fundus and the greater curvature of the stomach roll up through the diaphragm, forming a pocket alongside the esophagus. Acute paraesophageal hernia is a medical emergency

A patient with a history of chronic hypertension is being evaluated in the emergency department for a blood pressure of 200/140 mm Hg. Which patient assessment question is the priority? A. Is the patient pregnant? B. Does the patient need to urinate? C. Does the patient have a headache or confusion? Correct D. Is the patient taking antiseizure medications as prescribed?

The nurse's priority assessments include neurologic deficits, retinal damage, heart failure, pulmonary edema, and renal failure. The headache or confusion could be seen with hypertensive encephalopathy from increased cerebral capillary permeability leading to cerebral edema. In addition, headache or confusion could represent signs and symptoms of a hemorrhagic stroke. Pregnancy can lead to secondary hypertension. Needing to urinate and taking antiseizure medication do not support a hypertensive emergency.

The nurse has administered 3% saline to a patient with hyponatremia. Which one of these assessment data will require the most rapid response by the nurse? 1. There are crackles audible throughout both lung fields. 2. There is sediment and blood in the patient's urine. 3. The blood pressure increases from 120/80 to 142/94. 4. The patient's radial pulse is 105 beats/minute.

There are crackles audible throughout both lung fields.

High calorie, high protein diet

This diet is used for those who are malnourished and can take food by mouth. Encourage foods that the patient like in order to help with compliance of eating

Valsalva Maneuver

This is straining, this can cause hemorrhoids.

PN

This is used when the GI tract cannot be used for the ingestion, digestion, and absorption of essential nutrients. The composition is customized to meet the needs of each patient. Formed of dextrose and protein in the form of amino acids. Pharmacy will add in needed electrolytes

Is the reduction of platelets in the blood, which can result in prolonged or spontaneous bleeding because of the blood's inability to clot properly. Platelet count <100,000

Thrombocytopenia. What is it?

Platelet disorders can be inherited, but the vast majority are acquired. The causes of acquired disorders include autoimmune diseases, increased platelet consumption, splenomegaly, marrow suppression, and bone marrow failure. This can be caused by an autoimmune disorder, enzyme deficiencies, or as a result of HIT therapy

Thrombocytopenia: what causes it?

A 50-year-old woman weighs 95 kg and has a history of tobacco use, high BP, high sodium intake, and sedentary lifestyle. When developing an individualized care plan for her, the nurse determines that the most important risk factors for peripheral artery disease (PAD) that must be modified are

Tobacco use and high BP Significant risk factors for peripheral artery disease include tobacco use, hyperlipidemia, elevated levels of high-sensitivity C-reactive protein, diabetes mellitus, and uncontrolled hypertension; the most important is tobacco use. Other risk factors include family history, hypertriglyceridemia, hyperuricemia, increasing age, obesity, sedentary lifestyle, and stress.

Complications of Oxygen Therapy

Toxicity: Too much O2 sat administered over a long period of time. (48 hours above 50%) can administer vitamin c (may help)

Stomatitis (inflammation of mouth) etiology

Trauma, pathogens, irritants,. renal, liver and hematologic diseases. Side effect of many cancer chemotherapy

Treat underlying cause, administer corticosteroid (Glucocorticoid and mineralcorticoid replacement: Hydrocortisone), take it at the same time every morning, don't stop taking abruptly Trying to correct the hormone imbalances and prevent adverse effects that may occur from these states. Can see low BP, because of the decrease in the adrenal hormones, so we would be monitoring for that and treating that

Tx of Addison's disease

Tx the underlying cause, surgery or drugs to suppress cortisol, reducing dose of exogenous glucocorticoids by gradual tapering to avoid life-threatening adrenal insufficiency Trying to correct the hormone imbalances and prevent adverse effects that may occur from these states, can be caused by the patient being on high doses of corticosteroids, so we may try to taper the patient off

Tx of Cushing Syndrome:

Tx: for Hemorrhage with PUD

Tx: IV fluids, NG, and saline or water lavage; oxygen, treatment of potential shock including monitoring of VS and UO; may require endoscopic coagulation or surgical intervention

Formally know as adult onset or non-insulin dependent diabetes occurs when the pancreas is not producing enough insulin or does not use it effectively.

Type 2 DM

Not genetic and is related to lifestyle and occurs gradually over time.

Type 2 DM

Symptoms are often nonspecific, may be similar to the other type of Diabetes, but also will show signs of recurrent infections, fatigue, prolonged wound healing, and visual changes.

Type 2 DM

Platelets in aging

Unchanged but possible increase in adhesiveness

A priority consideration in the management of the older adult with hypertension is to

Use careful technique in assessing the BP of the patient because of the possible presence of an auscultatory gap Careful technique is important in assessing BP in older adults. In some older people, there is a wide gap between the first Korotkoff sound and subsequent beats; such a wide interval is called an auscultatory gap. Failure to inflate the cuff high enough may result in a serious underestimate of systolic BP.

Enteral feedings:

Use them for patients that have GI function but may have an eating disorder, oral or nose fractures, head and neck cancer, neuro or psychiatric conditions, extensive burns, chemo or radiation therapy, and critical illness.

Percutaneous endoscopic gastrostomy(PEG) tube:

Used long term, placed surgically into the fully functioning non-obstructed stomach, placed surgically on outside of body

what is Glaucoma

Visual changes and loss due to increase intraocular pressure (IOP)

The nurse is caring for a preoperative patient who has an order for vitamin K by subcutaneous injection. The nurse should verify that which laboratory study is abnormal before administering the dose? A. Hematocrit (Hct) B. Hemoglobin (Hgb) C. Prothrombin time (PT) D. Partial thromboplastin time (PTT)

Vitamin K counteracts hypoprothrombinemia and/or reverses the effects of warfarin (Coumadin) and thus decreases the risk of bleeding. High values for either the PT or the international normalized ratio demonstrate the need for this medication.

Laryngitis: Collaborative Care

Vocal rest Humidification Proton Pump Inhibitor - Why? Vanceril inhalation - what will you teach?

This is a related to Hemophilia, but it involves a deficiency of the von Willebrand coagulation protein, a protein that is crucial for platelet adhesion. Factor VIII is synthesized in the liver and circulates as a complex with von Willebrand factor (vWF). This is considered the most common congenital bleeding disorder.

Von Willebrand disease

This leads to increased gastric secretion and produces tissue damage, leading to Peptic ulcer disease

What does H. Pylori lead to?

What is a thyroid storm?

When T3 spills into the blood it can lead to a storm. It occurs with hyperthyroidism

One sentence about one thing you have learned about caring for a patient who has a tracheostomy:

When caring for a patient who has a tracheostomy sterile technique is performed in the hospital and it is a two person job, one to hold the tracheostomy in place while another person changes the straps.

Give in the morning when the patient normally secretes those hormones

When should corticosteroids be given? Patient teaching points.

Complications of surgery

When we're doing surgery it's because the GERD is being caused by a hiatal hernia. - Gastric or esophageal injury, splenic injury, pneumothorax, perforation, bleeding, infection, and pneumonia, High risk for respiratory complications because we're in the area of the airway and the lungs. -Could end up with too tight of a sphincter, with difficulty swallowing and dysphagia after

Can cigarette and cigar smoking also contribute to GERD?

Yes

Is obesity a risk factor for GERD?

Yes

Common causes of secondary peritonitis include

a ruptured appendix, perforated gastric or duodenal ulcer, severely inflamed gallbladder, and trauma from gunshot or knife wounds

Lorcaserin (Belviq)

a selective serotonin (5-HT) agnoist that suppresses appetite and creates a sense of satiety

A patient receives aspart (NovoLog) insulin at 8:00 AM. At which time would the nurse anticipate the highest risk for hypoglycemia? A. 10am B. 12am C. 4pm D. 2pm

a. 10am The rapid-acting insulins peak in 30min to 3 hours. The patient is not at a high risk for hypoglycemia at the other listed times (12am, 2pm, 4pm), although hypoglycemia may occur.

9. Chronic inflammatory disease of airways that causes hyperresponsiveness, mucosal edema, and mucus production

a. Asthma (reversible)

6. Air pollutants, allergens, environmental factors

a. Asthma triggers

8. Preventable and treatable slowly productive disease

a. COPD

9-11 mg/dl

a. Calcium

16. Decrease airway inflammation

a. Corticosteroid

17. Max dose to be given in 1 hour is 10-20 meq

a. Diluted potassium

2. Dry mucous membranes, decreased urine output, prolonged skin tenting over the sternum

a. Fluid-volume deficit/ hypovolemia

4. Rhythmic breathing x3, followed by held inspiration and forcefully expiration making a "who" sound

a. Huff

18. Used for Na and volume replacement

a. Hypertonic solution

7. Complete or partial collapse of lung due to an accumulation of air in the pleural space

a. Pneumothorax

3.5-5.0 meq

a. Potassium

3. Tachypnea, crackles in the lungs, pink and frothy sputum, restlessness, agitation

a. Pulmonary edema

A patient with a recent diagnosis of heart failure has been prescribed furosemide (Lasix) in an effort to physiologically do what for the patient? a. Reduce preload. b. Decrease afterload. c. Increase contractility. d. Promote vasodilation.

a. Reduce preload. Diuretics such as furosemide are used in the treatment of HF to mobilize edematous fluid, reduce pulmonary venous pressure, and reduce preload. They do not directly influence afterload, contractility, or vessel tone.

15. Utilized for acute attacks

a. Short-acting beta 2 agonists, anticholinergics

135-145

a. Sodium

14. Using light pressure with the index and middle fingers, the nurse cannot palpate any of the patient's superficial lymph nodes. How should the nurse respond to this assessment? a. Record this finding as normal. b. Reassess the lymph nodes using deeper pressure. c. Ask the patient about any history of radiation therapy. d. Notify the health care provider that x-rays of the nodes will be necessary.

a. Superficial lymph nodes are evaluated by light palpation but they are not normally palpable. It may be normal to find small (<1.0 cm), mobile, firm, nontender nodes. Deep lymph nodes are detected radiographically.

5. Cover mouth and nose when sneezing, proper disposal of tissue, and medication regimen

a. TB

In the immediate postoperative period, a nurse should immediately report: a. a systolic blood pressure lower than 90 mm Hg. b. a temperature reading between 97F and 98F. c. respirations between 20 and 25 breaths/min. d. all of the above assessments.

a. a systolic blood pressure lower than 90 mm Hg.

The nurse recognizes that primary manifestations of systolic failure include: a. decreased EF and increased PAWP b. decreased PAWP and increased EF. c. decreased pulmonary hypertension associated with normal EF d. decreased afterload and decreased left ventricular end-diastolic pressure

a. decreased EF and increased PAWP Rationale: Systolic heart failure results in systolic failure in the left ventricle (LV). The LV loses its ability to generate enough pressure to eject blood forward through the aorta. This results in increased pulmonary artery wedge pressure (PAWP). The hallmark of systolic failure is a decrease in the left ventricular ejection fraction (EF).

Clinical manifestations of diabetic ketoacidosis include (select all that apply): a. flushed, dry skin b. moist, clammy skin c. labored breathing d. abdominal pain

a. flushed, dry skin c. labored breathing d. abdominal pain

secondary peritonitis occurs when

abdominal organs perforate or rupture and release their contents (bile, enzymes, and bacteria) into the peritoneal cavity

Symptoms of IBS include

abdominal pain or discomfort and alterations in bowel patterns, are intermittent and may occur for years. patients often report a history of GI infections and food intolerance

What is the most common symptom of peritonitis?

abdominal pain, a universal sign is tenderness over the involved area

peptic ulcer develop only in

acid environment, and h. pylori is also associated with development

The most common restrictive surgeries include

adjustable gastric banding and vertical sleeve gastrectomy

An enteral feeding misconnection

an inadvertent connection between an enteral feeding system and a nonenteral system such as an iv line,a peritoneal dialysis catheter, or a trachesotomy tube cuff

One of the primary etiologic factors in GERD is

an incompetent LES, lets gastric contents move from the stomach to the esophagus when the patient is supine or has an increase in intraabdominal pressure

serums transferrin level is

an indicator of protein status

Gastritis

an inflammation of the gastric mucosa, affects the stomach

Gastroenteritis

an inflammation of the mucosa of the stomach and small intestine

Dronabinol (Marinol)

an orally active cannabinoid that is used alone or in combination with other antiemetics for the prevention of chemotherapy induced emesis, only used when other therapies are ineffective

Vitamins

are organic compounds required in small amounts for normal metabolism

The abnormal inflammatory response in the lungs occurs primarily in the: a. airways. b. parenchyma. c. pulmonary vasculature. d. areas identified in all of the above.

areas identified in all of the above

Most surgical patients are encouraged to be out of bed: a. within 6 to 8 hours after surgery. b. between 10 and 12 hours after surgery. c. as soon as it is indicated. d. on the second postoperative day.

as soon as it is indicated

What are two important safety concerns of tubes?

aspiration and dislodged tubes

The acute ulcer

associated with superficial erosion and minimal inflammation, short duration and resolves quickly when the cause is identified and removed

6. Components of normal hemostasis in the order of occurrence, beginning with the first component and ending with the last component. a. Lysis of clot b. Vascular response c. Plasma clotting factors d. Platelet plug formation

b, d, c, a.

A patient with a diagnosis of heart failure has been started on a nitroglycerin patch by his primary care provider. What should this patient be taught to avoid? a. High-potassium foods b. Drugs to treat erectile dysfunction c. Nonsteroidal antiinflammatory drugs d. Over-the-counter H2 -receptor blockers

b. Drugs to treat erectile dysfunction The use of erectile drugs concurrent with nitrates creates a risk of severe hypotension and possibly death. High-potassium foods, NSAIDs, and H2-receptor blockers do not pose a risk in combination with nitrates.

What should the nurse recognize as an indication for the use of dopamine (Intropin) in the care of a patient with heart failure? a. Acute anxiety b. Hypotension and tachycardia c. Peripheral edema and weight gain d. Paroxysmal nocturnal dyspnea (PND)

b. Hypotension and tachycardia Dopamine is a β-adrenergic agonist whose inotropic action is used for treatment of severe heart failure accompanied by hemodynamic instability. Such a state may be indicated by tachycardia accompanied by hypotension. PND, anxiety, edema, and weight gain are common signs and symptoms of heart failure, but these do not necessarily warrant the use of dopamine.

The nurse is preparing to administer digoxin to a patient with heart failure. In preparation, laboratory results are reviewed with the following findings: sodium 139 mEq/L, potassium 5.6 mEq/L, chloride 103 mEq/L, and glucose 106 mg/dL. What should the nurse do next? a. Withhold the daily dose until the following day. b. Withhold the dose and report the potassium level. c. Give the digoxin with a salty snack, such as crackers. d. Give the digoxin with extra fluids to dilute the sodium level.

b. Withhold the dose and report the potassium level. The normal potassium level is 3.5 to 5.0 mEq/L. The patient is hyperkalemic, which makes the patient more prone to digoxin toxicity. For this reason, the nurse should withhold the dose and report the potassium level. The physician may order the digoxin to be given once the potassium level has been treated and decreases to within normal range.

A patient who is hypoglycemic will display the following clinical manifestations. Select all that apply: a. warm, dry skin b. moist, clammy skin c. confusion and irritability d. weakness

b. moist, clammy skin c. confusion and irritability d. weakness

If nonamphetamines are used they should

be only used short term (for 3 months or less)

Why is skin care around the tube site important?

because the action of the digestive juices irritates the skin

What is the gold standard for diagnosis of H. pylori infection?

biopsy of the antral mucosa and testing for urease

Instructions for type 1 diabetic patients who are ill include: a. If unable to eat, stop taking insulin b. Take insulin and supplement food intake with low carbohydrate drinks. c. Take insulin and supplement food intake with high carbohydrate drinks. d. Check blood sugar every hour.

c. Take insulin and supplement food intake with high carbohydrate drinks.

Which assessment finding should the nurse caring for a patient with thrombocytopenia communicate immediately to the health care provider? A. There are purpura on the oral mucosa B. There are large bruises on the patient's back C. The patient is difficult to arouse D. The platelet count in 52,000/uL

c. The patient is difficult to arouse Difficulty in arousing the patient may indicate a cerebral hemorrhage, which is life threatening and requires immediate action. The other information should be documented and reported but would not be unusual in a patient with thrombocytopenia (which is caused by a decrease production of platelets).

The primary nursing goal in the immediate postoperative period is maintenance of pulmonary function and prevention of: a. laryngospasm. b. hyperventilation. c. hypoxemia and hypercapnia. d. pulmonary edema and embolism

c. hypoxemia and hypercapnia

A compensatory mechanism involved in HF that leads to inappropriate fluid retention and additional workload of the heart is: a. ventricular dilation b. ventricular hypertrophy c. neurohormonal response d. sympathetic nervous system activation

c. neurohormonal response Rationale: The following mechanisms in heart failure lead to inappropriate fluid retention and additional workload of the heart: activation of the renin-angiotensin-aldosterone system (RAAS) cascade and release of antidiuretic hormone from the posterior pituitary gland in response to low cerebral perfusion pressure that results from low cardiac output.

if treatment is delayed, peritonitis

can be fatal

Chronic disease-related malnutrition, or secondary PCM, is assocaited with

conditions that impose inflammation of a mild to moderate degree. occurs when tissue needs are not met even though the dietary intake would be satisfactory under normal conditions. (ex. organ failure, cancer, rheumatoid arthritis, obesity, and metabolic syndrome)

In AGB the band is

connected to a subcutaneous port and can be inflated or deflated to change the stoma size to meet the patients needs as weight is lost

For the patient with chronic reflux, a jejunostomy (j-tube) with

continuous feedings may be necessary to reduce the risk of aspiration

Liposuction is used for

cosmetic purposes and not for weight reduction

When a PACU room scoring guide is used, a patient can be transferred out of the recovery room with a minimum score of: a. 5. b. 6. c. 7. d. 8

d. 8

A stable patient with acute decompensated heart failure (ADHF) suddenly becomes dyspneic. Before positioning the patient on the bedside, what should the nurse assess first? a. Urine output b. Heart rhythm c. Breath sounds d. Blood pressure

d. Blood pressure The nurse should evaluate the blood pressure before dangling the patient on the bedside because the blood pressure can decrease as blood pools in the periphery and preload decreases. If the patient's blood pressure is low or marginal, the nurse should put the patient in the semi-Fowler's position and use other measures to improve gas exchange.

Instructions for type 1 diabetic patients who want to exercise include: a. Assess you exercise effect on you blood sugar. b. Hold your insulin until after you exercise c. Limit your exercise to avoid hypoglycemia. d. Eat extra carbohydrates before exercise

d. Eat extra carbohydrates before exercise

After having an MI, the nurse notes the patient has jugular venous distention, gained weight, developed peripheral edema, and has a heart rate of 108/minute. What should the nurse suspect is happening? a. ADHF b. Chronic HF c. Left-sided HF d. Right-sided HF

d. Right-sided HF An MI is a primary cause of heart failure. The jugular venous distention, weight gain, peripheral edema, and increased heart rate are manifestations of right-sided heart failure.

A major postoperative nursing responsibility is assessing for cardiovascular function by monitoring: a. arterial blood gases. b. central venous pressure. c. vital signs. d. all of the above.

d. all of the above

A wound that has hemorrhaged carries an increased risk of infection, because: a. reduced amounts of oxygen and nutrients are available. b. the tissue becomes less resilient. c. retrograde bacterial contamination may occur. d. dead space and dead cells provide a culture medium.

dead space and dead cells provide a culture medium.

In some cases a nasogastric (NG) tube and suction are used to

decompress the stomach

Because patients with evidence of metaplasia on an initial endoscopic examination are at risk for

esophageal cancer, a surveillance endoscopy every 2 to 3 years is often recommended

Bleeding from the esophagus is most likely due to

esophagitis, mallory-weiss tear, or esophageal varcies

Proteins

essential component of a well balanced diet and are obtained from both animal and plant sources

herniotomy

excision of the hernia sac

Anthropometric measurements are

gross measures of fat and muscle contents

The most common bacterial pathogen associated with tonsillitis and adenoiditis is: a. group A, beta-hemolytic streptococcus. b. gram-negative Klebsiella. c. Pseudomonas. d. Staphylococcus aureus.

group A, beta-hemolytic streptococcus.

Criteria guidelines for bariatric surgery include

having a BMI of 40 kg/m or a BMI of 35 kg/m with one or more severe obesity-related medical conditions (e.g. hypertension, type 2 diabetes mellitus, heart failure, sleep apnea)

Most common side effects of lorcaserin (belviq)

headache, dizziness, fatigue, nausea, dry mouth, and constipation

Qsymia can increase

heart rate

In acute gastritis patients with alcohol abuse what is commonly associated?

hemorrhage

The three major complications of chronic PUD are

hemorrhage, perforation, and gastric outlet obstruction

What is the most common complication of chronic complicaiton?

hemorrhoids

Hiatal Hernia

herniation of a portion of the stomach into the esophagus through an opening, or hiatus, in the diaphgram, in the upper GI tract

Acyclovir, an antiviral agent, is recommended for: a. herpes simplex infection. b. rhinitis. c. sinusitis. d. bronchitis.

herpes simplex infection

When caring for an alert and oriented elderly patient with a history of dehydration, the home health nurse will teach the patient to increase fluid intake 1. when the patient feels thirsty. 2. if the oral mucosa feels dry. 3. as soon as changes in level of consciousness (LOC) occur. 4. in the late evening hours.

if the oral mucosa feels dry.

large perforation require

immediate surgical closure

Treatment of appendicitis is

immediate surgical removal if the inflammation is localized. antibiotics and fluid resuscitation are administered before surgery

Straining contributes to

incontinence because it weakens the pelvic floor muscles

Neuromuscular blockers are given to patients who are on ventilators in ARF to accomplish all of the following except: a. maintain positive end-expiratory pressure (PEEP). b. maintain better ventilation. c. increase the respiratory rate. d. keep the patient from fighting the ventilator.

increase the respiratory rate

In a fasting state, Ghrelin is

increased

During the acute exacerbation of an ulcer, the patient often complains of

increased pain and nausea and vomiting, and some may have evidence of bleeding

Cholinergic drugs (e.g. bethanechol { urecholine})

increases LES pressure, improve esophageal emptying in the supine position, and increase gastric emptying

topiramate

induces a sense of satiety

The main goal of interprofessional care of iron-deficiency anemia is to treat the underlying disease that is causing reduced intake (e.g., malnutrition, alcoholism) or absorption of iron. It is important to recognize groups of individuals who are at an increased risk for the development of iron-deficiency anemia. Diet teaching, with an emphasis on foods high in iron and how to maximize absorption, is important for these patients.

iron deficiency anemia

The Roux-en-Y gastric bypass (RYGB) procedure

is a combination of restrictive and malabsorptive surgery, irreversible, involves creating a small gastric pouch and attaching it directly to the small intestine using a Y-shaped limb of the small bowel. Food bypasses 90% of the stomach, the duodenum, and a small segment of jejunum

Peripheral Parenteral Nutrition (PPN)

is administered through a peripherally inserted catheter or vascular access device, which uses a large vein

The waist-to-hip ratio (WHR)

is another tool used to assess obesity. a method of describing the distribution of both subcutaneous and visceral adipose tissue

Central PN

is indicated when long-term support is necessary or when the patient has high protein and caloric requirements

Nissen Tx

is to treat the hiatal hernia. The fundus of stomach is wrapped around distal esophagus. The fundus is then sutured to itself. The goal of surgical therapy is to reduce reflux by enhancing the integrity of the LES.

When upper GI bleeding is less profuse, infusion of

isotonic saline solution followed by packed RBCs permits restoration of the hematocrit more quickly and does not create complication related to fluid volume overload. The use of supplemental oxygen may help increase blood oxygen saturation

If a patient vomits with a nasogastric tube in place,

it may require repositioning, notify the surgeon immediately

Characteristics of the Mycobacterium tuberculosis include all of the following except: a. it can be transmitted only by droplet nuclei. b. it is acid-fast. c. it is able to lie dormant within the body for years. d. it survives in anaerobic conditions

it survives in anaerobic conditions.

Do not obtain residual volumes for EN delivered through a

jejunostomy tube

You can minimize the occurrence of nausea or vomiting by

keeping the immediate environment quiet, free of noxious odors, and well ventilated

Orlistate (xenical) is associated with

leakage of stool, flatulence, diarrhea, and abdominal bloating -severe liver injury has been reported in patients

Postoperatively, the nurse monitors urinary function. An abnormal outcome that should be reported to the physician is a 2-hour output that is: a. less than 30 mL. b. between 75 and 100 mL. c. between 100 and 200 mL. d. greater than 200 mL

less than 30 mL.

Most patients with GERD can successfully manage this condition through

lifestyle modifications and drug therapy

what is hypoxia?

low O2 to tissues and cells

Esophageal cancer

malignant neoplasm of the esophagus and is not common

Complications of esophageal diverticula include

malnutrition, aspiration, and perforation

fecal incontinence from fecal impaction usually involves after

manual disimpaction of the hard feces and cleansing enemas

Using antiemetics before determing the cause of n/v can

mask the underlying disease process and delay diagnosis and treatment

Histamine, a mediator that supports the inflammatory process in asthma, is secreted by: a. eosinophils. b. lymphocytes. c. mast cells. d. neutrophils.

mast cells

erythropoietin in aging

may be decreased

Self-monitoring

may involve keeping a record of the type and time food was consumed and how the person was feeling when eating

The daily addition of a multivitamin preparation to the PN generally

meets the vitamin requirements

Good sources of protein

milk and milk products, eggs, fish, meat, poultry, grains, legumes, nuts, seeds

The most common food triggers of EE are

milk, eggs, wheat, rye, and beef

Also teach the patient with GERD to avoid

milk, especially at bedtime, since it increases gastric acid secretion

Simple carbohydrates come in two forms:

monosaccharides and disaccharides

Stress-related mucosal disease (SRMD) also called physical stress ulcers

occurs in patients who have severe burns or trauma or major surgery

palliative treatment for oral cancer is

often the best management when the prognosis is poor, the cancer is inoperable, or the patient decides against surgery

In ulcerative colitis and crohn's disease, manifestations are

often the same (diarrhea, bloody stools, weight loss, abdominal pain, fever, and fatigue) Bloody stools are more common with ulverative colitis and weight loss is more common in crohn's disease

The nurse caring for a patient who is at risk for malignant hyperthermia subsequent to general anesthesia would assess for the most common early sign of: a. hypertension. b. muscle rigidity ("tetany-like" movements). c. oliguria. d. tachycardia

oliguria

A patient who has been receiving diuretic therapy is admitted to the emergency department with a serum potassium level of 3.1 mEq/L. Of the following medications that the patient has been taking at home, the nurse will be most concerned about 1. oral digoxin (Lanoxin) 0.25 mg daily. 2. metoprolol (Lopressor) 12.5 mg orally daily. 3. ibuprofen (Motrin) 400 mg every 6 hours. 4. lantus insulin 24 U subcutaneously every evening.

oral digoxin (Lanoxin) 0.25 mg daily.

Obesity is associated with an increased incidence of

osteoarthritis because of the stress put on weight bearing joints, especially the knees and hips

Lipectomy (adipectomy)

performed to remove unsightly flabby folds of adipose tissue, emphasize to the patient that surgical removal does not prevent obesity from recurring, especially if lifetime eating habits remain the same

Central PN can also be given using

peripherally inserted central catheters (PICCs) that are placed into the basilic or cephalic vein and then advanced into the distal end of the superior vena cava

Dietary fiber supplements or builk-forming laxatives such as

psyllium in metamucil increase stool bulk, firm stool consistency, and promote the sensation of rectal filling.

One of the major dangers associated with deep venous thrombosis is: a. pulmonary embolism. b. immobility because of calf pain. c. marked tenderness over the anteromedial surface of the thigh. d. swelling of the entire leg owing to edema.

pulmonary embolism

Antacids produce

quick but short-lived relief of heartburn, should be taken 1 to 3 hours after meals and at bedtime

Aphthous stomatitis (canker Sore) etiology

recurrent and chronic form of infection secondary to systemic disease, trauma, stress, or unknown cause

Hemoglobin changes with aging

remain normal, with possible decline in men.

The nurse assesses a patient for a possible pulmonary embolism. The nurse looks for the most frequent sign of: a. cough. b. hemoptysis. c. syncope. d. tachypnea.

tachypnea

Common causes of constipation include

taking in sufficient dietary fiber or fluids, decreasing physical activity, and ignoring the defecation urge

head and neck squamous cell carcinoma (HNSCC)

term used for cancers of the oral cavity, pharynx, and larynx

Parenteral nutrition is used when

the GI tract cannot be used for the ingestion, digestion, and absorption of essential nutrients

The goal of HCl acid suppression treatment is to reduce

the acidity of the gastric refluxate

For the tubercle bacillus to multiply and initiate a tissue reaction in the lungs, it must be deposited in: a. the alveoli. b. the bronchi. c. the trachea. d. all of the above.

the alveoli

Tonsillitis and Adenoiditis: Chronic

tonsilectomy

As protein intake is severely reduced, the muscles, which are the largest store of protein in the body, becomes

wasted and flabby, leading to weakness and fatigability.

Contributing factors to incontienence include

weakness or disruption of the internal or external anal sphincter, damage to the pudendal nerve or other nerves that innervate the anorectum, damage to the anal tissue, and trauma to the puborectalis muscle

Breath sounds that originate in the smaller bronchi and bronchioles and are high-pitched, sibilant, and musical are called: a. wheezes. b. rhonchi. c. rales. d. crackles.

wheezes

An individual with GERD may also report respiratory symptoms including

wheezing, coughing, and dyspnea

Hemorrhage occurs when in esophageal cancer?

when the cancer erods through the esophagus and into the aorta

Is h. pylori a risk factor for stomach cancer?

yes

Roux-en-Y Gastric Bypass (RYGB) Disadvantages:

• Leak at site of anastomosis • Anemia: iron deficiency, cobalamin deficiency, folic acid deficiency Calcium deficiency • Complication of the procedure: "Dumping syndrome:" where the contents get "dumped" or emptied too quickly into the small intestine and overwhelms the ability to digest the nutrients • Irreversible

Gastric Ulcer S/S:

• Occur less commonly than Duodenal (20%) • Eating increases pain, occurs about 1 to 2 hours after meals. The pain is "burning or gaseous". • Pain located high in the epigastrium • Patient has normal to decreased gastric secretions associated with weight loss because pain when eating • Vomiting and weight loss

Roux-en-Y Gastric Bypass (RYGB) Description:

• Surgery on stomach to create a pouch (restrictive) • Small gastric pouch connected to jejunum • Remaining stomach and first segment of small intestine are bypassed (malabsorptive)


Ensembles d'études connexes

Protestant reformation webquest and video analysis

View Set

ESC5 Lesson 6 Text Two - 狄晓晴

View Set

Maintaining a Healthy Body Composition and Body Image Assignment

View Set

Vistas 2, Chapter 9, ea-¿Qué? and ¿cuál? 1. Activity #98 Textbook, ¿Qué? and ¿cuál? Estructura 9.3

View Set

Myers Psychology 8e - Chapter 01

View Set

Chapter 1.2: How are our Ecological Footprints Affecting the Earth?

View Set

Toddlers/Preschoolers-Pediatric Nursing

View Set